Download as pdf or txt
Download as pdf or txt
You are on page 1of 331

S.

No Topics Types Page Number

Type - 1 1

Error Spotting (New Pattern) Type - 2 9


1.
Type - 3 16

Type - 4 23

Type -1 34

Type - 2 45
2. Phrase replacement
Type - 3 54

Type - 4 59

Type - 1 74

Type - 2 81

Type - 3 89
3. Fill In the Blanks
Type - 4 97

Type - 5 114
Type - 6 125
Type - 1 138

Type - 2 144

4. Sentence Correction Type - 3 157

Type - 4 169

Type - 5 181

Type - 1 193

Type - 2 205
5. Jumbled Sentence
Type - 3 230

Type - 4 269

6. Reading Comprehension Type - 1 283

7. Cloze Test Type - 1 295

8. Theme Detection Type - 1 303

9. Synonyms and Antonyms Type - 1 319

www.ibpsguide.com https://estore.ibpsguide.com Page 1 of 331


Error Spotting (New Pattern) - Type – 1

Directions (1-30): There is a sentence given below which is divided into four or five parts. There is an error in any
part(s) of the sentence. From the given options, find out the part which contain(s) error.

1). Even with the (A)/ most sophisticated weapons,(B)/ it is all but impossible to(C)/ defeat a militia that is well(D)/
entrenched upon a civilian population.(E)
(a) Only A and C (b) Only B (c) Only D (d) Only A (e) Only E

2). Right now, far from halting (A)/ the spreading of Iran’s influence,(B)/ the war has deepened the Houthi (C)/

reliance on Iran, which has(D)/ an easy and cheap means of tormenting the Saudis.(E)
(a) Only A and B (b) Only B and C (c) Only C and D (d) Only D and A (e) Only C and A

3). It has reportedly suggested (A)/ Northern Ireland could be given (B)/ new powers allowing it to follow same
(C)/ regulatory regime as the EU in areas (D)/ such as agriculture and energy, where there is most cross-border
trade.(E)
(a) Only A and B (b) Only B and C (c) Only A and C (d) Only D and E (e) Only B and D

4). Keurig Green Mountain, a maker of coffee machines(A)/, recently tweeted that it has halted(B)/ advertising for
a Fox News programme(C)/ whose host had appeared to defend Roy Moore,(D)/ a Senate candidate accused of
dating and assaulting teenagers.(E)

(a) Only A and B (b) Only B and C (c) Only B and D (d) Only D and E (e) Only C and E

5). In China, however, the migrants (A)/ most frequently grumbling about, (B)/ and treated with the greatest

hostility,(C)/ are not foreigners but other Chinese,(D)/ rural folk who moved to the cities in search of a better life.
(E)

(a) Only B and D (b) Only B and E (c) Only C and D (d) Only A and E (e) Only A and D

6). A few volunteers who have tried (A)/ to setup shelters for people that have found(B)/ themselves suddenly

homeless inside sub-zero(C)/ temperatures have been ordered by police to close them.(D)

(a) Only A and B (b) Only A and D (c) Only C and D (d) Only B and C (e) Only C and A

www.ibpsguide.com https://estore.ibpsguide.com Page 2 of 331


7). MiFID2 will make tradings more transparent (A)/ and oblige banks to charge clients separately in (B)/ research;

PSD2 will expose banks(C)/ to more competition from technology companies,(D)/ and each other, in everything
from payment services to budgeting advice.(E)
(a) Only A and B (b) Only B and C (c) Only C and D (d) Only D and E (e) Only E and A

8). Yemen’s infrastructure has been (A)/ crumbling for years, so it is difficult (B)/ for a visitor to tell among
buildings (C)/ that are falling down by neglect(D) and those half-levelled by explosions.(E)
(a) Only A and B (b) Only B and E (c) Only C and D (d) Only C and A (e) Only D and E

9). The more secular south adopted Marxism (A) and aligned itself with the Soviet Union.(B) Political feuds led to

further wars in 1972(C) and 1979, but economic hardship and the end of the cold war brought the sides
together.(D)

(a) Only A and B (b) Only B (c) Only C (d) Only C and B (e) Only A and D

10). Talks overseen by the UN led to (A)/ a plan of 2014 for a new constitution (B) enshrining a federal system
and a (C)/ parliament split between northerners and southerners.(D)
(a) Only A and C (b) Only B (c) Only C and D (d) Only D and B (e) Only A

11). In the south the Saudis along with the Emiratis,(A)/ which are the largest foreign force upon the (B)/ ground,
have built an unwieldy alliance (C)/ of Salafists, southern secessionists and other militias.(D)
(a) Only A (b) Only B (c) Only C and D (d) Only D and A (e) Only B and C

12). Ben Carson, the celebrated neurosurgeon (A)/ and unsuccessful presidential candidate, had no(B)
experience of political office or (C)/ housing policy before Donald Trump nominated him to lead the Department of

Housing and Urban Development.(D)/


(a) Only A and D (b) Only B and C (c) Only C and A (d) Only D and B (e) Only A and B

13). For anyone concerned that (A)/ American consumers have suffered from not (B)/ having enough financial
regulations on the beat,(C)/ there is now clear evidence of too much.(D)

(a) Only A and B (b) Only B and C (c) Only C and D (d) Only D and A (e) Only C and A

www.ibpsguide.com https://estore.ibpsguide.com Page 3 of 331


14). Mr. Cordray did little to resolve differences as(A) his tenure included disputed settlements, notably of car

finance,(B) that strayed far from the specified mission of the bureau, and heated debates(C) upon the regulation
of high-interest “payday” lenders.(D)
(a) Only A and B (b) Only B and C (c) Only C and D (d) Only D and B (e) Only C and A

15). On waking he seems to have grabbed his (A)/ phone to attack CNN, give air to an old conspiring (B)/ theory
and broadcast propaganda of a (C)/ hitherto obscure band of British xenophobes to his 43.6m Twitter followers(D)
(a) Only A and B (b) Only B and C (c) Only C and D (d) Only D and A (e) Only C and A

16). The nine justices of the Supreme Court (A)/ are used to applying 18th-century (B)/ principles to the America

that (C)/ would bewilder the constitution’s framers.(D/)


(a) Only A (b) Only B and C (c) Only C (d) Only D (e) Only D and A

17). Russians who think like Mr. Volkov have long looked to America (A)/ as the model of how a free press should
be because for them, (B)/ it is maddening to watch the news organisations they so admire (C)/ building Mr. Putin
into an all-powerful Bond villain, therein elevating (D)/ his stature among the many Russians who credited him
with making Russia a geopolitical player again.(E)
(a) Only A and B (b) Only B and C (c) Only C and D (d) Only D and E (e) Only E and A

18). As an illustration of what is ailing congressional Democrats, (A)/ Nancy Pelosi’s recent attempt to defend an
88-year-old (B)/ party grandee who was alleged to have shown up (C)/ to work in his pyjamas, fondled

generations of female (D)/ employees and to have asked at least one of them to “touch it”, was hard to beat. (E)/
(a) Only A and B (b) Only B and C (c) Only C and D (d) Only D and E (e) Only E and A

19. One custom in Mexico’s era of one party rule (A)/ was the dedazo (big finger), the president’s (B)/ choice of
his successor, that would (C)/ inevitably be elected upto a single six-year term.(D)

(a) Only A and B (b) Only B (c) Only C and D (d) Only D and A (e) Only C and A

20). Back in July, Nicolás Maduro’s big problem (A)/ was an opposition-backed rebellion with his (B)/ plan to

replace Venezuela’s elected parliament (C)/ with a hand-made constituent assembly (D)

(a) Only A and B (b) Only B and C (c) Only C and D (d) Only D and B (e) Only C and A

www.ibpsguide.com https://estore.ibpsguide.com Page 4 of 331


21. Juan Orlando Hernandez, Honduras’s (A)/ president, boasts off that he has brought stability (B)/ and security,

but his run (C)/ for re-election has caused turmoils. (D)
(a) Only A and B (b) Only B and C (c) Only C and D (d) Only D and B (e) Only C and A

22). Argentines have given up hope to find (A)/ alive the 44 crew upon the ARA San Juan, (B)/ the most modern
of the navy’s three (C)/ submarines, which disappeared on November 15th. (D)
(a) Only A and B (b) Only B (c) Only C (d) Only D and A (e) Only C and A

23). On October 30th the wheelchair held Mr. Rizvi and (A)/ around 5,000 supporters gathered at Faizabad

interexchange, (B)/ an important junction on the road between Islamabad,(C) the capital, and the nearby city of

Rawalpindi.(D)
(a) Only A and B (b) Only B and C (c) Only C and D (d) Only D and A (e) Only C and A

24). Following a brief hiatus during its testing programme,(A) North Korea launched a missile in the small
(B)hours of November 29th that, unlike any previous missile,(C) appeared to be able to strike any city of
America(D)
(a) Only A and B (b) Only B and C (c) Only C and D (d) Only D and A (e) Only C

25). The past 20 years have seen around 50 (A) attempts to pass law in different Australian(B) states to allow
doctors to help(C) terminally ill people ending their lives(D)
(a) Only A and D (b) Only B and C (c) Only B and D (d) Only C and A (e) Only A and B

26). The plot was made for Bollywood, a princess (A) so beautiful that a lustful prince besieged a (B) spectacular
fortress to catch her, and so virtituous (C) that she hurls herself into a fire rather than surrender. (D)

(a) Only A and B (b) Only B and C (c) Only C (d) Only D and A (e) Only C and A

27). The day after the disaster Cai Qi, the Communist Party (A) chief of Beijing municipality, announced a citywide

(B) fire-safety inspection and this quickly morphed upto (C) mass evictions, starting in the shantytowns. (D)
(a) Only A and B (b) Only B and C (c) Only C and D (d) Only D (e) Only C and A

28). The cartoons depicts the party (A) like a rabbit that is weak and small (B) at first but ultimately overcomes (C)
countless hurdles to become strong.(D)

(a) Only A and B (b) Only B and C (c) Only C and D (d) Only D and A (e) Only C and A

www.ibpsguide.com https://estore.ibpsguide.com Page 5 of 331


29). When it is finished, America’s imposing (A) new embassy in Lebanon will be its second-biggest (B) in the
world and yet it was France, not America, which (C)/ stepped up to resolve Lebanon’s latest political crisis. (D)
(a) Only A and B (b) Only B and C (c) Only C and D (d) Only D and A (e) Only C and A

30). On November 20th, in defiance of a law (A) introduced by his predecessor, Mr. Lourenço (B) fired a police
chief and (C) the head of the intelligence agency. (D)
(a) Only A and B (b) Only B and C (c) Only C (d) Only D (e) Only C and A

Answers:

1). Correct Answer is: (e)


Error in part E: Replace 'upon' with 'in'

2). Correct Answer is: (b)


Error in part B: Replace 'spreading' with 'spread'
Error in part C: Replace 'Houthi' with 'Houthi’s'
3). Correct Answer is: (c)
Error in part A: The correct phrase is ‘suggested that’
Error in part C: ‘follow the same’ is correct usage.

4). Correct Answer is: (b)


Error in part B: Replace 'has' with 'had'
Error in part C: Replace 'for' with 'in'

5). Correct Answer is: (b)


Error in part B: Replace 'grumbling' with 'grumbled'
Error in part E: Replace 'moved' with 'move'

6). Correct Answer is: (d)


Error in part B: Replace 'that' with 'who'

Error in part C: Replace 'inside' with 'in'

7). Correct Answer is: (a)


Error in part A: Replace 'tradings' with 'trading'

Error in part B: Replace 'in' with 'for'

8). Correct Answer is: (c)


Error in part C: Replace 'among' with 'between'

Error in part D: Replace 'by' with 'through'

www.ibpsguide.com https://estore.ibpsguide.com Page 6 of 331


9). Correct Answer is: (c)

Error in part C: Replace 'feuds' with 'feuding'


10). Correct Answer is: (b)
Error in part B: Replace 'of' with 'in'

11). Correct Answer is: (b)


Error in part B: Replace 'which' with 'who'
12). Correct Answer is: (c)
Error in part A: Replace 'the' with 'a'
Error in part C: Replace 'of' with 'in'

13). Correct Answer is: (c)

Error in part C: Replace 'regulations' with 'regulators'


Error in part D: Replace 'much' with 'many'

14). Correct Answer is: (d)


Error in part B: Replace 'of' with 'in'
Error in part D: Replace 'upon' with 'over'
15). Correct Answer is: (b)
Error in part B: Replace 'conspiring' with 'conspiracy'
Error in part C: Replace 'of' with 'from'

16). Correct Answer is: (c)


Error in part C: Replace 'the' with 'an'
17). Correct Answer is: (d)

Error in part D: Replace 'therein' with 'thereby'


Error in part E: Replace 'credited' with 'credit'
18). Correct Answer is: (e)

Error in part A: Replace 'is ailing' with 'ails'


Error in part E: Replace 'was' with 'is'

19). Correct Answer is: (c)

Error in part C: Replace 'that' with 'who'


Error in part D: Replace 'upto' with 'to'

20). Correct Answer is: (d)

Error in part B: Replace 'with' with 'against'


Error in part D: Replace 'made' with 'picked'

21). Correct Answer is: (d)

www.ibpsguide.com https://estore.ibpsguide.com Page 7 of 331


Error in part B: Remove ‘off’

Error in part D: Replace 'turmoils' with 'turmoil'


22). Correct Answer is: (a)
Error in part A: Replace 'to find' with 'of finding'

Error in part B: Replace 'upon' with 'aboard'


23). Correct Answer is: (a)
Error in part A: Replace 'held' with 'bound'
Error in part B: Replace 'interexchange' with 'interchange'
24). Correct Answer is: (d)

Error in part A: Replace 'during' with 'in'

Error in part D: Replace 'of' with 'in'


25). Correct Answer is: (c)

Error in part B: Replace 'law' with 'laws'


Error in part D: Replace 'ending' with 'end'
26). Correct Answer is: (b)
Error in part B: Replace 'besieged' with 'besieges'
Error in part C: Replace 'virtituous' with 'virtuous'
27). Correct Answer is: (c)

Error in part C: Replace 'upto' with 'into'


Error in part D: Replace 'shantytowns' with 'shantytown'
28). Correct Answer is: (a)

Error in part A: Replace 'depicts' with 'depict'


Error in part B: Replace 'like' with 'as'
29). Correct Answer is: (c)

Error in part C: Replace 'which' with 'that'


Error in part D: Replace 'up' with 'in'

30). Correct Answer is: (c)

Error in part C: Replace 'a' with 'the'

www.ibpsguide.com https://estore.ibpsguide.com Page 8 of 331


Error Spotting - Type – 2

Directions (31-60): In each of the following questions, a sentence is provided which is divided into five parts and
one of the parts is given in bolds. Consider the part given in bold to be grammatically correct, choose the option

from below which contains the part(s) having error.

31). The National Health Service is planning to pay recruitment agencies up to £100m to find 5,000 (A)/ doctors,

about half of them in overseas, to plug worsening staffing gaps and they plan to do (B)/ it with a recruitment drive,
over the next three-and-a-half years, which is intended to address a (C)/growing shortage of general practitioners

in England, as well as fulfill a pledge by Jeremy Hunt, (D)/the health secretary, to make GPs available to the

public seven days a week by 2020.(E)


(a) Only A and B (b) Only B and C (c) Only C and A (d) Only D and B

(e) The sentence is grammatically correct.

32). The US and UK governments have almost doubled (A)/ their requests to obtain data from technology, (B)/
media and telecom companies over the past three years (C)/ and this highlights a growing regulatory burden for
businesses (D)/ which are preparing for many more requests under tough new EU privacy rules. (E)/

(a) Only B (b) Only C (c) Only D (d) Only E (e) The sentence is grammatically correct.

33). The resignation of the third Scottish Labour leader in as many (A)/ years has created an opportunity for
leftwing supporters of (B)/ UK Labour leader Jeremy Corbyn to tighten his grip on the party (C)/ both north and

south of the border as the departure of the more moderate Kezia Dugdale could tilt (D)/ the balance of Labour’s
national executive committee decisively in favour of Mr. Corbyn. (E)/

(a) Only A and D (b) Only C and A (c) Only D and E (d) Only C (e) Only E

34). John Penrose, a backbench Conservative MP, attacked Ofgem (A)/ for only consulting on a “safeguard tariff”

to (B)/ protect 2.2m “vulnerable” customers in spite of promises made (C)/ by Theresa May, the prime minister,

before the June (D)/ general election to reduce energy bills for 17m households by £100. (E)/
(a) Only B (b) Only D (c) Only C and B (d) Only D and E (e) All are correct

35). Annual growth in consumer borrowing, which included (A)/ overdrafts, credit cards and car finance, dropped
below (B)/ double digits for the first time since April 2016 during (C)/ July, according to the latest money (D)/ and

debt statistics published by the Bank of England. (E)/

www.ibpsguide.com https://estore.ibpsguide.com Page 9 of 331


(a) Only A (b) Only B (c) Only A and B (d) Only D and E (e) Only D

36). Emmanuel Macron’s plummeted approval ratings (A)/ are a reminder that youth and a presidential bearing

are (B)/ not enough to secure popular support for pro-business (C)/ reforms in France yet Mr. Macron, who is
unveiling a (D)/ contentious jobs market bill today, might have to endure unpopularity for a while. (E)/
(a) Only B (b) Only A (c) Only A and B (d) Only B and E (e) Only D

37). The AfD and Chancellor Angela Merkel’s conservative CDU/CSU bloc (A)/ are engaged in a fierce fight for
the support of former Soviet immigrants (B)/ in the September 24 poll and such a contest is unprecedented as the
(C)/ Russian-Germans were once considered the Christian Democrats’ most loyalist constituency (D)/ but surveys

suggest many are switching to the AfD, attracted by its pro-Kremlin stance and hardline on Muslim refugees. (E)
(a) Only B (b) Only B and C (c) Only D (d) Only C and E (e) Only E

38). Shipbrokers in London said almost 40 cargoes of petrol had been booked or were being negotiated (A)/ so far
this week, well upon the usual volume, and traders were asking for flexibility to deliver to (B)/ the Atlantic
seaboard or the Gulf Coast depending on when ports reopen as the wreak made by (C)/Harvey has dented more
than a fifth of US oil refining capacity or roughly 4m barrels a day, (D)/sparking fears for fuel shortages and driving
wholesale petrol prices up by 20 per cent in the past week to the highest level in two years.(E)/

(a) Only A (b) Only A and C (c) Only C and D (d) Only D (e) Only E

39). The administration is preparing to lean heavily on congressional committees to pull together what (A)/ Mr.

Cohn described as the “flesh and bones” of the tax reforms and the so-called big six — Mr. (B)/ Cohn, Mr.
Mnuchin and four key Republican policymakers — have drafted a memorandum (C)/ setting out their vision for
reform but it leaves a welter of big questions unanswered, and these will (D)/have to be hammered out by

returning lawmakers on two committees — the House ways and means committee and the Senate finance
committee. (E)/

(a) Only A (b) Only B (c) Only D and B (d) Only D and E (e) Only E

40). Under different governments of Brazil’s Workers’ party, which ruled for 13 years until former (A)/ president

Dilma Rousseff was impeached last year for manipulating the budget, the BNDES grew (B)/ bigger than the World

Bank as the main provider of long-term finance in Latin America’s largest (C)/ economy and this was achieved
because the bank lent money to companies ranging from JBS to all owned by Jorge Paulo Lemann, Brazil’s

www.ibpsguide.com https://estore.ibpsguide.com Page 10 of 331


richest man (D)/ and one of the controlling shareholders of AB InBev, the world’s largest brewer, at interest rates

sometimes half those of the central bank’s benchmark Selic lending rate. (E)/
(a) Only B (b) Only A (c) Only A and D (d) Only D and E (e) Only D
41). Despite his grenade-throwing campaign, Mr. Trump (A)/ has not carried his worst threats (B)/ because as a

candidate he spoke about (C)/ slapping 45% tariffs on all Chinese goods and rewriting (D)/ or ditching the North
American Free-Trade Agreement with Canada and Mexico.(E)/

(a) Only C (b) Only B (c) Only D (d) Only D and E (e) Only E

42). The bank frauds involving Punjab National Bank (PNB) and the (A)/ companies associated

with businessmen Nirav Modi and Mehul Choksi as well as (B)/ the Rotomac case wouldn’t have come
at a worse time (C)/ as he Indian banking system is already reeling under the pressure of growing NPAs, (D)/ or
nonperforming assets (less politely known as loans that are not going to be repaid), which will touch
nearly 10 lakh crore by March this year. (E)/

(a) Only B (b) Only C (c) Only D and B (d) Only D (e) Only E

43). In this age of digital connectivity, an (A)/ error in judgment on a local page in a single edition(B)/
of the newspaper has the potential to (C)/ hurt the publication as a whole, and its fallin is (D)/

never restricted to that geographical region alone. (E)/

(a) Only A (b) Only B (c) Only D (d) Only D and C (e) Only B and D

44). Funding has failed to keep on with the needs (A)/ of a system pressurised by an expanding and (B)/ ageing

population, and advancing medical (C)/ technology and adding to this are the cuts that (D)/
have been made to social care budgets across the country (E)/
(a) Only A and B (b) Only B and C (c) Only D and B (d) Only D (e) Only A

45). Elena Aguilar came to America illegally from El Salvador (A)/ in 1996 to escape her children’s violent father

as earthquakes (B)/in her home country in 2001 brought her good fortune for a sort,(C) she was in 290,000

Salvadoreans who received (D)/ “temporary protected status” (TPS) from the American government.(E)/
(a) Only B (b) Only B and C (c) Only C and D (d) Only D and E

(e) Only E

46). On July4th 2012 news of the discovery of the Higgs boson (A)/ by researchers at CERN, Europe’s particle-

physics laboratory, electrified science (B)/ and the wider public as this particle, generated inside (C)/ the lab’s

www.ibpsguide.com https://estore.ibpsguide.com Page 11 of 331


Large Hadron Collider (LHC), was the last missing piece (D)/ of the Standard Model, one of the most successful

theories physicists had devised.(E)/


(a) Only B (b) Only C and B (c) Only D and B (d) Only D (e) Only E

47). Locally based fast-food chains that (A)/ underwrite McDonald’s or KFC have done (B)/much better than the
new (C)/ arrivals but local consumer businesses face much (D)/the same problem as multinationals.(E)

(a) Only C (b) Only B and C (c) Only D and E (d) Only D (e) Only B

48). North and South Korea had not spoken to one (A)/ another in almost two years and a bigger fact is (B)/ that

southern officials had been attempting (C)/ to use the hotline between the two governments (D)/ regularly, but the

North had refused to pick up. (E)/


(a) Only A (b) Only B (c) Only D and B (d) Only D and C (e) Only C

49). First, a father had to petition the courts (A)/ to be allowed to adopt his own biological son(B)/, who was born
in America with the (C)/help of a surrogate, and thus was not (D)/automatically, considered his child in

Singaporean law.(E)/
(a) Only A (b) Only B (c) Only D and B (d) Only D and E (e) Only E

50). An Aadhaar card allowed your correspondent to (A)/ apply for and walk away with a driver’s license in under
half (B)/an hour and it provides proof of address and other data that in other (C)/ countries and in pre-Aadhaar

India, would require a stack of documents but civil libertarians have long (D)/ worried that the government or,
worse, crooks who gain access to the data will put Aadhaar to nefarius use. (E)/
(a) Only B and C (b) Only B (c) Only D and C (d) Only D and E (e) Only E

51). The story of Japan’s modernization began 150 years ago (A)/ this month, when a band of young samurai (B)/
and their allies overthrew the Tokugawa shogunate (C)/ and with it seven centuries of feudal rule because (D)/

under the shoguns (military rulers), merchant and cultural life, centered upon bustling Edo, had been far away

stagnant, as the stunning woodblock prints of Hokusai and Hiroshige attest. (E)/
(a) Only A (b) Only B (c) Only D and B (d) Only D and E (e) Only E

52). Mr. Zhang’s story exemplifies the profound transformation in Chinese (A)/ agriculture that has been unfolding
since the 1980s (B)/ as it involves a shift away from a preoccupation with producing (C)/ enough grain for the

www.ibpsguide.com https://estore.ibpsguide.com Page 12 of 331


country’s needs, towards boosting (D)/ rural incomes by encouraging farmers to grow more profitable crops and

use scarce arable land more efficiently. (E)/


(a) Only A (b) Only B (c) Only D and B (d) Only D and E (e) Only E

53). China’s leaders no longer pay single minded (A)/ homage to the cult of GDP but although their (B)/ goals
have become more eclectic, they often(C)/ pursue them in much of the same way, by setting measurable,

(D)/quantitative targets that are used to rate and motivate provincial officials.(E)/
(a) Only A (b) Only B (c) Only D and B (d) Only D (e) Only E

54). There are more than 65m people in China who (A)/ share his surname—some would love to connect to their

(B)/ family branches to his bountiful tree but (C)/ constructing an accurate lineage could be tough,(D)/ not only
because of the huge number of Lius.(E)/

(a) Only A (b) Only B (c) Only D (d) Only D and C (e) Only C

55). As it became clear that Donald Trump would (A)/ win the presidential election, in the early hours(B)/ of 9th
November 2016, Asian financial markets (C)/ tanked but within hours of his victory(D)/ investors changed courses.
(E)/

(a) Only B and E (b) Only C (c) Only D and B (d) Only D and C (e) Only E

56). America’s judges are supposed to be above party (A)/ politics and yet often are appointed by politicians
(B)/and then asked to rule on disputes that can sway elections (C)/ as on January 9th federal judges in North

Carolina (D)/gave the state two weeks to redraw its congressional map.(E)/
(a) Only A (b) Only B (c) Only C and B (d) Only C and E (e) Only E

57). Thanks largely to Kim Jong Un (aka “Little Rocket Man”), missile defence (A)/of the American homeland is a
hot topic and owing to this, next month the (B)/ Trump administration is expected to publish a review of the

nation’s defences against ballistic-missile (C)/ attack and funding of the Missile Defence Agency (MDA) is likely to

exceed $11bn for 2018, (D)/ over $3bn more than the president’s original request (assuming Congress can come
up with a deal on the overall budget(E)/

(a) Only A (b) Only B (c) Only D and B (d) Only D (e) Only C

www.ibpsguide.com https://estore.ibpsguide.com Page 13 of 331


58). Any political system that puts great power in the hands of the (A)/ single person must also reckon with the

problem that creates (B)/ and it is sensible to make provision to remove the king or (C)/ president from office if he
becomes incapacitated, (D)/ yet the existence of such a provision also risks inviting a coup. (E)/

(a) Only A (b) Only B (c) Only D and B (d) Only D and C (e) Only C

59). A fan of military history, Stephen Bannon may know of (A)/ Nikephoros, a Byzantine emperor who was (B)/

vanquished and decapitated by a Bulgar (C)/ khan who, for extra humiliation, then (D)/ fashioned his skull upto a
drinking cup.(E)/
(a) Only A (b) Only B (c) Only C and B (d) Only C and E (e) Only E

60). On January 23rd it will invite Canadians to disclose their (A)/ cannabis habits anonymously through an app
but its (B)/ nosiness is entirely professional as Canada’s government,(C)/ led by Justin Trudeau, plans to legalize

(D)/the recreational use of cannabis in July1st. (E)/


(a) Only C (b) Only B (c) Only C and B (d) Only D and E (e) Only E

ANSWERS:
31). Correct Answer is: (b)
Error in part B: Replace 'in' with 'from'

Error in part C: Replace 'with' with 'by'


32). Correct Answer is: (d)
Error in part E: Replace 'which' with 'that'

33). Correct Answer is: (d)


Error in part C: Replace 'his' with 'their'
34). Correct Answer is: (e)

35). Correct Answer is: (a)


Error in part A: Replace 'included' with 'includes'

36). Correct Answer is: (b)

Error in part A: Replace 'plummeted' with 'plummeting'


37). Correct Answer is: (c)

Error in part D: Replace 'loyalist' with 'loyal'

38). Correct Answer is: (e)


Error in part E: Replace 'for' with 'of'

39). Correct Answer is: (b)

www.ibpsguide.com https://estore.ibpsguide.com Page 14 of 331


Error in part B: Replace 'described' with 'describes'

40). Correct Answer is: (e)


Error in part D: Replace 'all' with 'these'
41). Correct Answer is: (b)

Error in part B: ‘carried his worst’ is incorrect usage. ‘carried out his’ is correct usage.
42). Correct Answer is: (b)
Error in part C: Replace 'wouldn’t' with 'couldn’t'
43). Correct Answer is: (c)
Error in part D: Replace 'fallin' with 'fallout'

44). Correct Answer is: (e)

Error in part A: Replace 'keep on' with 'keep up'


45). Correct Answer is: (c)

Error in part C: Replace 'for' with 'of'


Error in part D: Replace 'in' with 'among'
46). Correct Answer is: (e)
Error in part E: Replace 'had' with 'have'
47). Correct Answer is: (e)
Error in part B: Replace 'underwrite' with 'undercut'

48). Correct Answer is: (b)


Error in part B: Replace 'bigger' with 'greater'
49). Correct Answer is: (e)

Error in part E: Replace 'in' with 'under'


50). Correct Answer is: (e)
Error in part E: Replace 'nefarius' with 'nefarious'

51). Correct Answer is: (e)


Error in part E: Replace 'away' with 'from'

52). Correct Answer is: (a)

Error in part A: Replace 'the' with 'a'


53). Correct Answer is: (d)

Error in part D: Replace 'much of the' with 'much the'

54). Correct Answer is: (b)


Error in part B: Replace 'connect to their' with 'connect their'

55). Correct Answer is: (e)

www.ibpsguide.com https://estore.ibpsguide.com Page 15 of 331


Error in part E: Replace 'courses' with 'course'

56). Correct Answer is: (b)


Error in part B: Replace 'often are' with 'are often'
57). Correct Answer is: (d)

Error in part D: Replace 'of' with 'for'


58). Correct Answer is: (a)
Error in part A: Replace 'the' with 'a'
59). Correct Answer is: (e)
Error in part E: Replace 'upto' with 'into'

60). Correct Answer is: (e)

Error in part E: Replace 'in' with 'by'


Error Spotting - Type – 3

Direction (61-90): There is a sentence given below which is divided into four or five parts. There is an error in any
two parts of the sentence. From the given options, find out the errorless combination.

61). Despite the Paris agreement’s ambiguities and some setbacks,(A)/ including President Donald Trump’s
decision to yanky(B)/ America out of the deal, the air about self-congratulation(C)/ was still on show among those

who gathered(D)/ in Bonn this month for a follow-up summit.(E)/


(a) A-B-C (b) C-D-E (c) E-B-A (d) B-C-D (e) D-E-A

62). As Chinese officials admit, one of the (A)/ biggest threats against the country’s (B)/ financial stability is the
reckless (C)/ build-up of local government debt.(D)
(a) A- B (b) B – C (c) C – D (d) D – B (e) A- D

63). You may think that an airline’s most valuable (A)/ asset is the planes but with Monarch, Britain’s (B)/ fifth-

biggest carrier, which went burst in October, (C)/ creditors were keenest to claim slices of (D)/ airspace at

particular times of day (E)/


(a) A- B- C (b) B- D- C (c) A- C- E (d) D – E – A (e) A- B- D

64). Some clinics buy time for puberty-blockers, (A) which suppress the action upon (B) sex hormones but these
may (C) have harmful side-effects. (D)

(a) A-C (b) C- D (c) A-B (d) B-D (e) B-C

www.ibpsguide.com https://estore.ibpsguide.com Page 16 of 331


65). Climate scientists like Mr. Henderson have been discussing (A)/ negative-emissions technologies (NETs)
with economists and policy (B)/ wonks since the 1990s and fortunately, their debate has turned (C)/ lively since
the Paris agreement, the phrasing of which strongly(D)/ suggests that countries will need to invent new sinks as

well as cut emissions. (E)/


(a) A-C- B (b) C-D- A (c) A-B- D (d) B-D- E (e) B-C-D

66). By and large, women in Japan work: 68% of (A) those aged 15 to 64 are employed or looking for a job, (B) a
similar figure in America and the chart of the proportion of women (C) in work by age is still “m-shaped”, because

women drop (D) out of work when they marry or have kids before returning later on. (E)

(a) A-B-C (b) C-D-E (c) A-B-E (d) B-D-C (e) B-C-E

67). Nascar is not the first place one would look into (A) find a voice for the resistance, yet after Donald Trump (B)
railed against NFL players’ kneeling during the (C) national anthem, the biggest name in the sport took to Twitter
with a rebuttal. (D)
(a) A-C (b) C-D (c) A-B (d) B-D (e) B-C

68). Vladimir Putin appears to stride the globe like a (A)/ political goliath, a mix of Peter the Great and Joseph

Stalin (B)/ and whether along Syria or Ukraine, the Russian President (C)/ flexes his muscles and the world
seems to quake in anxiety. (D)/
(a) B-C (b) C-A (c) A-B (d) B-D (e) D-C

69). Mr. Smith’s National Party is now out of (A) government but the real villain (B) behind New Zealand’s
deteriorated(C) water quality are still at large: cows. (D)

(a) A-C (b) C-D (c) A-B (d) D-A (e) B- C

70). Throughout its steep climb away from (A) Delhi, the plane remained enveloped (B) in an acid fug and it was

(C) not until it reached cruised altitude that sunlight could break through. (D)
(a) A-B (b) C-B (c) A-D (d) B-D (e) A-C

71). The court that heard Yanzi’s case did not rule (A) that the treatment he was given was itself illegal (B)—only
that the clinician had lacked a license and engaged (C) in false advertising by offering a cure for something which

was not an illness.(D)

www.ibpsguide.com https://estore.ibpsguide.com Page 17 of 331


(a) B-C (b) A-D (c) D-B (d) C-D (e) B-A

72). In the five years following the end of the recession (A) since June 2009 wages and salaries rose by only
8.7%, while prices increased by 9.5% (B) and in 2014 the median worker’s inflation-adjusted (C) earnings, by a

measure, were no higher than they were in 2000.(D)


(a) A-B (b) C-D (c) C-B (d) A-C (e) D-B

73). She had mostly stopped cooking and cleaning for herself (A) and began, every so often, to get off lost on (B)
her way home against the neighborhood coffee shop (C)— the one she’d been to a least a hundred times. (D)

(a) A-B (b) C-B (c) A-D (d) B-D (e) B-C

74). For social scientists the apparent epidemic of propositioning, (A) pinching, groping and flashing which is (B)

gripping America brings a rare opportunity to observe a new norm, (C) around how men behave towards women,
being created in real times. (D)
(a) B-C (b) C-D (c) A-B (d) A-C (e) D-B

75). The buddy seat on Rick Kimberley’s (A) combine-harvester is a fine (B) vintage point from which (C) to
observe precision farm. (D)

(a) A-C (b) C-D (c) A-B (d) B-D (e) B-C

76). After he was busted in 1974, Jeffrey Edmondson, (A) a small-time dealer in marijuana, cocaine and (B)

amphetamines in Minneapolis, faced a daunting (C) bill by the taxman for all his illicit income. (D)
(a) D-C (b) B-D (c) C-B (d) A-C (e) B-A

77). If Doug Jones becomes the first Democrat (A) in a quarter of the century to win a Senate seat from (B)
Alabama on December 12th, the state would (C) have followed his own political evolution. (D)

(a) A-C (b) C-D (c) A-D (d) B-D (e) B-C

78). In the annals of (A) Latin American democracy, Marcelo Odebrecht, the (B) Brazilian construction magnet,(C)

will occupy a place of unique infamy. (D)

(a) A-C (b) A-D (c) D-B (d) B-D (e) B-C

www.ibpsguide.com https://estore.ibpsguide.com Page 18 of 331


79). The flurry of elections coming up in Latin America (A) will not only choose new leaders, this will also provide

a check-up on (B) the health of democracy itself, which in most countries (C) in the continent has been in place
for only a few decades. (D)
(a) A-C (b) C-D (c) A-B (d) B-D (e) B-C

80). Whether Mr. Mugabe formally hands over or is kept on as a (A) kind of ceremonial president is barely
relevant, though it would be neat (B) if the old man was ushered into as dignified a (C) retirement as soon as is
feasible in these ugly, humiliating circumstances.(D)
(a) B-C (b) B-D (c) C-A (d) C-D (e) A-D

81). Despite inheriting a powerful surname—not to mention a vast construction company, (A) Saudi Oger—he
often bemoan his lot and seemed to (B) live in the shadow of his father, Rafik, a former prime minister, (C) whose

assassination in 2005 (probably at the betest of Syria) propelled Saad into politics (D)
(a) D-C (b) A-D (c) A-C (d) B-D (e) B-C

82). Mr. Steinmeier’s remarks underscored the commitment to European union, (A) one of the twin pillars in
German foreign policy since 1945, (B) alongside participation in a multilateral world underwritten by America but
(C) warm words cannot disguise the fact that these days both pillars are shaky. (D)

(a) A-C (b) C-D (c) A-B (d) B-D (e) B-C

83). He even attended parliament in the outfit of the Magyar Garda, Jobbik’s (A) now disbanded and, to many,

deeply sinister, uniformed wing but (B) as Hungary starts to gear up for an election due to next (C) April, Mr. Vona
and Jobbik claim to had mellowed. (D)
(a) A-B (b) C-D (c) A-C (d) B-C (e) D- A

84). Hundreds of Western-trained Cypriot lawyers and (A) accountants earn a living by (B) handling the affairs in

Russian (C) and Ukrainian onshore companies. (D)

(a) A-C (b) C-D (c) A-B (d) B-D (e) B-C

85). The last to do so, Selahattin Demirtas, joint leader of (A) the pro-Kurdish Peoples’ Democratic Party (HDP)

and the (B) candidate of the 2014 presidential election, was thrown in (C) prison last year on spurious terrorism
charges (D)

(a) D-C (b) C-A (c) A-D (d) B-A (e) B-D

www.ibpsguide.com https://estore.ibpsguide.com Page 19 of 331


86). Over 500 days have passed since the BREXIT referendum (A) in June 2016, and less than 500 days are left
until Britain is (B) due to leave the European Union, yet progress (C) in the BREXIT negotiations has been
agonizingly slower. (D)

(a) B-C (b) A-D (c)D-B (d) A-C (e) B-C

87). Behind the grimy frosted windows of an (A) abandoned shop front on the backstreets of central (B) London
lies a plush modern office, full of banks with (C) computer screens monitoring Iran’s internet output.(D)
(a) D-B (b) A-D (c) C-B (d) C-D (e) B-A

88). Britain should have been better placed (A) than every other country to fight (B) off a populist fever (C) that is
spreading around the world. (D)

(a) A-B (b) C-A (c) D-B (d) A-D (e) C-D

89). They may transition from a male identity into (A) a female one, or vice versa, perhaps taking sex (B)
hormones and having surgery to make their (C) bodies match as they feel and want to be seen.(D)
(a) A-C (b) C-D (c) A-B (d) B-D (e) B-C

90). When rumors swirled in August that Baidu, (A)/ the Chinese online-search giant, was buying Toutiao, (B)/ the
scrappy news-aggregation platform reportedly quipped (C)/ off response that reports had mistaken the buyer for
the seller.(D)/

(a) A-D (b) C-B (c) D-B (d) C-D (e) A-C

ANSWERS:

61). Correct Answer is: (e)


Error in part B: Replace 'yanky' with 'yank'

Error in part C: Replace 'about' with 'on'

62). Correct Answer is: (e)


Error in part B: Replace 'against' with 'to'.

Error in part C: Replace 'the' with 'a'

63). Correct Answer is: (d)


Error in part B: Replace 'the' with 'its'

Error in part C: Replace 'burst' with 'bust'

www.ibpsguide.com https://estore.ibpsguide.com Page 20 of 331


64). Correct Answer is: (b)

Error in part A: Replace 'for' with 'with'


Error in part B: Replace 'upon' with 'of'
65). Correct Answer is: (a)

Error in part E: Replace 'cut' with 'cutting'


Error in part D: Replace 'lively' with 'livelier'
66). Correct Answer is: (c)
Error in part C: Replace 'in' with 'to'
Error in part D: Replace 'because' with 'as'

67). Correct Answer is: (b)

Error in part A: Replace 'into' with 'to'


Error in part B: Replace 'for' with 'of'

68). Correct Answer is: (c)


Error in part C: Replace 'along' with 'in'
Error in part D: Replace 'in' with 'with'
69). Correct Answer is: (d)
Error in part B: Replace 'villian' with 'villians'
Error in part C: Replace ' deteriorated ' with ' deteriorating'

70). Correct Answer is: (a)


Error in part C: Replace 'acid' with 'acrid'
Error in part D: Replace 'cruised' with 'cruising'

71). Correct Answer is: (e)


Error in part C: ‘license and had’ is correct usage
Error in part D: Replace 'which' with 'that'

72). Correct Answer is: (d)


Error in part B: Replace 'since' with 'in'

Error in part D: Replace 'a' with 'one'

73). Correct Answer is: (c)


Error in part C: Replace 'against' with 'from'

Error in part B: Omit ‘off’

74). Correct Answer is: (d)


Error in part B: Replace 'which' with 'that'

Error in part D: Replace 'times' with 'time'

www.ibpsguide.com https://estore.ibpsguide.com Page 21 of 331


75). Correct Answer is: (c)

Error in part C: Replace 'vintage' with 'vantage'


Error in part D: Replace 'farm' with 'farming'
76). Correct Answer is: (d)

Error in part B: Replace 'in' with 'of'


Error in part D: Replace 'by' with 'from'
77). Correct Answer is: (c)
Error in part B: Replace 'the' with 'a'
Error in part C: Replace 'would' with 'will'

78). Correct Answer is: (b)

Error in part B: Replace 'the' with 'a'


Error in part C: Replace 'magnet' with 'magnate'

79). Correct Answer is: (a)


Error in part B: Replace 'this' with 'it'
Error in part D: Replace 'on' with 'in'
80). Correct Answer is: (e)
Error in part B: Replace 'neat' with 'neater'
Error in part C: Replace 'was' with 'were'

81). Correct Answer is: (c)


Error in part B: Replace 'bemoan' with 'bemoaned'
Error in part D: Replace 'betest' with 'behest'

82). Correct Answer is: (b)


Error in part A: Replace 'the' with 'a'
Error in part B: Replace 'in' with 'of'

83). Correct Answer is: (a)


Error in part C: Replace 'to' with 'by'

Error in part D: Replace 'had' with 'have'

84). Correct Answer is: (c)


Error in part C: Replace 'in' with 'of'

Error in part D: Replace 'onshore' with 'offshore'

85). Correct Answer is: (c)


Error in part B: Replace 'the' with 'a'

Error in part C: Replace 'of' with 'in'

www.ibpsguide.com https://estore.ibpsguide.com Page 22 of 331


86). Correct Answer is: (d)

Error in part B: Replace 'less' with 'fewer'


Error in part D: Replace 'slower' with 'slow'
87). Correct Answer is: (b)

Error in part B: Replace 'on' with 'in'


Error in part C: Replace 'banks with' with 'banks of'
88). Correct Answer is: (d)
Error in part B: Replace 'every ' with 'any'
Error in part C: Replace 'a' with 'the'

89). Correct Answer is: (e)

Error in part A: Replace 'into' with 'to'


Error in part D: Replace 'As' with 'how'

90). Correct Answer is: (d)


Error in part A: Replace 'off' with 'in'
Error in part B: Replace 'the' with 'a'

Error Spotting - Type – 4

Directions (91-120): Following question consists of two sentences. Read each sentence to find out whether there
is any grammatical error in it and mark your answer accordingly from the given options.

91). I. Ukraine’s grubby politicians and oligarchs have tried to frustrate Western aim without openly defying them.
II. The EU should make plain that the benefits of the association pact depend on progress against graf.
(a) if there is an error only in the first sentence;
(b) if there is an error only in the second sentence;
(c) if there are errors in both sentences; and

(d) if there is no error in either of the sentences.

(e) If there are more than two errors in either of the sentence.

92). I. Some political theorists argue that the law draws legitimacy not just from voting, but also from public debate

before legislation is passed.


II. After this bill, a great fear of Founding Fathers, a politics of warring factions and interests, scrapping over the

public purse, looks closer than ever.

www.ibpsguide.com https://estore.ibpsguide.com Page 23 of 331


(a) if there is an error only in the first sentence;

(b) if there is an error only in the second sentence;


(c) if there are errors in both sentences; and

(d) if there is no error in either of the sentences.

(e) If there are more than two errors in either of the sentence.

93). I. In announcing this week that America recognized Jerusalem as the capital of Israel, and would start the
process of moving the American embassy there from Tel Aviv, President Donald Trump claimed to be honoring
Israel’s democracy.

II. One would manage ties with Israel and the other in East Jerusalem would deal with the Palestinian state, which

he should also recognize.


(a) if there is an error only in the first sentence;
(b) if there is an error only in the second sentence;
(c) if there are errors in both sentences; and
(d) if there is no error in either of the sentences.
(e) If there are more than two errors in either of the sentence.

94). I. Two letters can add up to a lots of money.

II. A competing vision of AI stresses simulations, in which machines teach themselves using synthetic data or in
virtual environments.
(a) if there is an error only in the first sentence;

(b) if there is an error only in the second sentence;


(c) if there are errors in both sentences; and
(d) if there is no error in either of the sentences.
(e) If there are more than two errors in either of the sentence.

95). I. America has had fraught relations with it for years; under Mr. Trump, frustration has turned to aggression.

II. The tariffs it keeps threatening would raise prices for its own consumers.
(a) if there is an error only in the first sentence;

(b) if there is an error only in the second sentence;


(c) if there are errors in both sentences; and
(d) if there is no error in either of the sentences.

(e) If there are more than two errors in either of the sentence.

www.ibpsguide.com https://estore.ibpsguide.com Page 24 of 331


96). I. A 5,000-strong crowd at the Communist Party rally at Nelspruit Rugby Club in Mpumalanga, South Africa’s
north-eastern province, picks up the chant.
II. The nub in the “state capture” argument is that Mr. Zuma and his friends are putting state-owned enterprises

and other governmental institutions to the hands of people who are allowing them looting public funds.
(a) if there is an error only in the first sentence;

(b) if there is an error only in the second sentence;


(c) if there are errors in both sentences; and
(d) if there is no error in either of the sentences.

(e) If there are more than two errors in either of the sentence.

97). I. If opinion polls are to be believed, the BJP has reason to worry.

II. India’s national parliament, the Lok Sabha, sits for barely 70 days a year
(a) if there is an error only in the first sentence;
(b) if there is an error only in the second sentence;

(c) if there are errors in both sentences; and


(d) if there is no error in either of the sentences.
(e) If there are more than two errors in either of the sentence.

98). I. Last year an 83-year-old Japanese man, who had survived prostate cancer and heart surgery, asked to
retire.

II. Japan’s government has granted the emperor’s wish to a manifest lack of enthusiasm.
(a) if there is an error only in the first sentence;
(b) if there is an error only in the second sentence;

(c) if there are errors in both sentences; and


(d) if there is no error in either of the sentences.

(e) If there are more than two errors in either of the sentence.

99). I. South Africa’s Constitutional Court may be the world’s most emotionally powerful building..

II. The ANC leadership election offers what ought to be an easy choice.

(a) if there is an error only in the first sentence;


(b) if there is an error only in the second sentence;

(c) if there are errors in both sentences; and

www.ibpsguide.com https://estore.ibpsguide.com Page 25 of 331


(d) if there is no error in either of the sentences.

(e) If there are more than two errors in either of the sentence.

100). I. For decades Cuban exiles in Miami dreamed the day that Fidel Castro would die.

II. For a while, he seemed to offer the prospect of far-reaching economic reform
(a) if there is an error only in the first sentence;

(b) if there is an error only in the second sentence;


(c) if there are errors in both sentences; and
(d) if there is no error in either of the sentences.

(e) If there are more than two errors in either of the sentence.

101). I. Everyone knew this year’s summit of the Gulf Co-operation Council (GCC) would be contentious

II. The Kuwaitis felt snubbed, though Qatar sent its emir, other members dispatch mere cabinet ministers.
(a) if there is an error only in the first sentence;
(b) if there is an error only in the second sentence;

(c) if there are errors in both sentences; and


(d) if there is no error in either of the sentences.
(e) If there are more than two errors in either of the sentence.

102). I. On December 4th Ali Abdullah Saleh, Yemen’s former dictator, was killed outside the capital Sana’a,
which has been paralyzed by a week of fighting.

II. The rebels will try to consolidate power in the capital, where tens of thousands of their supporters jubilated after
Mr. Saleh’s death.
(a) if there is an error only in the first sentence;
(b) if there is an error only in the second sentence;
(c) if there are errors in both sentences; and

(d) if there is no error in either of the sentences.

(e) If there are more than two errors in either of the sentence.

103). I. The catalyst of the latest upheaval is Mikheil Saakashvili, a former president of Georgia who entered

Ukrainian politics after the Maidan uprising to help fight corruption


II. The hope that Ukraine’s corrupt elites could themselves reform Ukraine and introduce the law’s rule was never

high.

www.ibpsguide.com https://estore.ibpsguide.com Page 26 of 331


(a) if there is an error only in the first sentence;

(b) if there is an error only in the second sentence;


(c) if there are errors in both sentences; and

(d) if there is no error in either of the sentences.

(e) If there are more than two errors in either of the sentence.

104). I. Thirty years ago, Ronald Reagan and Mikhail Gorbachev signed the Intermediate-Range Nuclear Forces
(INF) treaty, banished an entire category of destabilising weapons from Europe.
II. Steven Pifer, a former arms-control negotiator at the Brookings Institution, a think-tank, says that it makes no

sense to give Mr. Putin exactly what he wants.

(a) if there is an error only in the first sentence;


(b) if there is an error only in the second sentence;
(c) if there are errors in both sentences; and
(d) if there is no error in either of the sentences.
(e) If there are more than two errors in either of the sentence.

105). I. Russia’s recession had taken its toll on Yuri Dromashko, an entrepreneur from the Siberian city of Irkutsk.
II. In October, President Vladimir Putin ordered his officials to draw up the regulatory framework.

(a) if there is an error only in the first sentence;


(b) if there is an error only in the second sentence;

(c) if there are errors in both sentences; and


(d) if there is no error in either of the sentences.
(e) If there are more than two errors in either of the sentence.

106). I. A federal court in New York is not the obvious setting for a trial that has sent panic through the Turkish
establishment..

II. In the event of a guilty verdict, Halkbank faces the prospect of major fines over its alleged role in the sanctions-

busting.
(a) if there is an error only in the first sentence;

(b) if there is an error only in the second sentence;


(c) if there are errors in both sentences; and
(d) if there is no error in either of the sentences.

(e) If there are more than two errors in either of the sentence.

www.ibpsguide.com https://estore.ibpsguide.com Page 27 of 331


107). I. Stand in Fozzano (pictured), a hamlet of stone houses perched above the spectacular western coast of
Corsica, and it is easy to see why locals think of themselves as special.
II. Places like Corsica and Brittany, which also harbored a regionalist movement, could be given more autonomy

without threatening the French state.


(a) if there is an error only in the first sentence;

(b) if there is an error only in the second sentence;


(c) if there are errors in both sentences; and
(d) if there is no error in either of the sentences.

(e) If there are more than two errors in either of the sentence.

108). I. For much long the last man standing in Europe, Germany has suddenly become its Sleeping Beauty.

II. Germany’s conviction that the problems of the euro zone are merely those of ill-disciplined states, encouraged
by a recklessly expansionary monetary policy, has only grown.
(a) if there is an error only in the first sentence;
(b) if there is an error only in the second sentence;
(c) if there are errors in both sentences; and
(d) if there is no error in either of the sentences.
(e) If there are more than two errors in either of the sentence.

109). I. Germany’s conviction that the problems of the euro zone are merely those of ill-disciplined states,

encouraged by a recklessly expansionary monetary policy, has only grown


II. There is just enough time to find a fudged form of words that would allow Mrs May to meet her hopes of
sufficient progress.

(a) if there is an error only in the first sentence;


(b) if there is an error only in the second sentence;

(c) if there are errors in both sentences; and

(d) if there is no error in either of the sentences.


(e) If there are more than two errors in either of the sentence.

110). I. Any hope that a chancellor of the exchequer that is a former defence secretary might have taken a
sympathetic view of the plight of Britain’s squeezed armed forces was dashed in the budget on November 22nd

www.ibpsguide.com https://estore.ibpsguide.com Page 28 of 331


II. he government had promised not to go lower than 82,000 soldiers, but the current figure is around 77,400 and

there are fears that it could go as low as 60,000


(a) if there is an error only in the first sentence;

(b) if there is an error only in the second sentence;


(c) if there are errors in both sentences; and
(d) if there is no error in either of the sentences.
(e) If there are more than two errors in either of the sentence.

111). I. The country is more divided than it has been for decades, with the rich consolidating their power and

people who are born inside the wrong class or region seeing their chances of getting ahead declining.

II. The Social Mobility Commission produced a series of excellent reports that suggested sensible solutions such
as better early education for disadvantaged children

(a) if there is an error only in the first sentence;


(b) if there is an error only in the second sentence;
(c) if there are errors in both sentences; and
(d) if there is no error in either of the sentences.
(e) If there are more than two errors in either of the sentence.

112). I. Shirin Musa draws on bitter experience to inspire her work to help women caught between legal and
cultural worlds
II. England has an elaborate subculture where Islamic family law is practiced.

(a) if there is an error only in the first sentence;


(b) if there is an error only in the second sentence;

(c) if there are errors in both sentences; and


(d) if there is no error in either of the sentences.
(e) If there are more than two errors in either of the sentence.

113). I. Mention polygamy in Canada and what might come to mind is Bountiful, a suitably named town in British
Columbia.

II. The debate on the conflicting principles of human rights and religious freedom is shifting to Islamic immigrants.

(a) if there is an error only in the first sentence;


(b) if there is an error only in the second sentence;

(c) if there are errors in both sentences; and

www.ibpsguide.com https://estore.ibpsguide.com Page 29 of 331


(d) if there is no error in either of the sentences.

(e) If there are more than two errors in either of the sentence.

114). I. Commanding the plot lines in Hollywood films, covers of magazines and reams of newsprint, the contest

between artificial intelligence (AI) and mankind draws much attention.


II. One way to understand AI’s potential impact is to look at databases
(a) if there is an error only in the first sentence;

(b) if there is an error only in the second sentence;


(c) if there are errors in both sentences; and

(d) if there is no error in either of the sentences.

(e) If there are more than two errors in either of the sentence.

115). I. The most frenzying rush is for human talent, which is far more scarce than either data or computing
power.
II. If companies can lure the right people in AI, the effect is to extend their workforces exponentially.
(a) if there is an error only in the first sentence;
(b) if there is an error only in the second sentence;

(c) if there are errors in both sentences; and


(d) if there is no error in either of the sentences.
(e) If there are more than two errors in either of the sentence.

116). I. A decade ago the idea of paying real money against virtual items was strange and exotic.
II. A new twist on that model has been attracting the attention of regulators in recent weeks.
(a) if there is an error only in the first sentence;
(b) if there is an error only in the second sentence;
(c) if there are errors in both sentences; and

(d) if there is no error in either of the sentences.

(e) If there are more than two errors in either of the sentence.

117). I. Many split video games into smaller chunks and charge separately for each, selling a base game for $60,

then releasing extra downloadable content a few months later.


II. The backlash against loot boxes is not coming only from regulators.

(a) if there is an error only in the first sentence;

www.ibpsguide.com https://estore.ibpsguide.com Page 30 of 331


(b) if there is an error only in the second sentence;

(c) if there are errors in both sentences; and


(d) if there is no error in either of the sentences.

(e) If there are more than two errors in either of the sentence.

118). I. Rio Tinto, one of the world’s largest mining firms, is leading that transformation in its vast iron-ore
operations in the Pilbara region of Western Australia.
II. In a sign of their concern, some big shareholders railed for the possibility that Sir MickDavis, a deal making
supremo who once led Xstrata, another mining company (and who is now the chief executive of Britain’s

Conservative Party), would be named as chairman.

(a) if there is an error only in the first sentence;


(b) if there is an error only in the second sentence;
(c) if there are errors in both sentences; and
(d) if there is no error in either of the sentences.
(e) If there are more than two errors in either of the sentence.

119). I. If you look back at how people described Walmart a decade ago, it is eerily similar to how Amazon is
viewed now

II. Walmart is probably the most formidable adversary Amazon has ever faced.
(a) if there is an error only in the first sentence;
(b) if there is an error only in the second sentence;

(c) if there are errors in both sentences; and


(d) if there is no error in either of the sentences.
(e) If there are more than two errors in either of the sentence.

120). I. Despite Mr. Trump’s threats, America does not seem on the verge of crashing out of a system it helped to

construct, to rely entirely on bilateral trade deals and remedies

II. As the Trump administration kicks at the working leg of a limping institution, it is worth a recall that previous
American administrations have also felt frustrated with the WTO

(a) if there is an error only in the first sentence;

(b) if there is an error only in the second sentence;


(c) if there are errors in both sentences; and

(d) if there is no error in either of the sentences.

www.ibpsguide.com https://estore.ibpsguide.com Page 31 of 331


(e) If there are more than two errors in either of the sentence.

ANSWERS:
91). Correct Answer is: (a)

Explanation: ‘Aims’ is correct usage instead of ‘aim’ because it is talking of western aims.
92). Correct Answer is: (b)
Explanation: In second sentence ‘the founding fathers’ is correct usage because specific founding fathers are

being talked here.


93). Correct Answer is: (d)

Explanation: Both the sentences are grammatically correct.

94). Correct Answer is: (a)


Explanation: ‘a lots of money is incorrect usage. The correct usage here is ‘a lot of money’.

95). Correct Answer is: (d)


Explanation: Both the sentences are grammatically correct.
96). Correct Answer is: (e)
Explanation: ‘The nub in the’ is incorrect usage. The correct usage is ‘The nub of the’.
Also, ‘allowing them looting’ is incorrect usage. ‘allowing them to loot’ is correct usage.

Also, ‘in the hands’ is correct usage.


Hence, more than two errors are there in sentence 2nd. So, option e is correct.
97). Correct Answer is: (d)
Explanation: The sentences are correct.

98). Correct Answer is: (c)


Explanation: In, 1st ‘who had’ is incorrect usage. The correct usage is ‘who has’.
In, 2nd ‘wish to a’ is incorrect usage. ‘wish with a’ is correct usage.

99). Correct Answer is: (d)


Explanation: Both are correct.

100). Correct Answer is: (a)

Explanation: In first sentence, ‘dreamed of the day’ is correct usage.


101). Correct Answer is: (b)

Explanation: In 2nd, ‘dispatch’ is incorrect usage. The correct usage is ‘dispatched’.

102). Correct Answer is: (d)


Explanation: Both are correct.

103). Correct Answer is: (b)

www.ibpsguide.com https://estore.ibpsguide.com Page 32 of 331


Explanation: In 2nd, ‘law’s rule’ is incorrect usage. ‘rule of law’ is correct usage.

104). Correct Answer is: (a)


Explanation: In 1st, ‘banished’ is incorrect usage. ‘banishing’ is correct usage.
105). Correct Answer is: (b)

Explanation: ‘draw up a’ is incorrect usage. ‘draw up a’ is correct usage.


106). Correct Answer is: (d)
Explanation: Both are correct.
107). Correct Answer is: (b)
Explanation: ‘harbored’ is incorrect usage. The correct usage is ‘harbors’.

108). Correct Answer is: (a)

Explanation: ‘for much long’ is incorrect usage. ‘for so long’ is correct usage.
109). Correct Answer is: (d)

Explanation: Both are correct.


110). Correct Answer is: (e)
Explanation: In sentence 1, ‘that is a former’ is incorrect usage. ‘who is a former’ is correct usage.
‘dashed in’ is also incorrect. ‘dashed by’ is correct usage.
111). Correct Answer is: (c)
Explanation: In 1st, ‘born inside the’ is incorrect usage. ‘born in the’ is correct usage.
In 2nd, ‘that suggested’ is incorrect usage. The correct usage is ‘which suggested’.
112). Correct Answer is: (d)
Explanation: Both are correct.

113). Correct Answer is: (b)


Explanation: In 2nd, ‘debate on the’ is incorrect usage. ‘debate about the’ is correct usage.
114). Correct Answer is: (a)

Explanation: ‘plot lines in’ is incorrect usage. ‘plot lines of’ is correct usage.
115). Correct Answer is: (a)

Explanation: ‘frenzying rush’ is incorrect usage. The correct usage is ‘frenzying rush’.

116). Correct Answer is: (a)


Explanation: ‘money against virtual’ is incorrect usage. ‘money for virtual’ is correct usage.

117). Correct Answer is: (d)

Explanation: Both are correct.


118). Correct Answer is: (b)

Explanation: ‘railed for the’ is incorrect usage. ‘railed against the’ is correct usage.

www.ibpsguide.com https://estore.ibpsguide.com Page 33 of 331


119). Correct Answer is: (d)

Explanation: Both are correct.


120). Correct Answer is: (b)
Explanation: ‘worth a recall’ is incorrect usage. The correct usage is ‘worth recalling’.

Phrase replacement - Type – 1

Directions (121-150): In each question, there is a pair of words/phrases that highlighted. From the highlighted
word(s)/phrase(s), select the most appropriate word(s)/phrase(s) to form correct sentences. Then, from the

options given, choose the best one.

121). I. A long spell of calm, in which America’s stock market rose steadily without a big sell-off, ended abruptly

(a)/ briefly (b) this week.


II. In the medium term, America will have to get to grips (a) / grabs(b) with its fiscal deficit.
III. Policymakers tend to consider those who have dropped (a)/ came (b) out of the jobs market as lost to the
economy for good.
(a) bba (b) aab (c) baa (d) aba (e) aaa

122). I. There are spending pledges (a)/ pledge (b) on infrastructure, where wealthy Germany is surprisingly
deficient
II. Having hammered the “grand coalition” parties in last September’s inconclusive elections, they have been

telling pollsters that their support for Mrs. Merkel’s CDU/CSU alliance and the SPD is burgeoning (a)/ sliding (b)
III. Germany and Europe are better off without another six months of adrift (a) / drift (b).
(a) bbb (b) abb (c) bba (d) aba (e) aaa

123). I. In the late 1980s, as Mikhail Gorbachev launched (a)/ lunged (b) perestroika, Russia made peace with the

West.

II. Among other things, he still has to say whether the conspiracy extended (a) / extricated (b) to the Trump
campaign

III. . Angela Merkel successfully warned Mr. Putin that there would be consequences if he interfered (a) / inferred

(b) in German elections


(a) aba (b) aaa (c) aab (d) bba (e) bab

www.ibpsguide.com https://estore.ibpsguide.com Page 34 of 331


124). I. The new president of South Africa, Cyril Ramaphosa, has a hash (a) / heck (b)of a mess to clear up

II. Mr. Ramaphosa cannot begin to reform (a)/ rebound (b) South Africa without honest and capable public
servants running the state and its companies
III. Throughout Mr. Zuma’s time in power, useless ministers were allowed to booze (a) /doze (b) through

parliamentary sessions and preside over collapsing departments—as long as they were loyal.
(a) aba (b) aaa (c) aab (d) bba (e) bab

125). I. The Romans did themselves no good by using (a) / ensuing (b) lead for water pipes and sometimes even
as a food sweetener.

II. In 2015 the Institute for Health Metrics and Evaluation, a research institute in Seattle, estimated that exposure

(a) / furor (b) to lead globally caused about 500,000 deaths that year and 12% of developmental disabilities, such
as cerebral palsy and epilepsy

III. The dangers of lead have long been known because America pinned (a) / banned (b) it from paint 40 years
ago, and by the late 1990s leaded petrol had been phased out in almost all rich countries.
(a) aba (b) aaa (c) aab (d) bba (e) bab

126). I. If Fannie and Freddie were capitalised to the same standards as banks and forced to make desperate (a)
/ adequate (b) profits, taxpayers would be left with only the remote risks that markets find it hard to price

II. Republicans, meanwhile, may be tempted to hold out for more radical (a) / sadist (b) change
III. Today the problem sits with the Senate Banking Committee, which is considering a draft proposal to replace
them with multiple privately capitalised firms, whose equity holders would suffer first during any slump (a) / limp

(b)
(a) aba (b) aaa (c) aab (d) baa (e) bab

127). I. As we went to press, the cabinet was due to enter an extended session in order to hammer (a) / stammer
(b) out what kind of Brexit is best for Britain

II. It is involved in drawing up EU laws and regulations, particularly in areas such as energy, where it knows the

ropes (a)/ roars (b).


III. Britain is leaving (a) / weaving (b) just as the EU is becoming a looser union of countries moving at different

speeds.

(a) aba (b) aaa (c) aab (d) bba (e) bab

www.ibpsguide.com https://estore.ibpsguide.com Page 35 of 331


128). I. Using fake social-media personas, the Russians tried to depress (a) / empower (b) turnout among blacks

and Muslims, encourage third-party voting and convince people of widespread voter fraud
II. Anton Shekhovtsov, a Ukrainian political scientist, has studied the links Russia has cultivated with an assay (a)
/ array (b) of European parties.

III. It is in assuring such continued circulation that outfits like the IRA play a role, setting up automated accounts—
“bots”—that promulgate (a) / proliferate(b) messages to specific groups and individuals
(a) bba (b) aaa (c) aab (d) aba (e) bab

129). I. In America the main work of identifying which bots and troll accounts were shun (a) / run (b) by the IRA

has been done by Twitter and Facebook themselves.

II. At his first meeting with Mr. Putin, Mr. Macron publicly accused RT and Sputnik of being state propaganda (a) /
statute (b) channels

III. Teachers there are particularly impressed by the effect (a) / affect (b) of assignments that get the students to
create fake-news campaigns themselves; they dramatically improve students’ awareness of how disinformation
works, and how to recognise it
(a) aba (b) baa (c) aab (d) bba (e) bab

130). I. At a rally in the southern state of Karnataka the prime minister wagged (a) / waged (b) his raised finger,

accusing the local government, run by the rival Congress party, of creaming a 10% cut from every state contract
II. The Hindu-nationalist BJP had excoriated the secular Congress for pandering (a) / pondering (b) to religious
and ethnic minorities

III. The energetic prime minister has launched dozens of heavily promoted (a) / socialized (b) social programmes.
(a) aba (b) aaa (c) aab (d) bba (e) bab

131). I. In the shade of the jungle, a round-bellied orangutan drowns (a) / glides (b) towards the ground
II. Land clearance is forint (a) / rampant (b) along its eastern coast, as farmers take advantage of lax laws to

make room for cattle to feed Asia

III. Family farmers lament (a)/ latent (b) that trees obstruct the big machinery needed to keep their land productive
(a) aba (b) aaa (c) aab (d) bba (e) bab

132). I. Permissive (a) / permissible (b) forestry laws seem especially odd given the billions of dollars the
government spends planting trees, fighting climate change and conserving native species.

www.ibpsguide.com https://estore.ibpsguide.com Page 36 of 331


II. Two years ago, however, the BJP scored a surprising success in Assam, by far the most populous (a)/

propitious (b) state in the north-east.


III. But Labor, which won a state election late last year, has promised to reintroduce (a) / disorganize (b) the
legislation.

(a) aba (b) aaa (c) aab (d) bba (e) bab

133). I. Destiny is usually said to lurk in heavy grapes (a) / drapes (b) of purple velvet, in the wicked glint of a
crystal ball, behind a veil of heady incense or in the tuck of a gold-chiffon turban.
II. Other stores offer the Korean arts of face-reading, palm-reading—one entices (a) / stupefies (b) clients with a

detailed mapping of Barack Obama’s raised hand at his presidential swearing in—and saju.

III. A tarot reader assesses (a)/ accesses (b) the character of her clients first.
(a) aba (b) aaa (c) aab (d) baa (e) bab

134). I. Face-readers consider cues like posture, body language and tone of voice in assessing (a) / accessing (b)
a customer, much as people naturally assess physical appearance to guess someone’s emotional state
II. That saju and face-reading are recognised as academic pursuit (a) / pursuits (b) in Korea also lends them
some modern-day credibility
III. As palm lines and facial features evolve (a)/ dissolve (b) with age, so too, it is thought, does fortune.

(a) aba (b) aaa (c) aab (d) bba (e) bab

135). I. In the Chinese tradition the family’s future rests on that child’s shoulders but this one is failing (a) / flailing

(b) in his duties


II. All of which makes the question of how to revive the region more than a patricidal (a) / parochial (b) one.
III. Still, it is possible to exaggerate the region’s peculiarities (a) / rationalization (b) as other provinces have failing

SOEs
(a) aba (b) aaa (c) aab (d) bba (e) bab

136). I. Until (a) / Till (b) America gets a grand military parade, a drive along the wharf at Naval Station Norfolk, in
Virginia, is the next-best thing.

II. As global temperatures rise so does the likelihood of extreme weather, with calls for military assistance (a) /

optimism (b) in disaster relief.

www.ibpsguide.com https://estore.ibpsguide.com Page 37 of 331


III. On February 18th scientists involved in the federal government’s National Climate Assessment, a four-yearly

exercise mandated (a) / shrugged (b) by Congress, presented an update to the last report from November,
showing that sea levels are rising twice as fast as 25 years ago.
(a) aba (b) aaa (c) aab (d) bba (e) bab

137). I. Three days after Nikolas Cruz walked into Marjory Stoneman Douglas High School in Florida and shot
dead 14 students and three teachers, one of his former schoolmates impressed (a) / addressed (b) a gun-control
rally
II. President Donald Trump, who waited 20 hours before addressing America about the tragedy but is unlikely to

have missed the students on the Sunday talk shows, said he backed (a) / stopped (b) a bill to improve

background checks on those who buy firearms.


III. Mr. Trump might not wobble (a) / hobble (b) the armed forces’ efforts to deal with the consequences of climate

change
(a) aba (b) aaa (c) aab (d) bba (e) bab

138). I. Noemi Luna was a teenager when she first enticed (a) / realised (b) she was not living legally in the United
States.
II. Paul Secunda at Marquette University Law School in Wisconsin is convinced that a pro-Janus ruling would

have a devastating effect on public-sector unions, as it would deprive (a) / detest (b) them of a big chunk of their
income.
III. Their supporters say unions perform essential (a) / superficial (b) services for their members, who would be far

worse-off without them.


(a) aba (b) aaa (c) aab (d) baa (e) bab

139). I. People who worry about Donald Trump’s presidency worry especially about how he might importune (a) /
respond (b) to a national-security crisis

II. The president considers any rival, at times including the institutions over which he presides, an enemy to be

obliterated (a) / substantiated (b)


III. By slandering (a) / punishing (b) Barack Obama as a Muslim Marxist they inspired Mr Trump to question his

place of birth, by rubbishing climate scientists they prepared the way for the president’s wider assault on truth and

reason.
(a) aba (b) aaa (c) aab (d) bba (e) baa

www.ibpsguide.com https://estore.ibpsguide.com Page 38 of 331


140) I. With our tried-and-tested installation and logistic process, we can take care of everything from creating

move management programmes to co-ordinating (a) / co operating (b) product delivery and storage, as well as
ensuring everything runs smoothly all the way through to final installation.
II. In September last year Donald Trump said he might decertify (a) decipher (b) Colombia as a partner in the fight

against drugs, putting in jeopardy some of the $390m in American aid Colombia receives
III. Last month in Caquetá, a southern department, ex-members of the FARC forced (a) / asked (b) six officials of
the UN Office on Drugs and Crime (UNODC) out of their cars and stole their GPS devices and mobile phones
(a) aba (b) aaa (c) aab (d) bba (e) bab

141). I. It takes guts to challenge both Andrés Manuel López Obrador, a messianic, silver-tongued populist, and

the parking (a) / residual (b) political machine of Mexico’s governing Institutional Revolutionary Party (PRI).
II. Neither can the PRI’s candidate, José Antonio Meade, a competent (a) / complainant (b) and respected former

finance minister who is not a party member.


III. Although it achieved important reforms of education and energy, it has failed in fighting violent crime and
corruption and improving (a) stressing (b) the economy, the issues that matter most to the public.
(a) aba (b) aaa (c) aab (d) baa (e) bab

142). I. The United States has long alleged that its northern neighbor is soft on piracy, allowing vendors to sell

goods and cultural products that infringe (a) / tag (b) trademarks and copyrights of American and other firms.
II. The successes may be short-lived as UNODC estimates that growers replant 30% of the area downgraded (a) /
uprooted (b)on average

III. But that is just a quarter of the families that have signed simultaneous (a) / collective (b) agreements, which is
just the first step towards qualifying for help.
(a) aba (b) abb (c) aab (d) bba (e) bab

143). I. Gone are the days when the black flag fluttered (a) / cluttered (b) over a third of Iraq and almost half of

Syria.

II. It sold oil from the fields it captured, taxed and robbed the people it ruled and stole perhaps (a) / some (b)
$500m from Iraqi banks, making it the richest terrorist group in history.

III. Transactions involving IS cash, sometimes millions of dollars, could (a) / can (b) take weeks to complete

(a) aba (b) aaa (c) aab (d) bba (e) bab

www.ibpsguide.com https://estore.ibpsguide.com Page 39 of 331


144). I. On February 7th American-led forces killed scores (a) / score (b) of Russian mercenaries who were

fighting alongside pro-regime forces in eastern Syria.


II. There is a growing risk of new clashes involving the many external powers that have been drawn in to Syria:
Russia and Iran, which back the regime; America, which arms and supports the Kurds; Turkey, which has

intervened to repel (a) / dispel (b) them; and Israel, which is alarmed by the prospect of Iran and its Lebanese ally,
Hizbullah, implanting themselves in the Golan Heights.
III. The UN Children’s Fund says it no longer has words to describe (a) / explain (b) the suffering in Syria.
(a) aba (b) aaa (c) aab (d) bba (e) bab

145). I. Early last year the Emirates Red Crescent launched a humanitarian (a) / humane (b) mission in

Ghayda—by August the Emirati army had taken over the mission.
II. The Saudis and Emiratis run their own retention (a) / detention (b) centres and keep local officials in the dark.

Smugglers in Mahra complain of a loss of business, as the newcomers crack down.


III. In securing the ports, say analysts, the UAE’s goal is to enhance the position of its own port at Jebel Ali, the
region’s largest, either by stifling (a) / stratifying (b) competition or directing traffic in its direction.
(a) abb (b) aaa (c) aba (d) bba (e) bab

146). I. Although nearby towns in the region of Oromia, which surrounds (a) / enshrines (b) the capital, have been

hit by anti-government protests since late 2014, these streets have remained mostly quiet.
II. South Africans had only a few days to celebrate the resignation of Jacob Zuma and the swearing-in of a new
president, Cyril Ramaphosa, before the hangover (a) / hangout (b) set in.

III. Goolam Ballim, an economist at Standard Bank, reckons (a) / states (b) that if South Africa had had a better
economic manager than Mr. Zuma for the past nine years, its GDP would be as much as 25% larger
(a) aba (b) aaa (c) aab (d) bba (e) bab

147). I. Every year dozens of Masonic lodges (a) / lodgings (b) from across Africa come together to discuss the

issues of the day

II. Instead, Mr. Ramaphosa hopes to attract investment and disport (a) / spur (b) the economy so that South Africa
can grow its way out of its debt crisis.

III. Seven months after their prime minister was appointed in May 2017, fully 35% of the French could not name

him accurately (a) / perfectly (b) in a poll


(a) abb (b) aaa (c) aab (d) bba (e) bab

www.ibpsguide.com https://estore.ibpsguide.com Page 40 of 331


148). I. The black limousine with darkened windows hurtling (a) / hurling (b) through the gathering darkness

hits160kph (100mph) as it rushes him to his next campaign stop


II. The last polls published before a pre election gag (a) / gagging (b) rule came into effect on February 17th all
implied a hung parliament.

III. Mr. Berlusconi’s Forza Italia party enjoys (a) / enjoyed (b) support throughout the country, but the polls suggest
only about17% of decided voters back it
(a) aba (b) aaa (c) aab (d) bba (e) bab

149). I. As soon as the Syrian security forces began repressing (a) / depressing (b) anti-government protests he

had helped to organise in the spring of2011, the Kurdish activist went underground.

II. Unlike (a) / Much like (b) countless others, Mr Ibrahim made it out: he was released as part of an amnesty in
May 2013 and fled to Germany via Iraqi Kurdistan.

III. Workingwith the European Centre for Constitutional and Human Rights (ECCHR), a pressure group in Berlin,
and Anwaral-Bounni and Mazen Darwish, two lawyers who spent years in Syrian prisons before escaping to
Germany, the refugees have lunged (a) / lodged (b)criminal complaints against several high-level offi- cials for
crimes against humanity.
(a) aba (b) aaa (c) aab (d) bba (e) bab

150). I. Some of those who should really be tried for (a) / in (b) war crimes wind up with a terrorism conviction
instead
II. German laws (a) / law (b) does not provide for trial in absentia, and most of the accused wisely do not travel

outside Syria
III. The witnesses and their lawyers hope that some bigwigs (a) / bigbats (b) will eventually be caught.
(a) aba (b) aaa (c) aab (d) bba (e) bab

ANSWERS:

121). Correct Answer is: (e)

Explanation: Abruptly, means suddenly. Get to grips means to realize it. And in the last they have dropped out
not ‘came out’. Hence, e is true

122). . Correct Answer is: (b)

Explanation: ‘pledges’ is correct usage here.


‘burgeoning’ means creating. ‘Sliding’ means dying, which is correct here.

In last sentence, adrift is incorrect.

www.ibpsguide.com https://estore.ibpsguide.com Page 41 of 331


123). Correct Answer is: (b)

Explanation: Lunged – having lungs, incorrect here.


Extend- to further it.
Interfered - intervene in a situation without invitation or necessity

124). Correct Answer is: (e)


Explanation: Heck – Too heavy

Reform – Form again, generally for good


Doze – sleep
125). Correct Answer is: (c)

Explanation: Using – with use of

Exposure – removing cover/ revealing


Banned – Forfeited from doing something

126). Correct Answer is: (d)


Explanation: Adequate – Enough
Radical – Practical

Slump – Reduction/ Downwards travel


127). Correct Answer is: (b)
Explanation: Hammer – beat
Ropes – Environment
Leaving – to leave

128). Correct Answer is: (d)

Explanation: Depress – To submerge/ discourage


Array – wide range

Promulgate – officially declaring

129). Correct Answer is: (b)


Explanation: Run – Led by/ Executed by

Propaganda – Showcasing ideas of a campaign

Effect – having some result


130). Correct Answer is: (b)

Explanation: Wagged – move sideways or up down

Pandering- helping especially in wrong


Promoted- helping showcase something to more people or increase something

131). Correct Answer is: (d)

www.ibpsguide.com https://estore.ibpsguide.com Page 42 of 331


Explanation: Glides – Comes downwards

Rampant – dispersing heavily


Lament – showing sadness

132). Correct Answer is: (a)

Explanation: Permissive – Extremely tight


Propitious – offering service to gods
Reintroduce – Introducing again

133). Correct Answer is: (d)


Explanation: Drape- covered

Entice – Attract
Assess- Check
134). Correct Answer is: (a)

Explanation: Assess – check


Pursuit – journey to find something
Evolve – become more evident

135). Correct Answer is: (a)


Explanation: Failing - not succeeding
Parochial – complex

Peculiarities - Characters
136). Correct Answer is: (a)
Explanation: Failing - not succeeding

Parochial – complex
Peculiarities - Characters

137). Correct Answer is: (e)

Explanation: Addressed - say or write remarks or a protest to


Backed – supported

Hobble - Unbalanced

138). Correct Answer is: (d)


Explanation: Realised – understand

Deprive – not having something


Essential - Important
139). Correct Answer is: (e)

Explanation: Respond – Giving reaction

www.ibpsguide.com https://estore.ibpsguide.com Page 43 of 331


Obliterated – Ended

Slandering – Criticize
140). Correct Answer is: (b)
Explanation: Co-ordinating – working together

Decertify – Removing certifications


Forced is correct usage here as they were forcefully asked to do something

141). Correct Answer is: (d)


Explanation: Residual – minimal
Competent – giving competition
Improving – Improve condition

142). Correct Answer is: (b)


Explanation: Infringe – violate
Uprooted – removed from roots
Collective – All as one
143). Correct Answer is: (c)
Explanation: Fluttered – waving
Perhaps – Probably

Can is correct usage here.

144). Correct Answer is: (b)


Explanation: Scores- Numbers
Repel – Move away

Describe – explain
145). Correct Answer is: (c)
Explanation: Humanitarian – In aid of human
Detention – Where someone is detained or kept as punishment
Stifling – gagging/ unable to breathe

146). Correct Answer is: (b)

Explanation: Surrounds – covering from sides


Hangover is generally state of unable to recovering after something generally effects of drinking.

Reckons – Calculates

147). Correct Answer is: (a)


Explanation: Lodges – place to live

Spur – Stimulate

www.ibpsguide.com https://estore.ibpsguide.com Page 44 of 331


Perfectly – without errors

148). Correct Answer is: (b)


Explanation: Hurtling – fighting through

Gag – cap/close
Enjoys – Embraces
149). Correct Answer is: (c)
Explanation: Repressing – pushing down

Unlike – different
Lodged- filed

150). Correct Answer is: (a)

Explanation: Tried for is correct.


Law is correct.
Bigwigs – high position person

Phrase replacement - Type - 2

Directions (151-180): Read the following phrase carefully and choose the correct sentence which suits for the
phrase

151). Purple patch


I) I have hit a purple patch and that is the reason I am confident of scoring in next game as well.

II) He was going well in exams until he hit the purple patch.
III) Stop looking for the purple patch and start working for small successes.
IV) This purple patch will pull you down from the ladder.

a) Only I &II b) Only II and IV c) Only III & I d) All except II e) All are correct

152). Go whole hog

I) They said they wanted to go the whole hog with this experiment.
II) If you don’t go the whole hog with your studies, your result will suffer.

III) Even when I warned him to go whole hog, he did the work completely.
IV) His intentions of going the whole hog were evident in his performance.
a) Only III b) Only I & II & III c) Only II & IV d) All except IV e) All except III

www.ibpsguide.com https://estore.ibpsguide.com Page 45 of 331


153). Infra dig

I) The people felt that the newly appointed politician’s work was too much infra dig.
II) His thoughts to help me in infra dig impressed me.

III) His outing was infra dig by other players.

IV) I hated to work in such an environment as I regard it as deeply infra dig.


a) Only I &II b) Only IV & I c) Only II & III d) Only II & IV e) All are correct

154). Bottle out


I) The batsman bottled out at the right time anticipating that the delivery will go over his head.
II) After a long day’s work, he bottled out the plans he had with his friends.

III) I wish I could bottle out and leave this classroom right now.
IV) He wanted to bottle out, but her persistence and diligence made him stay.
a) Only I &II b) Only IV & I c) Only II & III d) Only II & IV e) All are correct

155). No truck with


I) The fights had gone so frequent that the husband wanted no truck with the wife.
II) The coach’s fallout with the players led him to no truck with the entire association.
III) I have no truck with the fact that he died.
IV) His sense of work had no truck with any of the delays.
a) Only I &II b) Only IV & I c) Only II & III d) Only II & IV e) All are correct

156). Pyrrhic victory


I) Though India won the match, it was a pyrrhic victory.

II) There is no reason of this fight because it will only result in a pyrrhic victory.
III) The elections were an exemplification of what a pyrrhic victory is.
IV) I wanted a pyrrhic victory at no losses.

a) Only I &II b) All except II c) All except III d) All except IV e) All are correct

157). Resting on one’s laurels

I) The Indian contingent paid a heavy price for resting on the captain’s laurels.
II) After being number one last year, the company is resting on its laurels.
III) Just when everyone thought he would rest on his laurels, he struck a mighty century.

IV) I want to rest on my laurels but can’t do that until I retire.

www.ibpsguide.com https://estore.ibpsguide.com Page 46 of 331


a) Only I &II b) Only IV & I c) Only II & III d) Only II & IV e) All are correct

158). Beggars belief

I) It is a beggars belief that they won the match last night.


II) Though the Ministers ensured that situation was under control, the media regarded it as a beggars’ belief.
III) The beggars’ belief this year came from within the company.
IV) I wanted to set beggars belief on fire.

a) Only I &II b) Only IV & I c) Only II & III d) Only II & IV e) All are correct

159). Last Straw

I) The penultimate over of the match was the last straw for the tigers.
II) His arduous order served as the last straw for his staff.

III) They took the last straw and won.


IV) The last straw was helpful for elections.
a) Only I &II b) Only IV & I c) Only II & III d) Only II & IV e) All are correct

160). With bells on


I) The Companies’ employees are waiting for the new boss with bells on.

II) The movie has created a new trend and the viewers are going with bells on.
III) After he died with bells on, his family was crushed.
IV) Indian Armed forces personnel are generally ready to die with bells on.

a) Only I &II b) Only IV & I c) All except III d) All except I e) All are correct

161). Wing it
I) In process of winging its reputation, the company lost customers.
II) As the defense of the opposition was strong the team had to wing it to score the goal.

III) I decide to wing it after entering the interview room.

IV) He couldn’t wing it as he was not prepared for any newer situations.
a) Only I &II b) Only IV & I c) All except IV d) All except I e) All are correct

162). Cotton on
I) I tried to cotton on to the customer’s needs but he was not ready to explain them any further.

II) His mind was made up to cotton on to my religious despite being an atheist.

www.ibpsguide.com https://estore.ibpsguide.com Page 47 of 331


III) The coach cottoned on to the bowlers’ problem and gave him extra attention ahead of the next match.

IV) She always cottons on to my problem, that’s why she is my best friend.
a) Only I &II b) Only IV & I c) Only II & III d) Only II & IV e) All are correct

163). By the book


I) His going by the book shows a character of dedication in him.
II) The officer goes by the book but for this case decided to wing it.

III) The team lost the match because they didn’t go by the book.
IV) They want employees who always go by the book.
a) Only I &II b) Only IV & I c) Only II & III d) Only II & IV e) All are correct

164). Cannot hold a candle to


I) The final was a one sided affair as the tigers couldn’t hold a candle to the Lakers.
II) I can’t hold a candle to him because he is my boss.
III) Though he is the topper of the class, he can’t hold a handle to me in sports.
IV) The Prime minister will surely win next years’ elections as the opposition can’t hold a candle to him and his
party.

a) Only I &II b) Only IV & I c) Only II & III d) Only II & IV e) All are correct

165). Dog in the manger


I) He came quickly because his boss ordered him something and made him the dog in the manger.

II) The batsman is a dog in the manger as team already has top five batsmen of the world.
III) They acted like a dog in the manger during the competition.
IV) After the divorce he was a dog in the manger.

a) Only I &II b) Only IV & I c) Only II d) Only III e) All are correct

166). Donkey’s years

I) The last month in jail seemed like donkey’s years.


II) The batsman is struggling as his donkey’s years are ending.

III) After the meeting his donkey’s years also died.


IV) My donkey’s year have begun after this victory.
a) Only I &III b) Only III & II c) Only I d) Only IV e) All are correct

www.ibpsguide.com https://estore.ibpsguide.com Page 48 of 331


167). Loose cannon

I) The bowler is a loose cannon and that is what makes him so dangerous.
II) She behaves like loose cannon after a few drinks.

III) Even after he was fired from the company, he remained calm and behaved like a loose cannon.

IV) His behavior like a loose cannon won him the elections because people like a stable leader.
a) Only I &II b) Only IV & I c) Only II & III d) Only II & IV e) All are correct

168). Straight as a die


I) He has so many friends because he is straight as die
II) I am straight as a die as my wife died last month.

III) The boss was straight as a die but employees took him for granted.
IV) The accident caused him a brain injury and made him straight as a die.
a) Only I &II b) Only IV & III c) Only I & III d) Only II & IV e) All are correct

169). Pipe down


I) The coach asked the players to pipe down and concentrate on the job.
II) I was made to pipe down when I was sitting alone.
III) He piped me down when I tried to call him.
IV) There is no need to pipe down now as the teacher is gone.
a) Only I &II b) All except I c) Only II & III d) All except II e) All are correct

170). Close quarters


I) The captain decided to keep an eye on the teams’ training in close quarters.

II) He wanted to go in close quarters but was kept outside the territory of officers.
III) The close quarters have died now because of the boss.
IV) I have closed quarters with him.

a) Only I &II b) Only IV & I c) Only II & III d) Only II & IV e) All are correct

171). Achilles heel

I) The officer has died because of traitor’s Achilles heel.


II) I have an Achilles heel, please rub it.
III) They wanted his Achilles heel to win the case.

IV) His wife is his new Achilles heel.

www.ibpsguide.com https://estore.ibpsguide.com Page 49 of 331


a) Only I &II b) Only IV & I c) Only II & III d) Only III & IV e) All are correct

172). Devil-may-care
I) The boss has a devil-may-care attitude which prevents him from taking risks in business.

II) The reader’s devil-may-care demands have forced him to stop writing.
III) The batsmen’s devil-may-care put the whole team at stake.

IV) His devil-may-care driver decided to go without papers.


a) Only I &II b) Only IV & I c) Only II & III d) Only III & IV e) All are correct

173). Eat one’s heart out

I) The loss in the previous series has made the captain eat his heart out.
II) The depression has lead him to eat his heart out.

III) After he topped the college he decided to eat his heart out of joy.
IV) My mother eats her heart out if I don’t get to home on time.
a) Only I &II b) Only IV & II c) All except I d) All except III e) All are correct

174). Double entendre


I) The only requirement of the employer is an employee without any double entendre.

II) His double entendre attitude tells his persistent character towards one thing.
III) No double entendre can save you from this opponent.
IV) The politicians are known for their double entendre.

a) Only I &II b) Only IV & II c) Only II & III d) Only I & IV e) All are correct

175). Hot on the heels


I) The detective was hot on the heels of the suspect.
II) The boss was hot on the heels as the work was incomplete.

III) After exams I want to be hot on the heels.

IV) Being hot on the heels is considered a great quality.


a) Only I &II b) Only IV & III c) Only I d) Only III e) All are correct

176). In spades
I) The bowler caught the batsmen in spades.

II) After he was found in spades, he couldn’t walk away.

www.ibpsguide.com https://estore.ibpsguide.com Page 50 of 331


III) My work is held by me in spades.

IV) There is no need to be in spades here.


a) Only I b) Only IV & I c) Only III d) Only II & IV e) All are correct

177). Jump the gun


I) His jumping the gun always makes his work erratic.
II) Go with the work, there is no need to jump the gun.

III) After the appointment he seems to be jumping the gun.


IV) My parents are jumping the gun over me.
a) Only I &II b) Only IV & I c) Only II & III d) Only II & IV e) All are correct

178). Knock back


I) I have been knocking back at them, but nothing seems to happen.
II) The player is good as he never lets a knock-back affect his performance.
III) They tried knock-back therapy but failed.
IV) If you don’t knock-back, what is the use of your fees.
a) Only I b) Only IV & I c) Only II d) Only III & IV e) All are correct

179). Run a tight ship


I) The new coach is known for running a tight ship.

II) He likes to be carefree and run a tight ship.


III) There is no need of running a tight ship here.
IV) If you hadn’t run a tight ship, we would have lost the case.
a) Only I &II b) Only IV & I c) All except III d) All except II e) All are correct

180). See red

I) He cried over the loss of his loved one’s life as his eyes saw red.

II) I need to see red if I have to pass this exam.


III) They went to him and decided to see red as he had threatened them earlier.

IV) The need to see red is enormous in some situations.


a) Only I &II b) Only IV & III c) Only II & IV d) Only I & III e) All are correct

www.ibpsguide.com https://estore.ibpsguide.com Page 51 of 331


ANSWERS:

151). Correct Answer is: c)


Explanation: The meaning of the phrase "the purple patch" is "a run of good luck". Only I and III talk of that good

luck run. Hence, option c is correct.


152). Correct Answer is: e)
Explanation: The meaning of the phrase "go whole hog" is "do something completely and thoroughly" and it has

been used properly in I, II and IV sentences.


153). Correct Answer is: b)
Explanation: The phrase "infra dig" means beneath one’s dignity. Only sentences I and IV correctly signify its

meaning. Hence, option b is correct.

154). Correct Answer is: e)


Explanation: The phrase "bottle out" means ‘to lose one’s nerve and decide not to do something’. In all the given
sentences, the usage is correct. Hence, option e is correct.

155). Correct Answer is: a)


Explanation: The phrase "no truck with" means ‘no association with’. Only sentences I and II correctly signify its
meaning. Hence, option a is correct.
156). Correct Answer is: d)
Explanation: The phrase "pyrrhic victory" means a victory where the victor’s losses are same as the losers. Only
sentences I, II and III correctly signify its meaning. Hence, option d is correct.
157). Correct Answer is: d)
Explanation: The phrase "pyrrhic victory" means a victory where the victor’s losses are same as the losers. Only

sentences I, II and III correctly signify its meaning. Hence, option d is correct.
158). Correct Answer is: a)
Explanation: The phrase "beggars belief" means too good to be true. Only sentences I and II correctly signify its
meaning. Hence, option a is correct.
159). Correct Answer is: a)

Explanation: The phrase "last straw" means ‘that one event which breaks one tolerable limit’. Only sentences I

and II correctly signify its meaning. Hence, option a is correct.


160). Correct Answer is: c)

Explanation: The phrase "with bells on" means ‘enthusiastically’. All except III correctly signify its meaning. Hence,

option c is correct.
161). Correct Answer is: d)

www.ibpsguide.com https://estore.ibpsguide.com Page 52 of 331


Explanation: The phrase "wing it" means ‘to improvise’. All except I signify its meaning correctly. Hence, option d

is correct.
162). Correct Answer is: e)
Explanation: The phrase "cotton on" means ‘to understand’. All sentences correctly signify its meaning. Hence,

option e is correct.
163). Correct Answer is: e)
Explanation: The phrase "By the book" means ‘following set rules’. All sentences correctly signify its meaning.

Hence, option e is correct.


164). Correct Answer is: e)

Explanation: The phrase "can’t hold a candle to" means ‘something being inferior to other thing or unable to

compete against the other thing’. All sentences correctly signify its meaning. Hence, option e is correct.
165). Correct Answer is: c)

Explanation: The phrase "dog in the manger" means ‘a person who has no need of, or ability to use, a possession
that would be of use or value to others, but who prevents others from having it.’. Only sentence II correctly
signifies its meaning. Hence, option c is correct.

166). Correct Answer is: c)


Explanation: The phrase "donkey’s years" means ‘long time. Only sentence I signifies its meaning. Hence, option
c is correct.

167). Correct Answer is: a)


Explanation: The phrase "loose cannon" means ‘an unpredictable or uncontrolled person who is liable to cause
unintentional damage’. Only sentences I and II correctly signify its meaning. Hence, option a is correct.

168). Correct Answer is: c)


Explanation: The phrase "straight as a die" means ‘entirely open and honest’. Only sentences I and III correctly
signify its meaning. Hence, option c is correct.

169). Correct Answer is: d)


Explanation: The phrase "pipe down" means ‘to stop talking/ be less noisy’. All except II correctly signify its

meaning. Hence, option d is correct.

170). Correct Answer is: a)


Explanation: The phrase "close quarters" means ‘close to something or someone’. Only sentences I and II

correctly signify its meaning. Hence, option a is correct.

171). Correct Answer is: d)


Explanation: The phrase "Achilles heel" means ‘weakest point’. Only sentences III and IV correctly signify its

meaning. Hence, option d is correct.

www.ibpsguide.com https://estore.ibpsguide.com Page 53 of 331


172). Correct Answer is: d)

Explanation: The phrase "devil-may-care" means ‘reckless and carefree attitude’. Only sentences III and IV
correctly signify its meaning. Hence, option d is correct.

173). Correct Answer is: d)

Explanation: The phrase "Eat one’s heart out" means ‘to be in a suffering situation’. All except III correctly signify
its meaning. Hence, option d is correct.

174). Correct Answer is: d)


Explanation: The phrase "double entendre" means ‘double standards’. Only sentences I and IV correctly signify its
meaning. Hence, option d is correct.

175). Correct Answer is: c)

Explanation: The phrase "hot on the heels" means ‘following closely’. Only sentence I correctly signifies its
meaning. Hence, option c is correct.

176). Correct Answer is: c)


Explanation: The phrase "in spades" means ‘to a high degree’. Only sentence III correctly signifies its meaning.
Hence, option c is correct.

177). Correct Answer is: a)


Explanation: The phrase "jumping the gun" means ‘acting before appropriate time’. Only sentences I and II
correctly signify its meaning. Hence, option a is correct.

178). Correct Answer is: c)


Explanation: The phrase "Knock-back" means ‘setback’. Only sentence II correctly signifies its meaning. Hence,
option c is correct.

179). Correct Answer is: d)


Explanation: The phrase "run a tight ship" means ‘being strict’. All sentences except II correctly signify its
meaning. Hence, option d is correct.

180). Correct Answer is: b)


Explanation: The phrase "see red" means ‘angry’. Only sentences IV and III correctly signify its meaning. Hence,

option b is correct.

Phrase replacement Type - 3

Directions (180-210): Which of the word A, B, C and D given below defines the phrase printed in bold in the

sentence. If all defines the phrase then mark E as the answer.


181). Maharashtra has been experiencing a hot wave leaving the entire region bone dry.

(a) Putrid (b) Pitified (c) Parthed (d) Parched (e) All are correct

www.ibpsguide.com https://estore.ibpsguide.com Page 54 of 331


182). The way he grasped the nettle, he deserves a fair chance from now on.
(a) Stratified (b) Marti (c) Procrastinate (d) Confront (e) All are correct

183). The army decided to play by the ear in absence of any resources and artillery.
(a) Extemporize (b) Exaggerate (c) Magnate (d) Finish (e) All are correct

184). His accident just after his release seems a poetic justice as had charges of running over his truck over poor
people sleeping by the road.
(a) Vivid (b) Denigrate (c) Ignition (d) Retribution (e) All are correct

185). The fact that he racks his brains despite being down and out makes him ideal for this job.
(a) Relegate (b) Mortify (c) Mull (d) Desire (e) All are correct

186). She seemed caught between a rock and a hard place when she was asked to sign up for a year job
contract.
(a) Reprimand (b) Fizz (c) Dizzy (d) Crunch (e) All are correct

187). Though I had prepared for this job thoroughly, now I am getting cold feet ahead of the joining.
(a) Falter (b) Degrade (c) Notion (d) Pickle (e) All are correct

188). The team who won the trophy entered the tournament with reputation of being the dark horse.
(a) Extort (b) Bad egg (c) Retract (d) Checker (e) All are correct

189). After he got to England, he found out that the job offered was a damp squib.
(a) Tinker (b) Macramé (c) Fiasco (d) Flaubert (e) All are correct

190). His behavior is too near the knuckle.


(a) British (b) Gorgeous (c) Mortifying (d) Bawdy (e) All are correct

191). This bowler has become a piggy in the middle between the coach and the captain.
(a) Biased (b) Dead (c) Awkward (d) Bawdy (e) All are correct

www.ibpsguide.com https://estore.ibpsguide.com Page 55 of 331


192). This battle royal is taking its toll on both the bidders.

(a) Admen (b) Gotten (c) Fierce (d) Anticipate (e) All are correct

193). The captain’s innings made cock-up of the entire match for the tigers.

(a) Thingy (b) Botch (c) Grout (d) Blaster (e) All are correct

194). Ajay Devgn’s latest movie drew a blank.

(a) Elytis (b) Girtin (c) Block (d) Enhance (e) All are correct

195). The boss was unable to fathom the employee’s the growing reputation.

(a) Jealous (b) Evict (c) Tremor (d) Assimilate (e) All are correct

196). The bowler went gung-ho at the fan’s reaction.


(a) Fervent (b) Tantrum (c) Embezzle (d) Winkle (e) All are correct

197). The boss is loose cannon these days.


(a) Eccentric (b) Powder keg (c) Billy bally (d) Trooper (e) All are correct

198). The fact that he went rogue at the last moment made me see red.
(a) Exemplify (b) Deserve (c) Guilty (d) Impatient (e) All are correct

199). The murderer must have been somewhere in back of beyond.


(a) Under (b) Top (c) Drawer (d) Backwoods (e) All are correct

200). The teacher’s bee’s knees work helped students.


(a) Rectify (b) Elite (c) Dry (d) Brown (e) All are correct

201). The employee started the work tout de suite as the orders were directly from the executives.
(a) right away (b) Perfectly (c) Accurate (d) Incessant (e) All are correct

202). They are waiting for the new boss with bells on.
(a) Bright side (b) Grey matter (c) Bated breath (d) Dry time (e) All are correct

www.ibpsguide.com https://estore.ibpsguide.com Page 56 of 331


203). They have invited the new coach to ginger up the players.

(a) Teach (b) Train (c) Fear (d) Inspire (e) All are correct

204). There are no hard and fast rules of dying in army.

(a) Gregarious (b) Grim (c) Abiding (d) Truce (e) All are correct

205). He regards me as infra dig in every aspect of life.

(a) Superior (b) Inferior (c) Posterior (d) Bawdy (e) All are correct

206). I try to stay away from her because she knows how to get my goat.

(a) Anger (b) Irritate (c) Mortem (d) Disregard (e) All are correct

207). The company knew the deal was a jam tomorrow.


(a) Bizarre (b) Wrong (c) Done (d) Fantasy (e) All are correct

208). He is a dead ringer to me.


(a) Brother (b) Enemy (c) Trickster (d) Lookalike (e) All are correct

209). The reserves are faffing about this season


(a) Babbling (b) Squabbling (c) Mooching (d) Brandying (e) All are correct

210). His sound bite struck me the most during the interview.
(a) Elation (b) Prepared (c) Dearth (d) Excerpt (e) All are correct

ANSWERS:
181). Correct Answer is: (d)

Explanation: Bone dry = extremely dry.

182). Correct Answer is: (d)


Explanation: Grasped the nettle = to face a situation boldly

183). Correct Answer is: (a)

Explanation: Decide to play by the ear = to improvise in absence of adequate resources.


184). Correct Answer is: (d)

Explanation: Poetic Justice = the fact of experiencing a fitting or deserved retribution/revenge for one's actions.

www.ibpsguide.com https://estore.ibpsguide.com Page 57 of 331


185). Correct Answer is: (c)

Explanation: Rack one’s brains = to concentrate.


186). Correct Answer is: (d)
Explanation: Between a rock and a hard place = to be in a fix or confusing situation.

187). Correct Answer is: (a)


Explanation: Cold feet = to hesitate.
188). Correct Answer is: (b)
Explanation: Dark horse = Underdog/unexpected winner.
189). Correct Answer is: (c)

Explanation: Damp Squib = a situation or event which is much less impressive than expected.

190). Correct Answer is: (d)


Explanation: Near the knuckle = indecent and offensive.

191). Correct Answer is: (c)


Explanation: Piggy in the middle = a person who is placed in an awkward situation between two others.
192). Correct Answer is: (c)
Explanation: Battle royal = fierce fight/opposition.
193). Correct Answer is: (b)
Explanation: Cock-up = to mess up.

194). Correct Answer is: (c)


Explanation: Draw a blank = to fail.
195). Correct Answer is: (d)

Explanation: To Fathom = to understand.


196). Correct Answer is: (a)
Explanation: Gung-ho = Enthusiastic.

197). Correct Answer is: (b)


Explanation: Loose cannon = psycho/trouble maker.

198). Correct Answer is: (d)

Explanation: See red = Angry/impatient.


199). Correct Answer is: (d)

Explanation: Back of beyond = remote and inaccessible place.

200). Correct Answer is: (b)


Explanation: Bee’s knees = Excellent person/thing/work.

201). Correct Answer is: (a)

www.ibpsguide.com https://estore.ibpsguide.com Page 58 of 331


Explanation: Tout de suite = immediately/at once.

202). Correct Answer is: (c)


Explanation: With bells on = enthusiastically.
203). Correct Answer is: (d)

Explanation: Ginger up = inspire.


204). Correct Answer is: (c)
Explanation: Hard and fast = inflexible.
205). Correct Answer is: (b)
Explanation: Infra dig = beneath or under someone or something.

206). Correct Answer is: (b)

Explanation: Get someone’s goat = irritate.


207). Correct Answer is: (d)

Explanation: Jam tomorrow = a pleasant thing which is often promised but rarely materializes.
208). Correct Answer is: (d)
Explanation: Near the knuckle = indecent and offensive.
209). Correct Answer is: (c)
Explanation: Faff about = to waste time.
210). Correct Answer is: (d)

Explanation: Sound bite = a short extract from a recorded interview or speech, chosen for its succinctness or
concision.

Phrase replacement - Type - 4


Directions (211-240): In given questions a paragraph is given with some phrases/idioms/words in bold. Choose
the options which can replace them correctly.

211). Local governments in the north-east diet their industrial champions by giving them preference in

procurement contracts. But their protectionism has not helped. The Paulson Institute’s Mr Song has looked at

sales within China of 36 types of industrial products from Liaoning. He finds that their market share has fallen in
30 of them since 2000, most by between a third and two-thirds. In the late 1990s exports from the province were

growing at roughly the national rate. Since then, they have been growing only two-thirds as fast. There has been a
decisive shift away from openness and trade towards local autarky. There are a few bright spots. JD.com, one of
China’s largest delivery companies, is investing 20bn yuan in Harbin, and has put its data-analytics division there.

But the broader lessons of the north-east are sobering

www.ibpsguide.com https://estore.ibpsguide.com Page 59 of 331


(a) hone/mode (b) mollycoddle/autarky (c) discipline/relish (d) molest/dependent (e) No Change

Required.

212). President Donald Trump’s policies, which include pulling America out of the Paris climate agreement to limit

global warming and denigrating coal, make all this more likely. In the past two months his administration has put
his climate-skeptical stamp on the national-security and defense strategies. These documents, which each
administration must draw up, lay out a high-level plan for keeping America safe. Under Barack Obama, they listed

climate change as a strategic threat to be assessed and countered. Yet in a distinction that is typical of this White
House, other parts of the government are carrying on with planning for a warmer planet regardless. As global
temperatures rise so does the likelihood of extreme weather, with calls for military assistance in disaster relief.

Last September the USS Wasp helicopter carrier was sailing from Norfolk to Japan when it was diverted to
hurricane-struck US Virgin Islands, Dominica and Puerto Rico. Melting sea ice in the Arctic opens up a new
theatre of operations, especially against a belligerent Russia

(a) Orating/concession (b) Assailing/corroboration (c) Countering/accord (d)


Championing/contradiction (e) No Change Required.

213). South Africans had only a few days to celebrate the resignation of Jacob Zuma and the swearing-in of a
new president, Cyril Ramaphosa, before the hangover set in. A new budget hiked taxes, cut public spending and

reminded people how big a mess Mr. Zuma left behind. The budget was presented by Malusi Gigaba, a finance
minister whose appointment by Mr. Zuma almost a year ago sparked protests against what was seen as a hostile
takeover of the Treasury, a department that had stayed professional even as corruption and incompetence

flourished elsewhere in the government. Embarrassingly, Mr. Gigaba’s budget was marred by the release of a
high-court ruling that he had lied under oath in his previous job as minister of home affairs (and photos of him
playing “Candy Crush” on his iPad in parliament). Mr. Ramaphosa kept him on until the budget so as not to

unsettle markets. He may not last much longer. Mr. Ramaphosa’s most urgent task is to kick out of his cabinet the
superiors and hacks put there by Mr. Zuma. This is harder than it sounds. Fire too many and he risks splitting the

ruling African National Congress (ANC), some pundits worry

(a) Antagonists (b) Cronies (c) Foes (d) Foible (e) No Change Required.

214). Many British Brexiteers were once also keen on the EEA option. It keeps the economic benefits of single-

market membership, but dumps the political baggage of ever-closer union and a would-be superstate. Moreover,
the EEA model for Britain is one the EU would be happy with. As far as Brussels is concerned, it works. Norway is

treated as a friend—unlike Switzerland, which in place of the EEA has a royal set of bilateral deals. The EU hates

www.ibpsguide.com https://estore.ibpsguide.com Page 60 of 331


the Swiss set-up, because it is not dynamically updated to changed single-market rules and there is no agreed

dispute-settlement mechanism. Diplomats in Brussels are clear that the Swiss model is not on offer to the British
(many say it would not now be given to the Swiss). Besides, it excludes most financial services. But most

Brexiteers now agree with Mrs. May in rejecting the EEA model. Although the government’s own analysis shows

that it would be the least costly form of Brexit, it crosses many “red lines” drawn by Mrs. May
(a) Half-ass (b) Facile (c) Brisk (d) Knotty (e) No Change Required.

215). In America, Facebook is steadily losing users under the age of25. Youngsters are spending more time on
other apps such as Snapchat, and Facebook-owned photo-sharing app Instagram, where their parents and
grandparents are less likely to lurk. While Instagram and the two messaging apps that Facebook owns,

Messenger and WhatsApp, help insulate the firm, “core” Facebook still accounts for at least 85% of the firm’s
revenue. Americans and Canadians are by far its most valuable audience, with an average revenue per user of

$86, four times more than the global average. If users continue to engage less with Facebook’s core network, it
could cause advertisers to leave over time. Yet most analysts and investors are still exuberant about future
prospects for Facebook, which with a market value of$521bn is the world’s sixth biggest publicly traded firm. They

may be underestimating some of the risks the firm faces. One challenge, which has been highlighted by the
Russia controversy, is its sloppiness. For a company whose sales pitch to advertisers is that it offers precision,
targeting and transparency superior to traditional media, including television, it is remarkable that it has struggled

to track the movement of ad dollars and content on its properties.


(a) Manifest (b) Bolt (c) Tattle (d) Quest (e) No Change Required.

216). In the Russian business community Sergei Galitsky served as a rare example of a self-made billionaire who
rose with relatively little state help and outside the natural-resources trade. He built his retail company, Magnit,

from scratch into Russia’s largest network; it has more than 16,000 stores. Rather than moving to Moscow, he
kept his headquarters in his hometown of Krasnodar, where he also founded a football club. On February 16th he
made a telling exit, announcing he would sell nearly all of his shares in Magnit—29.1%—to VTB, a state bank.

That Mr Galitsky decided to sell is the result of a tough business cycle and some strategic mistakes. More
remarkable is that he found a buyer not on the domestic private markets, or from among foreign investors. Selling
to a public sector bank reflects the growing role of the state in the Russian economy. Russia’s federal anti-

monopoly service puts its share at 70%. Yet the retail sector had largely been berated from the trend.

(a) Exposed (b) Farraginous (c) Debunked (d) Segregated (e) No Change
Required.

www.ibpsguide.com https://estore.ibpsguide.com Page 61 of 331


217). When Ken Frazier, chief executive of Merck, an American pharmaceutical giant, started his job in 2011, he

had a hard decision to make. The firm had promising new drugs—such as Januvia, for diabetes, and Gardasil, a
vaccine against cervical cancer. But the pharma industry was struggling with dismal returns on R&D and investors

were questioning if companies were overspending on science. Some surrendered and started buying in drugs

instead. But Mr Frazier opted to carry on backing his labs and promised publicly to spend on R&D for the long
term, not for the stock market’s immediate gratification. An opportunity to implement the pledge soon arrived.

Merck’s merger with another pharma firm, Schering-Plough, in 2009, had brought it an evident new cancer drug.

At first Merck’s scientists were unimpressed and relegated the drug to a list of assets to be licensed out. There
was widespread skepticism at the time about whether drugs that attacked cancer using the immune system would

work.

(a) Notorious (b) Illuminative (c) Obscure (d) Photogenic (e) No Change Required.

218). When Maria Veikhman, founder of SCORISTA, a Russian credit-scoring startup, was considering expansion
abroad, China immediately came to mind. She believes the scope there is vast, for two-fifths of Chinese have no
credit records. Ms Veikhman settled in Tianfu Software Park, a state-owned incubator in Chengdu, capital of
Sichuan province where city authorities “offer almost everything for free”. Complementary facilities range from
office space, basic furniture and logistics services to detailed grope on entrepreneurial methods. Chengdu aims to

catch up with Beijing, Shanghai, and Shenzhen, which at present are in a different entrepreneurial league—

together they have over a hundred unicorns, or private startups worth over $1bn. The south-western city allocated
200m yuan ($30m) in 2016 to an innovation-and startup fund for overseas founders, and hands out up to 1m yuan
in cash to well capitalised foreign startups and joint ventures.

(a) Deceit (b) Steering (c) Punter (d) Caveat (e) No Change Required.

219). In the 1990s a “third way” emerged from provincial Spain; creating a global retail bank with a deep presence
in many countries, allowing true economies of scale. The pioneer was Santander, a middle-weight bank from the
Bay of Biscay. Today it is the king of the euro zone: the bloc’s largest lender by market value, with 133m clients,

mainly in Brazil, Britain, Mexico and Spain. Its lofty position in Europe’s league table demonstrates that its

approach has, on balance, worked. Santander is run by Ana Botín, an optimistic character who took over from her
father, Emilio, on his death in 2014 (the Botín family no longer has a significant stake in the bank but its reputation

helped her win the top job). He had used integrity and charm to expand by means of acquisitions worth $80bn in

total, first in Spain, then across Latin America and in Britain, where it bought Abbey in 2004. As the financial crisis
struck in 2007 Santander seemed well-positioned to weather it. It did not run a big investment bank and had just

made an opportunistic acquisition of ABN AMRO’s arm in Brazil, giving it heft there for the first time.

www.ibpsguide.com https://estore.ibpsguide.com Page 62 of 331


(a) Guile (b) Veracity (c) Tripe (d) Candor (e) No Change Required.

220). Ten months ago the Trump administration took aim at steel and aluminium imports, giving itself a year to

decide whether they threatened national security and, if so, what to do about it. On February 16th it concluded
that America is indeed under threat. The president has until mid April to choose whether to respond. The reports
handed to Donald Trump by the Department of Commerce, which led the investigations, describe America as
effectively under siege. Its steel industry might struggle to respond to a crisis similar to the second world war, they

fret, as foreigners are filling a third of American demand for steel, even as 28% of national capacity lies idle. The
share of primary aluminium (the kind smelted from ore, rather than recycled metal) that is imported is 91%, and

61% of local smelting capacity lies callous. Doubters can point out that the Department of Defence requires a tiny

slice of American steel supply, and that America’s largest supplier for both metals, Canada, is an ally
(a) Warm (b) Fiery (c) Clairvoyant (d) Fervid (e) No Change Required.

221). Mr Kuroda also worries that his policy of negative interest rates, announced in January 2016, may
eventually turn counterproductive. The cut in rates raised the value of long-term assets held by Japan’s banks,

improving their balance sheets. But this one-time benefit must be set against an ongoing cost: negative rates hurt
the margin that banks earn between the interest rates they charge their borrowers and the lower rates they pay for
their funding. Since banks have not been able to pass on negative rates in full to their depositors, their funding

costs have fallen less than their loan rates. This narrower margin could straighten their financial standing and
eventually inhibit their lending. There is, however, “no evidence that such a thing is happening in Japan”,

Masazumi Wakatabe, an economist at Waseda University in Tokyo, told Bloomberg, a news agency, in
December. The improvement in the economy has increased the creditworthiness of borrowers, obliging banks to
write off fewer bad loans. And bank lending grew apace after negative rates were introduced in January 2016 (see
chart), even as firms have taken advantage of low borrowing costs to issue more of their own bonds and

commercial paper.
(a) Gnaw (b) Accentuate (c) Compose (d) Bolster (e) No Change Required.

222). Ilmars Rimsevics, for 17 years the governor of Latvia’s central bank, had been due to retire next year.
Instead, he is facing calls to resign. On February 17th Latvia’s anti-corruption authority detained him on suspicion

of demanding bribes of at least €100,000 ($123,000). That forbid international concern. Mr Rimsevics is a

member of the governing council of the European Central Bank (ECB) and privy to the most sensitive monetary-
policy decisions. The prime minister, Maris Kucinskis, says the allegations are so serious that Mr Rimsevics must

stand down. But he is staying put. Released on bail on February 19th, the central bank chief says the allegations

www.ibpsguide.com https://estore.ibpsguide.com Page 63 of 331


are a set-up to punish him for cracking down on lax practices. He also says he has received death threats.

Latvia’s outsized and ill-regulated off- shore banking industry has been a headache since the country regained
independence in 1991. During the global financial crisis ten years ago, Parex Bank, the largest independent bank

in the region, collapsed, prompting a budget-busting bailout—and the rescue of the Latvian economy by the IMF

and the European Union


(a) Proscribed (b) Interdicted (c) Gleamed (d) Vetoed (e) No Change Required.

223). Six months later Innovations for Poverty Action, an independent research outfit, checked up on them. Not
surprisingly, those who were preached at had become more religious (though Catholics, who accounted for seven
out of ten recipients, mostly stayed formally Catholic). They were not more satisfied with their lives, and had

become a little lucid about their economic status. Yet their incomes had increased by 9.2% compared with the
others. Because the sample is so large—more than 6,000 households—that is significant. Mr Karlan of
Northwestern University suggests the Bible lessons might have pushed people to think about their futures, and
encouraged them to take more control over their lives. (They earned more largely because they took on cash
jobs.) He is not sure, and is doing a follow-up study to see, if the income effect endures. For now, anyone
recalling nudges from grandma urging wakefulness through tedious sermons should consider that she may have
been right.
(a) Bertram (b) Tram (c) Dreary (d) Vivacious (e) No Change Required.

224). To donate an organ, one must share a blood-type with the recipient. Someone who would be willing to
donate a kidney to a friend or family member might be stymied by a difference in blood-type. Mr Roth

circumvented this problem by developing matching markets, in which one person donates to a compatible
stranger and in turn receives another stranger’s compatible organ for use by the donor’s ailing love done. Such
swap groups can include scores of donors and recipients, who might otherwise have died awaiting a transplant.

Yet demand for healthy organs vastly outstrips supply. Were it legal to buy and sell organs, many more people
might donate, helping to alleviate the deadly shortage. Moral qualms generally discourage governments from

legalising the trade. This is an example of what Mr Roth calls a “repugnant market”, one which is constrained by

popular is taste or moral unease. Repugnance, he laments, tilts the political playing field against ideas that unlock
the gains from trade. He recommends that economists spend more time thinking about such taboos, but mostly

because they are a constraint on the use of markets in new contexts.


(a) Gratifying (b) Inoffensive (c) Delectable (d) Congenial (e) No Change Required.

www.ibpsguide.com https://estore.ibpsguide.com Page 64 of 331


225). In the case of mothers, it is now believed that this process, called intergenerational epigenesis, is caused by

micro-RNAs from the parent getting into eggs as they form in a developing fetus. That makes sense. Eggs are
large cells, with room to accommodate these extra molecules. But intergenerational epigenetic effects can pass

down the male line as well. And how paternal micro RNAs come to be in an egg is a mystery, for the sperm that

would have to carry them there are tiny and have no spare room. Work by Jennifer Chan, a graduate student at
the University of Pennsylvania, has, however, shed light on the process. Ms Chan’s solution was described on

February 16th by her research supervisor, Tracy Bale of the University of Maryland, at the annual meeting of the
American Association for the Advancement of Science, in Austin, Texas. The crucial folly behind her study was
that micro-RNAs need not actually get inside sperm cells as they form. They could equally well be attached to
sperm just before sexual intercourse

(a) Dingy (b) Zaniness (c) Preposterousness (d) Acumen (e) No Change Required.

226). Take the fertilised egg of a pig. From each cell in the resulting embryo cutout a gene or genes that promote
the development of the animal’s heart. Inject human stem cells from a patient who needs a new heart into the
embryo and then place it into the womb of a sow. Wait nine months. The result is an adult pig with a heart made
of human cells. The pig can be slaughtered and the heart transplanted into the patient who provided the stem
cells, for whom the organ will be a genetic match. That, atleast, is the hope of a panel of researchers who
presented their work to the AAAS meeting. For, though this kind of biological melding may trip the disgust circuits,

the value of such a procedure, were it possible, is clear. First, transplantable organs are lavish and demand for
them is increasing. As life in general, and cars in particular, become safer, the supply of bodies with healthy

organs in them is shrinking. Meanwhile, people are living longer.


(a) Rife (b) Copious (c) Niggardly (d) Exuberant (e) No Change Required.

227). Calculating machines that run on quantum bits (known as qubits, for short) are, by some accounts, the
future of computation. Quantum computers have the theoretical advantage that they can solve with ease certain
mathematical problems, such as the factorization of large numbers, which are hard or impossible for classical

machines. This is possible thanks to a qubit’s ability to remain, through the discrepancies of quantum mechanics,

in many quantum states simultaneously. The more qubits a computer has, the more mind-bogglingly gigantic are
the calculations it can handle. Finance, medicine, chemistry and artificial intelligence are thus all expected to be

transformed by quantum computing. And where the future is, there surely will Google be also. The firm sets great
store by its quantum-computing project, which it calls Project Bristlecone. This is intended to develop a “quantum-
supremacy device”, i.e., one that is palpably and provably faster than a traditional computer of equivalent size at

solving particular mathematical problems.

www.ibpsguide.com https://estore.ibpsguide.com Page 65 of 331


(a) Normalities (b) Silences (c) Discords (d) Quirks (e) No Change Required.

228). Around 1936 three neurologists at Harvard Medical School raided the medicine cabinet, filling their boots

with morphine, barbiturates, ethers and even cobra venom. They injected those substances into (apparently)
willing volunteers and cemented primitive electrodes to their scalps and earlobes. They also collared a drunk and
wired him up. With pen and paper, they then recorded how the electrical signals in their volunteers’ brains
changed as the drugs began to take hold. This kind of gonzo science might meet a touch of resistance from the

institutional review board if proposed today, but the work of Gibbs, Gibbs and Lennox still stands. The trio
showed, without meaning to, that sedatives lower the activity of the brain through several clear stages, and that

each stage is obscure in that organ’s electrical readings. Their results have been refined over the years, of

course, to the extent that Emery Brown, a successor of theirs at Harvard, now thinks, as he told the AAAS
meeting, that statistical analysis of such electroencephalography (EEG) signals has become so good that it can

be used to make anesthesia safer and better.


(a) Discernible (b) Camouflaged (c) Unobtrusive (d) Faint (e) No Change Required.

229). Tsunami is a terrible thing. And part of their terror lies in their unpredictability. Even when a submarine
earthquake that may cause one is detected, the information that is needed to determine whether a giant wave has

actually been created takes time to gather. That is time unavailable for the evacuation of coastlines at risk.
Contrariwise, issuing a warning when no subsequent wave arrives provokes naiveté and a tendency to ignore
future evacuation calls. Such tsunami-warning systems as do exist rely on seismometers to detect earthquakes,

and tide gauges and special buoys to track a wave’s passage. That is reliable, but can often be too late to get
people away from threatened coastlines. What these warning systems cannot do reliably is predict immediately
whether a given earthquake will cause a tsunami. And that, in the view of some seismologists, is a scandal. For,
as the AAAS meeting learned from Gerald Bawden of NASA, Paul Huang of America’s National Tsunami Warning

Centre, Tim Melbourne of Central Washington University, and Meghan Miller of UNAVCO, a geoscience research
consortium, the tools for accurate tsunami prediction already exist. All that needs to happen is to connect them

up.

(a) Tick (b) Cartel (c) Dejection (d) Credence (e) No Change Required.

230). It was late, 1985, before John Perry Barlow got into computers. He’d bought a word processor to keep

things efficient as he ran the family ranch, and because he wanted a machine that would print out his lyrics for the
Grateful Dead really nicely on paper. (Two jobs, two hats.) But then he procured a Macintosh and a modem, the

first ever seen in Sublette County, Wyoming, and discovered that, through this little blinking box and the tendril of

www.ibpsguide.com https://estore.ibpsguide.com Page 66 of 331


a landline, he could join an extraordinary community. In the WELL (for Whole Earth ‘Lectronic Link), one of the

first virtual bulletin boards, he moved like a cave fish, blindly, among entities without bodies. They were things of
words alone, free floating wasps of thought, in a perpetual town meeting of unleashed opinions. Everything was

possible, and almost everything allowed, in a suddenly limitless world. As he wrote for the Grateful Dead in
“Cassidy”, his most famous song, he was “a child of boundless seas”.
(a) Wisps (b) Wholes (c) Cleans (d) Hearties (e) No Change Required.

231). The government has at least stopped selling power at half the price recommended by NEPRA, an
independent regulator, slowing the growth of the $8bn of debt weighing down state-owned power firms. But the
expense of importing coal and liquefied natural gas is helping to sap Pakistan’s dwindling foreign reserves. If, as

many suspect, the government is eventually forced to devalue the rupee, the cost of imported fuel will rise, further
pushing up power bills. Earlier this year the energy minister was moved to another job after cushioning against

building too many power plants that rely on imported fuel. Meanwhile, the construction of Diamer Basha Dam, a
4,500-megawatt hydropower project, has proceeded frustratingly slowly. In other words, increased generation is
no panacea for Pakistan’s economy. But with the lights back on, the other problems are at least plain to see
(a) Hasting (b) Approving (c) Reprimanding (d) Caveat
(e) No Change Required.

232). Mr. Trump claims that he and Mr. Xi are close. The same can hardly be said of public attitudes towards
each other’s countries. A study in 2016 by Zhang Kun and Zhang Mingxin of Huazhong University of Science and

Technology found that America was far down the list of countries about which the Chinese express favorable
opinions—below Germany, Britain, France, Canada, Australia and Russia. Things may have changed since then
because views of Mr. Trump are warmer in China than in most places. But opinions of America itself are unlikely
to have greened much. A survey in the same year by the Pew Research Centre in Washington also found that

only half of Chinese respondents were favorable to America—much less than the global median “favorability
rating” for the United States of64% then. American opinions of China are even cooler. Pew’s poll in 2017 found

more Americans expressed negative views about China than positive. Such attitudes might not affect policy but

they could make public dissatisfaction easy to ignite.


(a) Ameliorated (b) Savaged (c) Primeval (d) Immaculated (e) No Change Required.

233). Compared with others who enjoy similar status in China, Mr. Zhou speaks with unusual confidence, in a
manner refreshingly free of party jargon. But despite his outspokenness, he is highly trusted by the party’s

leadership. He has been central bank governor for 15 years, the only ministerial-level official to straddle three

www.ibpsguide.com https://estore.ibpsguide.com Page 67 of 331


administrations. He was due for retirement at the last big government reshuffle five years ago but was kept on, a

credit to his rare combination of financial knowledge and political skills. Now his governorship is ending. Almost
70, he has perhaps just weeks before retirement and appears to be trying to burnish his legacy. Overall, he is

likely to be well remembered. Abroad, Mr Zhou is seen as a savvy policymaker who has tried to open the
country’s financial system more widely to global market forces, for example by pushing his government (albeit
with limited success) to permit freer trade in China’s currency, the yuan
(a) Tarnish (b) Besmirch (c) Decipher (d) Asperse (e) No Change Required.

234). Mr. Murphy’s win was the more routine. A former ambassador and Goldman Sachs executive who spent

$20m of his own money, he brought in the big guns to stump with him: Barack Obama and Joe Biden; Cory
Booker, the state’s charismatic and ambitious junior senator; and perhaps biggest of all in New Jersey, Jon Bon
Jovi, a 1980s-rockstar and native son. The last round of polls had Mr. Murphy up by 14 points; he won by 13. New
Jersey has almost 900,000 more registered Democrats than Republicans and the state tends to elect governors
from the party that does not hold the White House. Mrs. Guadagno was lieutenant-governor to the unpopular and
scandal-ridden Chris Christie, forcing her into the crude position of talking up her experience and achievements in

office, while trying to keep Mr. Christie at a distance (he did not campaign with her). But neither candidate inspired
much excitement; even by off-year election standards, turnout was low. Mr. Northam, by contrast, dramatically
outperformed expectations. As election day drew nearer, his polling lead shrunk; the last round had him up by
three points, around half as much as in October.
(a) Felicitous (b) Dexterous (c) Bulging (d) Bungling (e) No Change Required.

235). Most county assessors in Illinois (and other states) are appointed rather than elected. Electing the overseer
of such a reliable source of tax revenue is supposed to prevent such a cozy system from evolving. In this case, it

does not seem to have worked. Mr. Berrios, who has held his office since 2010, has created “a self-dealing
racket”, says Daniel Biss, a Democratic candidate for governor. Chris Kennedy, another Democratic candidate,
likens the system to extortion and wants to ban assessors from an important role in a political party and all elected

officials from jobs as property tax lawyers. Mr. Berrios has always denied all the allegations levelled against him.
This system of reciprocal gift-giving is bad enough, but it is not in fact the worst thing about how property is valued
in Chicago. Despite being run by Democrats, Cook County’s property-valuation system became deeply accretive

after the housing bubble burst in 2008.


(a) Late (b) Atavistic (c) Burgeoning (d) Destining (e) No Change Required.

www.ibpsguide.com https://estore.ibpsguide.com Page 68 of 331


236). Mr. Trump’s insurgency contains at most one of those ingredients, in the form of long-standing unhappiness

among some working-class Republicans with the liberal immigration and trade policies favoured by the party’s
elite. This was also evident in the Tea Party movement, which was in part fuelled by anti-immigration sentiment,

and thus played Goldwater to Mr. Trump’s Reagan. Yet it is hard to see Trumpism, as both a populist economic

platform and oppositionist style of politics, as an enduring Republican response to this malaise, far less an
effective one. Unlike Roosevelt’s and Reagan’s projects, it appears too reductive to survive in a two-party system

in which success depends on coalition-building. Though Mr. Trump, who has never won a majority in a national
poll, has pandered to pro-business Republicans, his protectionism is benediction to them. As is his nativism to the
suburbanites who trounced Mr. Gillespie. The president may also be too toxic to effect the transformation his
critics fear. He has undercut or tainted most people who have worked with him—which matters because

Trumpism looks heavily dependent on Mr. Trump. Unlike Reagan, he has no army of pressure groups to push his
agenda. He also has little governing success to point to. Indeed, it is not clear—considering he is yet to make
good on populist promises to pull out of NAFTA, boost infrastructure spending and rewrite immigration laws—that

he even thinks of them as a governing agenda. He has filled that gap with ethno nationalist dog-whistling; where
he has tried turning the whistles into action—as in his attempted travel ban—he has been checked
(a) Anathema (b) Stout (c) Daintiness (d) Clemency (e) No Change Required.

237). In the arrivals hall of Belém’s airport the excitement is palpable. Hundreds of supporters of Jair Bolsonaro, a
seven-term congressman and would-be president, gather under the steady gaze of a squad of policemen. Some
hold banners with Mr. Bolsonaro’s campaign slogan: “Brazil above everything, God above everyone”. A few wear
“Godfather” T-shirts, with his face in place of Marlon Brando’s. When the candidate finally emerges through sliding

doors the crowd surges forward, straining for a glimpse. While bodyguards recede through the scrum, the crowd
hoists Mr. Bolsonaro aloft as if he were a homecoming hero. The visit to Belém, the sweltering capital of the

Amazonian state of Pará, is an early stop in Mr. Bolsonaro’s campaign to win the presidential election due in
October 2018. A religious nationalist and former army captain, he is anti-gay, pro-gun, and an apologist for
dictators who tortured and killed Brazilians between 1964 and 1985. He rails against the political elite, whose

venality has been exposed by the three-year Lava Jato (Car Wash) investigation
(a) Contrive (b) Expunge (c) Derive (d) Escalate (e) No Change Required.

238). If a week is a long time in politics, try a fortnight. A string of sexual-harassment scandals in Westminster and

beyond has been followed by some crass ministerial errors. The combination exposes the weakness of Theresa
May at a crucial moment in the Brexit negotiations. The growing reports of sexual harassment claimed a minister

when Sir Michael Fallon resigned as defense secretary over past behavior that had fallen below standard. Mrs

www.ibpsguide.com https://estore.ibpsguide.com Page 69 of 331


May faced criticism for replacing him with her chief whip, Gavin Williamson, a close ally with no ministerial

experience. She was already tottering because her old friend Damian Green, the deputy prime minister, is under
investigation by the Cabinet Office over claims of sexual harassment (and possession of porn on an office

computer), which he strenuously denies. Mark Garnier, a trade minister, is also being investigated. Ministerial

blunders have further fortified the prime minister. This week she summoned home the international development
secretary, Priti Patel, who resigned after improperly and secretly meeting senior Israeli officials, apparently without
telling either the Foreign Office or Mrs May in advance.

(a) Brawnied (b) Condensed (c) Debilitated (d) Misjudged (e) No Change
Required.

239). Businesses across the continent have to contend with frequent blackouts, known as dumsor in Ghana, from
the Asante words for “off and on”. They rely on expensive backup generators, so the electricity they use is among
the costliest in the world. The full impact of incessant and high-cost energy on Africa’s economy and society is

hard to measure, but it seems safe to say that this is the biggest single barrier to development. The World Bank
reckons that if they had continuous energy supplies, sub-Saharan Africa’s economies could be growing by two
percentage points a year faster, on average, than they do now. A more limited study looking at the impact of
blackouts on small firms in Ghana found that power shortages sometimes almost halved their revenues, and in
aggregate cost the economy about2% of GDP. Putting this right will require a huge investment in generating

capacity. But thanks to a happy combination of innovation and falling costs for renewable energy, Africa may now
be able to leapfrog ahead not once but twice, skipping both polluting fossil fuels and, often, the electricity grid
itself. Last year Africa added a record 4,400MW of renewable power capacity, roughly enough to meet Nigeria’s

current consumption, according to the International Energy Agency (IEA). This is partly due to falling costs: the
price of solar panels has come down by more than 80% since 2010, and that of wind turbines is also dropping
fast. A recent study led by Anton Eberhard of the University of Cape Town found that grid-connected wind and
solar renewable energy in South Africa is now among the world’s cheapest
(a) Dreaded (b) Intermittent (c) Unremitting (d) Chronic (e) No Change

Required.

240). As Emmanuel Macron and Muhammad bin Zayed, the president of France and the crown prince of the

United Arab Emirates (UAE), walked towards the Louvre Abu Dhabi (LAD) for its grand opening on November 8th,
their eyes were fixed on the magnificent silvery domed roof—as heavy as the Eiffel Tower—that appears to float
above the galleries. They might more usefully have gazed down at the floor. For there, in the entrance, is a map

of the UAE’s coastline. All along the shore, listed as if they were ports on an old parchment, are the names of

www.ibpsguide.com https://estore.ibpsguide.com Page 70 of 331


towns around the world that manufactured the hundreds of objects on display inside. Each one is spelled out in its

own language; 26 in all. There is Greek, Spanish, German, Chinese, Russian and Arabic. There is even one in
Hebrew, for Qa al-Yahud, the old Jewish quarter in Sana’a, Yemen, where the LAD’s medieval Torah was made.

Designed by Jean Nouvel, the building is a debacle. A 30-year contract, signed in 2007, will pay €974m ($1.1bn)

to the Louvre and its partner museums in France, which have lent the LAD 300 objects
(a) Exult (b) Fiasco (c) Dud (d) Rout (e) No Change Required.

ANSWERS:
211). (b)

Explanation: - First, passage wants to convey that they spoiled their industrial champions. Mollycoddle is synonym

for the same. Secondly, It talks of the independency it has. Autarky is synonym for the same. Hence, option b is
correct usage.

212). (d)
Explanation: - First, passage wants to convey that they championed coal. Championing is correct here. Secondly,
It talks of the contradiction it has. Hence, option d is correct usage.

213). (b)
Explanation: - Here the passage wants to convey the message that it kicked out the friends and mates. Hence,
option b is correct usage as it is synonym for the same.

214). (d)
Explanation: - Here the passage wants to convey the message that it had a difficult and hard working task. Hence,
option d is correct usage as it is synonym for the same.

215). (e)
Explanation: - Here the passage wants to convey the message that they were less likely to hide . Hence, no
change is required as lurk is synonym for the same.

216). (d)
Explanation: - Here the passage wants to convey the message that it been isolated from the rest. Hence, option d

is correct usage as it is synonym for the same.

217). (c)
Explanation: - Here the passage wants to convey the message about the undiscovered drug. Hence, option c is

correct usage as it is synonym for the same.

218). (b)
Explanation: - Here the passage wants to convey the message that it provided detailed guidance. Hence, option b

is correct usage as it is synonym for the same.

www.ibpsguide.com https://estore.ibpsguide.com Page 71 of 331


219). (a)

Explanation: - Here the passage wants to convey the message that he had used tricks and cunningness. Hence,
option a is correct usage as it is synonym for the same.

220). (e)

Explanation: - Here the passage wants to convey the message that it was a distant dream. Hence, option e is
correct as no change is required.

221). (a)
Explanation: - Here the passage wants to convey the message that it could erode their chances. Hence, option a
is correct usage as it is synonym for the same.

222). (c)

Explanation: - Here the passage wants to convey the message that it sparked international concern. Hence,
option c is correct usage as it is synonym for the same.

223). (c)
Explanation: - Here the passage wants to convey the message that it became little gloomy. Hence, option c is
correct usage as it is synonym for the same.

224). (e)
Explanation: - Here the passage wants to convey the message that it was a dull market. Hence, option e is correct
as no change is required.

225). (d)
Explanation: - Here the passage wants to convey the message about insight behind her study. Hence, option d is
correct usage as it is synonym for the same.

226). (c)
Explanation: - Here the passage wants to convey the message that there is a scarcity of transplantable organs.
Hence, option c is correct usage as it is synonym for the same.

227). (d)
Explanation: - Here the passage wants to convey the message traits of quantum mechanincs. Hence, option d is

correct usage as it is synonym for the same.

228). (a)
Explanation: - Here the passage wants to convey the message that each stage is observable. Hence, option a is

correct usage as it is synonym for the same.

229). (c)
Explanation: - Here the passage wants to convey the message that it provokes cynicism or suspicion. Hence,

option c is correct usage as it is synonym for the same.

www.ibpsguide.com https://estore.ibpsguide.com Page 72 of 331


230). (a)

Explanation: - Here the passage wants to convey the message bundles of thought. Hence, option a is correct
usage as it is synonym for the same.

231). (d)

Explanation: - The passage wants to talk of warning being given. Caveat is synonym for the same. Hence option d
is correct.

232). (a)
Explanation: - Here the passage wants to convey the message that it is unlikely to improve. Hence, option a is
correct usage as it is synonym for the same.

233). (e)

Explanation: - Here the passage wants to convey the message that it is trying to polish his legacy. Hence, option
e no change is required as the usage here is correct.

234). (d)
Explanation: - Here the passage wants to convey the message that it was an awkward position. Hence, option d is
correct usage as it is synonym for the same.

235). (b)
Explanation: - Here the passage wants to convey the message that the system backfired. Hence, option b is
correct usage as it is synonym for the same.

236). (a)
Explanation: - Here the passage wants to convey the message that it is a curse to them. Hence, option a is
correct usage as it is synonym for the same.

237). (a)
Explanation: - Here the passage wants to convey the message that bodyguards forged through the scrum. Hence,
option a is correct usage as it is synonym for the same.

238). (c)
Explanation: - Here the passage wants to convey the message that it has weakened the minister. Hence, option c

is correct usage as it is synonym for the same.

239). (b)
Explanation: - Here the passage wants to convey the message that it is discontinuous energy. Hence, option b is

correct usage as it is synonym for the same.

240). (a)
Explanation: - Here the passage wants to convey the message that building is a triumph. Hence, option a is

correct usage as it is synonym for the same.

www.ibpsguide.com https://estore.ibpsguide.com Page 73 of 331


Fill in the blanks - Type - 1

Directions (241-270): The following question consists of two sentences. Read each sentence to find out whether
there is any grammatical error in it and mark your answer accordingly from the given options.

241). A. Look into the _______ and it looks back into you.
B. I was falling into _________ in my dreams last night.
C. The tragedy has sent me into the ________ and I don’t think I can come out now.
(a) Danger (b) Abyss (c) Tailor (d) Mingler (e) Tinker

242). A. The Boss fired his staff in a very _______ manner.

B. I have been competing with her but can’t win because she just too _________ at delivering the speech.
C. My ________ ways impressed the interviewer.

(a) Perspicacious (b) Perspective (c) Enigmatic (d) Eloquent (e) Dotted

243). A. The bowler’s ________ nature has caused a lot of trouble for his team.
B. I can’t talk to him because he is ________ and never listens to what I have to say.
C. Thanks to the _________ boss, the employees have been submitting work before deadlines with a fear of losing
out on their jobs.

(a) Pugnacious (b) Dreadful (c) Exemplary (d) Detrimental (e) Drooping

244). A. The party decided to ____ with the opposition seeing a bright future.

B. The employees wanted to ________ and win the battle against their boss.
C. The team was just about to _____ new player’s appointment for next match when they heard the dismal news.
(a) Dingy (b) Nullify (c) Concur (d) Remedy (e) Ramify

245). A. There has been a split between them because they ____ each other in public.

B. Had I not __________ him, I would have got my money back.

C. You have to face the repercussions as you were the one who ______ with fun.
(a) Derided (b) Recited (c) Tried (d) Enticed (e) Dried

246). A. His _______ has caused the project to delay for so long.
B. The reason he didn’t allow you to speak was your ______.

C. He asked me to stop the tricks of _____ and come to point.

www.ibpsguide.com https://estore.ibpsguide.com Page 74 of 331


(a) Mongers (b) Usurer (c) Digression (d) Ramps (e) Packs

247). A. They have been _____ you from going to gym because of bad health.
B. He was _____ me before but now he is just forcing me to do it.

C. There is no way I am _______ him from lying low.


(a) Deriding (b) Preventing (c) Conditioning (d) Tormenting (e) Tooling

248). A. He has ______ my position in the team.


B. I have _________ any hope for the project to go on.

C. The employees _____ the Boss’s new plan by not conforming to the rules.

(a) Extorted (b) Freebied (c) Ramified (d) Jeopardized (e) Criticized

249). A. The theory will have ____ when it is made public.


B. I don’t know about any of ______ of the project hurting your sentiments.
C. They decided to drop the bowler for the game as his injury might have some serious ______ for him in future if
it is not looked after.
(a) Costanzias (b) Ramifications (c) Implications (d) Satires (e) Tools

250). A. The bowler has been _____ the batsmen’s nerves with his sheer pace.
B. I tried _______ him but his punches were so hard I could not stay till the end.
C. The idea of ______ is best suited when the odds are in your enemies favor.

(a) Booming (b) Fraying (c) Decomposing (d) Drooling (e) Stooping

251). A. The recent developments _____ his anger regarding policies.

B. I tried to stay away but he ____ his intentions in a manner to provoke me.
C. They have _____ the entire script very truthfully.

(a) Homified (b) Manifested (c) Enticed (d) Eccticed (e) Presided

252). A. The team’s ___ performance has lead to the captain’s ousting.

B. They took some ____ steps which stopped the project.

C. In times of _______ economy, everyone is going with the sanctions.


(a) Thrilling (b) Growing (c) Ratifying (d) Gratifying (e) Laming

www.ibpsguide.com https://estore.ibpsguide.com Page 75 of 331


253). A. He has ________ himself from any superstitions.

B. I to be_____ from her love in order to reach my goals.


C. They dying word of his father ______ him from the problems of his own.
(a) Denigrated (b) Emancipated (c) Remoted (d) Reminisced (e) Trooped

254). A. He kept _______ about the project being too long but no one cared.
B. I tried both laughing and _________ but nothing was working.
C. His mannerisms of _____ tell us why he is not in the team.
(a) Mailing (b) Wailing (c) Tailing (d) Gailing (e) Drailing

255). A. They have _______ their resources and now have to work elsewhere.
B. Expenditure this month _______ and therefore everyone gets a bonus.

C. The runs ______ because of the impeccable bowling spell.


(a) Doomed (b) Emancipated (c) Curtailed (d) Fried (e) Extemporized

256). A. It is necessary to keep one’s _____ in face of emergency.


B. She kept her _______ during interview.
C. The bowler lost his ______ in the last over.

(a) Temper (b) Poise (c) Drool (d) Attitude (e) Ticks

257). A. It is _______ for her to be there on Saturday.

B. They have made it _______ for us to have a third fast bowler.


C. The boss’s orders were ________ for the new guy to follow.
(a) Dumb (b) Damb (c) Relegator (d) Tinker (e) Imperative

258). A. The bowlers’ injury was ______.

B. There were ______ signs of a murder.

C. She had made it ______ that she wanted a divorce.


(a) Dissect (b) Palpable (c) Sheer (d) Prose (e) Hip

259). A. He is more _____ to accidents in winters.


B. I was _______ to fall to fast bowlers but now I am comfortable against them.

C. The Chief Justice said that even the law was _____ to corruption these days.

www.ibpsguide.com https://estore.ibpsguide.com Page 76 of 331


(a) Dore (b) Abide (c) Dade (d) Prone (e) Timber

260). A. Our ship ______ towards the dead end.


B. The bowler ____ short balls despite the pitch offering no pace.

C. He ____ towards me and hugged me to my surprise.


(a) Banged (b) Threw (c) Careened (d) Moved (e) Carried

261). A. Much to his _____, he was ousted from the company.


B. They received nothing but _______ for calling upon the boss during emergency.

C. The disaster has caused ___ among the government employees.

(a) Tithing (b) Spyri (c) Chagrin (d) Chime (e) Adage

262). A. He has been taking my words _______.


B. He did everything ______ to make sure I rested him for next match.
C. The charity called for tougher ________ to protect Britain's remaining natural forests
(a) Retiringly (b) Dingily (c) Flippantly (d) Truly (e) Mattingly

263). A. His view of the party is ______

B. The _______ comments made by him led to ousting from the team.
C. He got maximum marks despite his answers being written in a _______ manner.
(a) Engorged (b) Derided (c) Jaundiced (d) Fettered (e) Deterred

264). A. He is so _______ that he even speaks ill in front of his boss.


B. His ________ behavior has led the Parliament to reject his policies.

C. Everyone likes him despite his _______ mannerisms.


(a) Gigue (b) Helmed (c) Obdurate (d) Groped (e) Dangerous

265). A. The king tried to _____ any leftover population.


B. I wanted to call him but his plans were to ______ mine.

C. The bowlers’ dream was to ________ the batsmen after being done so in the previous game.

(a) Hanker (b) Obliterate (c) Aghast (d) Dread (e) Mingle

266). A. His memories are too ________ for me.

www.ibpsguide.com https://estore.ibpsguide.com Page 77 of 331


B. The _______ economy has made the government do things that one would not have dreamed of.

C. The interviewer’s _____ questions made me sad.


(a) Obdurate (b) Poignant (c) Repealing (d) Tooling (e) Wiping

267). A. The house is in a _____ place.


B. He has _____ himself from the rest of the team after tonight’s loss.
C. I tried to go there but he has ____ himself from any contact.
(a) Ambient (b) Creptive (c) Secluded (d) Derided (e) Trooped

268). A. His boss’s comments have made him ________.

B. I had to be _______ to make sure my family survived.


C. The reason he doesn’t have many friends is because of his _______ nature.

(a) Dolmen (b) Jagging (c) Objective (d) Vindictive (e) Subjective

269). A. He has been treating the class with ____.


B. There has been _____ on coach’s part and that’s the reason team is losing.
C. He had to change his _____ in face of the war.
(a) Facie (b) Limp (c) Levity (d) Lenient (e) Impulse

270). A. I could not ________ his reason of leaving me.


B. The boss tried to ________ how the situation had gone so bad.

C. The bowler was up to ________ the coach’s words.


(a) Tepee (b) Fathom (c) Blabber (d) Valiance (e) Droop

ANSWERS:
241). Correct Answer is: (b)

Explanation: Option b is the most appropriate answer which fits in all three statements.

Abyss - something without bottom.


242). Correct Answer is: (d)

Explanation: Option d is the most appropriate answer which fits in all three statements.

Eloquent – Speaking beautifully and forcefully.


243). Correct Answer is: (a)

Explanation: Option a is the most appropriate answer which fits in all three statements.

www.ibpsguide.com https://estore.ibpsguide.com Page 78 of 331


Pugnacious – having a quarrelsome or aggressive nature.

244). Correct Answer is: (c)


Explanation: Option c is the most appropriate answer which fits in all three statements.

Concur- to agree with someone


245). Correct Answer is: (a)
Explanation: Option a is the most appropriate answer which fits in all three statements.

Derided – to make fun of someone


246). Correct Answer is: (c)
Explanation: Option c is the most appropriate answer which fits in all three statements.

Digression – a departure from the main issue

247). Correct Answer is: (b)


Explanation: Option b is the most appropriate answer which fits in all three statements.

248). Correct Answer is: (d)


Explanation: Option D is the most appropriate answer which fits in all three statements.
Jeopardize – put something at risk

249). Correct Answer is: (c)


Explanation: Option c is the most appropriate answer which fits in all three statements.
Implications - The action or state of being involved in something

250). Correct Answer is: (b)


Explanation: Option b is the most appropriate answer which fits in all three statements.
Fraying – to wear out

251). Correct Answer is: (b)


Explanation: Option b is the most appropriate answer which fits in all three statements.
Manifested - demonstrated

252). Correct Answer is: (e)


Explanation: Option e is the most appropriate answer which fits in all three statements.

Laming- weakening

253). Correct Answer is: (b)


Explanation: Option b is the most appropriate answer which fits in all three statements.

Emancipated – freeing from something

254). Correct Answer is: (b)


Explanation: Option b is the most appropriate answer which fits in all three statements.

Wailing - crying

www.ibpsguide.com https://estore.ibpsguide.com Page 79 of 331


255). Correct Answer is: (c)

Explanation: Option c is the most appropriate answer which fits in all three statements.
Curtailed - reduce in extent or quantity

256). Correct Answer is: (b)

Explanation: Option b is the most appropriate answer which fits in all three statements.
Poise - balance, hold (oneself) steady

257). Correct Answer is: (e)


Explanation: Option e is the most appropriate answer which fits in all three statements.
Imperative – something that cannot be avoided

258). Correct Answer is: (b)

Explanation: Option b is the most appropriate answer which fits in all three statements.
Palpable - evident

259). Correct Answer is: (d)


Explanation: Option D is the most appropriate answer which fits in all three statements.
Prone - likely or liable to suffer from

260). Correct Answer is: (c)


Explanation: Option c is the most appropriate answer which fits in all three statements.
Careened – rushing in uncontrollable manner.

261). Correct Answer is: (c)


Explanation: Option c is the most appropriate answer which fits in all three statements.
Chagrin- annoyance

262). Correct Answer is: (c)


Explanation: Option c is the most appropriate answer which fits in all three statements.
Flippantly – disrespectfully

263). Correct Answer is: (c)


Explanation: Option c is the most appropriate answer which fits in all three statements.

Jaundiced – viewing something in a biased manner

264). Correct Answer is: (c)


Explanation: Option c is the most appropriate answer which fits in all three statements.

Obdurate- stubborn

265). Correct Answer is: (b)


Explanation: Option b is the most appropriate answer which fits in all three statements.

Obliterate – completely destroy

www.ibpsguide.com https://estore.ibpsguide.com Page 80 of 331


266). Correct Answer is: (b)

Explanation: Option b is the most appropriate answer which fits in all three statements.
Poignant – if something is poignant it makes you sad

267). Correct Answer is: (c)

Explanation: Option c is the most appropriate answer which fits in all three statements.
Secluded - undisturbed

268). Correct Answer is: (d)


Explanation: Option D is the most appropriate answer which fits in all three statements.
Vindictive - revengeful

269). Correct Answer is: (c)

Explanation: Option c is the most appropriate answer which fits in all three statements.
Levity – non-serious behavior in serious situation

270). Correct Answer is: (b)


Explanation: Option b is the most appropriate answer which fits in all three statements.
Fathom – Understand

Fill in the blanks Type - 2


Directions (271-300): In each of the following sentences there is one blank space. Below sentence there are four

words denoted by (a), (b), (c) and (d). Find one word that to be fitted in both the sentences I and II and another
word that to fit in sentence III and to make it meaning fully complete, In case no word is fit to these sentences,
option 'e' is the answer.

271). I) We are always dying to ______ before the politicians.


II) He wanted to ________ but my father stopped him.

III) The markets have a tendency to ______ after an attack.


(a) Abstain / Gimp (b) Board / trump (c) Genuflect / Slump (d) Mandan / Aghast (e) None of these

272). I) The boss sued him for ____.


II) I can handle anything but ______.

III) He was filled with ____ after he had ousted the player from the team.

(a) Tantrum / Empident (b) Slander / Remorse (c) Hum / Incipit (d) Diction / Vole (e) None of these

273). I) His ______ nature is what makes it difficult to work with him.

www.ibpsguide.com https://estore.ibpsguide.com Page 81 of 331


II) The players do everything on time because the coach is _____

III) After the announcement of results, I was _____ with pain.


(a) Punctilious / Racked (b) Shrewd/ Frond (c) Peek / Mucky (d) Cheeky /Intrepid (e) None of these

274). I) The bowler has been bowling an ____ line and length.
II) He gave ______ answers to the interviewer’s questions.
III) He would ____ it his honor if we go there.
(a) Agnate / tingle (b) Eminent/ Rangel (c) Insidious / Teem (d) Impeccable / Deem (e) None of these

275). I) The coach was ______ and the players knew it.

II) His ____ nature won him the job.


III) Where do these tributaries ______ from?

(a) Dig / Magnate (b) Demure/ Emanate (c) Emmy / Assail (d) Horde /Dinette (e) None of these

276). I) Even though the coach knew of the bowler’s injuries, he decided to ______ it.
II) Why did you ________ him upon hearing of his expulsion?
III) North Korea has long been putting up a _______ for the world and doing nuclear experiments behind it.
(a) Dowl / Gape (b) Dale/ Male (c) Condone / Facade (d) Investigate /Bit (e) None of these

277). I) He grew _______ as soon as he came to know he had topped.


II) His ______ behavior has helped the team keep calm in difficult situation.

III) Everyone knows the ______ of Mr. Modi.


(a) Dingy / Extract (b) Shriek/ Excerpt (c) Mike / Story (d) Exuberant /Fable
(e) None of these

278). I) The magician________ a strong man out of the box.

II) He batted well the bowler ______ a brilliant last spell to remove him.

III) He couldn’t stop _____me after yesterday’s performance.


(a) Horsed / Praising (b) Barged/ Scorning (c) Conjured / Extolling (d) Instinet /Revering (e) None of

these

279). I) The blast caused ______ among the crowd.

II) The batsman’s fall caused _______ in the dressing room.

www.ibpsguide.com https://estore.ibpsguide.com Page 82 of 331


III) You will face the _________ of your actions.

(a) Doom / Signs (b) Hysteria/ Repercussions (c) Fall / Avengers (d) Shriek /Differences (e) None of
these

280). I) The captain tried everything but could not ______ the coach.
II) Seeing the lion leap, I stayed back and tried to _______ it.
III) The _______ workload made him commit suicide.
(a) Kiff / Huge (b) Falter/ poor (c) Bale / Extreme (d) Mollify /Sheer (e) None of these

281). I) He is very _____ as he speaks ill even in front of his boss.

II) The coach started to behave in a ______ manner when the captain did not listen.
III) He has been _______ about the project for past 5 months.

(a) Jupiter / Wrangling (b) Sinus/ Flailing (c) Meek / Angling (d) Churlish /Babbling (e) None of these

282). I) The teacher was unable to decipher his _______ behavior.


II) The coach has a reputation of giving out _________ decisions which even the captain is unable to comprehend.
III) He was very _____ for coming late and that’s why the teacher pardoned him.
(a) Boyous / Facade (b) Abstruse/ Contrite (c) Girly / Grate (d) Churlish /Apposite (e) None of these

283). I) The employees have been trying to ______ him but he listens to no one.
II) The Captain went to _____ the coach knowing that he can fooled.

III) Plato was an _________ philosopher.


(a) Mole / Orated (b) Dale/ Impeccable (c) Pluck/ Educated (d) Cajole / Erudite (e) None of these

284). I) The players started to ____ at lunch after a tiring afternoon session.
II) The kid began to ______ as soon as her mother left.

III) He went to the interview _______ but still managed to clear the round.

(a) Wobble / Killingly (b) Wrangle/ Daringly (c) Gobble / Impromptu (d) Hopper /Foolishly (e) None of
these

285). I) His _______ nature has energized the entire team in times of crises.
II) The boss told his to be _____ this year as there was not much work load.

III) He _______ from going there because of heavy media presence.

www.ibpsguide.com https://estore.ibpsguide.com Page 83 of 331


(a) Maine / Flailed (b) Boisterous/ Abstained (c) Stouting / Ruptured (d) Facde /Frunged (e) None of

these

286). I) The coach tried to _____ the bowler but captain kept insulting him.

II) His father used to ______ him to much but after his failure the attitude has changed.
III) Media has been strongly ________ the minister’s actions.
(a) Moddle / Disdaining (b) Wobble/ Mourning (c) Coddle / Denouncing (d) Grope /reverence (e)

None of these

287). I) Seeing him in a ___ mood his son came and started to play in front of him.

II) Stop being _____ , we will find a way out.


III) I was _______ by her beauty at the very first sight.

(a) Sinuous / Disdained (b) Impituous/ Scorned (c) Peeky / Esteemed (d) Dolorous /Enthralled (e)
None of these

288). I) The batsman took the team home _______.


II) The family of nine is run by him ____________.
III) I want ________ answers, I don’t have much time.

(a) Peggingly / Short (b) Suo-moto / Terse (c) truly / Immediate (d) Alone /hum (e) None of these

289). I) He is ______ of voting against BJP.

II) The coach was ________ about keeping the bowler out of the team for the next match.
III) He looks ________ after yesterday’s hard work.
(a) Tendentious / Wan (b) Frail/ Dale (c) Pluck / Dole (d) Done /won
(e) None of these

290). I) The media wanted to _________ the entire team for losing to Bangladesh, but the last match saved them.

II) After trying everything and failing he decided to _______ his son to improve his work.
III) The court ______ to remove all the barriers put up outside government buildings.

(a) Pan / Ordained (b) Shrug/ Asked (c) Move / opted (d) Ask /Maled

(e) None of these

291). I) He has ______ me from captaincy.

www.ibpsguide.com https://estore.ibpsguide.com Page 84 of 331


II) After his credentials were found wrong, he was ______ by the principal.

III) I was _______ by the fact that they left so late.


(a) Thrown / Examed (b) Relegated/ Perturbed (c) Promoted / Esteemed

(d) Instigated /Revered (e) None of these

292). I) The victory was ______ considering a weak opposition.


II) I am a ________ person living by doing savings and paying loans on time.
III) His ________ behavior always gets him into fights.
(a) Quintessential / Moving (b) Asking/ cool (c) Quotidian / Pugnacious

(d) True /Dole (e) None of these

293). I) They decided to ____ behind the walls as it was safe there.

II) Why did you __________ yourself from your own friends?
III) His ________ speech got him a standing ovation.
(a) Joke / Dying (b) Die/ Scorny (c) Hide / Sheered (d) Shroud /Impeccable
(e) None of these

294). I) Why are they fighting over such a ______ sum?

II) I wanted to go for the run, but considering the _____ total of opposition, I decided to avoid the risk.
III) There is no ________ of his losing now.
(a) Meager / kill (b) Small/ pine (c) Paltry / Spectre (d) Intradent /Daint

(e) None of these

295). I) His bowling has been too ______ showing his inability to bowl long spells.

II) He has decided to take some rest now and will appear only in a ______ manner.
III) His _______ behavior caused him his job.

(a) Sporadic / Repugnant (b) cheeky/ fool (c) dying / folly (d) Dead /true

(e) None of these

296). I) His tragic life has made him the epitome of a _______ person.

II) The company was in _______ after the project managers’ demise
III) He has been trying to _____ us in order to obtain the secret files.

(a) Exam / Clout (b) distract/ Reprimand (c) Wretch / Subterfuge

www.ibpsguide.com https://estore.ibpsguide.com Page 85 of 331


(d) Insinuate /Trail (e) None of these

297). I) His ____ for the world cup is what motivates him.
II) The workers were unable to ______ themselves in face of drought.

III) The sheer ease with which fraudulent practices have been carried out and the length of time
over which they continued suggest that the ____ is much deeper
(a) Quench / Rot (b) Muster/ glop (c) Erika / Kine (d) Anton /Reel

(e) None of these

298). I) His ______ is what earned him the title of The Butter Applier.

II) There is no need of using your ______ on me as it won’t work.


III) The two Prime Ministers called for ______ the infrastructure of support to terrorism

(a) Petrification / Minding (b) Enigma/ Drooling (c) Sycophancy / Dismantling


(d) Training /Moving (e) None of these

299). I) He decided to _______ his opponent because he was also injured in a similar fashion last season.
II) They wanted to _____ him but he rejected.
III)Mr. Modi’s decision to stick to protocol,and not welcome Mr. Trudeau as ______as he has
tended to do for many foreign visitors, was a signal.
(a) Support / puzzlingly (b) Kill/ Anglingly (c) Succor / Effusively (d) Help / Wranglingly (e) None of
these

300). I) He was _______ that the team will win in the finals.
II) They need to be ______ and not listen to negative comments.

III) They have been ________ their new product everywhere.


(a) Sure / Drubbing (b) Dole/ Asking (c) Dead/ Showing (d) Sanguine /Touting (e) None of these

ANSWERS:
271). Correct Answer is: (c)

Explanation: Genuflect – to bend the knee

Slump – to fall suddenly


272). Correct Answer is: (b)

Explanation: Slander – Speaking false

www.ibpsguide.com https://estore.ibpsguide.com Page 86 of 331


Remorse – Feeling of sorrow after doing something

273). Correct Answer is: (a)


Explanation: Punctilious – If you are punctilious you want everything exactly and properly

Racked – In extreme pain


274). Correct Answer is: (d)
Explanation: Impeccable – if something is impeccable it is perfect without any faults

Deem – to consider
275). Correct Answer is: (b)
Explanation: Demure – serious and quiet

Emanate – Originate from

276). Correct Answer is: (c)


Explanation: Condone- to regard a harming action as harmless
Façade – to see only the façade means a fake front to fool the observer

277). Correct Answer is: (d)


Explanation: Exuberant –full of life
Fable – short story
278). Correct Answer is: (c)
Explanation: Conjured – to make something happen as if done magically
Extolling – to praise
279). Correct Answer is: (b)
Explanation: Hysteria - Exaggerated or uncontrollable emotion or excitement

Repercussions - an unwelcome one


280). Correct Answer is: (d)
Explanation: Mollify – if you mollify someone, you try to make them less upset.
Sheer – extreme
281). Correct Answer is: (d)

Explanation: Churlish – bad-tempered and rude

Babbling – talk foolishly


282). Correct Answer is: (b)

Explanation: Abstruse – Hard to understand

Contrite – Showing sorrow for wrongdoing


283). Correct Answer is: (d)

Explanation: Cajole – flattering

www.ibpsguide.com https://estore.ibpsguide.com Page 87 of 331


Erudite – a learned and scholarly person

284). Correct Answer is: (c)


Explanation: Gobble – Eat noisily and quickly.

Impromptu – Something said or done without thinking


285). Correct Answer is: (b)
Explanation: Boisterous – Full of energy

Abstained – kept away from


286). Correct Answer is: (c)
Explanation: Coddle – protect

Denounce – Criticize

287). Correct Answer is: (d)


Explanation: Dolorous – Sad
Enthralled – Attracted

288). Correct Answer is: (a)


Explanation: Suo-moto – no its own
Terse – Concise/ using few words
289). Correct Answer is: (a)
Explanation: Tendentious – Having one certain strong opinion
Wan – tired and looking pale
290). Correct Answer is: (a)
Explanation: Pan – Criticize

Ordained – Officially ordered


291). Correct Answer is: (b)
Explanation: Relegated - Assign an inferior rank.
Perturbed – Disturbed
292). Correct Answer is: (c)

Explanation: Quotidian – Common

Pugnacious - Quarrelsome
293). Correct Answer is: (d)

Explanation: Shroud - Hide

Impeccable – Absolutely correct


294). Correct Answer is: (c)

Explanation: Paltry – small

www.ibpsguide.com https://estore.ibpsguide.com Page 88 of 331


Spectre – The idea that something bad might happen in future

295). Correct Answer is: (a)


Explanation: Sporadic – Irregular

Repugnant – Unpleasant behaviour


296). Correct Answer is: (c)
Explanation: Wretch – Unhappy

Subterfuge – A trick in order to achieve something


297). Correct Answer is: (a)
Explanation: Quench – Satisfy (thirst)

Rot – decay, decline

298). Correct Answer is: (c)


Explanation: Sycophancy – Flattering
Dismantling – Destroying/ Removing

299). Correct Answer is: (c)


Explanation: Succor – help
Effusively – enthusiastically
300). Correct Answer is: (d)
Explanation: Sanguine – Hopeful
Tout – Act of encouraging sale of something

Fill in the blanks - Type - 3

Directions (301-330): Four sentences are given with a blank in each. Five words are also given. The blank in each
sentence can be filled by one or more of the four words given. Similarly, each word given in the choices can go

into any number of sentences. Identify the number of sentences each word can go into and mark as your answer
the maximum number of sentences any word can go into.

301). A. The underlying problems of the project are _________.


B. He has __________ intentions which no one is able to decipher.

C. The murderer had a very _________ behavior before the incident.

D. The batsman played a/an __________ role in the victory.


(a) Instigated (b) Precursor (c) Inconspicuous (d) Sham

(e) Eponymous

www.ibpsguide.com https://estore.ibpsguide.com Page 89 of 331


302). A. I was ______ by his way of batting today.
B. They were _________ by the fact that I topped the class.
C. He ______ me with his speaking skills despite not studying in school.

D. The magician _____ the audience by taking out the rat from the cat’s mouth.
(a) Daunted (b) Flabbergasted (c) Extemporized (d) Grazed

(e) Raged

303). A. We are always dying to ______ before the politicians.

B. He wanted to ________ but my father stopped him.

C. After the surgery, the doctors told him that he could ________ no more.
D. The queen is remorseful that she did not ______ before her superiors.

(a) Dole (b) Renounce (c) Foam (d) Genuflect (e) Petty

304). A. There is no need to ________ him, he won’t listen.


B. The coach failed to _______ his team regarding the new rules.
C. I decided to ______ him after he was suspended for bad behavior.
D. You can’t _______ someone these days.

(a) Gump (b) Wail (c) Extemporize (d) Move (e) Edify

305). A. He grew _______ as soon as he came to know he had topped.

B. His ______ behavior has helped the team keep calm in difficult situation
C. It is imperative that the employee be of ________ qualities as it will help the entire company.
D. He was _______ after getting the new car.

(a) Apparent (b) Appalling (c) Exuberant (d) Timber (e) Hop

306). A. The boss gave wrong speech exemplifying his _________ character with work.

B. The coach’s authority grew _____ and even the players did not support him.
C. He kept talking like a girl, showing his ______ abilities.

D. It is a mystery that he is getting _______.

(a) Croat (b) Effete (c) Dingy (d) Denounce (e) Rangy

307). A. His ________ actions proved he is not suited for captaincy.

www.ibpsguide.com https://estore.ibpsguide.com Page 90 of 331


B. After the _______ thief was caught red handed, the police thrashed him.

C. His _______ nature is making it difficult for the project to pass.


D. He has been acting like a/an _______ of late.
(a) Adage (b) Imbecile (c) Grope (d) Tippy (e) Sulk

308). A. The summit _____ with the President giving out a wonderful speech.
B. Yesterday’s match _________ with both the batsmen at 99.
C. The project is _________, give it to the executives for the checking.
D. He __________ his birthday with a fine glass of champagne.

(a) Stocked (b) Made (c) Grossed (d) Enshrined (e) Culminated

309). A. I) The teacher was unable to decipher his _______ behavior.

B. The coach has a reputation of giving out _________ decisions which even the captain is unable to comprehend.
C. The jury found the criminal’s actions _______ considering his good reputation.
D. The theory given by him is ________, no one buys it.
(a) kuku (b) Kirk (c) Abstruse (d) Jostle (e) Mingle

310). A. The employees have been trying to ______ him but he listens to no one.

B. The Captain went to _____ the coach knowing that he can fooled.
C. He tried to ______ me by his talking and even offered me money.
D. He got the name of Mr. Butter Applier because all he does is _______ people.

(a) Joke (b) Renounce (c) Cajole (d) fool (e) Denounce

311). A. His _______ nature has energized the entire team in times of crises.

B. The boss told his to be _____ this year as there was not much work load.
C. The Minister has a very ________ behavior and is therefore able to gather support quickly.

D. He got the project done with his ________ and impeccable speech.

(a) Boisterous (b) Hum (c) digressing (d) snaring (e) flailing

312). A. He has been _________ by greed for money and power.

B. The captain’s legacy is now ____________ by the match fixing scandal.


C. His dream was ________by a dark horror of his reality.

D. The author has done justice with this book as it has not been _______ by any unknown references.

www.ibpsguide.com https://estore.ibpsguide.com Page 91 of 331


(a) Stomped (b) Dumbfound (c) Tainted (d) Subtle (e) Banned

313). A. The deal has been _______ in secrecy.


B. The entire scandal of fixing has been _________ from the public as well as the officials.

C. His files were _____ by his wife for aiding him against the tax officers.
D. His thoughts are _______ in his diary which no one can see.
(a) frossed (b) Shrouded (c) killed (d) Groped (e) Biden

314). A. There was a tinge of __________ in his harsh words.

B. His ______ speech was enough for the audience to boo him.

C. He lost his temper among the crowd and there was a ______ in things he said.
D. I know what I said exemplified ________ but I am sorry for it.

(a) Amorite (b) Nazify (c) Bulge (d) Rancor (e) Dissipate

315). A. The coach ______the bowler for not bowling Yorkers in the final over.
B. I was ________ by the boss because of the mistakes I made in the project.
C. They ______ me to take revenge with my brother.
D. He was disheartened at being ________ by his own friends.

(a) Groped (b) Disportioned (c) Rebuked (d) Snared (e) Bugged

316). A. He decided to _________ the company of the despicable employees.

B. I wanted to _____ the house of dogs, but they were too cute too let go.
C. Being the captain, only he had the authority to ______ the team of the bowler.
D. You can’t ______ the class of him like this.

(a) Dunk (b) Adage (c) Rebuke (d) Purge (e) Mince

317). A. His house looked ______ and old.

B. The ______ painting was sold for a huge sum of 1million.


C. His clothes were so ______ that even a dog won’t smell them.

D. The king’s palace is now just an old and ______ place.

(a) Vale (b) Shabby (c) Natty (d) Collocial (e) Quoiet

318). A. His ________ ways were the reason of his tenure termination.

www.ibpsguide.com https://estore.ibpsguide.com Page 92 of 331


B. Being _________ is a quality no likes to see in his/her partner.

C. The coach reprimanded the bowler for his ______ way of living.
D. The ________ manner with which he arrived at the party made everyone dislike him.
(a) Jay (b) Strained (c) Wedged (d) Horned (e) Unkempt

319). A. Steve Smith was at the _______ of his career when caught for ball tampering.
B. He said he wanted to reach the _______ of the Mount Everest.
C. The ________ of hatred was reached when they walked out of the meeting disregarding the boss.
D. I am at the _________ of this class and no one can dismantle me.

(a) Infringement (b) Culmination (c) Bong (d) Zenith (e) Edge

320). A. The entire set up was a ______ for the batsmen to hit the bowler in air.

B. They walked deep into the enemies’ _______ unaware of it completely.


C. I was caught in a ________ as the boss questioned me about the project.
D. The show was a ________ for the murderer.
(a) Frond (b) Dole (c) Hill (d) Snare (e) Make up

321). A. After I proposed to her, she ______ me completely.

B. The boss ________ his secretary’s piece of advice but regretted it the following day.
C. The bowler is not happy since the coach ________ him last week.
D. I _______ him in revenge of what he did to me.

(a) Snubbed (b) Dismantled (c) Grinded (d) Snared (e) Nabbed

322). A. It is time for you to _______ the entire project report.

B. The bowler decided to _______ the mistakes made by him on field by training entire night.
C. He knew the answer he wrote was wrong but could not ____________ it as the teacher took sheet from him.

D. The police condemned their officer’s actions and said that they will soon _________ the grave horror done by

him.
(a) Clout (b) Rectify (c) Improve (d) Remake (e) Degrade

323). A. The ____________ grew so big that emergency that alerts were issued in the nearby villages.
B. The Indian team, that performed brilliantly for the entire match, seemed caught in a ______ after the captain’s

wicket fell.

www.ibpsguide.com https://estore.ibpsguide.com Page 93 of 331


C. The situation got the actor into a _______ of controversy.

D. The project created such havoc that the entire company is in a ______.
(a) Postage (b) Maelstrom (c) Wrangle (d) Bell (e) Storm

324). A. The ________ bowler was criticized for taking the night off.
B. He is so ________ that the boss himself applauded him last week.
C. These days being _____ is of no use, you need to be smart.
D. After gaining the riches, he changed from being ________ to a lazy person.
(a) Clear (b) Gumpy (c) Frond (d) Industrious (e) Denise

325). A. The witch cursed the man of being _____ his entire life.
B. He was ______ in his last days and no cared for him.

C. The _____ bowler wanted to retire after the worst series of his life.
D. He lives in a _____ condition up in the hills.
(a) Janis (b) Subterfuge (c) Volition (d) Gent (e) Forlorn

326). A. The bowler removed the batsmen on the very first ball to _____ the rumors of his getting old and weak.
B. The staff decided to _______ the truth of their sales to make sure the company suffered after they left.

C. He decided to ________ the opposition’s agenda in public and media.


D. He could have come to me and told me about the errors but instead he chose to _______ my entire homework
to teacher.

(a) Stomp (b) Hum (c) Debunk (d) Jade (e) Chum

327). A. The ______ shown by team in absence of their top batsmen is tremendous.

B. His _____ skills in the interview impressed me a lot.


C. Even though the project report wasn’t finished, his ______ attitude made me pitch to the distributors

successfully.

D. The General is known for his _________ during the 1962 war.
(a) Horrendous (b) Bravado (c) Jaded (d) Clumsy (e) Chummy

328). A. There was a/an ______ smell in the kitchen after the fridge was opened.
B. His a/an _________ remarks against me made me hit him in public.

C. The coach has a way of provoking the team with his a/an________ proverbs.

www.ibpsguide.com https://estore.ibpsguide.com Page 94 of 331


D. The boss had a/an _______ attitude a year back but it changed after the company suffered huge losses.

(a) Stomping (b) Distorted (c) Engrossing (d) Acrid (e) Feeble

329). A. The new boss was called upon to revive the _____ business.

B. The _____ team was called upon after 10 years of obscurity.


C. Project is in a _______ state until the boss signs it.
D. The President gave assurance to people that he will improve the ______ economy of the country.
(a) Stomp (b) Clout (c) Moribund (d) Snarky (e) Gimpy

330). A. The teacher is too _____ with his students because of the conditions they live in.

B. The coach had a _____ behavior which led to his team playing in an unfair manner.
C. His ______ attitude is the reason you are lethargic today.

D. The staff begged the boss to be a bit _________ in his approach.


(a) Dingy (b) Lax (c) Distressed (d) Snarky (e) Gentle

ANSWERS:
301). Correct Answer is: (c)
Explanation: ‘Inconspicuous – Not easily noticed’ fits in the all the sentences correctly and explain the idea
completely.
302). Correct Answer is: (b)
Explanation: Flabbergasted (Astonished) - fits in the all the sentences correctly and explain the idea completely.

303). Correct Answer is: (d)


Explanation: ‘Genuflect – to bend the knee’ fits in the all the sentences correctly sand explain the idea completely.
304). Correct Answer is: (e)

Explanation: ‘Edify – to educate especially about moral conduct’ fits in the all the sentences correctly and explain
the idea completely.

305). Correct Answer is: (c)

Explanation: ‘Exuberant – Happy and lifelike’ fits in the all the sentences correctly and explain the idea completely.
306). Correct Answer is: (b)

Explanation: ‘Effete – Weak/ Worn out’ fits in the all the sentences correctly and explain the idea completely.

307). Correct Answer is: (b)


Explanation: ‘Imbecile - Stupid’ fits in the all the sentences correctly and explain the idea completely.

308). Correct Answer is: (e)

www.ibpsguide.com https://estore.ibpsguide.com Page 95 of 331


Explanation: ‘Culminated - Ended’ fits in the all the sentences correctly and explain the idea completely.

309). Correct Answer is: (c)


Explanation: ‘Abstruse – Hard to understand’ fits in the all the sentences correctly and explain the idea

completely.
310). Correct Answer is: (c)
Explanation: ‘Cajole - Flatter’ fits in the all the sentences correctly and explain the idea completely.
311). Correct Answer is: (a)
Explanation: ‘Boisterous – Full of life’ fits in the all the sentences correctly and explain the idea completely.
312). Correct Answer is: (c)

Explanation: ‘Tainted – If you are tainted by something, your purity is affected by it’ fits in the all the sentences

correctly and explain the idea completely.


313). Correct Answer is: (b)

Explanation: ‘Shrouded - Hidden’ fits in the all the sentences correctly and explain the idea completely.
314). Correct Answer is: (d)
Explanation: ‘Rancor – feeling of bitter unforgiving hatred’ fits in the all the sentences correctly and explain the
idea completely.
315). Correct Answer is: (c)
Explanation: ‘Rebuked – speaking severely or angrily’ fits in the all the sentences correctly and explain the idea
completely.
316). Correct Answer is: (d)
Explanation: ‘Purge – Get rid of’ fits in the all the sentences correctly and explain the idea completely.

317). Correct Answer is: (b)


Explanation: ‘Shabby – old and bad in condition’ fits in the all the sentences correctly and explain the idea
completely.

318). Correct Answer is: (e)


Explanation: ‘Unkempt – untidy and not looking after oneself’ fits in the all the sentences correctly and explain the

idea completely.

319). Correct Answer is: (d)


Explanation: ‘Zenith - top’ fits in the all the sentences correctly and explain the idea completely.

320). Correct Answer is: (d)

Explanation: ‘Snare - Trap’ fits in the all the sentences correctly and explain the idea completely.
321). Correct Answer is: (a)

Explanation: ‘Snubbed - Ignored’ fits in the all the sentences correctly and explain the idea completely.

www.ibpsguide.com https://estore.ibpsguide.com Page 96 of 331


322). Correct Answer is: (b)

Explanation: ‘Rectify – make something correct’ fits in the all the sentences correctly and explain the idea
completely.

323). Correct Answer is: (b)

Explanation: ‘Maelstrom – whirlpool/uncontrollable situation’ fits in the all the sentences correctly and explain the
idea completely.

324). Correct Answer is: (d)


Explanation: ‘Industrious – Hard working’ fits in the all the sentences correctly and explain the idea completely.
325). Correct Answer is: (e)

Explanation: ‘Forlorn – deserted and unhappy’ fits in the all the sentences correctly and explain the idea

completely.
326). Correct Answer is: (c)

Explanation: ‘Debunk – to show people that something is not good as it is being told’ fits in the all the sentences
correctly and explain the idea completely.
327). Correct Answer is: (b)
Explanation: ‘Bravado- Courage/confidence’ fits in the all the sentences correctly and explain the idea completely.
328). Correct Answer is: (d)
Explanation: ‘Acrid - bitter’ fits in the all the sentences correctly and explain the idea completely.

329). Correct Answer is: (c)


Explanation: ‘Moribund – No longer operating effectively’ fits in the all the sentences correctly and explain the idea
completely.

330). Correct Answer is: (b)


Explanation: ‘Lax - lenient’ fits in the all the sentences correctly and explain the idea completely.

Fill in the blanks - Type – 4

Directions (331-360): The following statements have two blanks which are to be filled with the options given

below. Choose the correct combination which can most suitably complete the sentence without altering the
meaning of the statement. If none of the combinations makes an impact, choose (e) i.e. “No combination fits” as

your answer.

331). You might think that, in China of all places, the centre reigned supreme. At a Communist Party congress

last month, its leader, Xi Jinping, acquired even greater power by filling jobs with his allies. Yet local governments

www.ibpsguide.com https://estore.ibpsguide.com Page 97 of 331


will not always heed him, even with his new______. True, they will put on a show if it does not cost much—as

when officials from the _________ government of Henan province went to pay homage to a Paulownia tree planted
by Mr. Xi, who had urged delegates at the congress to “follow the leadership core”, ie, him.
(I) Gump

(II) Clout
(III) Shroud
(IV) Sycophantic
(V) Enmity
(a) I-IV (b) I-III (c) II- IV (d) II-V (e) No combination fits

332). Some clinics buy time with puberty-blockers, which ______the action of sex hormones. But these may have
harmful side-effects. Furthermore, most gender-dysphoric children will probably not become transgender adults.

Studies are scarce and small, but suggest that, without treatment, a majority will end up comfortable in their birth
sex, so treatment would be harmful. Unfortunately, no one knows how to tell which group is which. Yet some trans
activists have thrown caution to the wind. Specialists, who start by trying to help gender-dysphoric children settle
in their birth identities, rather than making a speedy______, risk being labeled transphobes and forced out of their
jobs. Few are willing to say that some such children may actually be suffering from a different underlying problem,
such as anorexia or depression.

(I) Degrees
(II) Ingestion
(III) Jinx

(IV) Suppress
(V) Switch
(a) I-II (b) II-III (c) II- IV (d) IV-V (e) No combination fits

333). The marginal cost of reducing emissions is currently far lower than the marginal cost of taking carbon

dioxide straight from the atmosphere. But climate is not a short-term game. And in the long term, ignoring the

need for negative emissions is _______at best. The eventual undertakings, after all, will be_______. The median
IPCC model assumes sucking up a total of810bn tonnes of carbon dioxide by 2100, equivalent to roughly20 years

of global emissions at the current rate. To have any hope of doing so, preparations for large-scale extraction

ought to begin in the 2020s.


(I) Complacent

(II) Folly

www.ibpsguide.com https://estore.ibpsguide.com Page 98 of 331


(III) Pittance

(IV) Gargantuan
(V) Agnostic
(a) I-V (b) II-III (c) I- IV (d) II-V (e) No combination fits

334). Mr. Turnbull is already under pressure. His government has lost its majority in the lower house, after the
forced resignation of two MPs from the governing coalition found to be dual nationals and thus in _____of the
constitution. At least until by elections next month, Mr. Turnbull must rely on support from independents. The
citizenship mess was not of Mr. Turnbull’s making, but it has taken a toll. The government trailed the opposition

by ten points in a recent opinion poll. Australians’ ringing endorsement of gay marriage gives him a chance to

reclaim some authority, by bringing about a change he has long advocated. Both houses of parliament are due to
________on November 27th for the year’s final session. How he shepherds the legislation to make good on his

promise may well prove pivotal not just for gay Australians but for his own career.
(I) Jostle
(II) Breach
(III) Reconvene
(IV) Wrangle
(V) Volition

(a) I-IV (b) II-III (c) I- IV (d) III-V (e) No combination fits

335). The generals permitted free elections in 2015, allowing the National League for Democracy to come to

power. Among its priorities are clamping down on corruption and broadening the tax base. Businessmen complain
that taxmen _______them for bribes, not revenue for the state. Humbler citizens, meanwhile, tend not to pay tax
on their income. Other taxes are routinely_____, too. In order to avoid paying property taxes, some buyers and

sellers, or landlords and tenants, create two contracts: one recording the actual transaction and a dummy to be
submitted for tax purposes, says Lachlan McDonald, an economist at the Renaissance Institute, a think-tank in

Yangon. Most Burmese donate money to Buddhist temples or other religious institutions as a matter of course.

Handing money over to the exchequer is a far less common activity.


(I) Gouge

(II) Drub

(III) Dodged
(IV) Neocene

(V) Zeal

www.ibpsguide.com https://estore.ibpsguide.com Page 99 of 331


(a) I-IV (b) I-III (c) II- IV (d) II-V (e) No combination fits

336). New Zealand is a rainy place, but farmers are also criticized for causing rivers to shrivel and groundwater to
fall in certain overburdened spots. One recent tally suggested that just 2,000 of the thirstiest dairies suck up as

much water as 60m people would—equivalent to the population of London, New York, Tokyo, Los Angeles and
Rio de Janeiro combined. Most is ____on the stony Canterbury region, including the Mackenzie Basin. Earlier this
year locals were forced to rescue fish and eels from ______which formerly constituted the Selwyn river, after
drought and over-exploitation caused long stretches to dry up. Dairies are trying to clean up their act. Farmers
have fenced off thousands of kilometers of rivers to prevent livestock from wading in.

(I) Agony

(II) Hosed
(III) Doused

(IV) Puddles
(V) Middy
(a) I-IV (b) I-III (c) II- IV (d) II-V (e) No combination fits

337). Pakistan has also made a big, controversial push for coal. Just like Delhi, Lahore has flailed feebly at
tackling toxic air. Only when public anger over the stink mounted in recent weeks did the city government reveal

that it had bought six pollution monitors some time ago, but had not yet installed them. As in India, Pakistan’s
state governments have been wary of forcing farmers, a crucial vote bank, to _____ their pyromania. In Nepal,
meanwhile, it is not poverty so much as delays in disbursing budgets, compounded by an anti-corruption agency

that terrifies bureaucrats, that is largely responsible for the failure to pave streets. Ignoring evidence that diesel
exhaust is carcinogenic, and keen to______ the truckers and farmers who use it most, the government has
maintained a steep price differential.

(I) Throw
(II) Begot

(III) Curb

(IV) Placate
(V) Emanate

(a) I-IV (b) II-III (c) III- IV (d) II-V (e) No combination fits

338). Under China’s _____fiscal system, local governments—those at provincial, city and county levels—have

strictly limited revenue-raising powers. Until 2014 they could not borrow or issue bonds without the centre’s

www.ibpsguide.com https://estore.ibpsguide.com Page 100 of 331


permission. Localities depend on fixed shares of certain taxes: 50% of value-added tax, for example, and 40% of

personal income tax. The share is never enough. Local governments receive roughly half of total tax revenues but
are responsible for two-thirds of government spending. In counties the _______is especially large. The result is
that local authorities are permanently dependent on transfers from the centre to keep afloat. These transfers

account for, on average, half of local spending. Such huge flows give the centre a lot of influence over lower tiers.
(I) Hum
(II) Peculiar
(III) Daubing
(IV) Crispy

(V) Discrepancy

(a) I-IV (b) I-III (c) II- IV (d) II-V (e) No combination fits

339). Strong demand, rather than a productivity boom, is driving the _______for workers. In the manufacturing
sector, for example, output per hour worked is just 0.1% higher than a year ago, and has not grown at all in the
past five years. Production and wages have picked up anyway. One reason is a cheapening dollar. On a trade-
weighted basis, the greenback fell by almost 9% between the start of the year and mid-September (it has since
recovered a little). A weaker dollar and a strengthening world economy have ________demand for American
goods. In the first three quarters of the year, goods exports were up by nearly 4% on 2016. At the same time, a

rebound in oil prices from their trough in early 2016 has set off another cycle of investment in the shale industry
(I) Scramble
(II) Wrangle

(III) Figgie
(IV) Krista
(V) Spurred

(a) II-V (b) II-III (c) I- V (d) II-IV (e) No combination fits

340). Rising incomes for lower- and middle earners may help reduce inequality, especially if wage growth for

higher-earners remains_____. A recent analysis by the Economic Policy Institute, a left-leaning think-tank, found
that real wages for the top 1% of earners fell by 3.1% in 2016, and were lower that year than they were in 2007.

As workers’ wages grow, companies’ profit margins may also come under pressure, reducing, somewhat, the

capital income of the rich. Texas Roadhouse, a chain of steak restaurants, recently warned investors that it
expects its wage bill to grow by 7-8% in 2017. After years of imbalance, a shift in economic power towards

www.ibpsguide.com https://estore.ibpsguide.com Page 101 of 331


workers is to be_____, so long as inflation remains low. Yet wage growth also helps determine the fate of

politicians, whether or not they deserve it.


(I) Suspected
(II) Mended

(III) Subdued
(IV) Engrossed
(V) Welcomed
(a) I-IV (b) I-III (c) III- V (d) II-V (e) No combination fits

341). In 1980 almost four-fifths of America’s business income ______to corporations, with the most of the rest

going to sole traders. But as the top rate of income tax fell in the 1980s, firms started to reorganize themselves as
S-corporations and partnerships, another type of pass-through business. Today, 95% of firms are pass through.

They make more profit and employ more people than ______companies do. They also have richer owners. Three-
fifths of pass-through income flows to the top 1% of earners, compared with two-fifths of corporate dividends. This
is partly because high-rolling law firms, hedge funds and consultancies are often partnerships.
(I) Accrued
(II) Conventional
(III) Subterfuge

(IV) Dismantle
(V) Tainted
(a) I-IV (b) I-III (c) I- II (d) II-V (e) No combination fits

342). For social scientists the apparent epidemic of propositioning, pinching, groping and flashing that is gripping
America brings a rare opportunity to observe a new____, around how men behave towards women, being created

in real time. First though, they must figure out the ______of the problem. The most extreme example—rape—is
hard enough to count. Government statistics produced by the Centres for Disease Control and Protection suggest

that one in five women and one in 60 men have been a victim of rape or an attempted rape in their lifetime. On

the other end of the scale, sexual harassment—a charge that rarely carries criminal punishment—is far more
common, and harder still to count. It is also a fairly new phenomenon, in the sense that there was no phrase that

described it until the late-1970s. That coincided with women’s growing importance in the labour force as their

share of jobs rose from 33% to 42% over the preceding two decades
(I) Usurp

(II) Norm

www.ibpsguide.com https://estore.ibpsguide.com Page 102 of 331


(III) Extent

(IV) Normalize
(V) Throw
(a) I-IV (b) I-III (c) II- IV (d) II-V (e) No combination fits

343). The buddy seat on Rick Kimberley’s combine-harvester is a fine _____point from which to observe precision
farming. The combine’s satellite navigation allows farmers to make the most of good weather and to reap in the
dark during peak harvesting periods. It is precise enough to trace the most efficient path to scoop up yellow,
_______corn stalks to within a couple of centimeters. This enables Mr. Kimberley, a 67-year old who farms near

Maxwell, Iowa, to harvest about 100 acres in a 14-hour day, helped only by a big trailer into which he discharges

his corn. Almost by accident, the silver-haired Mr. Kimberley has become a sought-after ambassador for modern
farming methods in China. He travels there regularly to talk about precision farming and other tricks of his trade.

(I) Excess
(II) Vantage
(III) Grope
(IV) Crinkly
(V) Belligerence
(a) I-IV (b) I-III (c) II- IV (d) II-V (e) No combination fits

344). After he was busted in 1974, Jeffrey Edmondson, a small-time dealer of marijuana, cocaine and
amphetamines in Minneapolis, faced a ______bill from the taxman for all his illicit income. He argued that he

should be allowed to deduct $100,000 worth of business expenses, and a court agreed. Enraged, Congress
revised the tax code in the early Reagan years, forbidding tax exemptions for drug traffickers. One unintended
consequence of Mr. Edmondson’s _____persists four decades later: cannabis operations, now legitimate in many

states, are forbidden from the usual business deductions and face crippling tax bills of as much as 70% of
revenue

(I) Daunting

(II) Agnostic
(III) Hum

(IV) Audacity

(V) Jostling
(a) I-IV (b) I-III (c) II- IV (d) II-V (e) No combination fits

www.ibpsguide.com https://estore.ibpsguide.com Page 103 of 331


345). With many Democrats outside Alabama arguing for a more belligerent, left-wing platform, there is a lot in

Mr. Jones’s _______to admire. Indeed, for those looking for national meaning in Alabama’s contest, that is only
the start. If Mr. Jones wins, it will persuade Democrats that they can take control of the Senate next year. With the
polls suggesting that Mr. Jones is running well in the suburbs of Birmingham and with non-whites—a similar

pattern to the recent Democratic sweep in Virginia—it might even convince them they have a _______path to
recovery across the South. That would transform American politics—not least in a region where single-party rule
has bred torpor and corruption. If the admired Mr. Jones loses to the reviled Mr. Moore, on the other hand, such a
recovery might seem unimaginable
(I) Abrasive

(II) Eminent

(III) Frail
(IV) Pragmatism

(V) Shimmering
(a) I-IV (b) I-III (c) II- IV (d) II-V (e) No combination fits

346). Chile’s distribution of income is more unequal than in any other member of the OECD (a grouping of mostly
rich countries), bar Ireland. Taxes and benefits do little to correct this, in contrast to most other OECD nations.
Chileans think their country is crime-ridden, even though its murder rate is among the lowest in Latin America. In

Ms. Bachelet’s second term, which began in 2014, the country entered a______. Chile’s potential GDP growth rate
has dropped over the past two decades from about 5% a year to 3-3.5%, in part because the workforce has aged
and regulation _________ investment. Over the past four years growth has averaged less than 2%, partly because

of a fall in copper prices.


(I) Wails
(II) Dole

(III) Dunk
(IV) Funk

(V) Dissuades

(a) I-IV (b) IV-V (c) II- V (d) II-III (e) No combination fits

347). In the annals of Latin American democracy, Marcelo Odebrecht, a Brazilian construction magnate, will

occupy a place of unique_______. From Mexico to Argentina and many places in between, his Brazilian
construction company bribed presidents, ministers and candidates to win public contracts, setting a example that

www.ibpsguide.com https://estore.ibpsguide.com Page 104 of 331


other firms followed. The damage to the public purses in padded contracts ran to over $3bn. The intangible cost

to the credibility and prestige of democratic politics in Latin America is incalculable.


(I) Enmity
(II) Infamy

(III) Nefarious
(IV) Begot
(V) Necessary
(a) I-IV (b) I-III (c) II- V (d) II-III (e) No combination fits

348). Overall, Latin America has performed reasonably well on turnout. Excluding blank or _______votes, the

regional figure in presidential elections has held steady at just over two-thirds throughout the past two decades,
close to the global average for established democracies. (Latino baró metro’s respondents began to express

record levels of indifference to democracy only last year, making it too early to tell whether this trend
_______declining participation.) However, this stability masks wide variations. Since 1995, countries in the
southern half of South America have boasted turnout rates in the mid-70s, with Uruguay leading the wayat87%.
Colombia has managed just 43%
(I) Agnostic
(II) Spoiled

(III) Foreshadows
(IV) Preponderance
(V) Procrastinate

(a) I-IV (b) I-III (c) II- III (d) II-V (e) No combination fits

349). If Mr. Mnangagwa succeeds in taking back the reins of government, his first task will be to consolidate

power within Zanu-PF. Whether Mr. Mugabe formally hands over or is kept on as a kind of ceremonial president is
barely relevant, though it would be neater if the old man were ushered into as dignified a retirement as soon as is

feasible in these ugly, humiliating circumstances. Mr. Mnangagwa’s main concern will be to ensure that Mrs.

Mugabe and herG40 are dismissed. Many have already been locked up. ________who will probably sink with her
include Saviour Kasuku were, who enacted the racist indigenization law; Ignatius Chombo, the finance minister;

Jonathan Moyo, a serial plotter and former regime_______; Patrick Zhuwao, a nephew of Mr. Mugabe; and the

head of the police, Augustine Chihuri.


(I) Gimps

(II) Bushy

www.ibpsguide.com https://estore.ibpsguide.com Page 105 of 331


(III) Bigwigs

(IV) Mouthpiece
(V) Toll
(a) I-IV (b) I-III (c) III- IV (d) II-V (e) No combination fits

350). Without the fig leaf of Mr. Hariri as prime minister, Hizbullah’s influence on Lebanon would be exposed—or
so hoped the Saudis as they beat the drums of war. Its officials blamed Iran and Hizbullah for a missile fired at
Riyadh from Yemen on November 4th. It was_______, but the Saudi government nevertheless called it an “act of
war”. On November 9th it told Saudis to leave Lebanon at once, raising concerns of an imminent ______into

chaos, or even an attack. The Lebanese held their breath. But if Saudi Arabia’s plan was to lure Israel and

America, which lists Hizbullah as a terrorist group, into a war in Lebanon, it miscalculated.
(I) Abrased

(II) Intercepted
(III) Descent
(IV) Throw
(V) Done
(a) I-IV (b) II-III (c) III- IV (d) II-V (e) No combination fits

351). Over the weekend members of parliament from the CDU, FDP and Greens published a joint appeal to take
up Mr. Macron’s recent suggestions for the future of Europe, including efforts to improve asylum policy and to
create a “more crisis-proof, stable and democratic” euro zone. However, agreeing on what is necessary to

achieve this will be tricky, particularly if, as still looks_____, the FDP takes the finance ministry, and its leadership
keeps the new government committed to balanced budgets. Moving closer to France also carries risks. Eastern
European nations, already _______of the EU’s criticism of their domestic politics and German demands to do

more for refugees, may feel excluded by a strengthened Paris-Berlin axis. Keeping the eastern neighbors on
board will be essential.

(I) Daunt

(II) Plausible
(III) Pleasing

(IV) Wary

(V) Biggie
(a) I-IV (b) I-III (c) II- IV (d) II-V (e) No combination fits

www.ibpsguide.com https://estore.ibpsguide.com Page 106 of 331


352). Ms. Le Pen herself is_____. France’s parliament this month removed her immunity from prosecution, letting

investigative judges question her for tweeting graphic images of a murdered American journalist, James Foley.
(Sharing violent images is illegal in France.) Earlier this year the European Parliament, where she also sat, lifted
her immunity. Her 27-year-old niece, Marion Maré- chal-Le Pen (who is currently taking a break from politics),

could eventually step in to reinvigorate the party, suggests Mr. Bouvet. The FN will have to respond somehow, as
it will soon face another rival. Next month the centre-right Les Républicains elects a new leader—almost certainly
Laurent Wauquiez, from its right______. His focus on issues such as immigration could win back voters who had
drifted to the FN.
(I) Abrasive

(II) Flailing

(III) Beleaguered
(IV) Fringe

(V) Dingy
(a) I-IV (b) I-III (c) III- IV (d) II-V (e) No combination fits

353). Hundreds of Western-trained Cypriot lawyers and accountants earn a living by handling the affairs of
Russian and Ukrainian offshore companies. The relationship has ________since the island became a base for
proto-capitalists from the former Soviet Union in the 1990s, thanks to a communist-era treaty on removing double

taxation. A relaxed attitude to transactions involving cash-filled suitcases also helped. Nicosia, the island’s capital,
and Limassol, its largest port, are these days home to an estimated 50,000-60,000 citizens of the former Soviet
Union. Limassol’s once-seedy waterfront boasts smart blocks of flats, shopping malls and a _____marina for the

billionaires’ super yachts. The wealthiest Russian and Ukrainian families flit between Cyprus, London and Paris.
(I) Dole
(II) Shamming

(III) Flourished
(IV) Glanced

(V) Gleaming

(a) I-IV (b) III-IV (c) II- IV (d) III-V (e) No combination fits

354). The bigger issue is whether Mr. Erdogan, who relishes power as much as he fears ______it, will allow a free

election to take place. The referendum on the new constitution was held in a climate of repression and
censorship, and prompted claims of extensive vote-rigging. The state of emergency, which allows Mr. Erdogan to

www.ibpsguide.com https://estore.ibpsguide.com Page 107 of 331


rule by decree, looks set to last indefinitely. Some of Mrs. Aksener’s associates acknowledge the risk of new

___campaigns and even arrests as the election draws closer. She says that would betray weakness.
(I) Marti
(II) Relinquishing

(III) Smear
(IV) Wrangle
(V) Tickling
(a) I-IV (b) I-III (c) II- IV (d) II-III (e) No combination fits

355). One explanation for the delays is ill preparedness and the slow process since the referendum of learning

precisely what leaving such a complex organization entails. But another is excessive rigidity and a premature
closing down of options. This has often made the talks between David Davis, the Brexit secretary, and Michel

Barnier, the EU’s chief Brexit negotiator, needlessly______. The EU 27 are by no means blameless in this.
Pretending that Brexit was a problem for Britain alone and that they had more pressing matters to attend to has
not helped. Insisting that the talks cannot move to a second phase until the divorce terms, including a large exit
bill for Britain, are settled was always likely to lead to____. Failure to progress to the second phase in December
would be a blow to the EU as well as to Britain
(I) Abrased

(II) Confrontational
(III) Normal
(IV) Deadlock

(V) End
(a) I-IV (b) I-III (c) II- IV (d) II-V (e) No combination fits

356). Northern Ireland is ______along without its government. Most of its functions are being
carried out by local civil servants under the supervision of Westminster. The economy has done relatively well,

with the rate of output growth catching up with the rest of the UK and wages rising at a decent pace, in part

because many workers are on the minimum wage, which went up in April. It has also been ______by the economy
of the Irish Republic, which grew by 5% last year. About a third of Northern Ireland’s exports go to the south. But

that also makes the province vulnerable to Brexit—which is being negotiated in the absence of its politicians.

(I) Abrasive
(II) Ticking

(III) Buoyed

www.ibpsguide.com https://estore.ibpsguide.com Page 108 of 331


(IV) Pick up

(V) Down
(a) I-IV (b) I-III (c) II- IV (d) II-V (e) No combination fits

357). Britain considers negotiating prisoner releases ______to paying ransom for hostages. Other governments
have fewer scruples. Last year the Obama administration secured the release of Jason Rezaian, a correspondent
for the Washington Post in Tehran, who had been sentenced for ______and spent18 months in detention. The
same day, America delivered $400m in cash which had been frozen in Iranian accounts. After Iran’s deal with six
world powers to limit its nuclear programme in return for the lifting of sanctions took effect in January 2016, Britain

released all but £70m ($90m) of the £728m in Iranian accounts it had frozen. When Mr. Johnson heads to Iran

later this year to plead for Ms. Zaghari-Ratcliffe’s release, the regime might be hoping that he takes the
outstanding funds with him.

(I) Antigen
(II) Groping
(III) Sinuous
(IV) Tantamount
(V) Espionage
(a) I-IV (b) I-III (c) II- V (d) IV-V (e) No combination fits

358). Since the referendum two of Britain’s three main parties have lost their leaders, Theresa May has fought a
_________election, the cabinet has been ________by infighting and Jeremy Corbyn, Labour’s hard-left leader, has

become prime-minister-in waiting. The less visible result is a constitutional revolution. Before the referendum,
Parliament was sovereign (though, as Brexiteers rightly pointed out, the EU kept encroaching on that
sovereignty). Now, for the first time in Britain’s long parliamentary history, most MPs feel obliged to vote for a

policy that they oppose—in other words, to give in to a populist revolution. Three quarters of MPs voted for
Remain.

(I) Volition

(II) Posse
(III) Botched

(IV) Magnate

(V) Paralyzed
(a) I-IV (b) I-III (c) II- IV (d) III-V (e) No combination fits

www.ibpsguide.com https://estore.ibpsguide.com Page 109 of 331


359). The Williams Institute, a think-tank in Los Angeles, recently came up with an estimate of 1.4m Americans—

0.6% of those aged 16-65. Moreover, young people say that gender matters less than it used to, which sits oddly
with the spreading belief that gender dysphoria can be severe enough to justify the ______and risks of
transitioning. But transgender identities raise more general questions, and not only for those cultural

conservatives, who regard them as ________the natural, perhaps God given, order but also for everyone else.
(I) Botch
(II) Upheaval
(III) Transgressing
(IV) Impeding

(V) Pegging

(a) I-IV (b) II-III (c) II- IV (d) III-V (e) No combination fits

360). Jeffrey Immelt, Mr. Welch’s _______successor, violated all three rules. To be fair, he did steer GE through a
sharp downturn in aviation following the September11th 2001 terrorist attacks and unwound its risky financial arm
after the global financial crisis. But on his watch GE’s core power business deteriorated to the point where the firm
now cannot generate enough cash to pay its promised dividend. His reliance on multiple accounting standards
and ______long-term service contracts led financial analysts to complain about a lack of openness.
(I) Bending

(II) Amiable
(III) Rotatable
(IV) Hopping

(V) Opaque
(a) I-IV (b) I-III (c) II- IV (d) II-V (e) No combination fits

ANSWERS:
331). Correct Answer is: (c)

Explanation: The combination of II-IV i.e. “clout, sycophantic” fits into the blanks respectively. Other combinations

are unfit for the given blanks.


Clout – Influence of power

Sycophantic – Behavior in order to gain advantage

332). Correct Answer is: (d)


Explanation: The combination of IV-V i.e. “Suppress, Switch” fits into the blanks respectively. Other combinations

are unfit for the given blanks.

www.ibpsguide.com https://estore.ibpsguide.com Page 110 of 331


333). Correct Answer is: (c)

Explanation: The combination of I-IV i.e. “Complacent, Gargantuan” fits into the blanks respectively. Other
combinations are unfit for the given blanks.
Complacent – Satisfied

Gargantuan - Huge
334). Correct Answer is: (b)
Explanation: The combination of II-III i.e. “Breach, Reconvene” fits into the blanks respectively. Other
combinations are unfit for the given blanks.
Breach – breaking law

Reconvene – resume

335). Correct Answer is: (b)


Explanation: The combination of I-III i.e. “Gouge, Dodged” fits into the blanks respectively. Other combinations

are unfit for the given blanks.


Gouge- Excavate
Dodged- evade
336). Correct Answer is: (c)
Explanation: The combination of II-IV i.e. “Hosed, Puddles” fits into the blanks respectively. Other combinations
are unfit for the given blanks.

Hosed- Water or spray with hose


Puddles- pool of rainwater
337). Correct Answer is: (c)

Explanation: The combination of III-IV i.e. “Curb, Placate” fits into the blanks respectively. Other combinations are
unfit for the given blanks.
Curb- limit/cut

Placate – make someone less angry


338). Correct Answer is: (d)

Explanation: The combination of II-V i.e. “Peculiar, Discrepancy” fits into the blanks respectively. Other

combinations are unfit for the given blanks.


Peculiar – Strange

Discrepancy – difference/inconsistent

339). Correct Answer is: (c)


Explanation: The combination of I-V i.e. “Scramble, Spurred” fits into the blanks respectively. Other combinations

are unfit for the given blanks.

www.ibpsguide.com https://estore.ibpsguide.com Page 111 of 331


Scramble – Rush/climb

Spurred – Propel
340). Correct Answer is: (c)
Explanation: The combination of III-V i.e. “Subdued, Welcomed” fits into the blanks respectively. Other

combinations are unfit for the given blanks.


341). Correct Answer is: (c)
Explanation: The combination of I-II i.e. “Accrued, Conventional” fits into the blanks respectively. Other
combinations are unfit for the given blanks.
Accrued – result

Conventional – traditional/usual

342). Correct Answer is: (e)


Explanation: The combination of II-III i.e. “Norm, Extent” fits into the blanks respectively. Other combinations are

unfit for the given blanks.


343). Correct Answer is: (c)
Explanation: The combination of II-IV i.e. “Vantage, Crinkly” fits into the blanks respectively. Other combinations
are unfit for the given blanks.
Vantage – viewpoint
Crinkly – Wrinkles

344). Correct Answer is: (a)


Explanation: The combination of I-IV i.e. “Daunting, Audacity” fits into the blanks respectively. Other combinations
are unfit for the given blanks.

Daunting – Intimidating/Demoralize
Audacity – Boldness
345). Correct Answer is: (e)

Explanation: The combination of IV-V i.e. “Pragmatism, Shimmering” fits into the blanks respectively. Other
combinations are unfit for the given blanks.

Pragmatism – practically

Shimmering – flicker/twinkle
346). Correct Answer is: (b)

Explanation: The combination of IV-V i.e. “Funk, Dissuades” fits into the blanks respectively. Other combinations

are unfit for the given blanks.


Funk – Panic

Dissuades – Discourages

www.ibpsguide.com https://estore.ibpsguide.com Page 112 of 331


347). Correct Answer is: (d)

Explanation: The combination of II-III i.e. “Infamy, Nefarious” fits into the blanks respectively. Other combinations
are unfit for the given blanks.
Infamy- Scandal/Disrepute

Nefarious – wicked or criminal


348). Correct Answer is: (c)
Explanation: The combination of II-III i.e. “Spoiled, foreshadows” fits into the blanks respectively. Other
combinations are unfit for the given blanks.
349). Correct Answer is: (c)

Explanation: The combination of III-IV i.e. “Bigwigs, Mouthpiece” fits into the blanks respectively. Other

combinations are unfit for the given blanks.


350). Correct Answer is: (b)

Explanation: The combination of II-III i.e. “Intercepted, Descent” fits into the blanks respectively. Other
combinations are unfit for the given blanks.
351). Correct Answer is: (c)
Explanation: The combination of II-IV i.e. “Plausible, Wary” fits into the blanks respectively. Other combinations
are unfit for the given blanks.
352). Correct Answer is: (c)

Explanation: The combination of III-V i.e. “Beleaguered, Fringe” fits into the blanks respectively. Other
combinations are unfit for the given blanks.
353). Correct Answer is: (d)

Explanation: The combination of III-V i.e. “flourished, gleaming” fits into the blanks respectively. Other
combinations are unfit for the given blanks.
354). Correct Answer is: (d)

Explanation: The combination of II-III i.e. “Relinquishing, Smear” fits into the blanks respectively. Other
combinations are unfit for the given blanks.

355). Correct Answer is: (c)

Explanation: The combination of II-IV i.e. “Confrontational, Deadlock” fits into the blanks respectively. Other
combinations are unfit for the given blanks.

356). Correct Answer is: (e)

Explanation: The combination of II-III i.e. “Ticking, Buoyed” fits into the blanks respectively. Other combinations
are unfit for the given blanks.

357). Correct Answer is: (d)

www.ibpsguide.com https://estore.ibpsguide.com Page 113 of 331


Explanation: The combination of IV-V i.e. “Tantamount, Espionage” fits into the blanks respectively. Other

combinations are unfit for the given blanks.


358). Correct Answer is: (d)
Explanation: The combination of III-V i.e. “Botched, Paralyzed” fits into the blanks respectively. Other

combinations are unfit for the given blanks.


359). Correct Answer is: (b)
Explanation: The combination of II-III i.e. “Upheaval, Transgressing” fits into the blanks respectively. Other
combinations are unfit for the given blanks.
360). Correct Answer is: (d)

Explanation: The combination of II-V i.e. “Amiable, Opaque” fits into the blanks respectively. Other combinations

are unfit for the given blanks.

Fill in the blanks Type - 5

Directions (361-390): In each of the questions given below a sentence is given with one blank. Below each
sentence FOUR words are given out of which two can fit the sentence. Five options are given with various
combinations of these words. You have to choose the combination with the correct set of words which can fit in
the given sentence.

361). The government’s announcement that the remains of 39 Indian workers, kidnapped four years ago by the
Islamic State, have been found near Mosul in Iraq, has brought a ________ closure to the episode.

A) Heartbreaking.
B) Excruciating
C) Painful

D) Laborious.
(a) A-C (b) B-C (c) C-D (d) A-B (e) B-D

362). Electoral dirty tricks are a regrettably time-honored tradition. That Donald Trump’s presidential campaign
may have used underhand means to _____voters is thus not the central issue

A) Lured

B) Influence.
C) Target.

D) Divert.

www.ibpsguide.com https://estore.ibpsguide.com Page 114 of 331


(a) A-C (b) B-C (c) C-D (d) A-B (e) B-D

363). Electronic voting machines represent such an opportunity and danger. But because too much capital is
invested in selling and replicating these systems, the opportunities and advantages are currently _________ more

than the dangers.


A) Round up
B) Turn off
C) Drummed up
D) Round down.

(a) B-D (b) B-C (c) C-D (d) A-B (e) A-C

364). It was recently _______ that an analytics firm, Cambridge Analytica, wrongfully gained access to the data of

50 million people swept up from Facebook via a 'personality test app', and used the data for targeted political
campaigning.
A) Obtained
B) Unveiled
C) Revealed.
D) Discussed.

(a) B-D (b) B-C (c) C-D (d) A-B (e) A-C

365). The Whistle Blowers Protection Act, 2011 _______ the complete framework to investigate alleged cases of

wrongdoing.
A) Set out.
B) Laid down

C) Lay aside
D) Lays down.

(a) B-D (b) B-C (c) C-D (d) A-B (e) A-C

366). The Pakistan Army ___________ mortar shelling & small arms firing across LOC in Balakote sector of

Poonch district in J & K late on Tuesday night , the police officials said on Wednesday.

A) Encountered into
B) Descended to

C) Ascended to

www.ibpsguide.com https://estore.ibpsguide.com Page 115 of 331


D) Resorted to.

(a) B-D (b) B-C (c) C-D (d) A-B (e) A-C

367). The Centre, in turn, urged a Five Judge Constitution bench headed by Chief Justice Dipak Mishra to allow

Ajay Bhushan Pandey, CEO of the UIDAI, to make a power point presentation to the court ______ the
apprehensions about the Aadhaar scheme.
A) To kindle
B) To stir up
C) To allay

D) To mollify.

(a) B-D (b) B-C (c) C-D (d) A-B (e) A-C

368). IT & Law Minister Ravi Shankar Prasad cautioned the social media giant & its CEO Mark Zuckerberg of
___________ under IT Act in case of any data breach came to light.
A) Consequences
B) Backlashing
C) Repercussions
D) Causes

(a) B-D (b) B-C (c) C-D (d) A-B (e) A-C

369). The PNB fraud has ______ the debate on bank privatization, often considered a solution for the poor

management in public sector banks


A) Rekindled
B) Bought back

C) Began
D) Worsened

(a) B-D (b) B-C (c) C-D (d) A-B (e) A-C

370). It is true that the RBI does not have all the powers over PSU banks that it has over private sector banks,

such as the power to ______ a banking license, merge a bank, shut down a bank, or penalize the board of

directors.
A) Enact

B) Invalidate

www.ibpsguide.com https://estore.ibpsguide.com Page 116 of 331


C) Authorize

D) Revoke.
(a) B-D (b) B-C (c) C-D (d) A-B (e) A-C

371). An Islamic State suicide bomber _____ on the road to a Shiite shrine in Afghanistan’s capital on
Wednesday, killing at least 29 people as Afghans celebrated the Persian new year.
A) Fall
B) Strike
C) Inflicted

D) Struck

(a) B-D (b) B-C (c) C-D (d) A-B (e) A-C

372). The Persian New Year, known in Afghanistan as Nowruz, is a national holiday, & the country’s minority
Shiites typically celebrate by _________ shrines.
A) Visiting
B) Worshiping
C) Meeting
D) Exploring.

(a) B-D (b) B-C (c) C-D (d) A-B (e) A-C

373). The Defense ____ contended that Farooq alias Farooq Tikal, is an innovation of CBI, as mentioned in CBI

charge sheet.
A) Council
B) Counsel

C) Advisor
D) Committee.

(a) B-D (b) B-C (c) C-D (d) A-B (e) A-C

374). It also said that the reasons recorded must be ______ by the Magistrate for permitting further detention.

A) Disregarded

B) Skimmed
C) Probed

D) Scrutinized.

www.ibpsguide.com https://estore.ibpsguide.com Page 117 of 331


(a) B-D (b) B-C (c) C-D (d) A-B (e) A-C

375). The video of a Maharashtra farmer destroying cauliflower crop on his land after he got a _____ amount for
the products has gone viral on social media.

A) Puny
B) Conspicuous
C) Paltry
D) Ample.
(a) B-D (b) B-C (c) C-D (d) A-B (e) A-C

376). ______ from the Rs. 13,000 crore scam in the PNB, the banking sector is vigilantly going after other problem
areas such as NPAs & bad loans to regain the lost trust of public.

A) Stinging
B) Smarting
C) Soothing
D) Dimming
(a) B-D (b) B-C (c) C-D (d) A-B (e) A-C

377). At a time when there is growing uncertainty about the health of many Western banks, the latest version of
the Financial Stability Report just released by the RBI is _____.
A) Repressing

B) Discouraging
C) Reassuring
D) Soothing.

(a) B-D (b) B-C (c) C-D (d) A-B (e) A-C

378). In an increasingly-open economy such as India’s there is a need to focus on the efficiency _____ aspect of

the tax system more than ever before.

A) Angle

B) Expect
C) Except

D) Aspect

www.ibpsguide.com https://estore.ibpsguide.com Page 118 of 331


(a) B-D (b) B-C (c) C-D (d) A-D (e) A-C

379). Indian banks remain ______ though capital adequacy ratios have fallen & NPAs have increased.
A) Fragile

B) Uncertain
C) Booming
D) Robust.
(a) B-D (b) B-C (c) C-D (d) A-D (e) A-C

380). Attorney General Vahanvati’s reported _______ that the govt should cancel the allocation granted in the

past for coal blocks that have not been operationalized since is a sensible one.
A) Declaration

B) Order
C) Suggestion
D) Recommendation
(a) B-D (b) B-C (c) C-D (d) A-D (e) A-B

381) On 10 Dec 2013, Uruguay became the first country in the world to fully ______ the sale, production &

distribution of marijuana.
A) Outlaw
B) Legalize

C) Illegalize
D) Validate.
(a) B-D (b) B-C (c) C-D (d) A-D (e) A-B

382). The Uttarakhand assembly passed the new Lokayukta Bill _______ on the Centre’s Lokpal recently.

A) Preceded on

B) Modelled on
C) Comply on

D) Differentiating on.

(a) B-D (b) B-C (c) C-D (d) A-D (e) A-B

www.ibpsguide.com https://estore.ibpsguide.com Page 119 of 331


383). India & the Maldives have agreed to strengthen cooperation to enhance maritime safety and security in the

Indian Ocean Region through joint _______ aerial & maritime surveillance.
A) Neglecting
B) Parading

C) Patrolling
D) Safeguarding
(a) B-D (b) B-C (c) C-D (d) A-D (e) A-B

384). Cyrus Mistry ______ stiff competition from five other writers to become the fourth winner of the $ 50,000

DSC prize for South Asian Literature for his book Chronicles of a corpse bearer.

A) Beat around the bush


B) Beat the clock

C) Beat off
D) Bear out
(a) B-D (b) B-C (c) A-B (d) A-D (e) C-D

385). For a certain age concessions admissible to various categories, please _______ the instructions regarding
concessions & relaxations.

A) Check in
B) Go through
C) Consider

D) Look through
(a) B-D (b) B-C (c) A-B (d) A-D (e) C-D

386). The Telangana C.M. would like a third front but it can only be a post poll ___ ________.
A) Collation

B) Coalition

C) Alliance
D) Committee.

(a) B-D (b) B-C (c) A-B (d) A-C (e) C-D

387). The National Intelligence Council’s Global Trends Report , published every five years, paints a future in

which US power will greatly diminish but no other individual state rises to ________ it.

www.ibpsguide.com https://estore.ibpsguide.com Page 120 of 331


A) Legalize

B) Authorize
C) Supplant
D) Supersede.

(a) B-D (b) B-C (c) A-B (d) A-C (e) C-D

388). Gandhi is remembered in South Africa now as an important leader of the second rank who ______ the
African movement.

A) Instigated

B) Instilled
C) Inspired

D) Initiated.
(a) B-D (b) B-C (c) A-B (d) A-C (e) C-D

389). When Telangana Chief Minister K. Chandrashekar Rao _______a non-BJP, non-Congress ‘federal’ front, he
was already defining the alliance negatively, in terms of its opponents rather than as a coming together of like-
minded parties.

A) Mooted
B) Introduced
C) Debated

D) Negotiated
(a) B-D (b) B-C (c) A-B (d) A-C (e) C-D

390). The German automaker Volkswagen, has been in the hot seat for installing software that covered up diesel
_____ during testing by the Environmental Protection Agency of the U.S.

A) Immision

B) Infusion
C) Emission

D) Transmission

(a) B-D (b) B-C (c) A-B (d) A-C (e) C-D

ANSWERS:

www.ibpsguide.com https://estore.ibpsguide.com Page 121 of 331


361). Correct Answer is: (a).

Explanation –Heartbreaking & painful best fits here. As the given sentence talks about the deaths of 39 Indian
workers which is painful. Laborious means work some, requiring much physical effort. Excruciating means
causing great pain it can’t fit there because article ‘a’ is used in the sentence.

362). Correct Answer is: (b)


Explanation – Both the words ‘Influence & target’ best suits here as it means to aim for as an audience or voters.

Lured will not fit here because we use first form (infinitive form) of the verb after ‘To’.
363). Correct Answer is: (e)
Explanation – Here it says that because too much capital invested, opportunities are increasing. So correct option

will be drummed up & round up which means to generate, encourage, to beget, to recreate.

364). Correct Answer is: (b)


Explanation – The given context is about disclosing the details of analytics firm. So, options ‘Unveiled & revealed’
will best fits here which means to open up, to expose, to disclose.

365). Correct Answer is: (d)


Explanation – The sentence talks about the Act that outlined the guidelines to investigate the cases. So, here
options ‘laid down & set out ‘are correct which means to promulgate the law, statutory prescribed by the law, to
formulate, establish.
Meaning of Lay aside is to set something aside, give up, abandon. And lays down means set, establish and can
be fitted here but it is in the present form and sentence talks about the framework which is already formulated.
366). Correct Answer is: (a)
Explanation – The correct options are ‘resorted to & descended to ‘which means to make an attack, to stoop,

perpetrate, transgress, resonate. Rest option doesn’t suits here.


367). Correct Answer is: (c)
Explanation – Only options ‘to allay & to mollify’ fits here which means to pacify, calm, relieve, to lessen, to
mitigate. As sentence talks about making the power point presentation to reduce the apprehensions about the
Aadhaar scheme.

Other options are antonyms of allay.

368). Correct Answer is: (e)


Explanation – The sentence talks about the effects of data breach. So correct options will be ‘Repercussions &

Consequences’ which means result, outcome, by product, aftermaths. Causes is an antonym of consequences.
Thoughblack lash has same meaning as that of repercussions but it is in “ing” form which is not grammatically
correct.

369). Correct Answer is: (d).

www.ibpsguide.com https://estore.ibpsguide.com Page 122 of 331


Explanation – The correct option is ‘D’. As sentence throws light on how PNB fraud has bought back the debate

on bank privatization, which is considered as a solution for increase in frauds, poor management in PSB.
370). Correct Answer is: (a).
Explanation – It says RBI doesn’t have all the powers over PSU Banks as it has over pvt banks such as to cancel

the banking license. So correct word will be ‘Invalidate & Revoke’ which means to cancel, to withdraw, to nullify.
Whereas other words means to approve, authorize.

371). Correct Answer is: (c)


Explanation – Options ‘inflicted & struck’ best fits here which means to blast.
372). Correct Answer is: (d)

Explanation – Only options ‘visiting & worshiping’ best fits here.So answer is option D.

373). Correct Answer is: (b)


Explanation - Counsel means a barrister, legal advisor, lawyer, legal practitioner
Whereas Council means a conference, conclave, committee, assembly.
So correct answer is Counsel & advisor which have same meaning and which best fits here.
374). Correct Answer is: (c)
Explanation – The sentence talking about the inspection of all the reasons/evidences so correct words will be
‘Scrutinized & probed’ which means to examine, study, research, check, analyze. Disregard & skim means to

ignore, neglect, overlook.

375). Correct Answer is: (e)


Explanation - Options A & C best fits here which means small, meagre, pitiful. Whereas meanings of other words
are noteworthy, full.

376). Correct Answer is: (d).


Explanation – The words ‘stinging & smarting’ means to feel shame, to suffer, become very upset by something
that has happened. So, Ans is option D. whereas other words means peaceful, comfort, diminishing.

377). Correct Answer is: (c).


Explanation –As the first part of the sentence talks about the uncertainty , risks in banking sector (negative part)

the second part of the sentence must positive as sentence starts with ‘AT A TIME’ so the correct words will be

‘reassuring & soothing’ which means to bring back the hope, inspirit, encourage. Whereas the other words means
discouraging, bring down, dissuade.

378). Correct Answer is: (d)

Explanation – The correct words are angle & aspect which means feature, facet, side, attribute. Whereas the
meaning of Expect – anticipate, predict, suppose, and believe.

And that of Except – excluding, not counting, besides, barring, other than.

www.ibpsguide.com https://estore.ibpsguide.com Page 123 of 331


379). Correct Answer is: (c).

Explanation – The correct option will be ‘C’. As the sentence says even though the NPAs & adequacy ratios
increased Indian bank remain strong or powerful.

380). Correct Answer is: (c).

Explanation – The correct words that best fits here are ‘suggestion & recommendation’ which means advice,
counsel, and proposal whereas rest words means statement, order, demand which doesn’t suits here.

381). Correct Answer is: (a).


Explanation – The correct answer is option A.
382). Correct Answer is: (b)

Explanation – The correct option is ‘B’. So the correct words will be ‘modelled on & comply on’ which means to

copy, abide by, to be designed in a way that copies another system.


383). Correct Answer is: (c)
Explanation – The correct words are ‘Patrolling & Safeguarding’ which means monitoring, guarding.

384). Correct Answer is: (e)


Explanation – The correct phrase is ‘bear out & beat off’ which means to defeat someone, to win.
Meaning of other phrases – beat around the bush – discuss a matter after coming to the point be ineffectual &
waste of time
Beat the clock– to complete the task quickly.

385). Correct Answer is: (a)


Explanation – The correct words that best fits here are ‘go through & look through’ which means to undergo,
search , turn inside out.

386). Correct Answer is: (b)


Explanation – The correct answer is option B. Coalition means alliance, Union, partnership. Whereas Collation
means a collection, a gathering

387). Correct Answer is: (e).


Explanation – The correct option is E which means substitute for, displace, replace.

388). Correct Answer is: (e).

Explanation – Option E is the correct answer.


389). Correct Answer is: C

Explanation – The correct words are mooted & introduced which means to bring up, put forward.

390). Correct Answer is: (e)


Explanation- The correct words which will fill the blanks are Emission and transmission which means out sending,

to send out. Whereas the meaning of immision is the act of immitting, injection, to put in, insert.

www.ibpsguide.com https://estore.ibpsguide.com Page 124 of 331


Fill in the blanks Type - 6

Directions (391-420): In each of the following questions a short passage is given with one of the lines in the

passage missing and represented by a blank. Select the best out of the five answer choices given, to complete
the passage.

391). The NDA government’s scheme to provide health cover of ₹5 lakh per year to 10 crore poor and vulnerable
families through the Ayushman Bharat-National Health Protection Mission has taken a step forward with the

Union Cabinet approving the modalities of its implementation. Considering the small window, just over a year,

available before the term of the present government ends, _____________________________________________.


(a) It is impossible to do provide such a large manpower.
(b) Will Modi government succeed in doing that?
(c) Urgent action is needed to roll out such an ambitious scheme.
(d) Opposition parties will get a chance to prove Modi government wrong.
(e) None of these.

392). A Bench of Justices Sandi Khanna and Changer Shekhar reinstated all 20 MLAs, holding that the opinion of

the Election Commission of India to the President to disqualify the legislators was________________
(a) “Vitiated and bad in law for failure to comply with the principles of natural justice”.
(b)“The government had claimed that the appointments would facilitate a smooth functioning”

(c) “However, a constitutional clause prohibits legislators or parliamentarians from holding any position with
monetary or other benefits”
(d) Known as ‘office of profit’, the clause is aimed at reducing conflict of interest situations for public

representatives.
(e) None of these.

393). The States, which have a statutory responsibility for provision of health care, have to act quickly and form
dedicated agencies to run the scheme___________________________

(a) This could be on the lines of legislation governing the rights to food and information.

(b) It will also enable much-needed regulatory control over pricing of hospital-based treatments
(c) The government should take the bold step of including these groups universally; the financial risk can be borne

by the taxpayer.

www.ibpsguide.com https://estore.ibpsguide.com Page 125 of 331


(d) Since the NHPM (National Health Protection Mission) represents the foundation for a universal health

coverage system that should eventually cover all Indians, it needs to be given a sound legal basis, ideally through
a separate law.

(e) None of the above

394). The MLAs had told the court that EC’s order disqualifying them for allegedly holding office of profit was
passed in “complete violation of natural justice”____________________
(a) The Delhi government, led by Chief Minister Arvind Kejriwal, appointed the parliamentary secretaries attached
to government ministries after coming to power in March 2015.
(b) As they were not given the opportunity to explain their stand before the poll panel.

(c) Since ‘Poll panel’s advice to President bad in law as it had failed to hear the lawmakers’
(d) The government had claimed that the appointments would facilitate a smooth functioning.
(e) None of the above.

395). Union Minister of State for Electronics and Information Technology K.J. Alphons has said that the biometrics
data of Aadhaar holders in the country will ____________________.
(a) witness panel discussions and interactions featuring academics, and technocrats.
(b) have no problem while filling up 10 pages of data for getting the American visa.

(c) remain safe and protected using the latest technology.


(d) affect the health of the citizens
(e) None of these.

396). Universal health coverage is defined by the WHO as a state when “all people obtain the health services they
need without suffering financial hardship when paying for them”.___________________

(a) Affordable housing, planned urban development, pollution control and road safety are some aspects vital for
reducing the public health burden.

(b) It is a challenging task to make all this a reality, and the government will have to work hard to put it in place.

(c) With its endorsement of the Sustainable Development Goals for 2030, India will have to constantly raise its
ambition during the dozen years to the deadline.
(d) This underscores the importance of raising not just core budgetary spending every year, but paying attention
to social determinants of health.
(e) None of the above

www.ibpsguide.com https://estore.ibpsguide.com Page 126 of 331


397). As part of its continuing campaign to prevent China from stealing American intellectual

property,_______________.
(a) President Donald Trump’s administration is considering restrictions on the number of Chinese citizens enrolled

at US colleges and universities.


(b) President Trump’s national security strategy calls for the government to consider restrictions on “foreign STEM
students from designated countries to ensure that intellectual property is not transferred to our competitors.”
(c) Singling them out would be both discriminatory and dangerous, potentially fuelling suspicion of law-abiding

Americans of Chinese descent.


(d) The gains to the US economy aren’t limited to Silicon Valley

(e) None of these.

398). The most sensible strategy to protect the country’s intellectual property isn’t_________________

(a) only un-American, but also self-defeating.


(b) Singling them out would be both discriminatory and dangerous, potentially fuelling suspicion of law-abiding
Americans of Chinese descent.
(c) true to form, wants to cut the program instead.
(d) to keep talented foreign students out, but to encourage them to stay in the US and put their knowledge to
use—by joining the workforce or starting a business.

(e) None of the above.

399). R. Chandrashekhar, president of Nasscom; and Arun Kumar, Chairman and CEO of KPMG India; spoke.
The digital summit, _______________________________ had witnessed panel discussions and interactions
featuring academics, and technocrats.

(a) Kris Gopalakrishnan, co-founder of Infosys and member of the State’s High-Power IT Committee, said all the
deliberations of the two-day summit would be compiled and submitted to both the Central and the State

governments for follow-up action.

(b) Over 2,100 delegates attended the two-day event.


(c) Leader of the Opposition Ramesh Chennithala said the State offered the most conducive investment
opportunity and the entire political leadership remained united to ensure it.
(d) which was inaugurated by Chief Minister Pinarayi Vijayan on Thursday,
(e) None of above.

www.ibpsguide.com https://estore.ibpsguide.com Page 127 of 331


400). According to its headline estimates, an annual transfer of Rs7,620, distributed to 75% of the population,

would push all but India’s absolute poorest above the 2011 – 12 _____________
(a) This debate was re-energized by the Economic Survey 2017, which included a full chapter on financing,

targeting, and distributing an Indian universal basic income.


(b) Tendulkar poverty line bring the national poverty rate down to 0.5%.
(c) The survey advocated a quasi-universal transfer, citing political and fiscal prudence, and suggested methods
ranging from affluence testing to self-targeting to exclude the top 25% of the income distribution.

(d) The survey’s chapter deserves praise for explaining the internal logic behind the pursuit of a universal basic
income and firmly embedding it within mainstream policy discourse

e) None of the above.

401). The government’s announcement that the remains of 39 Indian workers, kidnapped four years ago by the

Islamic State, ____________________


(a) remains found in a mass grave have been matched conclusively with DNA samples for all but one of the men
declared missing almost four years ago.

(b) Part of a group of construction labourers held by the IS shortly after the fall of Mosul in June 2014
(c) the Ministry of External Affairs could have shown more sensitivity by informing the families before Ms. Swaraj
made the rushed announcement in Parliament.

(d) have been found near Mosul in Iraq, has brought a painful closure to the episode
(e) None of the above.

402). Part of a group of construction labourers held by the IS shortly after the fall of Mosul in June 2014,
______________________________.
(a) Since then, there was no word from them, but for the version of Harjit Masih, the 40th hostage who had
managed to escape.
(b) With the recovery of the remains in Mosul after its recapture by Iraqi forces last year, the government must

retrace Mr. Masih’s steps in Iraq in a wider effort to end its investigations into the killings.

(c) they had last contacted their families in the middle of that month and said they were being held in a basement
while fighting raged outside

(d) Mr. Masih said he was the only one to escape from being gunned down by the IS captors, and had
subsequently fled with a group of Bangladeshi labourers.
(e) None of the above.

www.ibpsguide.com https://estore.ibpsguide.com Page 128 of 331


403). It is also not clear whether universal basic income can effectively outperform major social programmes
___________________.
(a) While there is much uncertainty about its final form, it is clear that the quest for universal basic income

involves navigating the tricky waters of weak state capacity and the urgency of improving India’s existing welfare
architecture.

(b) Finally, if universal basic income’s allure still proves too powerful to resist, a basic income trial can generate
much-needed evidence to effectively dismiss or champion the idea.
(c) This leaves scant space for a meaningful, budget-neutral basic income, while efforts to exclude the rich could

reintroduce a role for targeting methods that carry high associated costs for administrators and citizens

(d) The empirical literature on the PDS (public distribution system) and MGNREGA (Mahatma Gandhi National
Rural Employment Guarantee Act) reveals that both schemes have improved significantly in terms of targeting

performance, coverage, and impact on poverty reduction.


(e) None of the above.

404). on the positive side, the government followed every lead in the case, and reached out to governments in
Iraq, Syria and Turkey in the effort to get any available information. _______
(a) It turned out, for instance, that the prison in Badush had been destroyed by the IS early during its occupation,
something that should have been verified before such information was disseminated to distraught family
members.
(b) The biggest lesson from the tragic saga is the need for a thorough appraisal of procedures for Indian labour

going abroad, so that they are not duped or remain uninformed about the risks of going to conflict areas.
(c) However, it would have been more prudent for the authorities to be circumspect instead of unnecessarily

talking up the chances of finding the men alive


(d) Subsequent investigations, including visits by Minister of State for External Affairs V.K. Singh, revealed those
leads to be wrong

(e) None of the above.

405). The world has just learned how a data analytics firm, Cambridge Analytica, harvested the data of 50 million

Facebook users and used that information to feed strategies such as ‘behavioral micro targeting’ and
‘psychographic messaging’ for Donald Trump’s presidential campaign in the U.S. Chris Wylie,

_________________________.

www.ibpsguide.com https://estore.ibpsguide.com Page 129 of 331


(a) a former CA employee-turned-whistle-blower, set off a storm with revelations of how the company had

deployed a ‘psychological warfare’ tool for alt-right media guru Steve Bannon to try to sway the election in Mr.
Trump’s favour.

(b) who was suspended a few days ago following an undercover report by a British TV broadcaster, said the

company has used other dubious methods in projects worldwide — including honeytraps to discredit clients’
opponents.

(c) Facebook has said its policies in 2014, when a personality profiling app was run on its platform, permitted the
developer to scrape data not only from those who downloaded the app but also from the profiles of their Facebook
‘friends’

(d) The issue here is particularly prickly because politics and elections are involved.
(e) None of the above.

406). Social activist Anna Hazare on Friday started an indefinite hunger strike to press for his demand
________________.
(a) Besides implementation of the swaminathan commission report, which has suggested ways to address the
agrarian distress.

(b) Thousands of people will attend the protest.


(c) Hazare has been pressing for setting up of the Lokpal at the center & Lokayuktas in the states.
(d) to appoint a Lokpal at the Centre, nearly seven years after his anti-corruption movement caught the
imagination of millions of Indians and shook the then UPA government.

(e) None of these.

407). The Bharatiya Janata Party (BJP)-led National Democratic Alliance (NDA) is set to improve its tally in the

Rajya Sabha __________________.


(a) While 33 Rajya Sabha seats witnessed no election, 26 went to polls on Friday. Of the 33 elected unopposed,

16 were from the BJP, three from its allies and five from the Congress, among others.

(b) which has 91 seats in the 140-member state assembly.


(c) The seven JDS legislators had defied the party whip in the 2016 Rajya Sabha elections and are all set to join

the Congress party on Sunday.


(d) winning 28 of the 59 seats that went to polls this year.
(e) None of the above.

www.ibpsguide.com https://estore.ibpsguide.com Page 130 of 331


408). President Donald Trump has banned transgender people from the US military in most

circumstances______________________.
(a) and capped a week in which he touched off a confrontation with China by following through with his campaign

pledge to impose sweeping tariffs on Chinese imports, and ousted his national security adviser, who had been
seen as a moderating influence, in favour of John Bolton, a hero of the right
(b) individuals who the policies state may require substantial medical treatment, including medications and
surgery—are disqualified from military service except under certain limited circumstances.”

(c) the latest in a series of actions that may resonate with his most ardent supporters as crucial midterm elections
approach.

(d) The ban, which the president first proposed last summer, touched off a legal confrontation and reignited a
cultural debate that had already seen fierce disagreements and campaigns in some states to require that students
use school restrooms corresponding to their gender at birth.

(e) None of these.

409). With the recovery of the remains in Mosul after its recapture by Iraqi forces last year, ______________
(a) Since then, there was no word from them, but for the version of Harjit Masih, the 40th hostage who had
managed to escape.

(b) Mr. Masih said he was the only one to escape from being gunned down by the IS captors, and had
subsequently fled with a group of Bangladeshi labourers.
(c) the government must retrace Mr. Masih’s steps in Iraq in a wider effort to end its investigations into the killings
(d) This inquiry should also help answer larger questions about the operating procedure followed by the

authorities
(e) None of the above.

410). In India, a growing emphasis on Aadhaar-based direct benefit transfers, small basic income trials, and the
search for alternatives to flagship welfare schemes have created a fertile environment for discussion about

universal basic income. ____________

(a) This debate was re-energized by the Economic Survey 2017, which included a full chapter on financing,
targeting, and distributing an Indian universal basic income

(b) According to its headline estimates, an annual transfer of Rs7,620, distributed to 75% of the population, would
push all but India’s absolute poorest above the 2011-12 Tendulkar poverty line and bring the national poverty rate
down to 0.5%.

www.ibpsguide.com https://estore.ibpsguide.com Page 131 of 331


(c) The survey argued that by transferring benefits directly from state coffers into Aadhaar-linked bank accounts,

universal basic income could sidestep the exclusion, misallocation, and leakage found in major welfare schemes.
(d) However, should future Indian policymakers wish to implement a universal basic income, the Economic

Survey 2017’s proposal alone offers a weak foundation.

(e) None of the above.

411). The biggest lesson from the tragic saga is the need for a thorough appraisal of procedures for Indian labour
going abroad, _________________.

(a) it would have been more prudent for the authorities to be circumspect instead of unnecessarily talking up the

chances of finding the men alive


(b) so that they are not duped or remain uninformed about the risks of going to conflict areas.

(c) However, it would have been more prudent for the authorities to be circumspect instead of unnecessarily
talking up the chances of finding the men alive
(d) to minimize such a incidences.

(e) None of the above

412). Multiple accounts have it that entities like Amazon.com Inc., Google, Facebook Inc. and Apple Inc. are evil

entities. There is much evidence as well to support that narrative. ____________________.


(a) Julia’s essay led to a long investigation by The New York Times on what really happens to people Inside
Amazon, particularly white collar workers.

(b)In Amazon warehouses, employees are monitored by sophisticated electronic systems to ensure they are
packing enough boxes every hour
(c) This little bit from the New York Times sounded particularly poignant: Noelle Barnes, who worked in marketing

for Amazon for nine years, repeated a saying around campus: “Amazon is where overachievers go to feel bad
about themselves.”

(d) For the sake of this argument, let us consider Amazon—not for anything else, but because what started out as

an entity to deliver books is an altogether different beast. A beast that now wants to dominate the world.
(e) None of the above.

413). Prime Minister Narendra Modi, Kumar said, has entrusted the NITI Aayog with ensuring that minimum
support prices are effectively administered and the NITI Aayog is considering three distinct mechanisms

_______________.

www.ibpsguide.com https://estore.ibpsguide.com Page 132 of 331


(a) Mohapatra added that at a recent fair organized by the agriculture ministry (KrishiUnnatiMela), a display of the

electronic national agriculture market (eNAM) platform showed that none of the products was selling below MSP.
(b) Commenting on the price crash in agriculture, NITI Aayog member Ramesh Chand said that “while the current

situation is bad for farmers, it may not last long as price movements are often cyclical

(c) These include schemes on direct procurement by states at support prices, paying farmers the difference
between ruling market price and MSP, and a private stockiest scheme.

(d) The spectre of farm distress has finally begun to haunt policymakers and the government is doing everything it
can to address the situation, Rajiv Kumar, vice-chairman of central government think tank NITI Aayog, said on
Friday.

(e) None of the above.

414). A study by a real estate firm CBRE suggests 50% of occupations today will be gone by 2020

__________________.
(a) If this be the case, it is only a matter of time before most of us will either be without a job or trying to re-
imagine how ought we reinvent ourselves

(b) It is in the nature of disruption and economics


(c) If this sounds cruel, we might as well accept it and get around trying to figure how to deal with it
(d) The rest has happened due to automation. And to make things worse, a McKinsey reckons 45% of knowledge

work activity can be automated.


(e) None of these.

415). Small unconditional cash transfers targeted at vulnerable groups have been used by governments from
Pakistan to Zambia to successfully improve development outcomes ____________.
(a) which are backed by evidence of progressive targeting and low leakage levels. Relaxing eligibility criteria and
moving towards universalization of existing schemes—the PDS, in particular, has benefited from this approach
(b) and can help plug gaps in the patchwork of current social security programmes

(c) Finally, if universal basic income’s allure still proves too powerful to resist, a basic income trial can generate

much-needed evidence to effectively dismiss or champion the idea


(d) Until such data is available, the policy discourse must clearly distinguish between efforts to provide a

genuinely universal, unconditional, and livable income, and interventions that may derive inspiration from a
universal basic income but share little in the way of form or substance
(e) None of the above.

www.ibpsguide.com https://estore.ibpsguide.com Page 133 of 331


416). The most sensible strategy to protect the country’s intellectual property isn’t to keep talented foreign

students out, but to encourage them to stay in the US and put their knowledge to use ____________.
(a) That’s the purpose of the optional practical training program, which allows foreign students who earn degrees

in technical fields to work in the US for up to three years


(b) The Trump administration, true to form, wants to cut the program instead.
(c) by joining the workforce or starting a business.
(d) Whether the president imposes new quotas on Chinese students, the goal of some of his advisers seems

clear: to make “designated” foreigners unwelcome on US campuses.


(e) None of the above.

417). Farmers in several states are demanding fair prices and loan waivers, even as the central government has
set an ambitious target ________________

(a) Despite six decades of agricultural development, we still have a situation like this and it’s time to introspect
deeply,” Kumar said.
(b) to double farm incomes by 2022.
(c) paying farmers the difference between ruling market price and MSP, and a private stockist scheme.
(d) Kumar’s comments come in the wake of repeated farmer protests, beginning in June last year, following a
record harvest and a sharp fall in crop prices

(e) None of the above.

418). CA chief executive Alexander Nix, who was suspended a few days ago following an undercover report by a

British TV broadcaster, said the company has used other dubious methods in projects worldwide
___________________________.
(a) Facebook has said its policies in 2014, when a personality profiling app was run on its platform, permitted the
developer to scrape data not only from those who downloaded the app but also from the profiles of their Facebook
‘friends’

(b) This episode has brought to light several issues that need to be addressed. First, companies have been

collecting data and tailoring marketing campaigns accordingly


(c) Individuals often share their data without being aware of it or understanding the implications of privacy terms

and conditions
(d) including honey traps to discredit clients’ opponents.
(e) None of the above

www.ibpsguide.com https://estore.ibpsguide.com Page 134 of 331


419). Rajya Sabha election results declared Friday showed that the BJP won nine of the 10 seats in Uttar

Pradesh, and one each in Karnataka, Jharkhand and Chhattisgarh ________________.


(a) The BJP’s performance ensures that its dominance as the largest party in the Upper House continues, though

the NDA is still short of a majority


(b) The biggest loser from Uttar Pradesh is the Samajwadi Party (SP), which had six out of the 10 Rajya Sabha
seats that went to polls
(c) While 33 Rajya Sabha seats witnessed no election, 26 went to polls on Friday. Of the 33 elected unopposed,

16 were from the BJP, three from its allies and five from the Congress, among others
(d) The counting of votes was delayed in Uttar Pradesh, Jharkhand and Karnataka following complaints by

members from different political parties


(e) None of the above

420). The country’s first sign language dictionary, in the works for one and a half years, was launched by the
Minister for Social Justice and Empowerment Thaawarchand Gehlot on Tuesday _____________
(a) It may have only 3,000 words, but it is the first step to a potentially huge resource for the country’s 50 lakh
deaf citizens and almost 20 lakh people with speech disabilities.
(b) The dictionary, developed by the Indian Sign Language Research and Training Centre, has subtitled videos
depicting the various signs. It includes legal, medical, academic and technical terms
(c) The aim of developing the Indian Sign Language Dictionary was to remove communications barriers between
the deaf and hearing communities, said Mr. Gehlot
(d) But while infrastructure solves a part of the problem, it cannot completely solve another-behaviour change.

(e) None of the above.

ANSWERS:

391). Correct Answer is: (c)

Explanation –only option C best fits here.As the sentence talks about implementation of health scheme within a

short span of time only by taking proper action.


392). Correct Answer is: (a).

Explanation- Except option ‘A’ other options talks about other aspects of ‘office of profits and government’s role in

that’.
393). Correct Answer is: (d).

www.ibpsguide.com https://estore.ibpsguide.com Page 135 of 331


Explanation – The sentence talks about the action of The States for provision of health care so the correct option

should tell why this scheme is important or why government should take action on NHPM scheme.
394). Correct Answer is: (b)
Explanation – Only option ‘B’ fits here because it is giving the reasons as why the Electronic commission’s

decision was violating the laws


395). Correct Answer is: (c).
Explanation – Except option ‘C’ other options are talking about irrelevant things
396). Correct Answer is: (c).
Explanation – Only option ‘C’ talks about the health coverage rest talks about challenges government have to

face.

397). Correct Answer is: (a)


Explanation – Here we need to choose the sentence which talks about the appropriate action taken by President
Donald Trump to prevent china from stealing American Intellectual property and only option ‘A’ fits there.

398). Correct Answer is: (d)


Explanation- The given sentence talks about the strategy to protect the intellectual property rights and only option
‘D’ talks about that. So the answer will be option ‘D’.
399). Correct Answer is: (d)
Explanation – Only option D fits here rest are irrelevant.

400). Correct Answer is: (e).


Explanation – None of the option fits here.
401). Correct Answer is: (d).

Explanation – Only option connect the given sentence.


402). Correct Answer is: (c)
Explanation – Only ‘C’ talks about the labourers held by the IS.We can get a hint from the pronoun ‘they’

403). Correct Answer is: (d)


Explanation – Only option D talks about the schemes

404). Correct Answer is: (c)

Explanation – Sentence talks about government actions / lead in getting the information about the Indian people
and only option C talks about the effective steps government should have taken to get the details.

405). Correct Answer is: (a)

Explanation – Only option A gives information about U.S Chris Wylie.


406). Correct Answer is: (d)

www.ibpsguide.com https://estore.ibpsguide.com Page 136 of 331


Explanation – Only option ‘D’ talks about the demand of Anna Hazare which completes the sentence & add

meaning to the sentence. Remaining options tells about Lokpal, protest.


407). Correct Answer is: (d)
Explanation – The given sentence talks about the tally of BJP in Rajya Sabha so the correct option will be ‘D’.

408). Correct Answer is: (c)


Explanation – The sentence talks about the one of the actions taken by the U.S President Donald Trump and only

option ‘C’ is giving more details about that.


409). Correct Answer is: (c)
Explanation – Only option C fits here as it gives further details about the incidence.

410). Correct Answer is: (a)

Explanation – Only option ‘A’ best fits here.


411). Correct Answer is: (b)

Explanation – The sentence talks about the lesson government should learn from this incidence and only option B
highlighted that thing as why should govt should learn a lesson from this incidence.
412). Correct Answer is: (d)
Explanation – Only option ‘D’ support/gives the evidence as why Amazon. FB etc. are evil entity.
413). Correct Answer is: (c).
Explanation–Only option ‘C’ briefs about the mechanism.

414). Correct Answer is: (e).


Explanation – None of the option fits here.
415). Correct Answer is: (b)

Explanation– Only option B highlights the benefit of cash transfers targeted at vulnerable groups.
416). Correct Answer is: (c)
Explanation – Only option ‘C’ tells about the strategy to protect the intellectual property of the country.

417). Correct Answer is: (b)


Explanation – only option B talks about the target which best fits in the sentence.

418. Correct Answer is: (d)

Explanation – As the given sentence talks about the dubious methods used by the company only option D tells
about that.

419). Correct Answer is: (c)

Explanation – Only option C briefs more about the Rajya Sabha election .So, the correct Ans is C.
420). Correct Answer is: (e)

Explanation –The option C & B any can fills the gap but we can’t tell which will be the correct one so answer is E.

www.ibpsguide.com https://estore.ibpsguide.com Page 137 of 331


Sentence Correction Type - 1

Directions (421-435): Below is a given passage. Some sentences begin with a number (corresponding to the

question number) and some words are highlighted in bold. One of the highlighted words is grammatically

incorrect. Choose the word from the option as your answer. If all the highlighted words are correct, mark 'All
correct' as your answer

421. In the year since the British Commonwealth dropped the “British” bit in 1949, two people have held the title of

Head of the Commonwealth: George VI until 1952 and Elizabeth II until now. 422. The world has changed out in
recognition in that time, and the Commonwealth with it. 423. The Queen, though, had been a constant. She has

carried out her role assiduously. 424. But the title is not hereditary. So, what should happen next? The

question was being asked, discreetly and decorously, in London this week as part of a review by a seven person
“high-level group” of Commonwealth officials and former ministers. 425. The review is a sensible exercise in

succeeding planning. It is particularly important for an organisation that has been embodied for so long by one
person, but which has itself undergone transformative evolution during those years. 426. The eight member states
of 1949, all former parts of the British empire, have now become 53 independent states, the majority of them

republic not monarchies and some with no British connection at all. 427. Although the Commonwealth emerged
out with empire, words like British and Crown, which featured prominently in the founding London declaration 68

years ago, are nowhere to be seen in today’s Charter of the Commonwealth, adopted in 2013. 428. This is as it
should be. The Commonwealth is a classic piece of soft power. 429. It is a useful, often unsung, network of
values, mutualities and nations. 430. But it has outgrown both their original Britishness and any need for

allegiance to the British Crown. 431. There is no reason other than traditions why its head and embodiment

should invariably be the British monarch. 432. As the Commonwealth prepares for the next phase of its existence,
it makes sense to look more wide for a head who embodies the organisation in more contemporary and diverse

ways. 433. Traditionalists would still favour the future King Charles III. But the 21st-century Commonwealth can
do better. 434. It needs a new kind of head, with leadership skills, global appeal, multicultural sensitivities and the
ability to embody the Commonwealth’s immense power for good in new times. 434. There is no legal need for the

head to be inside the Commonwealth itself. The member nations can pick anyone they wish, providing they can

reach consensus. We have, therefore, a suggestion of our own. The next head of the Commonwealth should
be Barack Obama.

421). ?
(a) year (b) dropped (c) have (d) until (e) No correction required.

www.ibpsguide.com https://estore.ibpsguide.com Page 138 of 331


422). ?
(a) changed (b) in (c) that (d) with (e) No correction required.

423). ?
(a) though (b) had (c) out (d) assiduously (e) No correction required.

424). ?
(a) hereditary (b) should (c) was (d) part (e) No correction required.

425). ?
(a) sensible (b) succeeding (c) so long (d) transformative (e) No correction required.

426). ?
(a) member (b) parts (c) majority (d) republic (e) No correction required.

427). ?
(a) Although (b) with (c) featured (d) nowhere (e) No correction required.

428). ?
(a) as (b) should (c) piece (d) power (e) No correction required.

429). ?

(a) a (b) often (c) of (d) mutualities (e) No correction required.

430). ?

(a) their (b) Britishness (c) any (d) allegiance (e) No correction required.

431). ?

(a) reason (b) traditions (c) embodiment (d) invariably (e) No correction required.

432). ?

www.ibpsguide.com https://estore.ibpsguide.com Page 139 of 331


(a) for (b) sense (c) wide (d) contemporary (e) No correction required.

433). ?
(a) would (b) favour (c) future (d) can (e) No correction required.

434). ?
(a) kind (b) appeal (c) embody (d) in (e) No correction required.

435). ?

(a) need (b) inside (c) wish (d) suggestion (e) No correction required.

Directions (436-450): Below is a given passage. Some sentences begin with a number (corresponding to the

question number) and some words are highlighted in bold. One of the highlighted words is grammatically
incorrect. Choose the word from the option as your answer. If all the highlighted words are correct, mark 'All
correct' as your answer
436. Why should anyone wish to learn about religion? Religion is, by the phrase of the sociologist Linda
Woodhead, “a toxic brand”. 437. Though the public imagination the word summons up images of violence,
patriarchy and irrationalism. 438. The facile confidence of the “New Atheist” movement in the early years of this
century was pushing off an open door. 439. Religious studies nevertheless remain a surprisingly popular A-level
subject, although this may owe something to its reputation as an easy one. 440. A recent YouGov poll found that

the British public think that RE is a subject scarcely more important than Latin, which the public, wrongly, does not

care about at all. 441. The National Association of Teachers of Religious Education has just launched an
appeal for more teachers. 442. The association is quite righteous: religious education matters a great deal. 443.
At the very last it can function as a kind of ethnography, teaching people about the customs and beliefs of
different religious cultures – something that is obviously desirable in a multicultural society. 444. To know that
Muslims and Jews won’t eat pork, and that Hindus regard cows as sacred, is really just a part of civics. 445. There

is nothing specifying religious about such teaching, even if it is by convention part of religious education. 446. It

could just as well be taught from geography or history, subjects profoundly influenced by the beliefs and actions
of religious people. The real task of RE is much more ambitious. 447. The peculiar difficulty of religious education,

which distinguishes it from all other subjects, is that it deals with beliefs that are not true for the way that the facts

of other subjects can be. 448. The sciences deal in testable facts; pupils can and do rediscover by experiment the
truths of physics or of chemistry and biology. In history, economics and even literature, there are methods of

inquiry that will converge around the generally accepted picture of the world. 449. But there is no experiment that

www.ibpsguide.com https://estore.ibpsguide.com Page 140 of 331


could determine whether God is love, or whether Muhammad is his prophet. There is no experiment that can

determine the truth of a humanist belief in human rights. 450. These are the sorts of beliefs that can all appear
absurd from a hostile perspective, and where they flourish they are not taught as schoolroom propositions but

transmitted in thick cultural bundles of habit and ritual: that is why there are so many middle-aged agnostics who
still love to sing the hymns of their childhood.

436). ?
(a) should (b) about (c) by (d) of (e) No correction required.

437). ?

(a) Though (b) imagination (c) summons (d) images (e) No correction required.

438). ?
(a) facile (b) of (c) years (d) off (e) No correction required.

439). ?
(a) remain (b) surprisingly (c) although (d) something (e) No correction required.

440). ?
(a) found (b) think (c) scarcely (d) wrongly (e) No correction required.

441). ?
(a) of (b) just (c) appeal (d) more (e) No correction required.

442). ?
(a) is (b) righteous (c) matters (d) great (e) No correction required.

443). ?
(a) last (b) kind (c) about (d) obviously (e) No correction required.

444). ?
(a) that (b) won't (c) and (d) really (e) No correction required.

www.ibpsguide.com https://estore.ibpsguide.com Page 141 of 331


445). ?

(a) specifying (b) such (c) convention (d) education (e) No correction required.

446). ?

(a) could (b) from (c) profoundly (d) by (e) No correction required.

447). ?
(a) peculiar (b) which (c) for (d) can (e) No correction required.

448). ?

(a) sciences (b) pupils (c) inquiry (d) around (e) No correction required.

449). ?
(a) no (b) could (c) his (d) humanist (e) No correction required.

450).
(a) sorts (b) appear (c) where (d) in (e) No correction required.

ANSWERS:
421). Correct Answer is: (a)
Replace 'year' with 'years'

422). Correct Answer is: (b)


Replace 'in' with 'of'. The correct phrase is 'out of recognition'. If you say that someone or something has changed
out of all recognition, you mean that person or thing has changed so much that you can no longer recognize them.

423). Correct Answer is: (b)


Replace 'had' with 'has'.

424). Correct Answer is: (c)

Replace 'was' with 'is'.


425). Correct Answer is: (b)

Replace 'succeeding' with 'succession'.

426). Correct Answer is: (d)


Replace 'republic' with 'republics'.

427). Correct Answer is: (b)

www.ibpsguide.com https://estore.ibpsguide.com Page 142 of 331


Replace 'with' with 'of'.

428). Correct Answer is: (e)


429). Correct Answer is: (d)
Replace 'mutualities' with 'mutuality'.

430). Correct Answer is: (a)


The subject (it) is singular, so use the singular form 'its' in place of 'their'.
431). Correct Answer is: (b)
Replace 'traditions' with 'tradition'.
432). Correct Answer is: (c)

Replace 'wide' with 'widely'.

433). Correct Answer is: (a)


Replace 'would' with 'will'.

434). Correct Answer is: (e)


435). Correct Answer is: (b)
Replace 'inside' with 'from'.
436). Correct Answer is: (c)
Replace 'by' with 'in'.
437). Correct Answer is: (a)

Replace 'Though' with 'In'.


438). Correct Answer is: (d)
Replace 'off' with 'at'. If someone is pushing at an open door, they are finding it very easy to achieve their aims.

439). Correct Answer is: (a)


Here, 'religious studies' is one particular subject, therefore 'singular' in sense. So use singular verb 'remains' in
place of 'remain'.

440). Correct Answer is: (b)


'Public' here is singular, so replace 'think' with 'thinks'.

441). Correct Answer is: (e)

442). Correct Answer is: (b)


Replace 'righteous' with 'right'.

443). Correct Answer is: (a)

Replace 'last' with 'least'.


444). Correct Answer is: (c)

Replace 'and' with 'or'.

www.ibpsguide.com https://estore.ibpsguide.com Page 143 of 331


445). Correct Answer is: (a)

Replace 'specifying' with 'specifically'.


446). Correct Answer is: (b)
Replace 'from' with 'under'.

447). Correct Answer is: (c)


Replace 'for' with 'in'.
448). Correct Answer is: (e)
449). Correct Answer is: (b)
Replace 'could' with 'can'.

450). Correct Answer is: (a)

Replace 'sorts' with 'sort'.

Sentence Correction Type - 2

Directions (451-480): In each of the following questions, a statement and five choices are given. Select from

among the choices, the most logical complement and mark its number as your answer.
451). Even before Narendra Modi took over as India’s prime minister, he decided to invite the leaders of SAARC

(South Asian Association for Regional Cooperation) countries and Mauritius to attend his swearing-in ceremony.
(a) The neighbourhood remains a headache for the Modi government.
(b) He has visited all the countries in the neighbourhood except the Maldives.
(c) His first foreign visit after assuming office was to Bhutan.

(d) “Neighbourhood first” was the first policy decision of the Modi government.
(e) Most of the analyses of India’s neighbourhood woes revolve around whether a pro-India or a pro-China regime

is in power in these countries.

452). Coupled with some savvy branding, this new generation of banks became the darling of the industry.
(a) As profit margins in the business were coming under pressure, the reduction in the cost of operations using

these technological interventions was a great relief to banks, and was aggressively encouraged in the name of
introducing innovative banking.
(b) The private sector banks wanted to grow rapidly, and their focus was on the latest technology had to offer.

(c) ATMs mushroomed at every nook and corner.


(d) In the traditional banking model, the processing of a transaction was done at the field or the branch level.

www.ibpsguide.com https://estore.ibpsguide.com Page 144 of 331


(e) The fragmented model of today’s banking and due to the reduced human interventions, the monitoring and

supervision structure in banks has deteriorated, resulting in more incidents of leakages, deviations, violations and
manipulations.

453). The CPI-M-led Left Front in Tripura was left red-faced as the BJP managed to seal a historic 2/3rd majority,
thereby, closing the curtains for the 25-year-old regime in the northeastern state.
(a) The BJP campaign in Tripura was led by none other than the Prime Minister himself with the party putting up
Modi posters and cutouts all over Agartala and elsewhere.
(b) BJP's Tripura story also marks the rise of Biplab Deb, the state party chief, who has emerged a frontrunner for

the state's top job now.

(c) BJP also performed in Nagaland, where the Centre-ruling party's alliance with NDPP seems to be bearing
fruit.

(d) BJP left no stone unturned in securing all three states in the northeast.
(e) The party's North East Democratic Alliance convenor Himanta Biswa Sarma was headed to Shillong to try and
form a non-Congress government in Meghalaya.

454). A study, conducted by AC Nielsen and commissioned by Britannia across six metros, found that one in two
respondents said they hadn’t met family during festivals for the past three years.

(a) 76% of working professionals have cancelled a family holiday at least once in the last three years.
(b) WhatsApp was the new 'meeting place' for families, while 73% were connected on Facebook.
(c) Expensive tickets make travelling home difficult.

(d) Vacations taken by extended families have reduced, and nuclear families taking trips with friends is now the
trend.
(e) With more Indians settled abroad, vacation time doesn’t coincide so family trips are rarer.

455). The battle for eyeballs between online video streaming giants Netflix and Amazon Prime Video is set to

intensify.

(a) Amazon Prime subscribers who actively use Prime Video as opposed to the total Prime subs.
(b) Although Netflix continues to be the global leader in online video streaming, Germany and Japan are the other

two markets, where Amazon’s subscriber base has surpassed that of Netflix’s.

(c) Analysts expect Amazon’s local content investments, integration and deals with local telcos and expansion of
the Amazon Prime offering to be the key drivers for Amazon Prime Video’s growth.

(d) Both companies follow different business models.

www.ibpsguide.com https://estore.ibpsguide.com Page 145 of 331


(e) Amazon Studios is likely to bring in a strong new leader given their large content budgets.

456). Mirchi, India's No.1 radio station, has always championed contemporary issues which are relevant to
society.

(a) With the highest listenership across the country and a track record of developing innovative content, Mirchi
has been expanding and retaining its audiences and advertisers through the years.
(b) Mirchi has delighted listeners and the industry with its exciting properties such as Mirchi Music Awards (in
eight languages) & Mirchi Top 20.
(c) Mirchi Chandigarh organised a "Power Drive" in the city on 18th August led by its Gang of Girls which involved

a large group of women who rode through Chandigarh making a statement for women's safety in the city

especially at night.
(d) Mirchi was also the first Indian brand to go international with the launch of the brand in the UAE.

(e) At Mirchi we have always strongly supported progressive thinking.

457). Analysts, VCs, and founders who called Zomato 'the only defensible Indian unicorn,' the term conjures
nothing but disdain now.
(a) Zomato has suffered a security breach with over 17 million user records stolen from the food-tech company's
database.

(b) 10 years after he co-founded Zomato, Pankaj Chaddah has hung his boots at the food technology firm in what
is the first of its kind instance of a founder leaving a unicorn startup in recent times.
(c) Zomato has raised $200 million from Ant Financial, the payments affiliate of Chinese e-commerce giant

Alibaba.
(d) The fight between the two players - Zomato and Swiggy, is playing out as both are in the midst of raising new
capital.

(e) HSBC Securities & Capital Markets slashed the restaurant search & discovery venture's valuation by half.

458). Public sector banks have been value destroyers of late.

(a) When the inflation is low, it is best to cut interest rates so that businesses could borrow better and grow the
economy.

(b) A few fund managers have done well to stay away from PSU bank stocks despite their outperformance last

year.
(c) The fraud at Punjab National Bank damaged investor confidence in the sector.

(d) Some PSU banks may be cheap and offer value, but did not fit in to the growth philosophy.

www.ibpsguide.com https://estore.ibpsguide.com Page 146 of 331


(e) PSU bank stocks are once again at the receiving end.

459). Despite the downslide in urban markets, Flipkart was able to keep its nose ahead in the race with Amazon.
(a) Flipkart dominates in two of the biggest categories in the online retail — smartphones and fashion.

(b) Flipkart started scaling and scaling instead of having focus on improvement of existing operations,

services,handling customer base,delivery,quality etc.


(c) Krishnamurthy, Flipkart's CEO, believes that an omni-channel foray — selling both offline and online—could be
a game-changer in the long run, albeit not in every category or customer segment.

(d) Flipkart have enough funding.

(e) In addition to owning fashion portal Myntra, the company also acquired rival fashion portal Jabong.

460). India’s public distribution system (PDS) is in danger of being derailed in several States across the country.
(a) If people had cash reserves, the system might work better.
(b) Recent disruptions of the PDS have taken different forms, from compulsory biometric authentication to so-
called direct benefit transfer (DBT).
(c) PDS rice meant for those who failed the biometric test was siphoned off with abandon.
(d) Most of the State governments are under tremendous pressure from the Central government to impose

Aadhaar-based biometric authentication or move towards DBT.


(e) UIDAI is participating in the crusade to make Aadhaar-based biometric authentication compulsory in various
contexts.

461). All tobacco-related products have been placed in the 28% tax slab.
(a) Governments have resorted to a mix of policies which range from monitoring the pricing and taxation regime of

these products.
(b) Behavioural changes towards tobacco use is important.

(c) For appropriate pricing policies, taxation measures must be at the forefront.

(d) It has become imperative for policymakers to devise measures to effectively bridle their use.
(e) India has a distinct pattern of tobacco consumption in multiple forms such as cigarettes, bidis, chewing

tobacco and khaini.

462). Bitcoin, the popular digital currency whose meteoric rise in price last year has surprised many of its critics.

www.ibpsguide.com https://estore.ibpsguide.com Page 147 of 331


(a) Bitcoin enthusiasts still believe that the currency may only be consolidating to begin its next huge rally to the

sky.
(b) There could be some valid reasons to get worried about the strength of bitcoin’s price rally.
(c) The cryptocurrency has wowed markets with breakneck gains as investors flocked to an asset that exists only

in cyberspace.
(d) The digital currency, which is supposed to derive its value from its use as a medium of exchange, has had
negligible acceptance in the real world of commerce.
(e) The scorching-hot cryptocurrency has tentacles that stretch into many different parts of the investment
landscape, and some traders are finding out the hard way how much influence it can wield.

463). A strong and united judiciary is the sine qua non for a strong and vibrant democracy.

(a) Corrective measures need to be taken by the judiciary itself in order to set things in order.
(b) The judges are looked at as gods by the common man.
(c) Political parties are settling scores, by accusing each other of interference in the highest judiciary.
(d) Judicial discipline also requires maintaining a certain decorum because under the Contempt of Courts Act,
truth is not a valid defence.
(e) In a democracy, the judiciary is one of the most important pillars and it is the last ray of hope for the common

man.

464). Suicide rates in India have skyrocketed in recent year.

(a) There is a fear about talking to people because of the way they may perceive you.
(b) Seven per cent adolescents were found to be bullied and they felt disturbed by the comments of their peers,
family members or teachers.

(c) The Indian government has been criticized by the media for its mental health care system, which is linked to
the high suicide rate.

(d) In India, suicide was illegal and the survivor would face jail term of up to one year and fine under Section 309

of the Indian Penal Code.


(e) Apart from smoking cigarettes, use of other tobacco products was also found to be high.

465). Trump has been an aggressive salesman for American defence manufacturers during his foreign tours and

to visiting heads of foreign countries.

www.ibpsguide.com https://estore.ibpsguide.com Page 148 of 331


(a) The sale of weaponry, traditionally, is guided less by commercial considerations rather than strategic ones.

(b) The security establishment and Congress will not easily accede to major changes in existing U.S. laws in order
to further Mr. Trump’s ideas.
(c) India’s robust defence partnership with Russia is a major irritant for American officials.

(d) In a joint press conference with Norwegian Prime Minister Erna Solberg at the White House earlier this month,
U.S. President Donald Trump made up the name of a non-existent fighter plane, “F-52,” while lauding the F-35
fighter sale in a new defence deal with America’s NATO ally.
(e) Mr. Trump wants to make arms sale itself a strategy.

466). The Indian rupee hasn't had a good run in the last two months.

(a) The return ratios in the future aren't expected to go through the roof.
(b) Investment demand continues to be tepid.
(c) The government needs to realise that business ca only thrive where the business and tax climate are certain.
(d) Downturns in consumer services, finance and insurance, real-estate and business services outweighed the
upturns in information and communication and transport & storage.
(e) The dollar index has been trending lower and has slipped 2.6%.

467). Stocks of large IT companies are the traditional safe haven.

(a) Investors are rewarding companies that have been able to articulate as well as demonstrate confidence about
growth, something that’s lacking among large-sized IT companies.

(b) Since smaller companies were at a relatively low base with respect to margins, investors expect earnings
growth to remain high as well.
(c) The companies that are doing better and where earnings are estimated to grow in double-digits, valuations of
over 20 times forward earnings assume earnings growth to remain strong for some years to come.

(d) Stocks of large IT companies, the traditional safe haven, fell 3% and compared to the broad market, which has
fallen around 10%.

(e) In the backdrop of high valuations, it’s a bit odd that some of the stocks are being viewed as safe havens in a

risk-off environment.

468). Twitter and Facebook have been under scrutiny by international regulators.

(a) Informations are shared differently in both social media: on Facebook, one needs to connect first with people
before sharing a content, while on Twitter, one can get quick info without any obligation to follow anyone.

www.ibpsguide.com https://estore.ibpsguide.com Page 149 of 331


(b) Facebook and Twitter keep track of every user and every page that is built on with the help of an auto

generated ID.
(c) False news stories spread much more quickly and widely on social media than truthful ones.

(d) Supporting privacy is a highly limiting burden, both from technology and business standpoints.

(e) Twitter and other social media companies have done too little to prevent the spread of false content.

469). The government will soon have to bail the sugar industry out of its predicament.
(a) These sweet white crystals could rot teeth and cause people to pack on the pounds.

(b) “Sugar is the new tobacco” in the minds of the public, says David Turner, a global food and drink analyst

at market research firm Mintel.


(c) A majority of sugar stocks hit a soft patch in the wake of bumper production and limited buying.

(d) With the carry over stock, total availability of sugar is sufficient to meet the estimated domestic requirement.
(e) The improvement in sugarcane acreage for sugar year 2017-18 has resulted in higher levels of sugar
output.

470). A favourable verdict in the elections to the State Assembly in Karnataka is important for both the Congress
and the Bharatiya Janata Party (BJP).

(a) Karnataka has a weighty presence in the Union ministry, which includes Defence Minister Nirmala Sitharaman,
a Rajya Sabha member elected from here.
(b) Congress has not won any State election, barring Punjab, after 2014 and Karnataka was the first State BJP

came to power in South India.


(c) Congress is increasingly veering round a package of values such as respect to religious commitment, social
justice and equality, and human dignity as its mantras, without necessarily disowning secularism.

(d) Caste has always been an important factor in the political process of Karnataka since wider ideological
struggles.

(e) The BJP has clearly marked out Mr. Siddaramaiah, rather than the Congress, as its enemy number one.

471). A broad-based recovery in global growth may be gathering steam.

(a) Since the 2008 financial crisis the global economy has been propped up mainly by the unprecedented easy

money policies adopted by global central banks.


(b) India’s economy is projected to grow at 7.4% during the financial year 2019, and at an even faster pace of

7.8% the following year.

www.ibpsguide.com https://estore.ibpsguide.com Page 150 of 331


(c) The IMF, however, was not oblivious to the threats that could severely derail the broad-based economic

recovery.
(d) Countries will need to move beyond monetary stimulus to boost their economies.
(e) IMF has pointed out, the possible end to the era of abundant liquidity and debt-fuelled economic activity is

likely to cause disruption by affecting asset prices.

472). Reading comic books is detrimental to the mental development of teenagers.


(a) Comic books contain ideas that are immoral or may even actively encourage deviant behaviour.
(b) Comic books are so popular that they are effectively used to convey desirable knowledge to teenagers.

(c) Their popularity implies that students have less time to spend on reading their more important study material.

(d) Any reading indicates a willingness to spend time on the written word and should therefore be encouraged.
(e) Fascinatingly reading comics thought to improve fluid intelligence showed little or no positive correlations to

performance in memory and comprehension.

473). Microsoft has decided to use Intel’s new Myriad X vision processor units (VPU) for handling all artificial
intelligence (AI)-related processing.
(a) Intel’s VPUs are not the only option available when it comes to handling AI workload on PCs.

(b) AI is beginning to embed itself into all aspects of our lives.


(c) The new platform will allow AI-based applications to offload most of the workload from cloud to the Windows
10 device.

(d) Light AI tasks will be handled by CPU, while the more complex will be assigned to the VPU.
(e) Processing heavy data set on a Windows PC will put a lot of stress on the device’s central processing unit
(CPU) and graphics processing unit (GPU) and will slow down other apps and processes.

474). The stocks of Kaveri Seeds Co. fell 4% over the last three days.

(a) The company is planning an expansion in key cotton growing states such as Maharashtra, Andhra Pradesh,
Telangana and Gujarat.

(b) The possibility of lower crop acreage this year after a season of repeated pest attacks and crop losses.
(c) It alluded to the increasing share of non-cotton seeds business.
(d) Union government to initiate criminal action against the sellers and buyers of Bollgard3 as the genetically

modified cotton variety.

www.ibpsguide.com https://estore.ibpsguide.com Page 151 of 331


(e) It is being feared that indiscriminate use of GM cotton may result in the development of a super weed.

475). The attack by the Taliban gunmen at the Intercontinental Hotel in Kabul last weekend was a grim reminder
of the deteriorating security environment in Afghanistan.

(a) Over the last 16 years, civilian casualties have mounted to 31,000, increasing progressively to over 4,000 a
year.
(b) The U.S. has contributed significant blood and treasure, spending over a trillion dollars (considerably more if
long-term veterans’ care is included) and losing more than 2,400 lives in pursuing the longest war in its history.
(c) Without a clear division of power and responsibility, relations between the President and his Chief Executive

have remained strained.

(d) The Afghan security forces are losing nearly 7,000 men a year, an attrition rate difficult to sustain.
(e) It is hardly surprising that since end-2016, there are growing questions about the legitimacy of the present

arrangement.

476). Each year, environmentalists record a high number of dead turtles washing up ashore.
(a) There are as many as five species in Indian waters — Leatherback, Loggerhead, Hawksbill, Green and Olive
Ridley.
(b) Growing public interest in bycatch reduction programmes is motivated by factors such as an appreciation for

endangered species and concern for maintaining marine biodiversity.


(c) The eastern coastline is the feeding area for Olive Ridley, juvenile Hawksbills and Green turtles.
(d) They sustain internal injuries from fishing hooks or suffer serious external injuries from trawl nets and gill nets.

(e) Thousands of sea turtles are accidentally captured by mechanised boats.

477). The World Bank announced that it would revise the methodology it uses to calculate the ease of doing

business index.
(a) The ease of doing business index has become a popular tool tracked by governments trying to show the world

that they offer a favourable investment climate for private businessmen.

(b) Earnest ease of doing business would mean carefully tracing back the archaic legislation, checking if they are
still relevant in solving the problems.

(c) A move that is expected to affect the rankings of countries in the last four years.

(d) Some critics have pointed to Chile which has seen its ranking fluctuate widely based more on the ideology of
the government in power than on underlying business conditions.

(e) Many businesses, for example, may be able to bribe their way out of bad rules.

www.ibpsguide.com https://estore.ibpsguide.com Page 152 of 331


478). The government has decided to recapitalise public sector banks (PSBs).
(a) It may, at best, expand the credit portfolio of the bank, but these new loans will also add up to existing bad
loans.

(b) Frequent recapitalisation and letting free corporate loan defaulters has strengthened the popular belief that
loans taken from nationalised banks need not be repaid.
(c) Bank unions have been listing big-ticket defaulters on bank loans, but no action is being taken against these
defaulters by the powers that be.
(d) The government needs to do more to bring PSBs back on track.

(e) The banks face a mounting challenge posed by non-performing assets.

479). Television has made people passive by reducing the urge to spend time on more active and healthier

activities.
(a) Even before television became a popular mode of entertainment, people spent a substantial amount of their
time listening to the radio.
(b) People today hardly go out on picnics or treks.
(c) Television has merely fulfilled a need for entertainment of a better quality.
(d) The quality of the medium is such that it encourages people to sit and watch for long periods.

(e) No medium has ever dominated people's lives as television did.

480). The private lives of public figures must not be discussed in the press.

(a) The press has no right to invade the privacy of an individual, public or otherwise.
(b) Public figures must be used to curiosity about their private lives.
(c) The press has a pressing need to be relevant and balanced.

(d) Public figures are not very sensitive people.


(e) Public figures are too controversial.

ANSWERS:
451). Correct Answer is: (d)

Option (a) has a negative tone with respect to the given sentence. Options (b), (c) and (e) are too general

statements and cannot complement the given sentence. Only (d) correctly complements citing the reason behind
the given sentence.

452). Correct Answer is: (b)

www.ibpsguide.com https://estore.ibpsguide.com Page 153 of 331


Only option (b) correctly complements the given sentence. The given sentence is talking about a new generation

of banks highlighting that they have become the 'darling of the industry'. This new generation of banks is nothing
but the 'private sector banks' mentioned in option (b) that have 'latest technology' to offer.

453). Correct Answer is: (a)

Options (c), (d) and (e) does not talk about 'Tripura' in particular. So, all these sentences cannot compliment the
given sentence as it mainly focuses on 'Tripura'. Option (b) is a general statement. The only correct complement
to the given sentence is option (a).
454). Correct Answer is: (c)
It is not mentioned whether the 'two respondents' have nuclear or extended families or whether they are settled

abroad. So, both options (d) and (e) can be ruled out. Option (a) provides a statistics. Option (b) is a general

statement. The only logical complement is option (c) citing the reason behind what is mentioned in the sentence
given.

455). Correct Answer is: (b)


The only logical complement is option (b) which clearly indicates that ' Amazon’s subscriber base has surpassed
that of Netflix’s' in Germany and Japan which could be the reason for the 'battle for eyeballs'.
456). Correct Answer is: (c)
Options (a), (b) and (d) are too general statements which has nothing to do which the given sentence. In option
(e) it is not mentioned how the 'progressive thinking' is relevant to society. The only complementary sentence is

mentioned in option (c) as it mentions about women's safety - a 'contemporary issue' which is relevant to society.
457). Correct Answer is: (e)
Option (e) mentions the reduction in valuation of the food tech startup - which is the reason of the disdain. Hence,

it is a complementary sentence.
458). Correct Answer is: (c)
The given sentence has a negative tone. But the tone in option (a) and (e) are positive. Options (b) and (d) are
general statements. Option (c) appropriately complements the given sentence citing the reason - the fraud in the
Punjab National Bank.

459). Correct Answer is: (a)

Option (a) clearly provides the reason why Flipkart was able to keep its nose ahead in the race with Amazon. Both
Flipkart and Amazon are highly-valued companies with huge funds. Option (d) is too vague to complement the

given sentence.

460). Correct Answer is: (b)


Option (b) appropriately complements the given sentence containing the reasons (disruptions of the PDS due to

various forms) as to why the PDS is in danger in various States.

www.ibpsguide.com https://estore.ibpsguide.com Page 154 of 331


461). Correct Answer is: (d)

Option (a) is just another detailed version of the given sentence. Options (b), (c) and (e) are general statements.
The only correct complement to the given sentence is option (d) citing the reason for the 28% tax slab of tobacco
products, to bridle or to curb their use.

462). Correct Answer is: (c)


The given sentence has a positive tone but the sentences provided in options (b) and (d) has a negative tone.
Options (a) and (e) are general statements and not related to the given sentence. Only option (c) complements
the given sentence citing the reason behind the 'meteoric rise' as mentioned in the given sentence.
463). Correct Answer is: (e)

Only (e) correctly complements the given sentence. The other options are not related in any way to the given

sentence.
464). Correct Answer is: (c)

Option (a) and (b) doesn't give us a clear idea whether 'the fear of talking' or 'bullying' may lead to suicide.
Options (d) and (e) are general statements. Only option (c) correctly cites the reason behind the 'high suicide
rates' in India.

465). Correct Answer is: (d)


Option (d) clearly provides an example of what is mentioned in the given sentence and hence complements the
sentence. Other the other options contain general statements which are not related to the sentence provided.

466). Correct Answer is: (b)


The lowering of 'dollar index' will have a positive impact on Indian currency. Hence, option (e) cannot complement
the given sentence. Options (c) and (d) are general statements which cannot be linked to the sentence provided.

In option (a), the return ratios cannot justify the poor performance of the Indian rupee. Option (b) is a probable
reason for the poor performance of Indian currency as it clearly mentions 'the lack of investment demand'.

467). Correct Answer is: (d)

The given sentence has a positive note whereas sentences provided in options (a) and (e) have negative tones.
There is no mention of IT stocks in options (b) or (c), so they cannot complement the given sentence. The only

probable complementary sentence is (d) which indicates that IT stocks have done fairly well compared to others.

468). Correct Answer is: (e)


Options (a) and (b) are too general statements and cannot be the reasons as to why these social media platforms

were 'under scrutiny'. There is no mention of social media in option (d), hence it cannot compliment the given
sentence. Though option (c) seems likely, but it doesn't complement. Option (e) states in detail that social media
companies have done 'too little' to prevent the spread of false content, which is the probable reason behind the

'scrutiny by international regulator'.

www.ibpsguide.com https://estore.ibpsguide.com Page 155 of 331


469). Correct Answer is: (c)

The given sentence talks about the sugar industry as a whole, while sentences provided in options (a) and (b)
indicates the health implications of consuming sugar. The statement clearly mentions 'difficult situation' of the
sugar industry while sentences in options (d) and (e) have a positive tone. The only probable complement is

the sentence provided in option (c) as it mentions that sugar stocks have hit 'a soft patch'. Soft patch means a
period of weak economic performance amidst a trend of economic growth.
470). Correct Answer is: (b)
Only (b) complements the given sentence. Since Congress has not won any State election barring Punjab, a
victory here would help it cash it on elsewhere, boosting its campaign in other States. For BJP, Karnataka was the

first State it came to power in South India and its inability to repeat this performance may tell badly on its future in

the South
471). Correct Answer is: (a)

The given sentence has a positive tone, however the tones in options (c), (d) and (e) are negative. The sentence
is talking about 'global economy' while (b) talks about the 'Indian economy'. The only sentence having a positive
tone complementary with the given sentence is the sentence in option (a).
clearly cities the reason behind the 'detrimental effect' on mental development.
472). Correct Answer is: (e)
Sentence in option (a) is irrelevant as the issue is not the moral development of the teenagers. For option (c), the

fact that comic books distract students from their studies do not mean that they are detrimental to their mental
development. The given sentence has a negative tone, while the sentences in options (b) and (d) are positive.
The only probable complement is provided in option (e) that

473). Correct Answer is: (c)


Only the sentence in option (c) correctly complements the given sentence citing how the new platform will benefit
Windows 10 devices - a Microsoft product.

474). Correct Answer is: (b)


The low yield in production will directly hamper the market price for cotton. Hence, option (b) is a complementary

sentence.

475). Correct Answer is: (d)


Only (d) complements the given sentence giving an example of 'of the deteriorating security environment' in

Afghanistan. Option (a) provides statistics of the past which is not relevant. Sentences in options (b), (c) and (e)

are too general to complement the given sentence.


476). Correct Answer is: (d)

www.ibpsguide.com https://estore.ibpsguide.com Page 156 of 331


The thing to catch here is the sentence mentions that 'dead turtles' are washed ashore. Only (d) can complement

this as the resulting internal and external injuries lead to their death and hence they end up washing ashore.
However, if sea turtles are captured by boats they cannot get washed up ashore, hence (e) cannot complement
the given sentence.

477). Correct Answer is: (c)


Except (c), all the other sentences are too general to complement the given sentence. Only (c) is a clear
implication of the given sentence, hence, it complement the sentence provided.
478). Correct Answer is: (e)
Option (e) cites the correct reason behind the decision of the government to recapitalise public sector banks.

479). Correct Answer is: (d)

Option (a) cannot does not give a valid reason as to why television cannot be blamed for passivity - it merely
states that people spent a 'substantial amount of time' listening to the radio too. In option (b) there is no

connection made to television watching and not going on picnics and treks. The given sentence has a negative
tone while options (c) and (e) have a positive tone. Only option (d) shows how television inherently encourages a
lack of activity.
480). Correct Answer is: (a)
Only (a) correctly complements the given sentence. 'Being used to curiosity' is a vague reason for discussing
public figures in newspapers. To be relevant the press need not discuss the private lives of public figures, they

can take up other issues. Option (d) and (e) are vague and unsubstantial generalizations.

Sentence Correction Type - 3

Directions (481-510): Various statements are given for the questions below. Choose the options corresponding to
them mentioning the incorrect statements. If all the statements are correct then choose none of these as your

answer.

481). A. The legendary rôtisseur and owner of the three Michelin-star Parisian restaurant L’Arpège, Alain

Passard, is celebrated for his delectably tender slow-cooked meats for over three decades.
B. Then, in 2001, he made the revolutionary decision to take red meat off his menu, and put vegetables centre

stage, designed plates that enabled them to truly shine.

C. Today, he spends as much time farming as he does cooking; he also plays the saxophone.
(a) Only A (b) Only B and C (c) Only B (d) Only A and B (e) None of these.

www.ibpsguide.com https://estore.ibpsguide.com Page 157 of 331


482). A. Upholding the right of the terminally ill to die with dignity, the Supreme Court granted legal sanction for

passive euthanasia and execution of a living will.


B. In an unanimous judgment, a constitution bench headed by Chief Justice Dipak Misra accorded primacy to the
constitutional values of liberty, dignity, autonomy and privacy as it laid down procedural guidelines governing a

living will.
C. A living will is a document prepared by a person in their healthy/ sound state of mind under which he can
specify in advance whether or not he would like to opt for artificial life support, if he is in a vegetative state due to
an irreversible terminal illness in the future.
(a) Only C (b) Only B (c) Only A and B (d) Only B and C (e) None of these.

483). A. Anew research paper by the Nobel-winning economist Angus Deaton, based on self-reported data from
Gallup polls, suggests that people tend to be consistently optimistic about their futures even though this optimism

is perpetually frustrated by actual outcomes.


B. Among the youth, expectations for future well-being run far ahead of reported well-being today.
C. The gap diminishes with age, and in rich countries, those with 65 years of age expect their future well-being to
be worse than what they are facing today, Deaton’s analysis shows.
(a) Only A and B (b) Only A and C (c) Only B and C (d) Only C (e) None of these.

484). A. The rising inflow of foreign investment in India’s private hospitals can lead to concentration of market
power, hurt public interest, writes Gulzar Natarajan, a former civil servant in his popular blog, Urbanomics.
B. The changing pattern of ownership in Hyderabad, for instance, has led to a power struggle between the old

promoters and new investors in at least some hospitals, with the latter trying to force hospitals to set procedural
targets for doctors.
C. Natarajan uses the example of Hyderabad, which has attracted large foreign investors such as Malaysia’s IHH

Healthcare Bhd, to argue that tertiary medical costs can spike up significantly when a big foreign hospital chain
purchases stakes across several hospitals in the same city.

D. It may only be a matter of time before healthcare policy debates are taken over by a lobby of corporate care

providers, their financiers, and insurers, warns Natarajan.


(a) Only A (b) Only B (c) Only C (d) Only D (e) None of these.

485). A. The Reserve Bank of India (RBI) has asked commercial lenders to provide it with details of all letters of
undertaking—a form of credit guarantee by the centre of an alleged $2 billion fraud—issued by them in the past

several years, according to four bankers who have seen the directive.

www.ibpsguide.com https://estore.ibpsguide.com Page 158 of 331


B. RBI wrote to all banks a week ago asking for details of the LoUs they have written, including the amounts

outstanding, and whether the banks had pre-approved credit limits or kept enough cash on margin before issuing
the guarantees, the bankers said.
C. One of the bankers said the regulator had asked for details of LoUs issued as far back 2011.

(a) Only A and C (b) Only A and B (c) Only B and C (d) Only B
(e) None of these.

486). A. US President Donald Trump has threatened to impose high tariffs on countries like India and China if
they do not match his country’s tariffs on similar items.

B. India reacted cautiously to the threat, stating that commitments by the US and India to the World Trade

Organization (WTO) differ as the former is a developed country while the latter is a developing country, enjoying
more flexibility.

C. India’s ambassador to the WTO, J. S. Deepak, explained that there is an outcry by the US for raising duties on
two items but not against India for raising duties on 40 items in this year’s Union budget because India has raised
them within the ceiling rates committed to WTO.
(a) Only C (b) Only A (c) Only B (d) Only A and C (e) None of these.

487). A. The main events in a political campaign used to happen in the open: a debate, the release of a major TV

ad or a public event where candidates tried to earn a spot on the evening news or the next day’s front page.
B. That was before the explosion of Facebook, Twitter and YouTube as political platforms.
C. Now some of a campaign’s most pivotal efforts happen on the often-murky world of social media, where ads

can be targeted to ever-narrower slices of the electorate and run continuously with no disclosure of who is paying
for them.
D. Reporters cannot easily discern what voters are seeing, and hoaxes and forgeries spread instantaneously.

(a) Only A (b) Only B (c) Only C (d) Only D (e) None of these.

488). A. Fuelled by increasing smartphone penetration and affordable data prices in the country, the number of

“digital only consumers” of content is set to more than double to 4 million by 2020, a report by Federation of
Indian Chambers of Commerce and Industry (Ficci) and EY has said.

B. It noted that the proliferation of digital infrastructure will enable shifts in consumption patterns, which will be

based not as much on geographic, gender and age criteria but more on the ability to pay.

www.ibpsguide.com https://estore.ibpsguide.com Page 159 of 331


C. Tactical digital consumers—who consume both paid television (cable and DTH) and have at least one over the

top (OTT) subscription like Netflix, Hotstar or Amazon Prime Video or are driven by sachet pricing of content—
would provide high volume-lower value subscription base to content distributors.
(a) Only A (b) Only C (c) Only B and C (d) Only A and B (e) None of these.

489). A. US President Donald Trump will need to adopt an unfamiliar set of traits—patience, persistence, clear
goals and conditions—and be prepared to walk off when he meets North Korean leader Kim Jong Un, people who
have negotiated with Kim’s regime say.
B. Trump’s apparently impromptu decision to begin setting up talks contrasts with Kim’s situation—North Korea

has prepared deliberately for decades for a meeting with the sitting US president as a major step toward gaining

international legitimacy.
C. The White House said that Trump plans to meet with Kim within months, dispensing with decades of American

foreign policy by accepting a high-stakes invitation from the North Korean leader.
D. The summit, which the Trump administration hopes will lead to talks to wind down Kim’s nuclear weapons
program, could avert what has at times seemed to be an imminent war on the Korean Peninsula but the meeting
may also be a ploy by Kim to buy time to perfect his weapons and wriggle out of punishing economic sanctions.
(a) Only A (b) Only B (c) Only C (d) Only D (e) None of these.

490). A. Bitcoin has become high and dry over the past three months and the world is divided over the merit of
bitcoin as an asset given its volatility.
B. While the geeks are of the view that bitcoin is one of the disruptive technologies of the decade with its secure-

proof, decentralized mechanism of functioning, financial sceptics warn against holding it due to its volatility and
inadequate backing by regulators worldwide.
C. There have been attempts to predict the value of bitcoin earlier using various models and algorithms.

(a) Only A and C (b) Only B (c) Only B and C (d) Only A (e) None of these.

491). A. The political crisis in Maldives presents an opportunity for India to leverage its waning influence in the

island.
B. It makes strategic sense for New Delhi to engage with a nation which has, in the past few years, come from the

sway of China. Beijing has been persistently playing its political cards in an attempt to encircle India through its

‘string of pearls’ strategy.


C. The Prime Minister must show a greater appreciation of the unfolding events and reach out to our oceanic

neighbour.

www.ibpsguide.com https://estore.ibpsguide.com Page 160 of 331


D. However, it should be a guarded approach so as to not be seen as interference in Male’s domestic affairs.

(a) Only A (b) Only B (c) Only C (d) Only D (e) None of these.

492). A. India has been free of colonial rule for over 70 years and our Constitution protects the right to equality,

yet women have never really been free.


B. Our 'culture' and our 'society' treats women as second class citizens in independent India - an India in whose
struggle for independence our mothers and grandmothers played a substantial role.
C. While the battle for gender equality is far from being won, we have seen significant progress since then. Sati
has been banned, child marriage is illegal, property rights have been given to women, the domestic violence bill

has been passed, and women's literacy rate is higher.

(a) Only B (b) Only C (c) Only A and B (d) Only B and C (e) None of these.

493). A. The internet continues to create new challenges for parents, 1 in 3 of them reported being concerned
about their children’s digital habits, according to the Pew Research Center.
B. Kids are online, which means that parenting today involves knowing where they are going and what they are
doing there.
C. The best line of defense with any dangers lurking a child is the parent which means that they need to be where
their kids are.

(a) Only B (b) Only A and C (c) Only C (d) Only B and D (e) None of these.

494). A. Women make up only 14 percent of full professors in U.S. economics departments which is in stark

contrast to other social sciences where women typically receive over 60 percent of doctoral degrees, women in
economics receive only 35 percent of doctorates.
B. Women have made significant progress in certain STEM (science, technology, engineering and mathematics)

fields, traditionally dominated by men. But economics is not one of them.


C. Those advocating against gender parity presume that women economists need to be included when research

is being conducted and when policy is being debated because they bring a different perspective than men.

(a) Only B (b) Only A and C (c) Only A and B (d) Only C (e) None of these.

495). A. Equality of opportunity is interpreted narrowly. It essentially means that no one can wantonly, or merely

on the basis of caste, creed or language, deny a candidate his right to try and win a seat or an admission or a
public office.

www.ibpsguide.com https://estore.ibpsguide.com Page 161 of 331


B. But this is a very limited definition of opportunity. In reality opportunity is defined by an individual's worldview,

his access, social networks, parental influence and, of course, many of those things in turn are defined by his
economic position.
C. It is perhaps useful to deconstruct the simple and simplistic word opportunity in its various components. Surely

access to quality education is a prime component of opportunity.


(a) Only A and C (b) Only B (c) Only B and C (d) Only A (e) None of these.

496). A. French President Emmanuel Macron has indicated that France would not have a problem if the Narendra
Modi government shared some details of the Rafale deal with the Opposition provided the 'commercial

sensitiveness' of all the parties involved are protected.

B. Macron, who is on a four days visit to India, told the deal was a 'win-win' for both countries but added that
parties to commercial agreements are always keen that competitors don't get hold of the details.

C. The Congress, which accuses the Narendra Modi government of agreeing to a much higher price than the
UPA government's negotiated price of Rs. 526.1 crore per jet, had citied figures from the annual report of Rafale's
French manufacturer, Dassault Aviation, to support his claim.
(a) Only B (b) Only C (c) Only A and C (d) Only A and B (e) None of these.

497). A. Biplab Kumar Deb, who was sworn in as the chief minister of Tripura in the presence of Prime

Minister Narendra Modi and a host of CMs of BJP-ruled states, has a tough task ahead as Deb has no experience
of an administrator and his party, the BJP, is new to office in Tripura.
B. His predecessor, Manik Sarkar, had drawn appreciation even from non-CPM circles for his frugal lifestyle and

upright conduct in office, however, he had failed to address the problem of unemployment, for instance, and
aspirations among the youth, a failure the BJP tapped into in this election.
C. Tripura’s topography and geographical isolation have limited its ability to attract private investment and thereby

create well-paying industrial and service sector jobs.


(a) Only A (b) Only B and C (c) Only A and B (d) Only B (e) None of these.

498). A. Turbulence building over the Centre’s refusal to give Special Category Status (SCS) to Andhra Pradesh
is now threatening the cohesion of the NDA as TDP chief and Andhra Chief Minister, Chandrababu Naidu,

announced that his party was withdrawing its two ministers from the Narendra Modi government since it found the

Centre’s response to Andhra’s request “very hurtful and insulting”.

www.ibpsguide.com https://estore.ibpsguide.com Page 162 of 331


B. The SCS demand is a politically-sensitive issue that has its root in the bifurcation of united Andhra Pradesh in

2014. In lieu of accepting the bifurcation plan and to compensate for revenue losses, primarily due to Hyderabad
becoming the capital of the new state, Telangana, Andhra was promised SCS.
C. Until then, only Jammu and Kashmir, states in the Northeast and the Himalayan region has been accorded

SCS: Special status would ensure that 90 per cent of central funds for state projects routed through the National
Development Council under the Planning Commission is given as grants.
(a) Only C (b) Only A and C (c) Only B and C (d) Only B (e) None of these.

499). A. In Haryana, Congress workers and MLAs are taking to the streets in order to fry up pakoras, in a stir as

its perception that the BJP has outsourced employment generation to the people.

B. And Chief Minister Manohar Lal Khattar has bought some pakoras from them, in order to express his support
for the unemployed: perhaps he has a better sense of the moment than Amit Shah, who had originally

foregrounded the pakora.


C. While celebrating pakora-sellers can be read as an attempt to palm away the burden of job-creation to the
small entrepreneur, it is surprising that the choice of the pakora itself has not drawn politically correct criticism.
(a) Only B (b) Only A and C (c) Only B and C (d) Only A and B (e) None of these.

500). A. When Vladimir Putin was asked recently what historical event he would change if he had the power, he

said he would undo the collapse of the Soviet Union.


B. This was not a surprising answer, but it was a timely reminder of what motivates Kremlin policy.
C. Putinism embodies the feeling that Russia was robbed of its wealth and superpower status. The internal

failings and atrocities of the Soviet system are of little consequence in this account of history.
(a) Only A (b) Only C (c) Only A and C (d) Only A and B (e) None of these.

501). A. It would be silly to mourn the demise of NME, which is closing its print edition after 66 years, maintaining
only its painful digital existence.

B. This is not because popular music do not matter – that fight was over decades ago – nor even because the

magazine became so associated with one particular form.


C. But NME’s heyday was long past and, though the closure is bad news for staff, it is hard to imagine current

readers weeping.

D. As an ad-funded free title, no one thought it thrilling, essential or even surprising. The growing sense of profits
overruling passions didn’t help.

(a) Only A (b) Only B (c) Only C (d) Only D (e) None of these.

www.ibpsguide.com https://estore.ibpsguide.com Page 163 of 331


502). A. It is a moment that may not be as celebrated, or as headline-grabbing, as Frances McDormand’s Oscar
speech urging Hollywood A-listers to consider “inclusion riders” – contractual clauses insisting that certain
proportions of black and minority ethnic, disabled, women and LGBT workers be employed on their film projects.

B. But in its own, much less glitzy and high-profile way, Britain’s classical music world is making strides towards
unravelling a situation that has seen a centuries-long dominance of its ranks of musicians, conductors and
composers by one particular group: white man.
C. The power of unconscious bias – the unreflecting process by which racism and sexism are perpetuated even
by decent and fair-minded people – demands to be combated with something stronger than good intentions.

(a) Only A (b) Only B and C (c) Only A and C (d) Only B (e) None of these.

503). A. Boko Haram has long exploited the power of gender, not only abusing women but treating them as

pawns.
B. Hundreds more women and girls have been seized in smaller raids in recent years – as have large numbers of
men and boys – with scant attention paid.
C. What more dramatic rebuttal could there be to the state’s claims of victory than a carbon copy of its highest
profile failure.
(a) Only A and C (b) Only C (c) Only B (d) Only A and B (e) None of these.

504). A. Bumping on the household savings rate and nudging savers to park their surpluses in financial assets
have always been high on the agenda of Indian Finance Ministers.

B. Fully aware of this, different arms of the financial services industry — insurers, banks, intermediaries, mutual
funds — usually present a long laundry list through Budget demands. This year has been no exception.
C. But it is accommodating such piecemeal demands over the years that has led to such a complicated and

inconsistent smorgasbord of tax rules for investors. This does them more harm than good.
(a) Only A (b) Only A and C (c) Only A and B (d) Only b and C (e) None of these

505). A. Growing evidence of the link between poor sanitation practices and the spread of antibiotic resistance
underscore the necessity to scale up sewage treatment facilities and strictly regulate the sale and import of

antibiotics.

B. While efforts have been made to control the sale of unprescribed medicines and over-the-counter medication,
all such initiatives are bound to have limitations without large-scale awareness programmes and a multi-sectoral

approach covering both consumer and supplier.

www.ibpsguide.com https://estore.ibpsguide.com Page 164 of 331


C. The subject needs to be introduced to academic curricula and as TV spots, as was done in the polio

eradication programme.
(a) Only A and C (b) Only B (c) Only B and C (d) Only C (e) None of these.

506). A. With chicken-based fast foods becoming the staple of the youth and large segments of the working
population in our cities, one can well imagine the effect consumption of poultry farm-sourced chicken and eggs
must have on the health of our population, especially the most productive segment.
B. It is shocking that multinationals have no qualms in selling hazardous drugs labelled as the ‘last line of defence’
against disease and passing them off as ‘growth promoters to the developing countries in bags’.

C. How can one forget reports about developing countries having been used as an export destination for harmful

substances such as asbestos, DDT and toxic cattle dung from Europe?
(a) Only A and B (b) Only C (c) Only A and C (d) Only B (e) None of these.

507). A. In delivering his first State of the Union speech on Capitol Hill, President Donald Trump spoke of many
small victories that he chalked up to his administration’s record over the past year.
B. Yet the biggest surprise to many may have been the fact that they saw before them a Commander-in-Chief
who unwaveringly stuck to the script and eschewed his usual provocative style on social media.
C. While “Twitter Trump” has lashed out at Democrats on immigration reform and the federal government

shutdown earlier in January, “Teleprompter Trump” issued a generous call for bipartisanship in policymaking.
(a) Only A (b) Only C (c) Only A and B (d) Only B (e) None of these.

508). A. Sri Lanka’s local government elections, scheduled to be held on February 10, have elicited the interest of
a national election, and with good reason.
B. This is the first time the country will go on the polls in about three years since the President Maithripala

Sirisena–Prime Minister Ranil Wickremesinghe combine rose to power promising “good governance”, giving
voters a chance to say what they think of the performance of the government they elected to office.

C. Further, the two coalition partners in government — Mr. Sirisena’s-led Sri Lanka Freedom Party (SLFP) and

Mr. Wickremesinghe’s United National Party (UNP) — are contesting the island-wide local polls separately, with
their campaigns laying bare the deep fissures and insecurities by the coalition government.

(a) Only B (b) Only A and B (c) Only B and C (d) Only A and C (e) None of these.

509). A. Every riot today produces a set of staged narratives which are eerie to watch and strange to listen to.

www.ibpsguide.com https://estore.ibpsguide.com Page 165 of 331


B. A riot is no longer an act of production where the narrative focusses on causes and an act of consumption

where a variety of narratives create a quilt patch we call history.


C. The actual event is enacted in a limited space, while the narratives of the event spread out in oceanic circles
for consumption.

(a) Only A (b) Only B (c) Only A and B (d) Only B and C (e) None of these.

510). A. Western Europe, a group of developed economies and known for their compassion and humane nature,
has been liberal enough to allow in refugees.
B. However, ever since the inflow of refugees became a problem, there has been diplomatic wrangle even there

on absorbing the number of refugees.

C. Though India has allowed refugees in earlier, a change in policy now wouldn't be termed as unjustified.
(a) Only A (b) Only A and B (c) Only C (d) Only B and C (e) None of these.

ANSWERS:
481). Correct Answer is: (d)
Error in sentence A: The phrase 'for over three decades' suggests that we should use 'was' in place of 'is'
Error in sentence B: Replace 'designed' with 'designing'

482). Correct Answer is: (b)


Error in sentence B: Unanimous is pronounced as 'you-nanimous'. The rule has to do with the sound of the word
following "a" or "an," and not the letter. Hence, replace 'an' with 'a'.

483. Correct Answer is: (c)


Error in sentence B: Replace 'youth' with 'young'
Error in sentence C: The correct phrase is 'those above 65 years of age'

484. Correct Answer is: (a)


Error in sentence A: Replace 'hurt' with 'hurting'

485. Correct Answer is: (b)

Error in sentence A: Replace 'by' with 'at'. To be 'at the centre of something' means to be the main cause or
subject of something.

Error in sentence B: Since, the events mentioned have occurred in the past, replace 'have' with 'had'.

486. Correct Answer is: (d)


Error in sentence A: Replace 'high' with 'higher'

Error in sentence C: Replace 'by' with 'against'. The correct phrase is 'outcry against'.

www.ibpsguide.com https://estore.ibpsguide.com Page 166 of 331


487. Correct Answer is: (c)

Error in sentence C: Replace 'on' with 'in'


488. Correct Answer is: (a)
Error in sentence A: Replace 'prices' with 'pricing'

489. Correct Answer is: (a)


Error in sentence A: Replace 'off' with 'away'. To 'walk off' means to leave a place because you are angry or
unhappy about something while to 'walk away' means to casually or irresponsibly withdraw from a situation in
which one is involved or for which one is responsible.
490. Correct Answer is: (e)

All the sentences are grammatically correct.

491. Correct Answer is: (b)


Error in sentence B: Replace 'from' with 'under'

492. Correct Answer is: (c)


Error in sentence A: The correct phrase will be 'our Constitution protects our right to equality'.
Error in sentence B: Since two different subjects are provided (culture and society), plural verb 'treat' should be

used in place of singular verb 'treats'.


493. Correct Answer is: (b)
Error in sentence A: Replace 'them' with 'whom'
Error in sentence C: Replace 'with' with 'against'
494. Correct Answer is: (d)
Error in sentence C: Parity means 'equality'. The correct phrase will be '..advocating for gender parity....'.

495. Correct Answer is: (b)


Error in sentence B: Replace 'those' with 'these'
496. Correct Answer is: (d)

Error in sentence A: The presence of simple present tense form 'are' suggests that we should replace the past
tense form 'shared' with 'shares'.

Error in sentence B: Replace 'days' with 'day'

497. Correct Answer is: (a)


Error in sentence A: Replace 'of' with 'as'

498. Correct Answer is: (c)

Error in sentence B: Replace 'root' with 'roots'


Error in sentence C: The sentence is in present tense. So, replace 'then' with 'now'.

499. Correct Answer is: (b)

www.ibpsguide.com https://estore.ibpsguide.com Page 167 of 331


Error in sentence A: Replace 'as' with 'over'

Error in sentence C: The correct phrasal verb to use here is 'palm off' which means to try to get rid of something
by giving or selling it to someone under the pretense of it being something else.

500. Correct Answer is: (e)

All the sentences are grammatically correct.


501. Correct Answer is: (b)
Error in sentence B: Since the subject is singular (music), use singular verb - 'does' in place of plural verb - 'do'.
502. Correct Answer is: (d)
Error in sentence B: Since in the sentence, it clearly indicates 'one particular group', we must use 'men' in place of

'man'.

503. Correct Answer is: (c)


Error in sentence B: Replace 'more' with 'of'

504. Correct Answer is: (c)


Error in sentence A: The correct phrasal verb to use in the sentence is 'bumping up' which means to raise or
increase.

Error in sentence B: Replace 'through' with 'of'


505. Correct Answer is: (a)
Error in sentence A: The subject here is singular (growing evidence), so use singular verb 'underscores' in place
of plural verb 'underscore'.
Error in sentence C: The correct phrase is 'introduced in academic curricula'

506. Correct Answer is: (e)

All the sentences are grammatically correct.


507. Correct Answer is: (a)
Error in sentence A: Replace 'year' with 'years'

508. Correct Answer is: (c)


Error in sentence B: Replace 'on' with 'to'. the correct phrase is 'go to the polls'.

Error in sentence C: Replace 'by' with 'within'.

509. Correct Answer is: (b)


Error in sentence B: Replace 'and' with 'but'

510. Correct Answer is: (d)

Error in sentence B: Replace 'wrangle' with 'wrangling'


Error in sentence C: Replace 'wouldn't' with 'cannot'

www.ibpsguide.com https://estore.ibpsguide.com Page 168 of 331


Sentence Correction Type - 4

Directions (511-540): In the following questions, four set of sentences are given. Some of the sentences when
correctly sequenced form a coherent paragraph. Some of the sentences does not fit into the context of the

paragraph. Mark the out-of-context sentence(s) as your answer.

511). A. North Korea’s nuclear ambitions date back to the fallout of World War II, after the U.S. detonated nuclear
bombs over the Japanese cities of Hiroshima and Nagasaki.
B. These annual operations have traditionally caused consternation in Pyongyang.

C. The prospect of a thaw in relations between North and South Korea, which resume talks after two years, holds

out the hope of denuclearisation on the Peninsula.


D. Lending the move diplomatic heft is the U.S.’s consent to South Korean President Moon Jae-in’s proposal to

delay the controversial joint military exercises between the two allies.
(a) Only C (b) Only D (c) Only A (d) Only A and D (e) Only A and C

512). A. It's P2P structure has served as a template for any number of digital business.
B. Sean Parker was just 19 years old when he co-founded Napster in 1999.
C. The new Co-Impact initiative launched is meant to target precisely that kind of change.

D. The peer-to-peer (P2P) file-sharing network changed the shape of the global music industry, for better and for
worse.
(a) Only B (b) Only C (c) Only D (d) Only B and C (e) Only C and D

513). A. Any sign of exploitation of cheap labour would turn consumers against products in rich countries.
B. The WTO may also frown upon some tax concessions and subsidies.

C. Special economic zones failed to take off any large scale as they did not suit the political economy.
D. Resources were misdirected and revenues lost, considering that large companies set up shop in SEZs mainly

to milch tax breaks.

(a) Only A (b) Only B (c) Only C (d) Only D (e) None of the above.

514). A. They sprout superstition, involve nonsensical mumbo jumbo.

B. An impression that rituals are entirely redundant, optional extras at best, is a pervasive feature of modernist
consciousness that treats them as vestiges of a pre modern, archaic past, to be left behind as we become more

educated and rational.

www.ibpsguide.com https://estore.ibpsguide.com Page 169 of 331


C. All these formal acts and utterances are reiterated and performed publicly, theatrically, so that all relevant

people can participate in them.


D. This is partly because of the association of rituals with religion, but also because of the belief that they can’t
survive the test of reason — they are meaningless, empty of content, needlessly repetitive and time-consuming.

(a) Only A and B (b) Only B (c) Only C (d) Only B and C (e) Only D

515). A. The DMK had assumed that it would return to power if the Edappadi K. Palaniswami government fell.
B. The decision of Tamil superstar Rajinikanth to try his hand at politics has added a totally new dimension to the
Tamil Nadu political theatre, which has been in turmoil for more than a year now.

C. Given the experience of the DMK, Mr. Rajinikanth should know that if he avoids the dynastic shadow over his

politics, his family will be his personal asset; otherwise, it will become a political liability.
D. Except for a hiccup in 1988-90, Tamil Nadu politics has never been in such tumult and uncertainty since the

demise of Jayalalithaa in end 2016.


(a) Only A (b) Only D (c) Only C (d) Only A and C (e) Only B and C

516). A. Duolingo recognizes that humans are weird to enjoy, turning the stuff of life into bite-sized, recreational
competitions.
B. Duolingo also recognizes that the key to learning a new tongue is repetition.

C. The key to success of Duolingo, a language-learning app, lies in gamification.


D. So the app transforms language study into amusing diversion, replete with points, leaderboards, and video
game 'lives'.

(a) Only A (b) Only B (c) Only A and D (d) Only B and D (e) None of the above.

517). A. In other species, including some from Africa, fire promotes coppicing (generation of shoots).

B. Many acacia trees are adapted to fire, including most of those in Australia.
C. In some acacia species, fire stimulates germination of seeds.

D. In some acacias, the seeds are known to remain viable in the ground for upto sixty years.

(a) Only A (b) Only B (c) Only C (d) Only D (e) None of the above.

518). A. Among these materials are diamonds.

B. Rising so rapidly and from so deep a source, a kimberlite pipe brings up exotic materials from the depths of the
earth.

www.ibpsguide.com https://estore.ibpsguide.com Page 170 of 331


C. Such events have occurred at random through the history of the earth, and a kimberlite pipe could explode in

any number of places next year.


D. The source of a diamond is a kimberlite pipe, a form of diatreme (volcanic pipe) - a relatively small hole bored
through the crust of the earth by an expanding combination of carbon dioxide and water which rises from within

the earth's mantle and moves so fast driving magma to the surface that it breaks into the atmosphere at
supersonic speeds.
(a) Only A (b) Only B (c) Only C (d) Only D (e) None of the above

519). A. As NASA focuses considerable effort on a mission to send humans to Mars in the coming decades,

psychology researchers are looking at what types of personalities would work the best together on such a long

trip.
B. Typically, extroverts - who tend to be sociable, outgoing, energetic and assertive - are good to have to work on

teams because they speak up and engage in conversations about what needs to be done, which is good for
planning.
C. For example, extroverts tend to be talkative, but their gregarious nature may make them seem intrusive or
demanding of attention in confined and isolated environments over the long term, researchers say.
D. Now, a study finds that on long-term space missions - such as mission to Mars, which could take as long as
three years to complete a round trip - having an extrovert on board could have several disadvantages.

(a) Only B (b) Only B and D (c) Only C (d) Only A and C (e) Only D

520). A. Inside the temple itself there are no jostling tourists, no complaints about the heat or the hassles of travel,

no camera flashes going off every few seconds - just silence and solitude as we explore this beautiful ancient site.
B. At the temple of Ramses the Great at Abu Simbel, souvenir kiosks at the entrance are shuttered, and metal
turnstiles are still.

C. Tourism is a major contributor to Egypt's economy, yet the tourist sites we visit are all but deserted.
D. During normal times up to 3,000 foreign tourists would visit the temple on a busy day, but eight months after

the Egyptian revolution ousted President Hosni Mubarak from power, the number of visitor has plunged to about

150 a day.
(a) Only A (b) Only B (c) Only C (d) Only D (e) None of the above.

521). A. Regular, modern exercise, including authentic yoga, holds the key.
B. Changes in lifestyle and diet can prevent non-communicable diseases.

C. This is more than a personal choice.

www.ibpsguide.com https://estore.ibpsguide.com Page 171 of 331


D. So, dealing with non-communicable diseases needs novel thinking and innovative responses.

(a) Only A and D (b) Only C (c) Only A and C (d) Only D (e) Only B and C

522). A. The reluctance of the U.S. to embark on ‘new wars’, especially in Asia, does not, however, undermine its

geopolitical, geostrategic and geo-economic pre-eminence.


B. India would again need to be on its guard in 2018 as China consolidates its takeover of Gwadar (Pakistan) and
Hambantota (Sri Lanka) ports.
C. Much of the speculation about the extent of China’s rise is based on the common presumption that the U.S.
under President Donald Trump had surrendered its global leadership role.

D. It is not China’s rise, but the breakdown of the institution of the state, as is evident in Afghanistan and Syria,

that poses far more pressing problems for Asia.


(a) Only C (b) Only B (c) Only A (d) Only A and B (e) Only B and D

523). A. It is sad that INS Arihant, the indigenous and important platform within India’s nuclear triad, seems to
have suffered major damage on account of human error.
B. Proper maintenance by technically trained staff is as, if not more, important as the first stage as such
equipment has to always be in a fit and ready condition for deployment at short notice.C. The manufacturing and
the purchase of warships and submarines is just one step in a complex process.

D. It is hard to imagine the cost to the nation after this important asset, and which cost crores of rupees to
manufacture, is now lying crippled.
(a) Only A (b) Only B (c) Only C (d) Only D (e) None of the above

524). A. But because something is possible and potentially hazardous, doesn't mean it is worth worrying about.
B. This is a huge moral reason to work hard to prevent existential threats from becoming a reality.

C. If consciousness or intelligence are lost, it might mean that value itself becomes absent from the universe.
D. Human extinction means the loss of meaning generated by past generations, the lives of all future generations

(and there could be an astronomical number of future lives) and all the value they might have been able to create.

(a) Only A (b) Only B (c) Only C (d) Only D (e) None of the above

525). A. However, with a remarkable sense of conformity with the spoils system put in place by the Congress, the

ruling Alliance is also treating the gubernatorial office as retirement benefit for its veterans.

www.ibpsguide.com https://estore.ibpsguide.com Page 172 of 331


B. It could have been chosen eminent personalities outside the political realm in line with the recommendations of

the Sarkaria Commission on Centre-State relations and moved away from the beaten path of turning Raj Bhavans
into rehabilitations homes for over-the-hill politicians.
C. The new National Democratic Alliance government had a golden opportunity to demonstrate its democratic

crdentials while appointing new Governors.


D. In the country's multi-party polity, the role of Governors under the Constitution has been a sensitive aspect of
the Centre-State relations.
(a) Only A (b) Only B (c) Only C (d) Only D (e) None of the above

526). A. But that may mean little in a scenario where the political apparatus has been captured by finance capital,

which is increasingly the norm in democracies where unknown donors contribute astronomical sums to political
parties.

B. It has been pointed out that bitcoins, unlike a stock or a bond, are a purely speculative asset untethered to a
material basis of value.
C. While this is somewhat true, it doesn’t explain why bitcoins continue to remain attractive as a store of value.
D. A major reason seasoned speculators find bitcoins irresistible is its deflationary nature, which makes it
inflation-proof.
(a) Only A and C (b) Only B (c) Only D (d) Only C (e) Only A

527). A. A much-neglected pollutant in discussions about pollution is surface ozone (O3).


B. Natural levels of ozone in both the stratosphere and troposphere have a warming effect.

C. A recent study shows that O3 levels will continue to rise drastically, particularly in north India.
D. As a blanket of pollutants descends on large parts of north India, it is important to remember that killer
pollutants go beyond particulate matter.

(a) Only B and D (b) Only B (c) Only D (d) Only A and B (e) Only C

528). A. Even Saudi Arabia, Iran’s arch-enemy in the region, stayed unusually quiet.

B. Unlike the 2009 Green Movement, which was largely a product of the urban middle class youth in Tehran, the
recent unrest in Iran seems to reflect the economic grievances of the lower and working classes, alienated from

institutional politics and suffering heavily from the consequences of an unjust and unequal management of the

Iranian economy.
C. Obama administration considered it as an ‘opportunity for Iranian civic actors to enable and empower Iran’s

civil society space’.

www.ibpsguide.com https://estore.ibpsguide.com Page 173 of 331


D. As a result, these protests have been largely driven by disaffected young people in rural areas, towns and

small cities who seized a pretext to express their frustrations with economic woes that are caused by Iran’s
foreign policy, as the country has been largely involved in both the Syrian conflict and turmoil in Yemen.
(a) Only A (b) Only B and D (c) Only C (d) Only A and C (e) Only A and B

529). A. The broad mechanism is akin to China's own build-up of the BRI initiative.
B. The broader attempt is to institutionalize this into a structured trilateral format, and possibly a quadrilateral one
at a later stage.
C. China spent over a decade building capacities independently in its areas of interest.

D. For instance, it built a series of dual-use facilities across the Indian Ocean surrounding India which is popularly

referred to as its 'string of pearls' strategy.


(a) Only A (b) Only B (c) Only A and C (d) Only C (e) Only B and D

530). A. This seems to be a criticism that verges on curbing creative freedom.


B. The comments and retorts, criticism both constructive and mindless, are immediate.
C. Posting cartoons directly on social media is a different, and new kettle of fish.
D. The followers need not necessarily be kind and appreciative, it is a free-for-all situation, no holds barred.
(a) Only A (b) Only B (c) Only C (d) Only D (e) None of the above.

531). A. The centrality of MSPs in vote-bank politics is well-known, but the economics of it is not sufficiently
appreciated.

B. The MSP, the price at which the government offers to procure from farmers, is an economic policy tool which
requires technical acumen.
C. The levers in governments’ hands are import and export controls, buffer stocks management and minimum

support prices (MSPs).


D. Prime Minister Narendra Modi is under pressure to deliver on the 2014 poll promise of higher MSPs.

(a) Only A (b) Only B (c) Only C (d) Only D (e) None of the above.

532). A. It is a fact that transport undertakings are facing huge losses and any prolonged agitation will cripple

them further.

B. The State government, already riven by factionalism, is definitely unable to break the deadlock.
C. This is a clear case of harassing the common man who is largely dependent on public transport.

www.ibpsguide.com https://estore.ibpsguide.com Page 174 of 331


D. The bus strike in Tamil Nadu is taking a toll on the travelling public, even after the Madras High Court has

directed the employees to return to work.


(a) Only A (b) Only B (c) Only C (d) Only D (e) None of the above.

533). A. These could range from an incorrect portrayal of an ancestor (fictional or otherwise), of a politician (living
or dead), a religious figure (god forbid), a professional occupation (lawyers) and so on.
B. Against the backdrop of much censorship led by non-state actors, governments have done their bit to appoint
committees to suggest certification of films, if only to show how much they value democracy and the freedom of
expression.

C. There are some groups which don’t like a film because of some very genuine reasons.

D. On the religious front, Hindus, Muslims, Sikhs, and Christians have been equal rights censors when it comes to
defending their faith.

(a) Only A (b) Only B (c) Only C (d) Only D (e) None of the above

534). A. Patriotism can never be mixed with entertainment.


B.As citizens of this great land, it is our fundamental duty to respect the anthem.
C. It could have been made mandatory as it is not a burden being imposed on citizens.
D. But patriotism should be come from within the heart and one cannot be forced to express this.

(a) Only A (b) Only B (c) Only A and C (b) Only B and C (e) Only D

535). A. It is not only Indian tech firms whose employees get awarded H-1B visas, but it is to a great extent a visa

that Silicon Valley giants such as Microsoft, Intel, Amazon, Facebook and Qualcomm rely on for their staffing
needs.
B. The apparently endless cycles of heartache over the H-1B visa stem from a fundamental reality: that the visa

itself is designed to be a non-immigrant entry ticket into the U.S. economy, but over time it has metamorphosed
into a virtual pathway to permanent residency and citizenship, particularly in the case of Indian nationals.

C. Thus, there is a self-limiting dimension to any reform that purports to slash H-1B allocations, so that no

President or lawmaker would want to be seen as causing economic pain to the companies on whose coat-tails the
U.S.’s reputation as a global tech leader rides.

D. The most important reason for this is that most of these “speciality occupation” workers — primarily experts in

fields such as IT, finance, accounting, and STEM subjects — fill a real void in the U.S. labour force.
(a) Only A (b) Only b (c) Only C (d) Only D (e) None of the above.

www.ibpsguide.com https://estore.ibpsguide.com Page 175 of 331


536). A. Compared to modern American-style ice cream, traditional Italian ice cream (gelato) has less fat in the

base and less air churned into it during the freezing process.
B. Most bars have a fairly good selection, but for real choice go to a geleteria, where the range is a tribute to the
Italian imagination and flair for display.

C. Italian gelato is justifiably famous: a cono (cone) or a better-value coppa (cup) are indispensible accessories to
the evening passeggiata (stroll).
D. You'll sometimes have to go by the appearance rather than attempting to decipher their exotic names, many of
which don't mean much even to Italians; often the basics - chocolate, strawberry, vanilla - are best.
(a) Only A (b) Only B (c) Only C (d) Only D (e) None of the above

537). A. The Sports Authority of India (SAI) has, however, said that the androgen level can be brought down by
medical help and the athlete would still be able to compete in the female category in future.

B. We have also failed to nurture, protect and prepare the few talents that have emerged through the
inadequacies.
C. India might have saved itself a huge embarrassment by pulling out promising sprinter Dutee Chand from the
Glasgow Commonwealth Games athletics squad the last moment as she failed a gender-hormone test, but the
accident points to a serious malaise that hogs sports in our country.
D. That this incident proves that we have not evolved as a sporting nation despite being the second most

populous and the youngest is another matter.


(a) Only A (b) Only B (c) Only C (d) Only D (e) None of the above.

538). A. That its capital will be equally shared by the founding members was emphasized by India to prevent its
domination by China or any one of the members.
B. Appropriately, an Indian nominee will be the first President of the bank.

C. The setting up of the New Development Bank at the BRICS summit is a major victory for India, which had put
its full weight behind the move.

D. Significantly, the BRICS nations represent 40 percent of the world population and a fifth of the economy.

(a) Only A (b) Only B (c) Only C (d) Only D (e) None of the above.

539). A. To distinguish between toleration and cowardice or weakness of will, the agent must have some capacity

to enact his negative judgement.


B. Toleration is concerned with resisting the temptation to actively negate the things in question.

www.ibpsguide.com https://estore.ibpsguide.com Page 176 of 331


C. Toleration occurs when the agent could actively negate or destroy the person or object in question, but

chooses not to.


D. This is the case, for example, when tolerant governments consider groups who advocate violence,
discrimination and other tolerant practices.

(a) Only A (b) Only B (c) Only C (d) Only D (e) None of the above.

540). A. That the views of the Telengana government should have been sought before the state's boundaries are
altered is another point.
B. Polavaram - conceived over 50 years ago and grounded two decades ago - represents a few of the many

problems involved in the grand plan of interlinking rivers.

C. Smaller in size as compared to Tehri and Bhakra, Polavaram will go down in India's history as the largest land
submergence and displacement project.

D. Apart from 65,000 hectares of forestland set to be submerged, it involves displacement of 1.7 lakh tribals.
(a) Only A (b) Only B (c) Only C (d) Only d (e) None of the above.

ANSWERS:
511). Correct Answer is: (c)

After carefully reading the sentences, we find that when the sentences are ordered in the sequence of CDB, a
meaningful paragraph is formed. The main theme of the paragraph is the initiation of talks between North and
South Korea. The link between the second and the third sentence is the 'joint military exercise' which is referred to

in the third sentence as 'annual operations.' However, sentence A is talking about North Korea's nuclear ambitions
which is irrelevant with the context of the paragraph being formed.
512). Correct Answer is: (b)

Sentences when arranged in the sequence of BDA forms a meaningful paragraph and discusses about the P2P
file-sharing network co-founded by Sean Parker, while sentence C talks about the launch of Co-Impact initiative,

making no connection with the other sentences.

513). Correct Answer is: (e)


Going through the sentences we find that that the sentences when arranged in the sequence of CDAB forms a

coherent paragraph talking about the reasons behind the failed Special Economic Zones on any large scale.

514). Correct Answer is: (c)


Sentences when arranged in the sequence of BDA forms a coherent paragraph which mainly highlights why many

people don't perform rituals. Sentence B initiates the paragraph highlighting how people with 'modern

www.ibpsguide.com https://estore.ibpsguide.com Page 177 of 331


consciousness' treats rituals. Sentences D and A are the elaborating the reasons further. However, option C is

irrelevant to the theme of the paragraph.


515). Correct Answer is: (d)
The only two sentences that are interlinked are sentence B and sentence D. The latter part of sentence B speaks

about the 'turmoil' of politics in Tamil Nadu. Sentence D is a suitable continuation of sentence B. Sentences A
speaks about an individual topic which is not cannot be linked with any of the sentences. Sentence C also is
irrelevant to the central theme of the two interlinked sentences.
516). Correct Answer is: (b)
Sentence C is the first sentence, which introduces Duolingo and the reason for its success, 'gamification'.

Sentence A and sentence D elaborate on why and how 'gamification' works. So these three sentences are

connected in a proper sequence. Sentence B introduces a new point about Duolingo's methods, so it doesn't fit
into the sequence.

517). Correct Answer is: (d)


The sentences when arranged in the sequence of BCA forms a meaningful paragraph. Sentence B introduces the
idea of acacia trees being adapted to fire and is the first sentence. Sentences C and A, which gives examples of
this phenomenon, follow from sentence B. However, sentence D doesn't fit into the sequence as it is not an
example of the acacias' adaptation to fire.
518). Correct Answer is: (c)

At first glance, the main topic of these sentences may seem to be 'kimberlite pipes' - in which case sentence A
might seem to be the incorrect sentence. But on arranging these sentences in the correct sequence, one finds
that the topic is in fact 'sources of diamonds'. Sentence D introduces this source (kimberlite pipe) and explains

what it is; sentence B states that it brings up exotic materials from the earth; and sentence A states that among
these are diamonds. Sentence C goes off on a tangent about the explosions of kimberlite pipes, and doesn't talk
of diamonds at all.

519). Correct Answer is: (a)


Sentence A is clearly the introductory sentence, which states the subject - types of personalities on a mission to

Mars. Both sentences C and D state that extroverts would not be suitable for such a mission and can follow

sentence A in the sequence D and C. However, sentence B mentions the advantages of having extroverts
around.

510). Correct Answer is: (a)

All the sentences are about the lack of tourists in Egypt. Sentence C introduces this topic in general terms;
sentence B describes this situation at a specific tourist site; and sentence D explains the reason for the situation.

All three sentences are negative in tone : the author does not consider the lack of tourists to be a good thing. But

www.ibpsguide.com https://estore.ibpsguide.com Page 178 of 331


sentence A has a positive tone : the author seems to appreciate the silence and the lack of fellow tourists. So it

does not fit into the context.


521). Correct Answer is: (d)
Sentences in the sequence of BAC forms a coherent paragraph talking about the prevention of non-

communicable diseases while sentence D is about the requirement to deal with the non-communicable diseases,
making it unrelated to the other sentences.
522). Correct Answer is: (b)
Each sentence are interlinked to each other in the sequence of CAD. Sentence C explains how it is presumed that
China's rise is presumably due to the surrender of global leadership by U.S President Donald Trump. Sentence A

elaborates on sentence C by mentioning that this doesn't undermines 'the geopolitical, geostrategic and geo-

economic pre-eminence' of the U.S. Sentence D brings back to the point that one shouldn't be worried about
China's rise but the other political problems in Asia as faced by Afghanistan and Syria. Sentence B is totally

irrelevant to the other sentences mentioned here.


523). Correct Answer is: (e)
Sentences when arranged in the sequence of ADBC forms a coherent paragraph explaining how INS Arihant has
been left crippled after an accident and the lack of its maintenance.
524). Correct Answer is: (a)
Sentence D talks about the implications of human extinction and sentence C continues that point. Sentence B

concludes that it is thus necessary to prevent the threat of extinction from becoming a reality. Sentence A is in
contrast to the other sentences as it claims that a potentially hazardous threat is not worth worrying about.
525). Correct Answer is: (d)

Though sentence D is a decent opening sentence, it does not fit contextually with the remaining sentences as
they do not talk about the role of Governors. Sentence C is thus a better opening sentence than sentence D. 'It' in
sentence B refers to the new government and also gives an idea as to how the new government could make use

of the Sarkaria Commission recommendation. This should be followed by sentence A as it mentions what the new
ruling Alliance is doing instead.

526). Correct Answer is: (e)

Sentences when arranged in the sequence of BCD forms a coherent paragraph. Sentence B starts the paragraph
mentioning bitcoins to be 'a purely speculative asset' following sentence C which explains that despite this,

bitcoins are still attractive. Sentence D explains the 'reason' of why bitcoins are attractive and hence follows

sentence C. Sentence A is out of context with the other three sentences.


527). Correct Answer is: (b)

www.ibpsguide.com https://estore.ibpsguide.com Page 179 of 331


Sentences when arranged in the sequence of DAC forms a coherent paragraph about how the much-neglected

pollutant surface ozone (O3) will continue to rise. Sentence B is irrelevant to this main theme as it speaks about
natural levels of ozone and not surface ozone in particular.

528). Correct Answer is: (d)


Only sentences B and D can be interlinked to each other which mentions the recent crisis faced by Iran and how
protests are driven by disaffected young people in rural areas, towns and small cities. Both sentences A and C
are independent sentences which cannot be linked to B and D.
529). Correct Answer is: (b)

Sentences when arranged in the sequence of ACD forms a coherent paragraph. Sentence B is not a part of the

paragraph, the hint being the word 'trilateral' as there isn't any specific mention of the third country.
530). Correct Answer is: (a)

The coherent paragraph is the observation of an expert where he explains the ills of posting something directly on
social media without going through the processes of editorial judgement . Sentences when arranged in the order
of CBD forms a coherent paragraph. Sentence A is not a part of the coherent paragraph as it is an opinion of
another expert and is contracdictory to the theme of the paragraph as according to him cartoons need breathing
space to retain a sense of humour while delivering a political opinion.
531). Correct Answer is: (c)

The sentences when arranged in the sequence of DAB forms a coherent paragraph. Sentence D is the first
sentence highlighting the promise (to ensure higher MSPs) made by Modi during the 2014 election. Sentence A
follows sentence D. Sentence B follows sentence A. However, sentence C cannot be linked to any of the

sentences of the paragraph.


532). Correct Answer is: (e)
Sentences when arranged in the sequence of DBCA forms a coherent paragraph about the bus strikes in Tamil

Nadu and how it is a reason of worry for the common people.


533). Correct Answer is: (b)

The sentences when arranged in the sequence of CAD forms a coherent paragraph. The main theme of the

paragraph is how some groups or people take exception to a film. There is no indication anywhere about the
'censorship led by non-state actors' as mentioned in sentence B. Hence, this sentence is the odd one out.

534). Correct Answer is: (c)

Only sentences B and D can be interlinked with each other. Sentence A cannot be linked to the other two
sentence as it cannot comply with the main theme discussed in sentence B and sentence D. Sentence C is an

independent sentence and not relevant to the central idea.

www.ibpsguide.com https://estore.ibpsguide.com Page 180 of 331


535). Correct Answer is: (e)

The main theme of the paragraph is the H-1B visa. Sentences when arranged in the sequence of BDAC forms a
coherent paragraph.
536). Correct Answer is: (a)

Though all the sentences are about Italian ice cream (gelato), sentences B, C and D are written for the sake of
people who want to buy / eat it, while sentence A is written for the sake of someone who wants to make it or know
what it is made of. The correct sequence is CBD, with A being the incorrect sentence.
537). Correct Answer is: (a)
Sentence C is the first sentence in the paragraph as it reports the incident of sprinter Dutee Chand and points to

the problems faced by sports and sportspersons in this country. It is followed by sentence D as it mentions 'this

incident'. Sentence B continues in the same manner of reasoning and further highlights the failures of the system.
The tone in sentence A is more positive as compared to the rest of the sentences and talks about androgen levels

which are not mentioned in any other sentences.


538). Correct Answer is: (d)
Sentence C introduces the subject of the New Development Bank and India's role in setting it up. Sentence A
continues the topic of India's role in setting up this bank. Sentence B is the result of India's stand in this project.
Though sentence D talks about the BRICS nations, it is in no way related to the setting up of the bank.
539). Correct Answer is: (d)

Sentence B opens the paragraph by stating what toleration is concerned with. Sentence A discusses the condition
that needs to be met for something to be called toleration instead of cowardice or weakness. Sentence C clarifies
that condition. Sentence C mentions when toleration occurs, thus, they make a connected paragraph. Sentence D

is not an example of the point that the other three sentences are trying to make. An appropriate example would be
one in which a person has the ability to negate something but chooses not to.
540). Correct Answer is: (a)

Sentence B introduces the subject of Polavaram and mentions the problems involved. This will be followed by
sentence C and sentence D respectively, as sentence C mentions land submergence while sentence D talks

about the exact area of land that will be submerged. Sentence A talks about Telengana and does not mention

Polavaram.

Sentence Correction Type - 5

Directions (541-570): The following question consists of a sentence which is divided into three parts which
contains grammatical errors in one or more than one part of the sentence, as specified in bold in each part. If

www.ibpsguide.com https://estore.ibpsguide.com Page 181 of 331


there is an error in any part of the sentence, find the correct alternative to replace those parts from the three

options given below to make the sentence grammatically correct.

541). Bank nationalization was instrumental to expand (I)/ the footprint of Indian capital to the hinterland,

mobilizing savings for industry and infrastructure and (II)/ creating a large middle class for the lending to small
enterprises and big farmers.(III)
I. was instrumental in expanding

II. the footprints of Indian capital


III. through its lending
(a) Only II (b) Only I and II (c) Only I and III (d) Only III (e) No correction required.

542). Last month, one member of the six member monetary policy committee voted against a rate hike, another

gave up his call for rate cuts (I)/ while deputy governor in charge of monetary policy, Viral Acharya, also veered
more toward the hawkish camp; (II)/ still, the MPC voted to keep rates unchanged at its last meeting and flagged
the government’s decision to widen budget deficit targets as something that will have an inflationary impact.(III)

I. voted for a rate hike, another gave up his call


II. veered more toward the hawk camp
III. that must have an inflationary impact.
(a) Only I (b) Only III (c) Only I and II (d) Only II and III (e) No correction required.

543). The Supreme Court is likely to pronounce a verdict if (I)/ the Indian legal sector could be opened up for

practice by foreign lawyers and international law firms, (II)/ an issue that has been central to the debate about
globalization of the Indian legal sector.(III)

I. its verdict on whether


II. should be opened up for practice to
III. that had been central to the debate

(a) Only I (b) Only II (c) Only II and III (d) Only I and II (e) No correction required.

544). Banks are close to reaching an accord on resolving the tussle of payments due to them by Punjab National

Bank (PNB) against guarantees (I)/ the latter issued to jewellers Nirav Modi and Mehul Choksi as well as their

companies that are the subject of a Rs 13,000-crore fraud; (II)/ while PNB had agreed to honour claims due by
March-end, the payments will have conditions attached, said two senior officials with knowledge of the

development.(III)

www.ibpsguide.com https://estore.ibpsguide.com Page 182 of 331


I. on resolving the tussle over payments

II. that were the subject of a


III. PNB has agreed to honour claims
(a) Only III (b) Only I and II (c) Only I and III (d) Only I (e) No correction

required.

545). The government has brought the Pradhan Mantri Gramin Awas Yojana (PMAYG), which has a Rs 50,000
crore payout in this financial year, (II)/ under the direct benefit transfer (DBT) mechanism that will result in a DBT
of Rs 1.59 lakh crore in this fiscal, (II)/ greater than double that in the previous year and up 50% in just a

month.(III)

I. which has a Rs 50,000 crores payout in


II. mechanism which will result in a

III. more than double that in


(a) Only III (b) Only I and II (c) Only II (d) Only II and III (e) No correction required.

546). Businessmen once considered giants of the Saudi economy now wear ankle bracelets that track them,
princes who led military forces and appeared in glossy magazines are monitored by guards they do not command,
(II)/ families who flew on private jets cannot gain access to their bank accounts, even wives and children have

been forbidden to travel (II)/ following the Saudi government locking up hundreds of influential businessmen —
many of them members of the royal family — in the Riyadh Ritz-Carlto.(III)
I. now wear ankle bracelets to track them

II. cannot gain access inside their bank accounts


III. after the Saudi government locked up hundreds of
(a) Only I (b) Only III (c) Only I and III (d) All I, II and III (e) No correction required.

547). President Donald Trump had chosen what one expert calls "the blue ribbon option" when he assigned a

sensitive gun control proposal to a new panel on school safety, (I)/ part of a package the White House announced

in response to the school shooting in Parkland, Florida by putting Education Secretary Betsy DeVos in charge of
the panel (II)/ and left clues that a key proposal he's voiced support for — raising the purchase age for some

firearms — was now in doubt.(III)

I. Donald Trump had chosen what one expert calls "the blue ribbon option" when
II. in response towards the school shooting

III. proposal he's voiced support to

www.ibpsguide.com https://estore.ibpsguide.com Page 183 of 331


(a) Only I (b) Only II (c) Only III (d) Only I and II (e) No correction required.

548). In the states – largely confined from the Hindi heartland – where BJP garnered (I)/ nearly saturated seats,
there is an undeniable erosion of support as was evident in the recent assembly elections (II)/ in Gujarat where,

although BJP won, it faced a far more robust challenge from Congress.(III)
I. states – largely confined to the Hindi heartland
II. near saturation seats
III. it faced greater more robust challenge
(a) Only II (b) Only I and II (c) Only III (d) Only I and III (e) No correction required.

549). Persons booked under serious offences should be debarred from contesting elections (I)/ if charges have
been framed by a competent court, the offence is punishable (II)/ by imprisonment of at least five years and the

case is at least six months prior to the election , Chief Election Commissioner (CEC) OP Rawat said.(III)
I. booked for serious offences should be
II. if charges are framed by a competent court
III. at least five years or the case is at least six months
(a) Only I (b) Only II (c) Only III (d) All I, II and III (e) No correction required.

550). Three weeks ago, an U.S. agency sent the clearest signal that fossil fuels’ days are numbered; (I)/ true
enough, the carbon-burning economy has been declared to be in its death bed (II)/ umpteenth time ago but this
came with a time frame related to the ultimate killer: the battery. (III)

I. an U.S. agency sent the clearest signal


II. to be on its death bed
III. umpteenth times before but this

(a) Only I (b) Only II (c) Only III (d) All I, II and III (e) No correction required.

551). India’s monsoon rains are expected to be slightly below normal this year, (I)/ while parts of Australia’s

eastern grain belt could be drier as an El Nino weather pattern (II)/ may develop in the second half of 2018, a
private US-based weather forecaster said.(III)

I. were expected to be slightly below normal


II. belt could be dry as an
III. private US-based weather forecast

(a) Only II and III (b) Only I (c) Only I and II (d) Only III (e) No correction required.

www.ibpsguide.com https://estore.ibpsguide.com Page 184 of 331


552). The harsh reality as far as health care and insurance coverage are concerned (I)/ is that when the same
principles were enforced in the United States (II)/ in the form of Obamacare, premium costs shot up significantly,
but the quality of health care deteriorated.(III)

I. health care and insurance coverage is concerned


II. were enforced by the United States
III. shot up significantly, and the quality of health care
(a) Only II and III (b) Only I and II (c) Only I and III (d) Only I (e) No correction required.

553). Many architects who have been awarded the Pritzker are best known for buildings like the Lloyds’

headquarters, (I)/ vast museums like the Tate Modern, unforgettable set-pieces such as the Sydney Opera House
(II)/ but, last week, the Pritzker jury gave the 2018 prize to the Indian architect Balkrishna Doshi, who is best

known for very different designs.(III)


I. Most architects who have been awarded
II. unforgettably set-pieces such as the
III. who has been best known for very different designs.
(a) Only I (b) Only II and III (c) Only III (d) Only I and II (e) No correction required.

554). Ever since Abdulla Yameen Abdul Gayoom became the President of the island nation in 2013, (I)/ the
country has grown close to China, and has consistently used the oldest trick of the playbook (II)/ of small states:
playing big neighbours against each other, to get what it wants.(III)

I. of an island nation in 2013


II. has grown closer to China, and has consistently used the oldest trick in the playbook
III. playing big neighbours against each other

(a) Only III (b) Only II (c) Only II and III (d) Only I and II (e) No correction required.

555). For much of the world is busy worrying about losing jobs to automation in the future, (I)/ what has crept past

for over a decade is that automated systems (to most, Artificial Intelligence, or AI) already play a major role (II)/ in
whether or not -- and how -- we get the jobs we still have, and they are used by a steady growing number of

Indian and multinational companies.(III)

I. While much of the world is busy worrying about losing jobs


II. which has crept past for over a decade is that

III. and these are used by a steadily growing number

www.ibpsguide.com https://estore.ibpsguide.com Page 185 of 331


(a) Only II (b) Only I and II (c) Only I and III (d) Only III (e) No correction required.

556). In line with New Delhi’s policy of trying to build capacity of Palestine, (I)/ India signed six agreements worth
around $50 million with the Palestinian Authority that include setting up of a super speciality hospital in Beit Sahur,

(II)/ a centre for empowering women, procuring of equipments and machinery for the National Printing Press and
significant investment in the education section.(III)
I. In line of New Delhi’s policy of trying
II. India have signed six agreement worth
III. procurement of equipment and machinery

(a) Only I and II (b) Only I (c) Only II and III (d) Only III (e) No correction required.

557). Laughter or “a cackle” is an expression of freedom (I)/ and defending laughter that was meant to insult

someone and create hindrances, (II)/ especially in the country’s supreme legislative body, is stretching things a bit
too far.(III)
I. an expression for freedom
II. but defending laughter that was
III. especially by the country’s supreme legislative body
(a) Only I (b) Only II (c) Only III (d) All I, II and III (e) No correction required.

558). By a fifth of the world’s disease burden, a growing incidence of non-communicable diseases (I)/ such as
diabetes, and poor financial arrangements to pay for caring, (II)/ India brings on the rear among the BRICS

countries in health sector performance.(III)


I. With a fifth of the world’s disease burden

II. arrangements to pay for care


III. brings up the rear among the BRICS countries
(a) Only I (b) Only II and III (c) Only III (d) All I, II and III (e) No correction required.

559). The GST subsumes the multiple Central, State and local taxes and cesses levied on goods and services,
(I)/ unifies the country to a single market, thereby making it easier (II)/ to do business and ensure tax compliance

which will attract investors through more efficientive mop up revenues for the exchequer.(III)

I. subsumes in the multiple Central, State and local taxes


II. unifying the country into a single market

III. and more efficiently mop up revenues

www.ibpsguide.com https://estore.ibpsguide.com Page 186 of 331


(a) Only I and II (b) Only III (c) Only II and III (d) Only I and III (e) No correction required.

560). The Indian Space Research Organisation boosted its reputation further when it successfully launched (I)/ a
record 104 satellites in one mission from Sriharikota by relying on its workhorse Polar Satellite Launch Vehicle

rocket (II)/ but ISRO views the launch not as a mission to set up a world record but as an opportunity to make full
use of the capacity of the launch vehicle.(III)
I. its reputation far when it

II. by relying on the workhorse Polar Satellite Launch Vehicle rocket


III. not as a mission to set a world record
(a) Only III (b) Only I and II (c) Only II and III (d) Only I (e) No correction required.

561). Called to lay listed papers, Minister of State for Finance P Radhakrishnan (I)/ began to say "I beg to lay...." ,

Naidu immediately (II)/ quipped that the minister should just use the term "I rise" to lay the papers and not "beg
to".(III)
I. Calling to lay listed papers
II. began by saying "I beg to lay
III. quipped that the minister can just use
(a) Only II and III (b) Only II (c) Only I and III (d) Only I and II (e) No correction required.

562). Pressure mounted on Japan's Prime Minister Shinzo Abe (I)/ over a ballooning cronyism and cover-up
scandal, (II)/ as a new poll showed his support at the lowest level since his re-election in October.(III)

I. Pressure mounted over Japan's Prime Minister


II. due to a ballooning cronyism
III. as new polls show his support to the lowest level

(a) Only I (b) Only II (c) Only III (d) All I, II and III (e) No correction required.

563). The farmer's march which began a week ago from Nashik (I)/ and culminated in Mumbai was called off on

Monday evening after the state government gave specific commitment on two of the protesters' nine demands (II)/
and assured in writing that the rest will be looked onto by committees it would constitute.(III)

I. farmers' march which began a week ago


II. after the state government gave specific commitments
III. that the rest would be looked into

(a) Only I and III (b) Only II (c) Only II and III (d) All I, II and III (e) No correction required.

www.ibpsguide.com https://estore.ibpsguide.com Page 187 of 331


564). The tri-junction stretch of the boundary at Sikkim, though contested, (I)/ has witnessed a fewer tensions
than the western sector of the India-China boundary (II)/ even as India and Bhutan have carried on separate

negotiations with China.(III)


I. stretch of the boundary along Sikkim
II. witnessed few fewer tensions than the
III. even as India and Bhutan has carried on

(a) Only I and III (b) Only II (c) Only I (d) Only I and II (e) No correction required.

565). From the last decade there has been a flurry of interest –and not a little incredulity –(I)/ about claims, often

made by companies backed by billionaires (II)/ and run by bold physicists, that market-ready fusion reactors were
just around the corner.(III)
I. For the last decade there has been a flurry of interest
II. in claims, often made
III. or run by bold physicists
(a) Only I (b) Only II and III (c) Only III (d) Only I and II (e) No correction required.

566). The tragic – genuinely tragic – death of Kim Briggs, a pedestrian in London who was hit by a cyclist as she

crossed a busy road, (I)/ and died of her injuries, has now led to the proposed introduction of (II)/ an offence of
causing death by dangerous cycling, to match the existing offence of causing death by dangerous driving.(III)
I. as she was crossing a busy road

II. and died because of her injuries


III. an offence that causes death by dangerous cycling
(a) Only III (b) Only I and II (c) Only II (d) Only I and III (e) No correction required.

567). World Book Day coincided this year with the release of two surveys, one showing that fewer than half the

parents in the UK (I)/ read with their children daily, and another revealing that household spending on digital

entertainment (II)/ had for the first time surpassed that of the printed word.(III)
I. coincided this year on the release of two surveys

II. read to their children daily


III. first time surpassed that on the printed word
(a) Only II (b) Only III (c) Only I and III (d) Only II and III (e) No correction required.

www.ibpsguide.com https://estore.ibpsguide.com Page 188 of 331


568). A blast of winter in Britain and along much of northern Europe is unpleasant but (I)/ perfectly survivable and

affects less people outside the region, but a period of unseasonal warming (II)/ in the Arctic could be the harbinger
of something that changes the whole world for the worse.(III)

I. and across much of northern Europe is unpleasant


II. affects few people outside the region
III. changes the whole world to the worst
(a) Only II and III (b) Only I (c) Only II (d) Only I and II (e) No correction required.

569). For extending an olive branch to the Taliban at an international conference (I)/ in Afghanistan’s capital

Kabul, President Ashraf Ghani has boosted (II)/ the prospects for peace in his battle-scarred country.(III)

I. In extending an olive branch to


II. had boosted

III. in his battle-scarring country


(a) Only I (b) Only II and III (c) Only II (d) Only I and II (e) No correction required.

570). To keep alive its rich culture Iceland has built linguistic defences (I)/ to prevent erosion from outside
influence yet the technological revolution (II)/ of recent years has seen Iceland submerged into a rising tide of
English.(III)

I. keep alive the rich culture


II. for preventing erosion from outside influence
III. submerged by a rising tide of

(a) Only I and II (b) Only II and III (c) Only III (d) Only II (e) No correction required.

ANSWERS:

541). Correct Answer is: (c)


The highlighted phrases of part I and III of the sentence are incorrect. The correct phrase for part I is 'was

instrumental in expanding' and for part III is 'through its lending'. However, the highlighted phrase of part II of the

sentence is correct and needs no replacement.


542). Correct Answer is: (a)

There is a contrast in the sentence: the highlighted phrase of part I of the sentence is incorrect and should be

replaced by the phrase 'voted for a rate hike, another gave up his call'. The highlighted phrases of part II and III of
the sentence are correct and need no replacement.

543). Correct Answer is: (d)

www.ibpsguide.com https://estore.ibpsguide.com Page 189 of 331


The highlighted phrases in part I and II of the sentence are incorrect. The correct phrase of part I is 'its verdict on

whether' and of part II is 'should be opened up for practice to' respectively. The highlighted phrase of part III of the
sentence is correct and needs no replacement.

544). Correct Answer is: (c)

The highlighted phrases in part I and III of the sentence are incorrect. The correct preposition to follow tussle is
'over' and not 'of'. The sentence is in simple present tense, so in part III of the sentence 'had' should be replaced
by 'has'. Also, since the sentence is in simple present tense, the tense 'are' present in part II of the sentence is of

the correct form and needs no replacement.


545). Correct Answer is: (d)

When quoting sum (or any currency values) in written words, pluralization of units (thousands , hundreds) is not

acceptable, and hence the highlighted phrase in part I of the sentence is correct and needs no replacement.
However, the highlighted phrases of part II and III of the sentence are incorrect. The correct replacement of the

highlighted phrase of part II is 'mechanism which will result in a' and part III is 'more than double that in'
respectively.
546). Correct Answer is: (b)
The only highlighted phrase which is incorrect is present in part III of the sentence. The correct replacement is '
after the Saudi government locked up hundreds of'. The other highlighted phrases are correct and need no
replacement.

547). Correct Answer is: (a)


The entire sentence is in simple present tense, hence the usage of 'had chosen' in part I of the sentence is
incorrect. Replace 'had chosen' with 'chosen' to make the sentence grammatically correct.

548). Correct Answer is: (b)


The correct phrase in part I of the sentence is 'confined to' which means to enclose within bounds; limit or restrict.
And the correct replacement of the highlighted phrase of part III of the sentence is 'near saturation seats'. The

highlighted phrase of part III of the sentence is correct and needs no replacement.
549). Correct Answer is: (a)

The only incorrect phrase is present in part I of the sentence. Replace 'under' with 'for' to make the sentence

grammatically correct.
550). Correct Answer is: (d)

U.S. is pronounced as 'you-yess' so the correct article to use is 'a' and not 'an'. In part II of the sentence, the

correct phrase is 'on its death bed' which means close to death. The correct phrase in part III of the sentence is
'umpteenth times before but this'.

551). Correct Answer is: (e)

www.ibpsguide.com https://estore.ibpsguide.com Page 190 of 331


The sentence is grammatically correct.

552). Correct Answer is: (c)


In part I of the sentence, the subject is singular - 'harsh reality' so replace the plural verb 'are' with singular verb
'is'. In part III of the sentence, there is no contrast so the use of 'but' is not justified. Replace 'but' with 'and'. The

highlighted phrase in part II of the sentence is correct and needs no replacement.


553). Correct Answer is: (a)
The only error is present in part I of the sentence. The correct replacement of the highlighted phrase is 'Most

architects who have been awarded'.


554). Correct Answer is: (b)

The only error is present in part II of the sentence. The correct replacement of the highlighted phrase is 'has

grown closer to China, and has consistently used the oldest trick in the playbook'.
555). Correct Answer is: (c)

The highlighted phrases in part I and III of the sentence is incorrect. The correct replacement for part I is 'While
much of the world is busy worrying about losing jobs' and for part III is 'and these are used by a steadily growing
number' respectively. The highlighted phrase of part II of the sentence is correct and needs no replacement.
556). Correct Answer is: (d)
The only error in present in part III of the sentence. Equipments (plural) is not parallel with machinery (singular).
The correct phrase therefore is 'procurement of equipment and machinery'.

557). Correct Answer is: (b)


The only error in present in part II of the sentence. The correct replacement of the highlighted phrase is 'but
defending laughter that was'. There is a contrast in the sentence so use 'but' in place of 'and'.

558). Correct Answer is: (d)


The correct phrase to use in part I of the sentence is 'With a fifth of the world’s disease burden'. The correct
phrase to use in part II of the sentence is ' arrangements to pay for care'. The correct idiom to use in part III of the

sentence is 'brings up the rear' which means 'to be the last in a line or group'.
559). Correct Answer is: (c)

The highlighted phrases of part II and III of the sentence are incorrect. The correct replacement for part II is

'unifying the country into a single market ' and for part III is 'and more efficiently mop up revenues '.
560). Correct Answer is: (a)

In part III of the sentence, the correct phrase is 'set a world record' and not 'set up a world record'. The other

highlighted phrases of part I and II of the sentence are correct and do not need any replacement.
561). Correct Answer is: (b)

www.ibpsguide.com https://estore.ibpsguide.com Page 191 of 331


There is an error only in part II of the sentence. The correct replacement of the highlighted phrase is 'began by

saying "I beg to lay'.


562). Correct Answer is: (e)
The sentence is grammatically correct.

563). Correct Answer is: (d)


It is evident that the sentence is talking about more than one farmer (in the latter part of the sentence it is

mentioned - two of the protesters'). So the correct phrase in part I of the sentence is 'farmers' march which began
a week ago'. In part II, the word following specific must be plural. So the correct phrase in part II of the sentence is
'after the state government gave specific commitments'. The correct phrase in part III of the sentence is 'that the

rest would be looked into'.

564). Correct Answer is: (b)


Only the highlighted phrase of part II of the sentence is incorrect. "tensions" is plural so it cannot be preceded by
'a'. Therefore the correct phrase is 'witnessed few fewer tensions than the'.

565). Correct Answer is: (a)


There is an error only in part I of the sentence. The correct phrase will be 'For the last decade there has been a
flurry of interest'.
566). Correct Answer is: (e)
The sentence is grammatically correct.

567). Correct Answer is: (d)


The phrases in part II and III of the sentence are incorrect. The correct phrase in part II will be 'read to their
children daily' and that of part III will be 'first time surpassed that on the printed word '.

568). Correct Answer is: (d)


The phrases in part I and II of the sentence are incorrect. The correct phrase in part I will be ' and across much of
northern Europe is unpleasant' and that of part II will be 'affects few people outside the region'.

569). Correct Answer is: (a)


There is an error only in part I of the sentence. The correct phrase will be 'In extending an olive branch to'.

570). Correct Answer is: (c)

There is an error only in part III of the sentence. The correct phrase will be 'submerged by a rising tide of'.

www.ibpsguide.com https://estore.ibpsguide.com Page 192 of 331


Jumbled Sentence – Type – 1

Directions (571-600): Sentences given in each question, when properly sequenced form coherent paragraph.

Each sentence is labelled with a letter. Choose the most logical order of sentences amongst the five choices
given to construct the paragraph.
571). A. With the relevant measure of price gains, the Consumer Price Index, reflecting an acceleration in inflation
for a sixth straight month in December, and that at the fastest pace in 17 months, the bank’s rate-setting panel
must have had little difficulty in choosing to remain on hold.
B. The RBI’s nominee, Michael Debabrata Patra, in fact voted to head off incipient price pressures by raising the

policy rate by 25 basis points.

C. This was probably best exemplified by the reversal in stance of the six-member panel’s hitherto most dovish
member, Ravindra H. Dholakia, to vote to stand pat on interest rates.

D. The Reserve Bank of India’s decision to keep the repo rate unchanged was no surprise given the focus with
which the Monetary Policy Committee has approached its mandate: of keeping inflation in check.
E. This the MPC did while retaining a ‘neutral stance’, which gives it the flexibility to change gears in either
direction.
(a) DABCE (b) ACBED (c) DACEB (d) ABCED (e) None of these.

572). A. Private investments, the principle engine of growth, are out of steam.
B. It is now well recognised that there is an investment slowdown in India, which is delaying a full-blooded
recovery in the economy.

C. The fall is so severe that it has more than offset the government’s macroeconomic stimulus of increased public
investments.
D. The Survey recommends urgent prioritisation of investment revival to arrest more lasting growth impacts, with

policy focus on both big and small companies, creating a conducive environment for the smaller industries to
prosper and invest, with their ‘animal spirits’ conjured back.

E. The slowdown started five years ago, and is, as Economic Survey 2018 notes, the most severe in India’s

history.
(a) BACED (b) BEDCA (c) ACBED (d) BECDA (e) None of these.

573). A. But its finale, in mid-ocean, was a rescue operation that had a thorny side to it.
B. Among this motley group of seven hostages was a Maldivian cabinet minister and his mother-in-law.

www.ibpsguide.com https://estore.ibpsguide.com Page 193 of 331


C. ‘Operation Cactus’, the code name for India’s military intervention in the Maldives in 1988, following an

attempted coup d’état against the government of President Maumoon Abdul Gayoom and his request for help,
was spontaneous and swift.
D. On learning about the swift landing of an Indian Army parachute brigade at Hulule airport adjacent to the island

capital of Male, the mercenaries of the Sri Lankan rebel group, the People’s Liberation Organisation of Tamil
Eelam (PLOTE), ran for cover after randomly grabbing people from the mainland and holding them hostage on
board a hijacked merchant vessel, MV Progress Light, that had been anchored in Male harbour.
E. As the hijacked ship moved out of Male harbour, the Indian rescue plan fell into place equally swiftly.
(a) CDBAE (b) CADBE (c) CDEAB (d) CBEAD (e) None of these.

574). A. The UK has been unlawfully breaching nitrogen dioxide limits since 2010.
B. Finally a minister is committing to a clean air strategy that restricts diesel use “to ensure our air is properly

breathable”.
C. So says the environment secretary Michael Gove.
D. The government has been taken to court and lost three times.
E. Britain needs to do more to clean up its dirty air as it is a “major public health scandal”.
(a) EABDC (b) EBADC (c) ECBAD (d) ECADB (e) None of these.

575). A. Litigation in countless cases, the overhang of a 1990s arms deal and actions that undermined judicial
investigations have marred Mr. Zuma’s decade-long presidency.
B. Calls for the anti-apartheid veteran’s exit acquired momentum after South African Deputy President Cyril

Ramaphosa was elected leader of the African National Congress in December.


C. But the controversy that has come to define his tenure is the questionable access an immigrant Indian
business family, the Guptas, gained with ANC apparatchiks and state institutions, a nexus widely dubbed as ‘state

capture’.
D. With Jacob Zuma appearing to be finally willing to resign as President of South Africa, a protracted power

struggle could soon draw to a close.

E. The financial dealings of the Guptas and their interface with the government in South Africa have tarnished the
reputation of top global accountancy and public relations firms.

(a) BADCE (b) DBCEA (c) DBACE (d) BACED (e) None of these.

576). A. Stephen Hawking was a brilliant, complex man and scientist.

B. Diagnosed at 21 with amyotrophic lateral sclerosis, he had been expected to live a few more years.

www.ibpsguide.com https://estore.ibpsguide.com Page 194 of 331


C. Hawking lasted another 55.

D. From his wheelchair, Hawking’s mind roamed the multiverses.


E. He made his name as a young Cambridge cosmologist with breakthroughs as awesome as anything religion
offers: proving that big bang theory must hold true and elucidating the link between gravity and quantum

mechanics.
(a) ABCDE (b) ACDBE (c) ABDCE (d) ABCED (e) None of these.

577). A. The actions of soldiers against the Rohingya in Myanmar bear the “hallmarks of genocide”, said a UN
Special Envoy on Human Rights.

B. Yet India, which has a proud, if patchy, history of helping resolve crises in the subcontinent, has done little

more than send basic aid including 500 tonnes of rice, pulses, cooking oil, tea, noodles, mosquito nets, and
biscuits.

C. The UN High Commissioner for Human Rights, Zeid Ra’ad Al Hussein, called it a “textbook example of ethnic
cleansing.”
D. Other than inking an “agreement” with Naypyidaw that normalcy and development would be promoted in the
restive Rakhine state, New Delhi’s only public position has been to announce that it intends to send the nearly
40,000 Rohingya in India back to Myanmar.
E. Since late last year, the crisis facing the Rohingya, Myanmar’s predominantly Muslim minority group, has

spiralled out of control and sent over 650,000 of the community fleeing to neighbouring Bangladesh to escape
violent attacks in Rakhine state on the border.
(a) BCDEA (b) EACBD (c) EABDC (d) EABCD (e) None of these.

578). A. From the same Kennedy Space Center launch pad that saw Apollo rockets soar towards the moon, a
new launcher, built by Elon Musk’s SpaceX company and currently the most powerful in the world, made its

dramatic maiden flight.


B. Hopes that the great pioneering days of US space flight, in the 1960s and 1970s, would soon return were

suddenly rekindled.

C. The sight of the giant Falcon Heavy rocket roaring into space after its launch last week will have brought, if
nothing else, a nostalgic tear to the eyes of many experienced space hands who had gathered to witness the

blast-off.

D. Musk had made it clear that Falcon Heavy’s launch should be a fun event and there was certainly an irreverent
aspect to the aiming of a sports car at a distant planet.

www.ibpsguide.com https://estore.ibpsguide.com Page 195 of 331


E. The fact that Falcon Heavy’s cargo was ferrying an electric sports car, built by a different Musk

company, Tesla, to Mars only added spice to an already extravagant event.


(a) CABED (b) CAEDB (c) CDEAB (d) CADEB (e) None of these.

579). A. The United Nations warns of unprecedented levels of suffering in a country that has already witnessed so
many crimes and such desperation.
B. After seven years of carnage, at the cost of half a million lives, the violence in Syria is not dwindling but
multiplying.
C. The mighty pursue power, territory and resources, while civilians pay in blood.

D. The IS threat is not over, despite high-profile captures.

E. The disintegration of Islamic State’s self-proclaimed caliphate has thrown these overlapping wars into sharper
relief.

(a) BCEDA (b) BCAED (c) EDBCA (d) ABCED (e) None of these.

580). A. Privacy is necessary for human society to function.


B. The problem is not that the information exists but that it reaches the wrong people.
C. Information on the internet could bring great benefits to society, and to individuals, when huge datasets can be
refined to yield information otherwise unavailable.

D. But once the information is gathered, a precautionary principle has to apply.


E. It is too much of a stretch to agree with John Perry Barlow, the internet rights pioneer who died this week,
when he quipped that “relying on the government to protect your privacy is like asking a peeping tom to install

your window blinds”; but it does not help when it appears that everything the public sector does with the huge
datasets it has will be overseen by the minister for fun.
(a) ABDEC (b) BACDE (c) BCDEA (d) ACDEB (e) None of these.

581). A. This romantic view, desiring the integration of all medical systems, is naive and unrealistic.

B. Would it not be ideal if traditional and scientific medical systems were integrated into one system of curative

medicine?
C. If the physician knew all systems, the patient could be treated with the best, instead of the patient choosing a

particular system.

D. All medical systems strive for one purpose — help the ill get well.
E. Traditional and scientific medical systems cannot be integrated.

(a) EADBC (b) DBCAE (c) DBAEC (d) CAEDB (e) None of these.

www.ibpsguide.com https://estore.ibpsguide.com Page 196 of 331


582). A. Alphabet has many subsidiaries (though only Google makes substantial profits).
B. The lucky winner this time is Alphabet, the holding company of Google.
C. The city of Toronto, is trying to solve its short-term problems by handing responsibility for part of its

infrastructure over to a giant private company.


D. Sensors and cameras everywhere will record the physical world – pollution levels, traffic flow, weather and so
on – and of course the behaviour of its human inhabitants.
E. One of them, Sidewalk Labs, has signed an agreement in principle to develop 800 acres of prime waterfront
land in Toronto as a hypermodern wired city where the roads and the buildings themselves will become as flexible

and responsive to their users’ whims as if they were made of pure software.

(a) CBEDA (b) ABCED (c) CBAED (d) CBDEA (e) None of these.

583). A. Now its rewards structure for 184 on-screen news stars has been described as “secretive” and
“unstructured” in a second report from PricewaterhouseCoopers.
B. In July came the revelation that two-thirds of its highest paid on-screen talent is male.
C. “Better than average”, blustered the corporation, but it has settled 115 unfair pay claims, and there are as
many outstanding.
D. The BBC is in a hole and it is time it stopped digging.

E. In October, its cumbersome bureaucratic 10-level staff pay grading system was found to be shot full of holes
and a 9.6% average gap between men and women’s earnings was exposed.
(a) EBDAC (b) DEBCA (c) DCABE (d) DBECA (e) None of these.

584). A. This extraordinary achievement was distinguished by its apparent ordinariness as Mr Kamprad himself
was famously mean with his personal expenses, at least in public, flying economy class and taking the tube to

visit his shops.


B. Ingvar Kamprad started his career as a schoolboy selling matches that he bought cheap in Stockholm and sold

round his home in the forests of southern Sweden.

C. It was a remarkable feat of self-assembly, which has touched the lives of millions of people.
D. When he died he was the eighth richest man in the world, and Ikea, the business empire he left behind, had

graduated from matchsticks to using 1% of all the timber felled in the world.

E. At one stage it was calculated that 10% of the children born in Europe had been conceived in an Ikea bed.
(a) BDAEC (b) BDEAC (c) BDCEA (d) BCDEA (e) None of these.

www.ibpsguide.com https://estore.ibpsguide.com Page 197 of 331


585). A. Last week a video was widely shared that apparently showed a balaclava-wearing “vigilante” attacking

what was said to be a drug dealer’s car somewhere in Bolton.


B. It was a vivid illustration of how damaging fake video – fake views – can be, to individuals and also to
communities.

C. It is hardly a surprise to learn that images are untrustworthy; forensic analysis of still and video images has
been around for decades but fakes are getting smarter all the time.
D. In the end, the police were able to show that the video was not real, but had been staged in order to suggest
the police were not keeping the streets safe.
E. Then at the weekend, the American glossy Vanity Fair became the centre of an angry protest for extensively

doctoring a group image of this year’s big stars.

(a) AECDB (b) ADBEC (c) ABDCE (d) ACEBD (e) None of these.

586). A. It represents a once-in-a-lifetime opportunity to see much of the work amassed by the only British
monarch to have had serious artistic taste.
B. There are great works by Titian, Holbein, Correggio and Dürer – and the magnificent series by Mantegna, the
Triumphs of Caesar, that normally occupies a distant orangery at Hampton Court.
C. The exhibition of Charles I’s art collection that opens at the Royal Academy of Arts in London is stupendous.
D. If you came from another planet – or from a state of historical ignorance – you would never know that the

aesthetics it celebrates are those of a disastrous, autocratic ruler whose reign ended in bloody civil war.
E. For all its splendour, the exhibition is also peculiar.
(a) CAEBD (b) CDEAB (c) CBAED (d) CABED (e) None of these.

587). A. As India seeks to pursue a multi-dimensional engagement with West Asia, Prime Minister Narendra
Modi’s latest visit to the region has merely underscored its growing salience in the Indian foreign policy matrix.

B. This is Mr. Modi’s fifth visit to West Asia in the last three and a half years and sustained high-level
engagements have ensured that India’s voice is becoming an important one in a region that is witnessing major

power rivalries playing out.

C. Underlining India’s credentials as a “very respected country in the international arena”, Palestinian President
Mahmoud Abbas had called for a potential Indian role in the West Asian peace process.

D. While much focus is often given to India’s ‘Act East’ policy, India’s ‘Look West’ policy too is evolving rapidly.

E. Mr. Modi’s Palestine visit — and the first visit by an Indian Prime Minister — coming just weeks after Israeli
Prime Minister Benjamin Netanyahu’s high profile visit to India, has been being looked at with significant interest.

(a) ECADB (b) EADBC (c) ADECB (d) ADBEC (e) None of these.

www.ibpsguide.com https://estore.ibpsguide.com Page 198 of 331


588). A. The concern now is that the problem may also affect climate change-related studies.
B. They interviewed over 100 women IPCC authors across the world and found that while 87% of women
reported a positive experience in the way they are treated, and in their ability to influence the report, there were

many barriers such as race, nationality and command over English that prevented women scientists from
contributing to the best of their capacity.
C. For many years, the under-representation of women in science has vexed science policymakers across the
world.
D. ‘Climate for women in climate science: Women scientists and the Intergovernmental Panel on Climate

Change’, a study published online in the Proceedings of the National Academy of Sciences, has looked at the

question of gender in the authorship of reports by the Intergovernmental Panel on Climate Change (IPCC), the
world’s most authoritative compendium on the science of climate change.

E. The authors, who are based at the University of Arizona, report that the proportion of female IPCC authors has
seen an increase from less than 5% in 1990 to about 20% in the most recent set of assessment reports.
(a) ECADB (b) EADBC (c) CADEB (d) ADBEC (e) None of these.

589). A. Many projects on adaptation begin by studying what climate impacts are expected, what kinds of
vulnerabilities exist locally and how these can be addressed in a given local context.

B. While there are ongoing efforts to reduce greenhouse gas emissions and restrict global warming to below 2°C
or even below 1.5°C, there are also efforts to help us live in a world where average global temperatures are rising.
C. However, a 2010 survey by James Ford and his colleagues of over 1,700 projects concluded that adaptation

projects were not helping the most vulnerable communities, and benefits were simply reaching those who had
been assisted earlier.
D. These projects on adaptation have been funded or implemented in a number of countries, either by individual

governments or with the help of external donor funds.


E. When several projects from the global Adaptation Fund, an international fund managed by the United Nations

climate secretariat to help developing countries with climate change adaptation projects, were analysed, they too

were found not to take into account unequal power structures.


(a) BDCEA (b) BADCE (c) BEDAC (d) BDACE (e) None of these.

590). A. In democracies, citizens have a right to know why, where and how the government spends public money.
B. Each year we, therefore, await the Prime Minister’s response to the President’s address in the Budget session

of Parliament.

www.ibpsguide.com https://estore.ibpsguide.com Page 199 of 331


C. This year, Prime Minister Narendra Modi’s speech was less about the financial implications of the intended

policies of his government, and more about blaming the Congress for the disasters that track Indians since 1947.
D. In the process, he neatly sidestepped the basic obligation he owes us: accountability.
E. Equally troubling is his rather rash allegation that the Congress was responsible for the partition of the country,

and for the division of Jammu and Kashmir.


(a) ABDCE (b) ADBCE (c) ACEDB (d) ABDEC (e) None of these.

591). A. The film stands out for focussing the spotlight on data science by showing that the art of data science is
more about asking the right questions than just having the data.

B. It is believed that social media and networking service companies such as Facebook may already have more

data than they are leveraging.


C. It is difficult to imagine the great volume of data we supply to different agencies in our everyday actions, bit by

bit through surfing the Internet, posting on social media, using credit and debit cards, making online purchases,
and other things where we share information about our identity.
D. There are infinite ways to slice and dice data, which itself is quite daunting as at every step, there is potential to
make huge mistakes.
E. The Hollywood film Moneyball (2011) is about the Oakland Athletics baseball team and the attempt by its
manager to put together a competitive team on a lean budget using data and computer analytics rather than

depending on mere biases to recruit new players.


(a) EADBC (b) EACBD (c) EABDC (d) EDCBA (e) None of these.

592). A. There has been an increase over the baseline cover of 20% at the turn of the century yet, tree cover is
not the same as having biodiverse, old-growth forests.
B. The Environment Ministry’s ‘India State of Forest Report 2017’ based on satellite imagery, may present a net

positive balance in the form of 24.4% of India’s land area under some form of forest or tree cover, but this is but a
broad-brush assessment.

C. However, such an estimate listing very dense, moderately dense, open and scrub forests mapped through

remote sensing does not really provide deep insights into the integrity of the green areas.
D. According to the report, forest and tree cover together registered a 1% rise over the previous estimate two

years ago.

E. The emphasis in environmental policy to raise forest cover to 33% of the geographical area will yield some
dividends.

(a) BADCE (b) BDACE (c) BDCEA (d) BDECA (e) None of these.

www.ibpsguide.com https://estore.ibpsguide.com Page 200 of 331


593). A. But the Singularity is not what humanity needs to worry about right now.
B. Machine learning (a more precise term for AI) will certainly continue to surpass human capabilities in specific
domains such as medical diagnosis and facial recognition.

C. Iconic AI from pop culture such as HAL 9000 and Agent Smith epitomise a Manichaean obsession with the
idea of ‘superintelligence’ (‘the Singularity’) that could prove to be good or evil, vested as it is with the power to
turn humans into either immortals or slaves oppressed by parasitic machines.
D. Public discourse around artificial intelligence (AI) is often hijacked by themes that belong in fantasy rather than
the real world.

E. But an AI that can match human intelligence in all respects is unlikely because it is impossible for AI

technology to replicate that which makes human intelligence what it is — its embodiment in a biological substrate
refined by millions of years of evolutionary feedback loops.

(a) BEDCA (b) DBEAC (c) BDCEA (d) DCABE (e) None of these.

594). A. Electoral reforms in the hands of politicians is a classic example of a fox guarding the henhouse.
B. Unsurprisingly, the Supreme Court, over the last few decades, has readily stepped in to introduce electoral
reforms.
C. The Supreme Court’s recent decision on information disclosure (Lok Prahari v. Union of India) paves a way for

future constitutional interventions in India’s party funding regime, including the scheme of electoral bonds.
D. While there are many policies that both major parties disagree with each other on, they form a remarkable tag
team when it comes to electoral reforms.

E. However, most of these interventions are directed at candidates, and rarely at the parties.
(a) ABECD (b) ADBEC (c) ABDEC (d) ADCEB (e) None of these.

595). A. People don’t seem to realise the strategic role played by public sector banks in the economy.
B. These banks fulfil social commitments such as catering to unbanked areas and lending to the weaker sections

under government-sponsored schemes.

C. The recent exposure of huge corporate defaults has become an occasion to bash government-owned banks
and their staff as being inept and dishonest.

D. They also finance huge infrastructural projects that cause an asset-liability mismatch in their balance sheets.

E. Unlike private sector banks that focus on less risky, personal segment loans, public banks lend to the
productive sectors such as agriculture, small and medium industries and businesses.

(a) CBEDA (b) CBADE (c) CAEBD (d) CABDE (e) None of these.

www.ibpsguide.com https://estore.ibpsguide.com Page 201 of 331


596). A. After the devastating school shooting in Newtown, Connecticut, in December 2012, a tearful Barack
Obama, then President, mooted legislation to tighten the regulation of gun ownership.
B. After decades of campaigning to bring about common-sense gun control in the U.S., it appears that a group of

children may succeed where even Presidents have failed.


C. Their courage is to be doubly applauded, for they appear undaunted by the depressing history of America’s
227-year-old lethal love affair with guns, built on the constitutional right to bear arms, overlaid with a myriad state-
level laws that make gun ownership easy.
D. That was speedily seen off by conservative lawmakers.

E. Following Friday’s deadly school shooting in Parkland, Florida, in which 17 people including 14 students were

killed, survivors took to the streets in a relatively rare show of anger directed at President Donald Trump and
Congress for not doing more to promote gun control.

(a) ECADB (b) ECBAD (c) BECDA (d) BECAD (e) None of these.

597). A. When a language dies, a way of life dies, a way of thinking disappears, a connection between word and
world is lost.
B. There is a hypocrisy and ambivalence which captures modernity’s attitude to the obsolescent and near extinct.
C. Newspapers often report the death of a last tribal speaker, scarcely mentioning the death of a culture that

preceded it.
D. The death of a language in particular has a particular poignancy.
E. Often, today, we mourn the death of the last speaker, treating him as a vestige of an entire past.

(a) DAECB (b) DEABC (c) DABCE (d) ADECB (e) None of these.

598). A. Rule of law is the fundamental principle of governance of any civilised liberal democracy.

B. It is the anti-thesis of arbitrariness.


C. Yet, the Uttar Pradesh government looks somewhat determined to disregard the first principles of the criminal

justice system.

D. Police encounters have become routine in U.P., and in December, Chief Minister Yogi Adityanath introduced in
the State Assembly the Uttar Pradesh Control of Organised Crime Bill, 2017 on the pattern of the regressive

Maharashtra Control of Organised Crime Act (MCOCA).

E. Such legislation does not promote the rule of law, but is itself a kind of violence, though a legitimate one with
due authority of law.

(a) BCDEA (b) DEABC (c) ABCED (d) BDECA (e) None of these.

www.ibpsguide.com https://estore.ibpsguide.com Page 202 of 331


599). A. The launch site was named the Thumba Equatorial Rocket Launching Station (TERLS), or India’s first
space port.
B. During the 1950s and 1960s, the Cold War between the U.S. and the USSR accelerated as they entered the

space race, which eventually turned into a bitter rivalry and raised the risk of space weaponisation.
C. It was against this backdrop that the Indian space programme was born in 1963 with the launch of Nike-
Apache sounding rockets from Thumba in Thiruvananthapuram.
D. When Sputnik 1 was launched by the Soviets in 1957, little did anyone in India imagine that within five decades
the country would become a powerhouse in the outer space arena and a major proponent of its peaceful use.

E. Thankfully, this was avoided after the Outer Space Treaty of 1967.

(a) BECAD (b) DBECA (c) BEDCA (d) DBCAE (e) None of these.

600). A. Each succeeding week brings fresh evidence of how anarchic the international global order has become.
B. This is one cause for many of today’s turmoils.
C. For instance, currently the U.S. is seen to be incapable of playing a balancing role in Asian affairs, and to have
ceded ground to China.
D. The unfortunate aspect is that while there is greater clarity on the new challenges that nations face, the
international system is unable to come up with sustainable solutions to deal with these multiple challenges.

E. Quite a few nations, including many of the newer ones, are seeking a new salience in the affairs of their region,
aiming to establish their dominance.
(a) ECDAB (b) ADCBE (c) AEDCB (d) AEBDC (e) None of these.

ANSWERS:
571). Correct Answer is: (c)

The correct sequence is DACEB.


572). Correct Answer is: (a)

The correct sequence is BACED.

573). Correct Answer is: (b)


The correct sequence is CADBE.

574). Correct Answer is: (d)

The correct sequence is ECADB.


575). Correct Answer is: (c)

The correct sequence is DBACE.

www.ibpsguide.com https://estore.ibpsguide.com Page 203 of 331


576). Correct Answer is: (d)

The correct sequence is ABCED.


577). Correct Answer is: (b)
The correct sequence is EACBD.

578). Correct Answer is: (a)


The correct sequence is CABED.
579). Correct Answer is: (b)
The correct sequence is BCAED.
580). Correct Answer is: (e)

The correct sequence is ABCDE.

581). Correct Answer is: (b)


The correct sequence is DBCAE.

582). Correct Answer is: (c)


The correct sequence is CBAED.
583). Correct Answer is: (d)
The correct sequence is DBECA.
584). Correct Answer is: (c)
The correct sequence is BDCEA.

585). Correct Answer is: (b)


The correct sequence is ADBEC.
586). Correct Answer is: (d)

The correct sequence CABED


587). Correct Answer is: (d)
The correct sequence is ADBEC.

588). Correct Answer is: (c)


The correct sequence is CADEB.

589). Correct Answer is: (d)

The correct sequence is BDACE.


590). Correct Answer is: (e)

The correct sequence is ABCDE.

591). Correct Answer is: (b)


The correct sequence is EACBD.

592). Correct Answer is: (c)

www.ibpsguide.com https://estore.ibpsguide.com Page 204 of 331


The correct sequence is BDCEA.

593). Correct Answer is: (d)


The correct sequence is DCABE
594). Correct Answer is: (b)

The correct sequence is ADBEC.


595). Correct Answer is: (c)
The correct sequence is CAEBD.
596). Correct Answer is: (d)
The correct sequence is BECAD.

597). Correct Answer is: (a)

The correct sequence is DAECB.


598). Correct Answer is: (e)

The correct sequence is ABCDE.


599). Correct Answer is: (b)
The correct sequence is DBECA.
600). Correct Answer is: (d)
The correct sequence is AEBDC.

Jumbled Sentence Type - 2

Directions (Q601-700): In the following passage, a set of 7 sentences is given below. The first and the last
sentences are given in bolds. The remaining sentences (being the 2nd, 3rd, 4th, 5th and 6th sentence

respectively) are jumbled up and marked as A, B, C, D and E. Rearrange the sentences in a proper sequence
and answer the following questions.

Directions (Q601-605):
Cricket is engulfed in one of its worst controversies in recent times, and at the centre of it is a bit of yellow tape
that Australian cricketer Cameron Bancroft used on the ball during the just-concluded Test match between South
Africa and Australia.

A) If that was all there was to it, the matter might have ended with a reprimand and a penalty.
B) The admission of premeditated ball tampering has evoked outrage across the cricketing world, with even
the Australian prime minister weighing in.
C) But after Bancroft was caught on camera, Australian captain Steve Smith admitted that the act of ball
tampering was masterminded by himself and other seniors in the team.

www.ibpsguide.com https://estore.ibpsguide.com Page 205 of 331


D) Ball tampering is generally used to aid reverse swing once the ball gets older
E) This has been tried by teams in the past, including using such blatant tools as bottle caps. While earlier
such methods often went unnoticed, in the television era allegations of ball tampering have surfaced fairly
regularly
The International Cricket Council (ICC) also has rules against ball tampering, which is regarded as an offence
that carries a 100% fine of the match fee and up to four demerit points.

601). Which of the following should be the fifth sentence after the rearrangement?

(a) B (b) A (c) C (d) E (e) D

602). Which of the following should be the SECOND sentence after the rearrangement?

(a) D (b) B (c) E (d) A (e) C

603). Which of the following should be the third sentence after the rearrangement?
(a) B (b) C (c) A (d) E (e) D

604). Which of the following should be the SIXTH sentence after the rearrangement?

(a) E (b) A (c) D (d) C (e) B

605) Which of the following should be the FOURTH sentence after the rearrangement?
(a) A (b) B (c) C (d) D (e) E.

Directions (Q606-610):

Is the government’s push to increase lending to micro and small enterprises making a difference?
A) Indian banks haven’t turned any friendlier towards small businesses even though the current bad loan
mess has arisen from large borrowers.
B) The share of micro and small loans in total non-food credit has hardly moved; in fact, it has slipped a bit in
the last three years
C) The short answer is no.
D) Small businesses continue to be seen with a beady eye and data from the banking sector only underscores
this trend.
E) In fact, it has slipped a bit in the last three years.
The share of loans to micro and small enterprises was 12.63% as of 3 March, down from 13.33% at the end of

fiscal year 2014-15. The year-on-year growth in such loans too is slowing down.

www.ibpsguide.com https://estore.ibpsguide.com Page 206 of 331


606) Which of the following should be the FOURTH sentence after the rearrangement?

(a) E (b) A (c) D (d) C (e) B

607). Which of the following should be the FIFTH sentence after the rearrangement?

(a) B (b) A (c) C (d) E (e) D

608).Which of the following should be the SECOND sentence after the rearrangement?
(a) D (b) B (c) E (d) A (e) C

609). Which of the following should be the SIXTH sentence after the rearrangement?

(a) E (b) A (c) D (d) C (e) B

610). Which of the following should be the THIRD sentence after the rearrangement?
(a) E (b) A (c) D (d) C (e) B

Directions (Q. 611-615):


The Maharashtra Legislative Assembly today unanimously passed a resolution recommending the Centre to
rename the Kolhapur airport as the 'Chhatrapati Rajaram Maharaj Airport'

A) The Chair then put the resolution to vote which was passed unanimously.
B) Fadnavis said the resolution of the House would be sent to the Centre
C) Chief Minister Devendra Fadnavis moved the resolution in the Lower House to rename the airport after the
member of the erstwhile royal family of Kolhapur
D) The civic body of Kolhapur had passed a resolution to this effect earlier.
E) There was a long pending demand from the people and leaders of the district for renaming the Kolhapur
airport after Chhatrapati Rajaram Maharaj.
The Fadnavis cabinet had also decided in January this year to rename the airport after Chhatrapati Rajaram

Maharaj, who is credited with launching various infrastructure projects, including the airstrip in the erstwhile
princely state of Kolhapur

611). Which of the following should be the FIFTH sentence after the rearrangement?
(a) E (b) A (c) D (d) C (e) B

612). Which of the following should be the FOURTH sentence after the rearrangement?

www.ibpsguide.com https://estore.ibpsguide.com Page 207 of 331


(a) B (b) A (c) C (d) E (e) D

613).Which of the following should be the SECOND sentence after the rearrangement?
(a) D (b) B (c) E (d) A (e) C

614). Which of the following should be the SIXTH sentence after the rearrangement?
(a) E (b) A (c) D (d) C (e) B

615). Which of the following should be the THIRD sentence after the rearrangement?

(a) E (b) A (c) D (d) C (e) B

Directions (Q. 616-620):

The Delhi High Court verdict setting aside the disqualification of 20 Aam Aadmi Party MLAs in Delhi is a searing
indictment of the manner in which the Election Commission handled the complaint that they held offices of profit
while serving as parliamentary secretaries.
A) For a body vested with the crucial power to determine whether lawmakers have incurred disqualification in
certain circumstances and advise the President or the Governor suitably, this is an embarrassing moment
B) The court has not reviewed its decision on merits. Rather, it has ruled that the EC violated the principles of
natural justice while adjudicating a lawyer’s complaint against the legislators
C) It failed to offer an oral hearing on the merits of the complaint and chose to hide under the specious argument
that notices had been issued to the MLAs to respond to documents that the EC had summoned from the Delhi
government
D) After saying in its order of June 2017 that it would fix a date for the next hearing, the commission issued two
notices seeking replies but fixed no date; instead, it proceeded to give its decision on January 19, 2018.
E) Further, Election Commissioner O.P. Rawat, who had recused himself at an earlier point, rejoined the process
without intimation to the legislators.
And another vitiating factor was that Election Commissioner Sunil Arora, who had not heard the matter and
assumed office only in September 2017, had signed the order. It is a basic feature of judicial or quasi-judicial
processes that someone who does not hear a matter does not decide on it.

616). Which of the following should be the FIFTH sentence after the rearrangement?
(a) E (b) A (c) D (d) C (e) B

617). Which of the following should be the FOURTH sentence after the rearrangement?

www.ibpsguide.com https://estore.ibpsguide.com Page 208 of 331


(a) B (b) A (c) C (d) E (e) D

618).Which of the following should be the SECOND sentence after the rearrangement?
(a) D (b) B (c) E (d) A (e) C

619). Which of the following should be the SIXTH sentence after the rearrangement?
(a) E (b) A (c) D (d) C (e) B

620). Which of the following should be the THIRD sentence after the rearrangement?

(a) E (b) A (c) D (d) C (e) B

Directions (Q. 621-625):

Cricket is a sport, but it is also a code of honor. The phrase ‘it’s not cricket’ refers to any act that is not fair.

A) The seemingly innocuous application of saliva and sweat, and more interventionist acts such as
B) Yet, like a few other things wrong with the game, ball-tampering remains one of its murkiest secrets
C) pressing chewed lozenges, throwing the ball hard on the surface, the use of nails or abrasive dust from the
turf, and in some cases the use of bottle openers have plunged a knife into the game’s heart even as they
enhanced many a fast bowler’s ability to extract reverse-swing.
D) The act, caught on camera, and the subsequent admission of guilt by the fielder and Smith have tarred them
and their fellow accomplices in the leadership group, including vice-captain David Warner and coach Darren
Lehmann
E) This past weekend, Steve Smith’s Australian team went one step further on that road to infamy, prompting its
opening batsman Cameron Bancroft to scuff the ball with a yellow tape laden with dirt-granules from the pitch
during the course of the third Test against hosts South Africa at Cape Town’s Newlands Ground
The entire episode has also raised questions about the manner in which a powerhouse such as Australia goes

about playing its cricket.

621). Which of the following should be the FIFTH sentence after the rearrangement?

(a) E (b) A (c) D (d) C (e) B

622). Which of the following should be the FOURTH sentence after the rearrangement?

(a) B (b) A (c) C (d) E (e) D

www.ibpsguide.com https://estore.ibpsguide.com Page 209 of 331


623).Which of the following should be the SECOND sentence after the rearrangement?

(a) D (b) B (c) E (d) A (e) C

624). Which of the following should be the SIXTH sentence after the rearrangement?

(a) E (b) A (c) D (d) C (e) B

625). Which of the following should be the THIRD sentence after the rearrangement?
(a) E (b) A (c) D (d) C (e) B

Directions (Q. 626-630):

Earlier this month, the government cut royalties that local seed companies pay to Monsanto, for the second time
in two years

A) In February, for instance, the anti-trust regulator, the Competition Commission of India, decided to probe into
anti-competitive practices by Monsanto.
B) This follows previous attempts to defang Monsanto.
C) At the centre of all this is the pink bollworm infestation plaguing cotton farmers. Even though Bollgard 2, or
BG-2, Monsanto’s second generation insecticidal technology for cotton, was supposed to protect crops
against the pink bollworm, the pest has grown resistant to the toxins produced by this trait.
D) As a result, farmers now spend more on pesticides to control infestations.
E) This, along with the high cost of BT seeds, is driving farmers to indigence.

One solution suggested by the National Seed Association of India is for the government to encourage a move

back to Bollgard, the first iteration of Bt cotton, as Monsanto hasn’t patented BG in India

626). Which of the following should be the FIFTH sentence after the rearrangement?

(a) E (b) A (c) D (d) C (e) B

627). Which of the following should be the FOURTH sentence after the rearrangement?

(a) B (b) A (c) C (d) E (e) D

628).Which of the following should be the SECOND sentence after the rearrangement?

(a) D (b) B (c) E (d) A (e) C

www.ibpsguide.com https://estore.ibpsguide.com Page 210 of 331


629). Which of the following should be the SIXTH sentence after the rearrangement?

(a) E (b) A (c) D (d) C (e) B

630). Which of the following should be the THIRD sentence after the rearrangement?

(a) E (b) A (c) D (d) C (e) B

Directions (Q. 631-635):


Elections to the Rajya Sabha would have been dull, predictable affairs if not for stories of intrigue and betrayal.
A) In this round of biennial elections to the Upper House of Parliament, the element of drama was provided by
the 10th seat from Uttar Pradesh, eventually won by the Bharatiya Janata Party over the Bahujan Samaj Party
through a combination of cross-voting and last-minute switch of loyalties.
B) The battle for the 10th seat, which the BJP could not have won on its own, supposedly held long-term
implications for the political churn that U.P. is now witnessing
C) Chief Minister Yogi Adityanath sought to paint the result less as a satisfying win for his party and more as a
disturbing result for the BSP, saying the SP took votes from others but did not return them
D) . The BJP saw it as a test of the new bonding between the SP and the BSP, which won for the SP two Lok
Sabha by-elections recently.
E) More than getting an additional seat, the BJP was hoping that the outcome would sow the seeds of distrust
between the SP and the BSP
Although clearly unhappy with the defeat, BSP leader Mayawati was unwilling to let it jeopardise the nascent
understanding with the SP, and said the loss would not affect her party’s growing proximity to the SP

631). Which of the following should be the FIFTH sentence after the rearrangement?

(a) E (b) A (c) D (d) C (e) B

632). Which of the following should be the FOURTH sentence after the rearrangement?

(a) B (b) A (c) C (d) E (e) D

633). Which of the following should be the SECOND sentence after the rearrangement?

(a) D (b) B (c) E (d) A (e) C

634). Which of the following should be the SIXTH sentence after the rearrangement?

(a) E (b) A (c) D (d) C (e) B

www.ibpsguide.com https://estore.ibpsguide.com Page 211 of 331


635). Which of the following should be the THIRD sentence after the rearrangement?

(a) E (b) A (c) D (d) C (e) B

Directions (Q. 636-640):

What makes ISIS different from other jihadist groups is that it’s an insurgency as well as a proto-state at the same
time
A) Al-Qaeda, the most powerful force in the global jihadist landscape till the rise of ISIS, is largely a hit-and-run
outfit
B) Osama bin Laden didn’t create a state.
C) He was always at the mercy of foreign governments or intelligence agencies — Saudi Arabia and Pakistan
during the Afghan civil war, Sudan in the early days of al-Qaeda, Taliban during the most important
operational phase and again Pakistan in his final years
D) The way it operates is different from other organizations
E) Nor did he declare himself as the Caliph of the world’s Muslims.

Abu Bakr al-Baghdadi, on the other side, established a proto-state that at one point of time was as big as the
United Kingdom, ruling over about 2 million people. ISIS used both asymmetric and conventional warfare tactics
in the battlefield

636). Which of the following should be the FIFTH sentence after the rearrangement?
(a) E (b) A (c) D (d) C (e) B

637). Which of the following should be the FOURTH sentence after the rearrangement?

(a) B (b) A (c) C (d) E (e) D

638).Which of the following should be the SECOND sentence after the rearrangement?
(a) D (b) B (c) E (d) A (e) C

639). Which of the following should be the SIXTH sentence after the rearrangement?
(a) E (b) A (c) D (d) C (e) B

640). Which of the following should be the THIRD sentence after the rearrangement?
(a) E (b) A (c) D (d) C (e) B

www.ibpsguide.com https://estore.ibpsguide.com Page 212 of 331


Directions (Q. 641-645):
The US government has added seven Pakistani firms to a list of foreign entities that pose a "significant risk" to its
national security and interests by allegedly engaging in nuclear trade, denting Islamabad's bid to join the elite

Nuclear Suppliers Group (NSG)


A) The list prepared by the US BIS has declared that all seven Pakistani firms are "reasonably believed to be
involved, or to pose a significant risk of being or becoming involved, in activities contrary to the national
security or foreign policy interests of the United States".
B) A total of 23 entities were added to the list that was published in the US Federal Register last week by the
country's Bureau of Industry and Security (BIS). Besides the Pakistani firms, the list includes 15 entities from
South Sudan and one from Singapore
C) Now all 23 entities face stringent export control measures, which could prevent them from international trade
D) Among the seven Pakistani companies, three are listed for "their involvement in the proliferation of
unsafeguarded nuclear activities that are contrary to the national security and/or foreign policy interests of the
US
E) Two are accused of procuring supplies for nuclear-related entities already on the list and the remaining two
are accused of acting as fronts for listed entities. An eighth Pakistani entity is based in Singapore.

The End-user Review Committee (ERC) of the US Department of Commerce determined that Mushko Logistics

Pvt Ltd, Singapore, and Mushko Electronics Pvt Ltd, Pakistan, should be added to the list on the grounds that
they procured items for several Pakistani entities on the entity list.

641). Which of the following should be the FIFTH sentence after the rearrangement?
(a) E (b) A (c) D (d) C (e) B

642). Which of the following should be the FOURTH sentence after the rearrangement?
(a) B (b) A (c) C (d) E (e) D

643).Which of the following should be the SECOND sentence after the rearrangement?
(a) D (b) B (c) E (d) A (e) C

644). Which of the following should be the SIXTH sentence after the rearrangement?
(a) E (b) A (c) D (d) C (e) B

www.ibpsguide.com https://estore.ibpsguide.com Page 213 of 331


645). Which of the following should be the THIRD sentence after the rearrangement?

(a) E (b) A (c) D (d) C (e) B

Directions (Q. 646-650):

The Centre on Monday reached out to Anna Hazare, who is on an indefinite hunger strike here since March 23,
with its emissary, Maharashtra Minister Girish Mahajana, assuring the social activist that most of his “demands
will be taken care of”.
A) He said the Union Budget had taken care of the demand to give a better price for farmers for their yield
B) “Tomorrow [Tuesday], we will give a written proposal to Anna ji ,” Mr. Mahajan said. “We are hopeful that he
will end his indefinite hunger strike tomorrow
C) The Minister said he was representing both the Centre and the State government. However, there was no
word from the Hazare camp on the meeting
D) Mr. Mahajan said that of the 11 main demands, which include appointment of a Lokpal and giving better prices
for farmers, seven or eight had been agreed upon. “Most of his demands will be taken care of,” he said.
E) The Chief Minister [Devendra Fadnavis] is in the loop of what is happening. Even the central leadership [of the
BJP], including party president Amit Shah, is looking into the matter,” Mr. Mahajan said.
The Lokpal demand would be resolved at a third meeting.
646). Which of the following should be the FIFTH sentence after the rearrangement?

(a) E (b) A (c) D (d) C (e) B

647). Which of the following should be the FOURTH sentence after the rearrangement?
(a) B (b) A (c) C (d) E (e) D

648).Which of the following should be the SECOND sentence after the rearrangement?

(a) D (b) B (c) E (d) A (e) C

649). Which of the following should be the SIXTH sentence after the rearrangement?

(a) E (b) A (c) D (d) C (e) B

650). Which of the following should be the THIRD sentence after the rearrangement?

(a) E (b) A (c) D (d) C (e) B

Directions (Q. 651-655):

www.ibpsguide.com https://estore.ibpsguide.com Page 214 of 331


Will laying down procedural safeguards to curb false accusations work against the interest of protecting the

oppressed from discrimination and caste-based atrocities?

A) On the face of it, it is difficult to fault the court’s approach.


B) This is the salient question that arises from the Supreme Court verdict that has taken note of the perception
that the Scheduled Castes and the Scheduled Tribes (Prevention of Atrocities) Act, 1989, is being rampantly
misused to settle personal scores and harass adversaries
C) It is settled law that the mere scope for misuse of an Act is not a ground to invalidate it
D) Constitution courts seek to preserve the spirit of such legislation on the one hand and to evolve guidelines to
prevent its misuse on the other
E) This is precisely what the two-judge bench has aimed to do.

It has ruled that Section 18, which bars grant of anticipatory bail to anyone accused of violating its provisions, is
not an absolute bar on giving advance bail to those against whom, prima facie, there is no case

651)Which of the following should be the FIFTH sentence after the rearrangement?
(a) E (b) A (c) D (d) C (e) B

652). Which of the following should be the FOURTH sentence after the rearrangement?
(a) B (b) A (c) C (d) E (e) D

653).Which of the following should be the SECOND sentence after the rearrangement?

(a) D (b) B (c) E (d) A (e) C

654). Which of the following should be the SIXTH sentence after the rearrangement?

(a) E (b) A (c) D (d) C (e) B

655). Which of the following should be the THIRD sentence after the rearrangement?

(a) E (b) A (c) D (d) C (e) B

Directions (Q. 656-660):

China is no stranger to reform.

www.ibpsguide.com https://estore.ibpsguide.com Page 215 of 331


A) But the scale of reform pushed through this month is comparable to that of 1998 when Zhu Rongji as Premier
shut or merged 15 ministries as part of a major liberalisation drive.
B) Beijing has also created a powerful anti-corruption agency, while the Vice President, till now holding a
ceremonial post, is expected to play an active role in policymaking.
C) Over the past three decades the structure of the government has changed at least half a dozen times
D) The stamp of Xi Jinping, re-elected President for five more years with no term limit, is visible in these reforms
E) This time, Prime Minister Li Keqiang has closed six ministries, two ministry-level agencies and seven vice
ministry-level departments

A big decision is the empowerment of the Environment Ministry, which will fight air, water and soil pollution, a top
priority for Mr. Xi. Two of his close aides have been appointed to key posts — Wang Qishan, an anti-corruption

crusader, is now the Vice President, and Liu He, the President’s top fiscal adviser, is a Vice Premier.

656). Which of the following should be the FIFTH sentence after the rearrangement?

(a) E (b) A (c) D (d) C (e) B

657). Which of the following should be the FOURTH sentence after the rearrangement?
(a) B (b) A (c) C (d) E (e) D

658).Which of the following should be the SECOND sentence after the rearrangement?
(a) D (b) B (c) E (d) A (e) C

659). Which of the following should be the SIXTH sentence after the rearrangement?

(a) E (b) A (c) D (d) C (e) B

660). Which of the following should be the THIRD sentence after the rearrangement?
(a) E (b) A (c) D (d) C (e) B

Directions (661 - 665):


The NDA government’s scheme to provide health cover of ₹5 lakh per year to 10 crore poor and vulnerable
families through the Ayushman Bharat-National Health Protection Mission has taken a step forward with the

Union Cabinet approving the modalities of its implementation.

www.ibpsguide.com https://estore.ibpsguide.com Page 216 of 331


A) The States, which have a statutory responsibility for provision of health care, have to act quickly and form
dedicated agencies to run the scheme.
B) Considering the small window, just over a year, available before the term of the present government ends,
urgent action is needed to roll out such an ambitious scheme
C) Since the NHPM represents the foundation for a universal health coverage system that should eventually
cover all Indians, it needs to be given a sound legal basis, ideally through a separate law.
D) For a start, the apex council that will steer the programme and the governing board to operationalise it in
partnership with the States need to be set up.
E) This could be on the lines of legislation governing the rights to food and information.
Such legislation would strengthen entitlement to care, which is vital to the scheme’s success. It will also enable

much-needed regulatory control over pricing of hospital-based treatments

661). Which of the following should be the FIFTH sentence after the rearrangement?
(a) E (b) A (c) D (d) C (e) B

662). Which of the following should be the FOURTH sentence after the rearrangement?

(a) B (b) A (c) C (d) E (e) D

663).Which of the following should be the SECOND sentence after the rearrangement?
(a) D (b) B (c) E (d) A (e) C

664). Which of the following should be the SIXTH sentence after the rearrangement?
(a) E (b) A (c) D (d) C (e) B

665). Which of the following should be the THIRD sentence after the rearrangement?

(a) E (b) A (c) D (d) C (e) B

Directions (Q. 666-670):


The Maldivian government’s decision to lift the state of emergency after 45 days, just ahead of the expiry of its
second self-imposed deadline, comes as cold comfort for those concerned about the turn of events in the islands

over the past couple of months.


A) In a dramatic turn of events Mr. Yameen had then ordered the arrest of two judges, as well as hundreds of
activists and politicians including former President Abdul Gayoom, and imposed a state of emergency.

www.ibpsguide.com https://estore.ibpsguide.com Page 217 of 331


B) In a statement India said the withdrawal of the emergency is but “one step”, and much more must be done to
restore democracy in the Maldives.
C) The opposition, mostly in exile and led by former President Mohamad Nasheed, says the emergency was
lifted
D) The remaining judges overturned the February 1 release order, under what is seen to be coercion by the
security forces, which had locked down the Majlis (parliament) and court buildings
E) only because President Abdulla Yameen has established total control over the judiciary and parliament since
the February 1 court verdict that cancelled the sentencing of 12 opposition leaders and ordered their release
Therefore, lifting the emergency does not automatically amount to status quo ante.

666). Which of the following should be the FIFTH sentence after the rearrangement?

(a) E (b) A (c) D (d) C (e) B

667). Which of the following should be the FOURTH sentence after the rearrangement?

(a) B (b) A (c) C (d) E (e) D

668).Which of the following should be the SECOND sentence after the rearrangement?
(a) D (b) B (c) E (d) A (e) C

669). Which of the following should be the SIXTH sentence after the rearrangement?
(a) E (b) A (c) D (d) C (e) B

670). Which of the following should be the THIRD sentence after the rearrangement?

(a) E (b) A (c) D (d) C (e) B

Directions (Q. 671-675):


Russia made headlines for all the wrong reasons this week, when a clutch of countries led by the U.S. expelled
more than 100 of its diplomats and intelligence officers over suspicion that the Kremlin was behind a nerve agent

attack on a Russian spy and defector to the U.K., Sergei Skripal, and his daughter Yulia, in Salisbury on March 4.
A) In a sense this move, seen as the most dramatic, concerted such purge since the Cold War years, has been
coming for some time.
B) Besides the U.S., 14 member-states of the European Union, including Germany, France, Poland, the Czech
Republic, Lithuania, Italy, Denmark, the Netherlands and Latvia, undertook coordinated expulsions, with
Australia also joining them

www.ibpsguide.com https://estore.ibpsguide.com Page 218 of 331


C) Last week the U.K. led the way when it expelled 23 Russian diplomats, but the week before that the U.S. had
slapped Russia with sanctions against multiple individuals and entities for interfering in the 2016
U.Spresidential election through covert online propaganda, including fake news.
D) .Beyond these specific charges lie other alleged violations: in Afghanistan, President Donald Trump’s senior-
most ground commander has accused Russia of arming Taliban militants; on the North Korean crisis Mr.
Trump mentioned in January that Russia was helping Pyongyang avoid UN sanctions
E) And the EU last year voted to extend into 2018 sanctions that prohibit its businesses from investing in Crimea.
Has Russia truly gone rogue, and is this its grand strategy to reclaim its superpower status?
The answer is yes and no. To an extent the U.S. response, significant though it may appear on the surface,
signals to Russia an inconsistent application of any principles of bilateral and multilateral engagement

671). Which of the following should be the FIFTH sentence after the rearrangement?
(a) E (b) A (c) D (d) C (e) B

672). Which of the following should be the FOURTH sentence after the rearrangement?
(a) B (b) A (c) C (d) E (e) D

673).Which of the following should be the SECOND sentence after the rearrangement?
(a) D (b) B (c) E (d) A (e) C

674). Which of the following should be the SIXTH sentence after the rearrangement?
(a) E (b) A (c) D (d) C (e) B

675). Which of the following should be the THIRD sentence after the rearrangement?
(a) E (b) A (c) D (d) C (e) B

Directions(Q. 676-680)
As widely anticipated, the Board of directors of trouble-ridden Fortis Healthcare Limited (FHL) has cleared the de-

merger of its hospitals business (“Fortis Hospitals”) into Manipal Hospital Enterprises Private Limited (Manipal
Hospitals).
A) The funds will be utilized by Manipal Hospitals to finance the acquisition of 50.9% stake in SRL (20.0% from
FHL and 30.9% from other investors for which discussions are currently on).
B) The remaining FHL will be an investment holding company with 36.6% stake in SRL.
C) As part of the proposed transaction, Ranjan Pai and TPG will invest ₹3,900 crore into Manipal Hospitals.

www.ibpsguide.com https://estore.ibpsguide.com Page 219 of 331


D) The resultant entity Manipal Hospitals will be a publicly-traded company listed on NSE and BSE
E) The board, which had an elongated session on Tuesday, also approved the sale of its 20% stake in SRL
Limited (SRL) to Manipal Hospitals.
In addition, the investment will support the proposed acquisition of hospital assets owned by RHT Health Trust
(RHT) and the growth of the hospitals and the diagnostics businesses.

676). Which of the following should be the FIFTH sentence after the rearrangement?
(a) E (b) A (c) D (d) C (e) B

677). Which of the following should be the FOURTH sentence after the rearrangement?

(a) B (b) A (c) C (d) E (e) D

678).Which of the following should be the SECOND sentence after the rearrangement?

(a) D (b) B (c) E (d) A (e) C

679). Which of the following should be the SIXTH sentence after the rearrangement?
(a) E (b) A (c) D (d) C (e) B

680). Which of the following should be the THIRD sentence after the rearrangement?

(a) E (b) A (c) D (d) C (e) B

Directions (Q. 681-685):


Thirty six paintings line the walls of Art Houz at the fourth edition of the exhibition, I Rise.

A) Gayathri Devi is from Coimbatore and some of her works reflect the seashore she visited on a vacation
B) They are the works of seven women artists - Arathi Rajagopalan, Geetha Shree Rajanna, Gayathri Devi,
Ishrath Humairah, Ruchi Bagrecha, Shoba and Surekha
C) One of her works is called ‘Shore’ where she uses the acrylic pour technique. “I pour acrylic colours on canvas
and then tilt it in specific angles to control the flow of the paint.
D) The entire process should take less than an hour as the colour will dry fast,” she explains
E) She again combines the ocean and seasons in her series of three paintings titled, ‘Show of Seasons’
The predominant use of orange, green and blue represent autumn, spring and monsoon respectively

681). Which of the following should be the FIFTH sentence after the rearrangement?
(a) E (b) A (c) D (d) C (e) B

www.ibpsguide.com https://estore.ibpsguide.com Page 220 of 331


682). Which of the following should be the FOURTH sentence after the rearrangement?
(a) B (b) A (c) C (d) E (e) D

683).Which of the following should be the SECOND sentence after the rearrangement?
(a) D (b) B (c) E (d) A (e) C

684). Which of the following should be the SIXTH sentence after the rearrangement?
(a) E (b) A (c) D (d) C (e) B

685). Which of the following should be the THIRD sentence after the rearrangement?
(a) E (b) A (c) D (d) C (e) B

Directions (Q. 686-690):


Karnataka was supposed to be the Bharatiya Janata Party’s point of entry into southern India
A) A relatively new entrant to the Congress, he has created his own space in the faction-ridden party and in the
wider public sphere by traversing caste divides and resisting communal polarisation
B) But after its historic victory in the 2008 Assembly election,
C) Thus, the single-phase election on May 12 could witness a face-off between the BJP and the Congress, with
the Janata Dal (Secular) a distant third.
D) Now, far from expanding to the neighbouring States, the party is struggling to return to power in Karnataka in
the face of a determined defensive battle by the politically savvy Congress Chief Minister, Siddaramaiah
E) the party lost its way in the State, and the Congress staged a comeback five years later.
The BJP’s challenge is mounted by the old warhorse B.S. Yeddyurappa, its most valuable asset and arguably
also its greatest liability

686). Which of the following should be the FIFTH sentence after the rearrangement?
(a) E (b) A (c) D (d) C (e) B

687). Which of the following should be the FOURTH sentence after the rearrangement?
(a) B (b) A (c) C (d) E (e) D

www.ibpsguide.com https://estore.ibpsguide.com Page 221 of 331


688).Which of the following should be the SECOND sentence after the rearrangement?

(a) D (b) B (c) E (d) A (e) C

689). Which of the following should be the SIXTH sentence after the rearrangement?

(a) E (b) A (c) D (d) C (e) B

690). Which of the following should be the THIRD sentence after the rearrangement?
(a) E (b) A (c) D (d) C (e) B

Directions (Q. 691-695):

Silicon Valley has its own rules


A) The companies which are the most valuable are the companies with the most information, not necessarily the
most products, or the most sales.
B) It’s the new economy, after all.
C) ‘Number of Daily Active Users’, ‘Time spent’, ‘Level of engagement’ and ‘Behavioral insights’ are the new
measures of value
D) These companies are valued on the basis of the data that they are harvesting off us and, consequently, how
well they know us.
E) They are valued on the basis of how deeply they have infiltrated our private lives.
Strip away all the financial jargon and you will realise that the primary reason why the market values these
companies so highly is that they have built up an astronomical

691). Which of the following should be the FIFTH sentence after the rearrangement?

(a) E (b) A (c) D (d) C (e) B

692). Which of the following should be the FOURTH sentence after the rearrangement?
(a) B (b) A (c) C (d) E (e) D

693).Which of the following should be the SECOND sentence after the rearrangement?
(a) D (b) B (c) E (d) A (e) C

694). Which of the following should be the SIXTH sentence after the rearrangement?
(a) E (b) A (c) D (d) C (e) B

www.ibpsguide.com https://estore.ibpsguide.com Page 222 of 331


695). Which of the following should be the THIRD sentence after the rearrangement?
(a) E (b) A (c) D (d) C (e) B

Directions (Q. 696-700):


To curb rampant corruption, the Maharashtra government has decided to bring a Bill on the lines of what the Bihar
government has, wherein property of those accused in criminal cases and of corrupt officers will be confiscated
and attached to the government.
A) Also, most officials indulge in corrupt practices, and the existing law is not effective to punish the guilty
B) Speaking in the state legislative council, chief minister Devendra Fadnavis on Wednesday said they had
issued a GR asking government officers to declare and submit details of their assets
C) However, response to this decision has been lukewarm.
D) Government employees are reluctant to declare their assets.
E) Therefore, on the lines of Bihar, we have decided to table a bill in the next winter session. Our Home
department is already in the process of preparing the draft bill," he added.
Presently, government officers booked for corruption or disproportionate assets continue to hold on to their ill-
gotten wealth even as a long trial goes on in court.

696). Which of the following should be the FIFTH sentence after the rearrangement?
(a) E (b) A (c) D (d) C (e) B

697). Which of the following should be the FOURTH sentence after the rearrangement?
(a) B (b) A (c) C (d) E (e) D

698).Which of the following should be the SECOND sentence after the rearrangement?
(a) D (b) B (c) E (d) A (e) C

699). Which of the following should be the SIXTH sentence after the rearrangement?

(a) E (b) A (c) D (d) C (e) B

700). Which of the following should be the THIRD sentence after the rearrangement?
(a) E (b) A (c) D (d) C (e) B

www.ibpsguide.com https://estore.ibpsguide.com Page 223 of 331


ANSWERS:

601). Correct Answer is: (e)


The correct sequence is ACBDE in the order of 2nd, 3rd, 4th, 5th and 6th sentence respectively.
602). Correct Answer is: (d)

The correct sequence is ACBDE in the order of 2nd, 3rd, 4th, 5th and 6th sentence respectively.
603). Correct Answer is: (b).
The correct sequence is ACBDE in the order of 2nd, 3rd, 4th, 5th and 6th sentence respectively.
604). Correct Answer is: (a).
The correct sequence is ACBDE in the order of 2nd, 3rd, 4th, 5th and 6th sentence respectively.

605). Correct Answer is: (b).

The correct sequence is ACBDE in the order of 2nd, 3rd, 4th, 5th and 6th sentence respectively.
606). Correct Answer is: (c)

The correct sequence is CADBE in the order of 2nd, 3rd, 4th, 5th and 6th sentence respectively.
607). Correct Answer is: (a)
The correct sequence is CADBE in the order of 2nd, 3rd, 4th, 5th and 6th sentence respectively.
608). Correct Answer is: (e)
The correct sequence is CADBE in the order of 2nd, 3rd, 4th, 5th and 6th sentence respectively
609). Correct Answer is: (a)

The correct sequence is CADBE in the order of 2nd, 3rd, 4th, 5th and 6th sentence respectively
610). Correct Answer is: (b)
The correct sequence is CADBE in the order of 2nd, 3rd, 4th, 5th and 6th sentence respectively

611). Correct Answer is: (a)


The correct sequence is CABED in the order of 2nd, 3rd, 4th, 5th and 6th sentence respectively
612). Correct Answer is: (a)

The correct sequence is CABED in the order of 2nd, 3rd, 4th, 5th and 6th sentence respectively.
613). Correct Answer is: (e)

The correct sequence is CABED in the order of 2nd, 3rd, 4th, 5th and 6th sentence respectively

614). Correct Answer is: (c)


The correct sequence is CABED in the order of 2nd, 3rd, 4th, 5th and 6th sentence respectively

615). Correct Answer is: (b)

The correct sequence is CABED in the order of 2nd, 3rd, 4th, 5th and 6th sentence respectively
616). Correct Answer is: (c)

The correct sequence is ABCDE in the order of 2nd, 3rd, 4th, 5th and 6th sentence respectively

www.ibpsguide.com https://estore.ibpsguide.com Page 224 of 331


617). Correct Answer is: (c)

The correct sequence is ABCDE in the order of 2nd, 3rd, 4th, 5th and 6th sentence respectively.
618). Correct Answer is: (e)
The correct sequence is CABED in the order of 2nd, 3rd, 4th, 5th and 6th sentence respectively

619). Correct Answer is: (c)


The correct sequence is CABED in the order of 2nd, 3rd, 4th, 5th and 6th sentence respectively
620). Correct Answer is: (b)
The correct sequence is CABED in the order of 2nd, 3rd, 4th, 5th and 6th sentence respectively
621). Correct Answer is: (a)

The correct sequence is BACED in the order of 2nd, 3rd, 4th, 5th and 6th sentence respectively

622). Correct Answer is: (c)


The correct sequence is BACED in the order of 2nd, 3rd, 4th, 5th and 6th sentence respectively.

623). Correct Answer is: (b)


The correct sequence is BACED in the order of 2nd, 3rd, 4th, 5th and 6th sentence respectively
624). Correct Answer is: (c)
The correct sequence is BACED in the order of 2nd, 3rd, 4th, 5th and 6th sentence respectively
625). Correct Answer is: (b)
The correct sequence is BACED in the order of 2nd, 3rd, 4th, 5th and 6th sentence respectively

626). Correct Answer is: (a)


The correct sequence is BACED in the order of 2nd, 3rd, 4th, 5th and 6th sentence respectively
627). Correct Answer is: (c)

The correct sequence is BACED in the order of 2nd, 3rd, 4th, 5th and 6th sentence respectively.
628). Correct Answer is: (b)
The correct sequence is BACED in the order of 2nd, 3rd, 4th, 5th and 6th sentence respectively

629). Correct Answer is: (c)


The correct sequence is BACED in the order of 2nd, 3rd, 4th, 5th and 6th sentence respectively

630). Correct Answer is: (b)

The correct sequence is BACED in the order of 2nd, 3rd, 4th, 5th and 6th sentence respectively
631). Correct Answer is: (d)

The correct sequence is ABDCE in the order of 2nd, 3rd, 4th, 5th and 6th sentence respectively

632). Correct Answer is: (e)


The correct sequence is ABDCE in the order of 2nd, 3rd, 4th, 5th and 6th sentence respectively.

633). Correct Answer is: (d)

www.ibpsguide.com https://estore.ibpsguide.com Page 225 of 331


The correct sequence is ABDCE in the order of 2nd, 3rd, 4th, 5th and 6th sentence respectively

634). Correct Answer is: (d)


The correct sequence is ABDEC in the order of 2nd, 3rd, 4th, 5th and 6th sentence respectively
635). Correct Answer is: (e)

The correct sequence is ABDCE in the order of 2nd, 3rd, 4th, 5th and 6th sentence respectively
636). Correct Answer is: (a)
The correct sequence is DABEC in the order of 2nd, 3rd, 4th, 5th and 6th sentence respectively
637). Correct Answer is: (a)
The correct sequence is DABEC in the order of 2nd, 3rd, 4th, 5th and 6th sentence respectively.

638). Correct Answer is: (a)

The correct sequence is DABEC in the order of 2nd, 3rd, 4th, 5th and 6th sentence respectively
639). Correct Answer is: (d)

The correct sequence is DABEC in the order of 2nd, 3rd, 4th, 5th and 6th sentence respectively
640). Correct Answer is: (b)
The correct sequence is DABEC in the order of 2nd, 3rd, 4th, 5th and 6th sentence respectively
641). Correct Answer is: (c)
The correct sequence is BACDE in the order of 2nd, 3rd, 4th, 5th and 6th sentence respectively
642). Correct Answer is: (c)

The correct sequence is BACDE in the order of 2nd, 3rd, 4th, 5th and 6th sentence respectively.
643). Correct Answer is: (b)
The correct sequence is BACDE in the order of 2nd, 3rd, 4th, 5th and 6th sentence respectively

644). Correct Answer is: (a)


The correct sequence is BACDE in the order of 2nd, 3rd, 4th, 5th and 6th sentence respectively
645). Correct Answer is: (b)

The correct sequence is BACDE in the order of 2nd, 3rd, 4th, 5th and 6th sentence respectively
646). Correct Answer is: (a)

The correct sequence is BCDEA in the order of 2nd, 3rd, 4th, 5th and 6th sentence respectively

647). Correct Answer is: (e)


The correct sequence is BCDEA in the order of 2nd, 3rd, 4th, 5th and 6th sentence respectively.

648). Correct Answer is: (b)

The correct sequence is BCDEA in the order of 2nd, 3rd, 4th, 5th and 6th sentence respectively
649). Correct Answer is: (b)

The correct sequence is BCDEA in the order of 2nd, 3rd, 4th, 5th and 6th sentence respectively

www.ibpsguide.com https://estore.ibpsguide.com Page 226 of 331


650). Correct Answer is: (b)

The correct sequence is BCDEA in the order of 2nd, 3rd, 4th, 5th and 6th sentence respectively
651). Correct Answer is: (c)
The correct sequence is BACDE in the order of 2nd, 3rd, 4th, 5th and 6th sentence respectively

652). Correct Answer is: (c)


The correct sequence is BACDE in the order of 2nd, 3rd, 4th, 5th and 6th sentence respectively.
653). Correct Answer is: (b)
The correct sequence is BACDE in the order of 2nd, 3rd, 4th, 5th and 6th sentence respectively
654). Correct Answer is: (a)

The correct sequence is BACDE in the order of 2nd, 3rd, 4th, 5th and 6th sentence respectively

655). Correct Answer is: (b)


The correct sequence is BACDE in the order of 2nd, 3rd, 4th, 5th and 6th sentence respectively

656). Correct Answer is: (e)


The correct sequence is CAEBD in the order of 2nd, 3rd, 4th, 5th and 6th sentence respectively
657). Correct Answer is: (d)
The correct sequence is CAEBD in the order of 2nd, 3rd, 4th, 5th and 6th sentence respectively.
658). Correct Answer is: (e)
The correct sequence is CAEBD in the order of 2nd, 3rd, 4th, 5th and 6th sentence respectively

659). Correct Answer is: (c)


The correct sequence is CAEBD in the order of 2nd, 3rd, 4th, 5th and 6th sentence respectively
660). Correct Answer is: (b)

The correct sequence is CAEBD in the order of 2nd, 3rd, 4th, 5th and 6th sentence respectively
661). Correct Answer is: (d)
The correct sequence is BDACE in the order of 2nd, 3rd, 4th, 5th and 6th sentence respectively

662). Correct Answer is: (b)


The correct sequence is BDACE in the order of 2nd, 3rd, 4th, 5th and 6th sentence respectively.

663). Correct Answer is: (a)

The correct sequence is BDACE in the order of 2nd, 3rd, 4th, 5th and 6th sentence respectively
664). Correct Answer is: (a)

The correct sequence is CAEBD in the order of 2nd, 3rd, 4th, 5th and 6th sentence respectively

665). Correct Answer is: (c)


The correct sequence is BDACE in the order of 2nd, 3rd, 4th, 5th and 6th sentence

666). Correct Answer is: (b)

www.ibpsguide.com https://estore.ibpsguide.com Page 227 of 331


The correct sequence is BCEAD in the order of 2nd, 3rd, 4th, 5th and 6th sentence respectively

667). Correct Answer is: (d)


The correct sequence is BCEAD in the order of 2nd, 3rd, 4th, 5th and 6th sentence respectively.
668). Correct Answer is: (b)

The correct sequence is BCEAD in the order of 2nd, 3rd, 4th, 5th and 6th sentence respectively
669). Correct Answer is: (c)
The correct sequence is BCEAD in the order of 2nd, 3rd, 4th, 5th and 6th sentence respectively
670). Correct Answer is: (d)
The correct sequence is BCEAD in the order of 2nd, 3rd, 4th, 5th and 6th sentence.

671). Correct Answer is: (c)

The correct sequence is BACDE in the order of 2nd, 3rd, 4th, 5th and 6th sentence respectively
672). Correct Answer is: (c)

The correct sequence is BACDE in the order of 2nd, 3rd, 4th, 5th and 6th sentence respectively.
673). Correct Answer is: (b)
The correct sequence is BACDE in the order of 2nd, 3rd, 4th, 5th and 6th sentence respectively
674). Correct Answer is: (a)
The correct sequence is BACDE in the order of 2nd, 3rd, 4th, 5th and 6th sentence respectively
675). Correct Answer is: (b)

The correct sequence is BACDE in the order of 2nd, 3rd, 4th, 5th and 6th sentence.
676). Correct Answer is: (d)
The correct sequence is EDBCA in the order of 2nd, 3rd, 4th, 5th and 6th sentence respectively

677). Correct Answer is: (a)


The correct sequence is BACDE in the order of 2nd, 3rd, 4th, 5th and 6th sentence respectively.
678). Correct Answer is: (b)

The correct sequence is BACDE in the order of 2nd, 3rd, 4th, 5th and 6th sentence respectively
679). Correct Answer is: (a)

The correct sequence is BACDE in the order of 2nd, 3rd, 4th, 5th and 6th sentence respectively

680). Correct Answer is: (b)


The correct sequence is BACDE in the order of 2nd, 3rd, 4th, 5th and 6th sentence.

681). Correct Answer is: (c)

The correct sequence is BACDE in the order of 2nd, 3rd, 4th, 5th and 6th sentence respectively
682). Correct Answer is: (c)

The correct sequence is BACDE in the order of 2nd, 3rd, 4th, 5th and 6th sentence respectively.

www.ibpsguide.com https://estore.ibpsguide.com Page 228 of 331


683). Correct Answer is: (b)

The correct sequence is BACDE in the order of 2nd, 3rd, 4th, 5th and 6th sentence respectively
684). Correct Answer is: (a)
The correct sequence is BACDE in the order of 2nd, 3rd, 4th, 5th and 6th sentence respectively

685). Correct Answer is: (b)


The correct sequence is BACDE in the order of 2nd, 3rd, 4th, 5th and 6th sentence.
686). Correct Answer is: (b)
The correct sequence is BEDAC in the order of 2nd, 3rd, 4th, 5th and 6th sentence respectively
687). Correct Answer is: (e)

The correct sequence is BEDA C in the order of 2nd, 3rd, 4th, 5th and 6th sentence respectively.

688). Correct Answer is: (b)


The correct sequence is BEDAC in the order of 2nd, 3rd, 4th, 5th and 6th sentence respectively

689). Correct Answer is: (d)


The correct sequence is BEDAC in the order of 2nd, 3rd, 4th, 5th and 6th sentence respectively
690). Correct Answer is: (a)
The correct sequence is BEDAC in the order of 2nd, 3rd, 4th, 5th and 6th sentence.
691). Correct Answer is: (c)
The correct sequence is BACDE in the order of 2nd, 3rd, 4th, 5th and 6th sentence respectively

692). Correct Answer is: (c)


The correct sequence is BACDE in the order of 2nd, 3rd, 4th, 5th and 6th sentence respectively.
693). Correct Answer is: (b)

The correct sequence is BACDE in the order of 2nd, 3rd, 4th, 5th and 6th sentence respectively
694). Correct Answer is: (a)
The correct sequence is BACDE in the order of 2nd, 3rd, 4th, 5th and 6th sentence respectively

695). Correct Answer is: (b)


The correct sequence is BACDE in the order of 2nd, 3rd, 4th, 5th and 6th sentence.

696). Correct Answer is: (a)

The correct sequence is BCAED in the order of 2nd, 3rd, 4th, 5th and 6th sentence respectively
697). Correct Answer is: (b)

The correct sequence is BCAED in the order of 2nd, 3rd, 4th, 5th and 6th sentence respectively.

698). Correct Answer is: (b)


The correct sequence is BCAED in the order of 2nd, 3rd, 4th, 5th and 6th sentence respectively

699). Correct Answer is: (c)

www.ibpsguide.com https://estore.ibpsguide.com Page 229 of 331


The correct sequence is BCAED in the order of 2nd, 3rd, 4th, 5th and 6th sentence respectively
700). Correct Answer is: (d)
The correct sequence is BCAED in the order of 2nd, 3rd, 4th, 5th and 6th sentence.

Jumbled Sentence Type - 3

Directions (701- 705): A set of six sentences is given below. The first sentence of the passage is given in bolds.

Rearrange the remaining sentences in a logical sequence and then answer the following questions

The Maldivian government’s decision to lift the state of emergency after 45 days, just ahead of the expiry of its

second self-imposed deadline, comes as cold comfort for those concerned about the turn of events in the islands
over the past couple of months.
A) In a statement India said the withdrawal of the emergency is but “one step”, and much more must be done
to restore democracy in the Maldives.
B) The remaining judges overturned the February 1 release order, under what is seen to be coercion by the
security forces, which had locked down the Majlis (parliament) and court buildings
C) Therefore, lifting the emergency does not automatically amount to status quo ante.
D) The opposition, mostly in exile and led by former President Mohamad Hashed, says the emergency was

lifted only because President Abdulla Yamens has established total control over the judiciary and
parliament .
E) Since the February 1 court verdict that cancelled the sentencing of 12 opposition leaders and ordered their

release. In a dramatic turn of events Mr. Yameen had then ordered the arrest of two judges, as well as
hundreds of activists and politicians including former President Abdul Gayoom, and imposed a state of

emergency.

701). Which of the following should be the THIRD sentence after the rearrangement?
(a) (E) (b) (C) (c) (A) (d) (B) (e) (D)

702). Which of the following should be the LAST sentence of the passage after rearrangement?
(a) (E) (b) (C) (c) (A) (d) (B) (e) (D)

703). Which of the following should be the SECOND sentence of the passage after rearrangement?
(a) (E) (b) (C) (c) (A) (d) (B) (e) (D)

www.ibpsguide.com https://estore.ibpsguide.com Page 230 of 331


704). Which of the following should be the FIRST sentence of the passage after rearrangement?

(a) (E) (b) (C) (c) (A) (d) (B) (e) (D)

705). Which of the following should be the FOURTH sentence of the passage after rearrangement?

(a) (E) (b) (C) (c) (A) (d) (B) (e) (D)

Directions (706-710): A set of six sentences is given below. The first sentence of the passage is given in bolds.
Rearrange the remaining sentences in a logical sequence and then answer the following questions

In keeping with his reputation as the global disrupter-in-chief, US President Donald Trump’s unprecedented

acceptance of a face-to-face meeting with North Korea’s strongman Kim Jong-un in May has created a political
whiplash

A) Nixon’s trip to China was a high-stakes gamble as there was no certainty that he would meet chairman
Mao Zedong
B) However, the run-up to the historic Nixon-Mao summit and beyond significantly changed the course of
international relations.
C) While most experts have only focused on the nuclear dimension, this initiative, if it is to materialize, is likely
to have international repercussions well beyond the nuclear realm.

D) In its most optimistic rendering, the Kim-Trump summit might be akin to the dramatic US opening up to
China by President Richard Nixon in 1971-72.
E) Infamously tilted towards Pakistan and China, turned a blind eye to the ongoing brutality of the Cultural

Revolution, supported China’s permanent membership of the UN Security Council, hastened the end of
the Cold War, and contributed to China’s eventual economic rise.

736). Which of the following should be the THIRD sentence after the rearrangement?
(a) (E) (b) (C) (c) (A) (d) (B) (e) (D)

707). Which of the following should be the LAST sentence of the passage after rearrangement?
(a) (E) (b) (C) (c) (A) (d) (B) (e) (D)

708). Which of the following should be the SECOND sentence of the passage after rearrangement?

(a) (E) (b) (C) (c) (A) (d) (B) (e) (D)

709). Which of the following should be the FIRST sentence of the passage after rearrangement?

www.ibpsguide.com https://estore.ibpsguide.com Page 231 of 331


(a) (E) (b) (C) (c) (A) (d) (B) (e) (D)

710). Which of the following should be the FOURTH sentence of the passage after rearrangement?

(a) (E) (b) (C) (c) (A) (d) (B) (e) (D)

Directions (711-715): A set of six sentences is given below. The first sentence of the passage is given in bolds.
Rearrange the remaining sentences in a logical sequence and then answer the following questions

India has much to learn from the opening of China’s onshore bond market

A) Contrast this with India, which has been on a path of gradual debt-market globalization for more than two

decades
B) Last year, Beijing allowed overseas investors to access local-currency securities via its so-called Bond

Connect
C) Over a 20-month period starting in April next year, as many as 386 bonds will enter the $53.73 trillion
benchmark, giving China a weight of 5.49% at full inclusion.

D) Yuan-denominated notes will end up with the fourth-largest presence, after those in US dollars, euros and
yen.
E) On Friday, Bloomberg LP announced the inclusion of Chinese government and policy-bank debt in the

Bloomberg Barclays Global Aggregate Index.


711). Which of the following should be the THIRD sentence after the rearrangement?
(a) (E) (b) (C) (c) (A) (d) (B) (e) (D)

712). Which of the following should be the LAST sentence of the passage after rearrangement?

(a) (E) (b) (C) (c) (A) (d) (B) (e) (D)

713). Which of the following should be the SECOND sentence of the passage after rearrangement?

(a) (E) (b) (C) (c) (A) (d) (B) (e) (D)

714). Which of the following should be the FIRST sentence of the passage after rearrangement?

(a) (E) (b) (C) (c) (A) (d) (B) (e) (D)

715). Which of the following should be the FOURTH sentence of the passage after rearrangement?

(a) (E) (b) (C) (c) (A) (d) (B) (e) (D)

www.ibpsguide.com https://estore.ibpsguide.com Page 232 of 331


Directions (716-720): A set of six sentences is given below. The first sentence of the passage is given in bolds.
Rearrange the remaining sentences in a logical sequence and then answer the following questions

In contrast, a failure of the summit might lead to a dangerous escalation in the region
A) Such a scenario might also bring North Korea closer into China’s fold and will most likely see the end of

South Korea’s rapprochement and a strengthening of the US-South Korean military alliance.
B) North Korea will most likely resume nuclear and long-range missile tests and also operationalize its
arsenal, while the US might feel compelled to take stringent actions, such as a quarantine of North Korean
ports and possibly military action against nuclear and missile targets.

C) This more belligerent approach in turn once again raises the spectre of a devastating nuclear confrontation
with global consequences.

D) First, unlike the previous historic summits, the level of preparation for the Kim-Trump summit is appallingly
poor.
E) While neither extreme scenario is likely to manifest, the prospect of failure far exceeds the chance of
success for a number of reasons.

716). Which of the following should be the THIRD sentence after the rearrangement?
(a) (E) (b) (C) (c) (A) (d) (B) (e) (D)

717). Which of the following should be the LAST sentence of the passage after rearrangement?

(a) (E) (b) (C) (c) (A) (d) (B) (e) (D)

718). Which of the following should be the SECOND sentence of the passage after rearrangement?
(a) (E) (b) (C) (c) (A) (d) (B) (e) (D)

719). Which of the following should be the FIRST sentence of the passage after rearrangement?

(a) (E) (b) (C) (c) (A) (d) (B) (e) (D)

720). Which of the following should be the FOURTH sentence of the passage after rearrangement?
(a) (E) (b) (C) (c) (A) (d) (B) (e) (D)

www.ibpsguide.com https://estore.ibpsguide.com Page 233 of 331


Directions (721-725): A set of six sentences is given below. The first sentence of the passage is given in bolds.

Rearrange the remaining sentences in a logical sequence and then answer the following questions

This is an odd policy choice

A) India, meanwhile, runs a current account deficit, implying a perennial reliance on overseas savings
B) It isn’t an idle query. The country’s basic balance of payments—the sum of the current account deficit and
net foreign direct investment—has turned negative

C) China has more than ample domestic savings to power its economy, but it wants to share credit risk
meaningfully with foreigners.
D) So why doesn’t the central bank allow more borrowing in domestic currency, which it can freely print, rather

than cling to riskier dollar inflows?


E) This is when crude oil, which India needs to import, is at only $66 a barrel. Should oil prices firm further,
and foreign portfolio investment in stocks and bonds ebb, the rupee would need to weaken to make Indian

assets attractive again

721). Which of the following should be the THIRD sentence after the rearrangement?
(a) (E) (b) (C) (c) (A) (d) (B) (e) (D)

722). Which of the following should be the LAST sentence of the passage after rearrangement?

(a) (E) (b) (C) (c) (A) (d) (B) (e) (D)

723). Which of the following should be the SECOND sentence of the passage after rearrangement?
(a) (E) (b) (C) (c) (A) (d) (B) (e) (D)

724). Which of the following should be the FIRST sentence of the passage after rearrangement?
(a) (E) (b) (C) (c) (A) (d) (B) (e) (D)

725). Which of the following should be the FOURTH sentence of the passage after rearrangement?
(a) (E) (b) (C) (c) (A) (d) (B) (e) (D)

Directions (726-730): A set of six sentences is given below. The first sentence of the passage is given in bolds.
Rearrange the remaining sentences in a logical sequence and then answer the following questions

www.ibpsguide.com https://estore.ibpsguide.com Page 234 of 331


While currency values can and do adjust relatively freely in developed economies, excessive exchange-rate

volatility in emerging markets tends to lead to a hard choice between capital controls and high local interest rates
A) That’s where aiming for index inclusion could help.

B) That’s what happened during the destabilizing taper-tantrum episode of 2013


C) But foreign money has already started leaving the Indian bond market; in the face of rising US interest
rates, especially a surging dollar Libor, India needs a new category of bond buyers.
D) This time around, the Reserve Bank of India is buying dollars to make sure the rupee doesn’t strengthen to

a point where investors lose confidence in its value.


E) Sovereign credit rating is the one benchmark Indian authorities obsess over endlessly. Indexes— both

equity and debt—don’t get as much attention

726). Which of the following should be the THIRD sentence after the rearrangement?
(a) (E) (b) (C) (c) (A) (d) (B) (e) (D)

727). Which of the following should be the LAST sentence of the passage after rearrangement?
(a) (E) (b) (C) (c) (A) (d) (B) (e) (D)

728). Which of the following should be the SECOND sentence of the passage after rearrangement?
(a) (E) (b) (C) (c) (A) (d) (B) (e) (D)

729). Which of the following should be the FIRST sentence of the passage after rearrangement?
(a) (E) (b) (C) (c) (A) (d) (B) (e) (D)

730). Which of the following should be the FOURTH sentence of the passage after rearrangement?
(a) (E) (b) (C) (c) (A) (d) (B) (e) (D)

Directions (731-735): A set of six sentences is given below. The first sentence of the passage is given in bolds.
Rearrange the remaining sentences in a logical sequence and then answer the following questions

Over the past several years, the world’s attention has been focused on China’s excessive credit buildup.
A) But the number of steps Beijing has taken to prevent an accident—from shuttering excess capacity in steel

and aluminum and stopping a one-way bet on yuan appreciation to squeezing easy liquidity and going

www.ibpsguide.com https://estore.ibpsguide.com Page 235 of 331


after sales of wealth management products masked as insurance—have all helped maintain a semblance

of calm.
B) Underneath, there still are fault lines, for instance in rapid growth of consumer credit

C) But the mess in India’s banking system, hobbled by more than $200 billion in stressed assets, is many

times more pressing than any challenge facing China.


D) One of the solutions for making banks healthy in the long run would be to let active, deep onshore bond

markets put a price on credit risk.


E) The price tag need not be prohibitive, provided India drops its historical aversion to being in debt to
foreigners. Mimicking China’s playbook on index inclusion would be a good starting point.

731). Which of the following should be the THIRD sentence after the rearrangement?
(a) (E) (b) (C) (c) (A) (d) (B) (e) (D)

732). Which of the following should be the LAST sentence of the passage after rearrangement?
(a) (E) (b) (C) (c) (A) (d) (B) (e) (D)

733). Which of the following should be the SECOND sentence of the passage after rearrangement?
(a) (E) (b) (C) (c) (A) (d) (B) (e) (D)

734). Which of the following should be the FIRST sentence of the passage after rearrangement?
(a) (E) (b) (C) (c) (A) (d) (B) (e) (D)

735). Which of the following should be the FOURTH sentence of the passage after rearrangement?
(a) (E) (b) (C) (c) (A) (d) (B) (e) (D)

Directions (736-740): A set of six sentences is given below. The first sentence of the passage is given in bolds.
Rearrange the remaining sentences in a logical sequence and then answer the following questions

As it turns out, the rather stormy relationship between Facebook and your data just got a bit worse.
A) Facebook, in its statement, denies that the data was collected covertly, and is adamant that the users

themselves gave the app permission to access contact details

B) “Call and text history logging is part of an opt-in feature for people using Messenger or Facebook Lite on
Android

www.ibpsguide.com https://estore.ibpsguide.com Page 236 of 331


C) This helps you find and stay connected with the people you care about, and provide you with a better

experience across Facebook.People have to expressly agree to use this feature,” says Facebook, in an
official statement.

D) It is believed that the data collected includes the time of calls, the names and numbers of call recipients,

and call duration, as well as text message metadata such as the recipient details and the time the
message was sent—none of which was done through the Facebook app itself.

E) The social media network is now facing raised eyebrows, and a lot of questions, after it was reported that
the Facebook app on Android phones had been collecting your call and SMS text message history for
quite some time now

736). Which of the following should be the THIRD sentence after the rearrangement?
(a) (E) (b) (C) (c) (A) (d) (B) (e) (D)

737). Which of the following should be the LAST sentence of the passage after rearrangement?
(a) (E) (b) (C) (c) (A) (d) (B) (e) (D)

738). Which of the following should be the SECOND sentence of the passage after rearrangement?
(a) (E) (b) (C) (c) (A) (d) (B) (e) (D)

739). Which of the following should be the FIRST sentence of the passage after rearrangement?
(a) (E) (b) (C) (c) (A) (d) (B) (e) (D)

740). Which of the following should be the FOURTH sentence of the passage after rearrangement?
(a) (E) (b) (C) (c) (A) (d) (B) (e) (D)

Directions (741-745): A set of six sentences is given below. The first sentence of the passage is given in bolds.

Rearrange the remaining sentences in a logical sequence and then answer the following questions

Global card companies like Visa Inc. and MasterCard Inc. are losing market share to upstarts in the world’s most

innovative payments market: India


A) The surge in UPI transactions has taken place since 2016 when the interface was set up by an umbrella
organization of Indian banks

www.ibpsguide.com https://estore.ibpsguide.com Page 237 of 331


B) The introduction of UPI in August 2016 led to the creation of a wealth of new innovative payment solutions,

and the adoption rates of UPI payments are truly spectacular,” US-based Fidelity National Information
Services Inc. said in a December 2017 report.

C) Transactions through India’s homegrown Unified Payments Interface (UPI)—which allows mobile apps run

by retailers, airlines and other firms to take payment directly from bank accounts—reached almost half the
value of debit and credit cards swiped at stores last month, central bank data shows.

D) UPI “opens up access to real-time by allowing payments to be directly integrated into external business
applications,” it said.
E) India’s payments system was alone among more than 40 countries tracked by Fidelity National to gain a

top score of five for innovation and customer value.

741). Which of the following should be the THIRD sentence after the rearrangement?
(a) (E)
(b) (C)
(c) (A)
(d) (B)
(e) (D)

742). Which of the following should be the LAST sentence of the passage after rearrangement?
(a) (E)
(b) (C)

(c) (A)
(d) (B)

(e) (D)

743). Which of the following should be the SECOND sentence of the passage after rearrangement?

(a) (E)
(b) (C)
(c) (A)

(d) (B)

(e) (D)

744). Which of the following should be the FIRST sentence of the passage after rearrangement?

www.ibpsguide.com https://estore.ibpsguide.com Page 238 of 331


(a) (E)

(b) (C)
(c) (A)

(d) (B)

(e) (D)

745). Which of the following should be the FOURTH sentence of the passage after rearrangement?

(a) (E)
(b) (C)

(c) (A)

(d) (B)
(e) (D)

Directions (746-750): A set of six sentences is given below. The first sentence of the passage is given in bolds.
Rearrange the remaining sentences in a logical sequence and then answer the following questions

After a five day wait, Facebook founder and CEO, Mark Zuckerberg, has finally broken his silence on the crisis
involving data firm Cambridge Analytica reportedly accessing information of more than 50 million Facebook users,

without their consent.


A) We have a responsibility to protect your data, and if we can’t then we don’t deserve to serve you,” wrote
Zuckerberg in a lengthy Facebook post, before adding, “I’ve been working to understand exactly what

happened and how to make sure this doesn’t happen again


B) The good news is that the most important actions to prevent this from happening again today we have
already taken years ago.

C) But we also made mistakes, there’s more to do, and we need to step up and do it
D) In what was a coordinated post timed with Zuckerberg’s post, Facebook COO Sheryl Sandberg said, “As

he said, we know that this was a major violation of people’s trust, and I deeply regret that we didn’t do

enough to deal with it.


E) We have a responsibility to protect your data - and if we can’t, then we don’t deserve to serve you.”

746). Which of the following should be the THIRD sentence after the rearrangement?
(a) (E) (b) (C) (c) (A) (d) (B) (e) (D)

www.ibpsguide.com https://estore.ibpsguide.com Page 239 of 331


747). Which of the following should be the LAST sentence of the passage after rearrangement?

(a) (E) (b) (C) (c) (A) (d) (B) (e) (D)

748). Which of the following should be the SECOND sentence of the passage after rearrangement?

(a) (E) (b) (C) (c) (A) (d) (B) (e) (D)

749). Which of the following should be the FIRST sentence of the passage after rearrangement?

(a) (E) (b) (C) (c) (A) (d) (B) (e) (D)

750). Which of the following should be the FOURTH sentence of the passage after rearrangement?

(a) (E) (b) (C) (c) (A) (d) (B) (e) (D)

Directions (751-755): A set of six sentences is given below. The first sentence of the passage is given in bolds.
Rearrange the remaining sentences in a logical sequence and then answer the following questions

India’s payments system was alone among more than 40 countries tracked by Fidelity National to gain a top score
of five for innovation and customer value.
A) Ola, a local rival to Uber Technologies Inc., Big Bazaar, a chain of stores run by billionaire Kishore Biyani,
and incumbent mobile wallet leader Paytm Mobile Solutions Pvt. are other users.
B) China’s Internet Banking Payment System scored two and Kenya’s PesaLink scored four

C) Amazon.com Inc. and Jet Airways India Ltd—the nation’s second-biggest airline by passengers—are
among firms that have integrated UPI into their apps in order to take payment from customers

D) Facebook Inc. is piloting a payments service in India with a UPI backbone for its WhatsApp Pay, which
has already drawn comparisons with the way WeChat reshaped payments in China

E) Currently, India’s payments market is worth less than $200 billion, dwarfed by China’s $27 trillion market
that’s now opening up to global players.

751). Which of the following should be the THIRD sentence after the rearrangement?
(a) (E) (b) (C) (c) (A) (d) (B) (e) (D)

www.ibpsguide.com https://estore.ibpsguide.com Page 240 of 331


752). Which of the following should be the LAST sentence of the passage after rearrangement?

(a) (E) (b) (C) (c) (A) (d) (B) (e) (D)

753). Which of the following should be the SECOND sentence of the passage after rearrangement?

(a) (E) (b) (C) (c) (A) (d) (B) (e) (D)

754). Which of the following should be the FIRST sentence of the passage after rearrangement?

(a) (E) (b) (C) (c) (A) (d) (B) (e) (D)

755). Which of the following should be the FOURTH sentence of the passage after rearrangement?

(a) (E) (b) (C) (c) (A) (d) (B) (e) (D)

Directions (756-760): A set of six sentences is given below. The first sentence of the passage is given in bolds.
Rearrange the remaining sentences in a logical sequence and then answer the following questions

One challenge in India is the dominance of cash, which accounts for some 70% of the country’s total transactions
by value.
A) The same shift may play out in India, where data usage for 300 million Indian smartphone users rose to 5-
10 GB a month from 1GB last year.
B) Advances in India’s digital payments market picked up pace after Prime Minister Narendra Modi’s shock
decision to invalidate 86% of the country’s currency in circulation

C) While the effects of the demonetization event of November 2016 are now waning, it would be safe to
conclude that the event has caused a permanent uplift in the share of non-cash transactions in the

economy,” said Credit Suisse.


D) However, China transitioned to digital on the back of rising mobile and data penetration, and that process
was hastened by e-commerce and social platforms, according to Credit Suisse

E) The number of UPI transactions have surged almost 57,000% since November 2016, while that of debit
and credit cards has risen 20%.

756). Which of the following should be the THIRD sentence after the rearrangement?
(a) (E) (b) (C) (c) (A) (d) (B) (e) (D)

757). Which of the following should be the LAST sentence of the passage after rearrangement?

www.ibpsguide.com https://estore.ibpsguide.com Page 241 of 331


(a) (E) (b) (C) (c) (A) (d) (B) (e) (D)

758). Which of the following should be the SECOND sentence of the passage after rearrangement?

(a) (E) (b) (C) (c) (A) (d) (B) (e) (D)

759). Which of the following should be the FIRST sentence of the passage after rearrangement?

(a) (E) (b) (C) (c) (A) (d) (B) (e) (D)

760). Which of the following should be the FOURTH sentence of the passage after rearrangement?

(a) (E) (b) (C) (c) (A) (d) (B) (e) (D)

Directions (761-765): A set of six sentences is given below. The first sentence of the passage is given in bolds.

Rearrange the remaining sentences in a logical sequence and then answer the following questions

The Mumbai city recorded the second highest temperature in March in a decade on Sunday, the India
Meteorological Department (IMD) said.
A) This made Sunday the second hottest day in March this decade, getting within a touching distance of 41.3
degrees Celsius recorded on 17 March 2011.

B) The mercury on Sunday soared to 41 degrees Celsius, IMD director Shubhangi Bhute told PTI on Monday.
C) The next few days are likely to be hot in Mumbai, with the temperatures likely to hover around 41 degrees

Celsius, according to the IMD.


D) A low-level inversion led to a spike in temperatures. The easterly winds don’t let the sea breeze effect to
set in, which is why the temperatures rise,” the official said.
E) . The highest ever temperature recorded this month in Mumbai was on 28 March 1956 when the mercury

reached 41.7 degrees C, Bhute said.

761). Which of the following should be the THIRD sentence after the rearrangement?
(a) (E) (b) (C) (c) (A) (d) (B) (e) (D)

762). Which of the following should be the LAST sentence of the passage after rearrangement?
(a) (E) (b) (C) (c) (A) (d) (B) (e) (D)

763). Which of the following should be the SECOND sentence of the passage after rearrangement?

www.ibpsguide.com https://estore.ibpsguide.com Page 242 of 331


(a) (E) (b) (C) (c) (A) (d) (B) (e) (D)

764). Which of the following should be the FIRST sentence of the passage after rearrangement?

(a) (E) (b) (C) (c) (A) (d) (B) (e) (D)

765). Which of the following should be the FOURTH sentence of the passage after rearrangement?

(a) (E) (b) (C) (c) (A) (d) (B) (e) (D)

Directions (766-770): A set of six sentences is given below. The first sentence of the passage is given in bolds.

Rearrange the remaining sentences in a logical sequence and then answer the following questions

Opinion polls published on Sunday in the United States and Germany indicated that a majority of the public were

losing trust in Facebook over privacy, as the firm ran advertisements in British and US newspapers apologizing to
users

A) Facebook founder and chief executive Mark Zuckerberg apologized for “a breach of trust” in
advertisements placed in papers including the Observer in Britain and the New York Times, Washington

Post and Wall Street Journal.


B) We have a responsibility to protect your information. If we can’t, we don’t deserve it,” said the
advertisement, which appeared in plain text on a white background with a tiny Facebook logo
C) Fewer than half of Americans trust Facebook to obey US privacy laws, according to a Reuters/Ipsos poll

released on Sunday, while a survey published byBild am Sonntag, Germany’s largest-selling Sunday
paper, found 60% of Germans fear that Facebook and other social networks are having a negative impact

on democracy
D) The world’s largest social media network is coming under growing government scrutiny in Europe and the
United States, and is trying to repair its reputation among users, advertisers, lawmakers and investors

E) This follows allegations that the British consultancy Cambridge Analytica improperly gained access to
users’ information to build profiles of American voters that were later used to help elect US President
Donald Trump in 2016.

766). Which of the following should be the THIRD sentence after the rearrangement?
(a) (E) (b) (C) (c) (A) (d) (B) (e) (D)

www.ibpsguide.com https://estore.ibpsguide.com Page 243 of 331


767). Which of the following should be the LAST sentence of the passage after rearrangement?

(a) (E) (b) (C) (c) (A) (d) (B) (e) (D)

768). Which of the following should be the SECOND sentence of the passage after rearrangement?

(a) (E) (b) (C) (c) (A) (d) (B) (e) (D)

769). Which of the following should be the FIRST sentence of the passage after rearrangement?

(a) (E) (b) (C) (c) (A) (d) (B) (e) (D)

770). Which of the following should be the FOURTH sentence of the passage after rearrangement?

(a) (E) (b) (C) (c) (A) (d) (B) (e) (D)

Directions (771-775): A set of six sentences is given below. The first sentence of the passage is given in bolds.
Rearrange the remaining sentences in a logical sequence and then answer the following questions

Ride-hailing firm Uber Technologies Inc. has agreed to sell its Southeast Asian business to bigger regional rival
Grab, the firms said on Monday, marking the US Company’s second retreat from an Asian market.
A) A shake-up in Asia’s fiercely competitive ride-hailing industry became likely earlier this year when Japan-
based SoftBank Group Corp’s Vision Fund made a multi-billion dollar investment in Uber. SoftBank also
invested in Grab.
B) As part of the transaction, Uber will take a 27.5% stake in Singapore-based Grab and Uber CEO Dara

Khosrowshahi will join Grab’s board.


C) The deal is the industry’s first big consolidation in Southeast Asia, home to about 640 million people, and

puts pressure on Indonesia’s Go-Jek, which is backed by Alphabet Inc’s Google and China’s Tencent
Holdings Ltd
D) For Grab, the deal is a boon for its meal-delivery service, which will now merge with Uber Eats. A more

robust food service will give Grab an advantage over Go-Jek, according to a person close to Grab
E) It will help us double down on our plans for growth as we invest heavily in our products and technology,”
Khosrowshahi said in a statement.

771). Which of the following should be the THIRD sentence after the rearrangement?
(a) (E) (b) (C) (c) (A) (d) (B) (e) (D)

www.ibpsguide.com https://estore.ibpsguide.com Page 244 of 331


772). Which of the following should be the LAST sentence of the passage after rearrangement?

(a) (E) (b) (C) (c) (A) (d) (B) (e) (D)

773). Which of the following should be the SECOND sentence of the passage after rearrangement?

(a) (E) (b) (C) (c) (A) (d) (B) (e) (D)

774). Which of the following should be the FIRST sentence of the passage after rearrangement?

(a) (E) (b) (C) (c) (A) (d) (B) (e) (D)

775). Which of the following should be the FOURTH sentence of the passage after rearrangement?

(a) (E) (b) (C) (c) (A) (d) (B) (e) (D)

Directions (776-780): A set of six sentences is given below. The first sentence of the passage is given in bolds.
Rearrange the remaining sentences in a logical sequence and then answer the following questions

Allegations that Prime Minister Narendra Modi’s official mobile application was sending personal user data to a
third party without their consent caused a furore on social media in India and drew criticism from the leader of the
opposition Congress party on Sunday.

A) Modi’s ruling Bharatiya Janata Party (BJP) denied the allegations and said the data was being used only
for analytics to offer all users the “most contextual content”.

B) A security researcher, who has previously highlighted some vulnerabilities in India’s national identity card
project and who tweets under the pseudonym Elliot Alderson, posted a series of tweets on Saturday
stating the app was sending personal user data to a third-party domain that was traced to an American
company.

C) The tweets, which come at a time of heightened sensitivity around the alleged misuse of personal data
amid the unfolding Facebook-Cambridge Analytica controversy, triggered a stir in India on social media

D) Hi! My name is Narendra Modi. I am India’s Prime Minister. When you sign up for my official App, I give all

your data to my friends in American companies,” wrote opposition Congress party chief Rahul Gandhi in a
Twitter message on Sunday
E) The BJP quickly responded on Twitter, saying Gandhi was trying to divert attention. The BJP has accused
the Congress of engaging Cambridge Analytica in India, a charge the opposition party has denied.

www.ibpsguide.com https://estore.ibpsguide.com Page 245 of 331


776). Which of the following should be the THIRD sentence after the rearrangement?
(a) (E) (b) (C) (c) (A) (d) (B) (e) (D)

777). Which of the following should be the LAST sentence of the passage after rearrangement?
(a) (E) (b) (C) (c) (A) (d) (B) (e) (D)

778). Which of the following should be the SECOND sentence of the passage after rearrangement?

(a) (E) (b) (C) (c) (A) (d) (B) (e) (D)

779). Which of the following should be the FIRST sentence of the passage after rearrangement?

(a) (E) (b) (C) (c) (A) (d) (B) (e) (D)

780). Which of the following should be the FOURTH sentence of the passage after rearrangement?
(a) (E) (b) (C) (c) (A) (d) (B) (e) (D)

Directions (781-785): A set of six sentences is given below. The first sentence of the passage is given in bolds.
Rearrange the remaining sentences in a logical sequence and then answer the following questions

The latest incident has led to a massive reaction from users as well as parts of the tech fraternity, with doubts
over how Facebook could let a third-party access their own user data without taking explicit consent.
A) The information was later transferred to third parties, including Cambridge Analytica, in violation of
Facebook’s policies. Zuckerberg outlined the timeline of incidents

B) Facebook had earlier confirmed that the data was initially collected by a professor for academic purposes

in line with its rules.


C) Many are angry at why Facebook didn’t react sooner, or do more, to ensure that user data was safe.

D) There is now a #deletefacebook campaign trending on social media, something backed by the WhatsApp
co-founder Brian Acton, whom as it turns out, Facebook made a billionaire with the WhatsApp acquisition.
E) This isn’t the first time Facebook has been facing the wrath of users. As recently as 2014, Facebook faced
a backlash after it was revealed that the social network was running secret psychological experiments on

users, and attempting to tweak the content that they see on their News Feeds and alter their reactions

781). Which of the following should be the THIRD sentence after the rearrangement?

www.ibpsguide.com https://estore.ibpsguide.com Page 246 of 331


(a) (E) (b) (C) (c) (A) (d) (B) (e) (D)

782). Which of the following should be the LAST sentence of the passage after rearrangement?
(a) (E) (b) (C) (c) (A) (d) (B) (e) (D)

783). Which of the following should be the SECOND sentence of the passage after rearrangement?

(a) (E) (b) (C) (c) (A) (d) (B) (e) (D)

784). Which of the following should be the FIRST sentence of the passage after rearrangement?

(a) (E) (b) (C) (c) (A) (d) (B) (e) (D)

785). Which of the following should be the FOURTH sentence of the passage after rearrangement?
(a) (E) (b) (C) (c) (A) (d) (B) (e) (D)

Directions (786-790): A set of six sentences is given below. The first sentence of the passage is given in bolds.
Rearrange the remaining sentences in a logical sequence and then answer the following questions

It’s been a busy day for former Karnataka chief minister H.D. Kumaraswamy.

A) A lot of complaints made to the 58-year-old are about candidate selection—for which Kumaraswamy’s
father and former prime minister H.D. Deve Gowda is the final authority

B) With barely two months to go for the Karnataka assembly elections, Kumaraswamy and JD(S) are
preparing to fight heavyweights, Siddaramaiah-led Congress and Narendra Modi-led Bharatiya Janata
Party (BJP).
C) The state president of the Janata Dal (Secular) or JD(S) has held back-to-back meetings with hundreds of

people from different constituencies at his Bengaluru home


D) Unlike the BJP and Congress, who have spent their campaigns targeting each other, the JD(S) strategy is

to ask people for an opportunity to govern the state.

E) Known to have a reputation as “Kingmakers” for having allied with both the Congress and the BJP in the
past when there have been fractured verdicts, the party is now seeking a majority on its own.

786). Which of the following should be the THIRD sentence after the rearrangement?
(a) (E) (b) (C) (c) (A) (d) (B) (e) (D)

www.ibpsguide.com https://estore.ibpsguide.com Page 247 of 331


787). Which of the following should be the LAST sentence of the passage after rearrangement?

(a) (E) (b) (C) (c) (A) (d) (B) (e) (D)

788). Which of the following should be the SECOND sentence of the passage after rearrangement?

(a) (E) (b) (C) (c) (A) (d) (B) (e) (D)

789). Which of the following should be the FIRST sentence of the passage after rearrangement?

(a) (E) (b) (C) (c) (A) (d) (B) (e) (D)

790). Which of the following should be the FOURTH sentence of the passage after rearrangement?

(a) (E) (b) (C) (c) (A) (d) (B) (e) (D)

Directions (791-795): A set of six sentences is given below. The first sentence of the passage is given in bolds.
Rearrange the remaining sentences in a logical sequence and then answer the following questions

Over the last few years, the emerging closeness between India and Australia has often made news, but such
enthusiasm has been, at times, short-lived. .

A) Historically, a key problem with Australia’s bilateral relationships has been the fundamental dichotomy of
its foreign policy, to wit, the misalignment of Australia’s economic and political-security interests

B) This translates into hard choices for Australia, an issue that has gained prominence in the eyes of many
international observers.
C) While Australia is reliant on the US for its defence and security through the Australia-New Zealand-US
treaty, its economy depends on China, which accounts for the lion’s share of bilateral trade and investment

Down Under
D) Australia and India are not exceptions in this regard. Moreover, the rebirth of the Quadrilateral Security

Dialogue (commonly known as the Quad) in November last year and the importance given to India in the

2017 Australian Foreign Policy White Paper, provide an even broader canvas for a sustainable, upward
trajectory in India-Australia relations.
E) However, with the Indo-Pacific in flux—results of China’s growing assertiveness as well as uncertainty
about American leadership—middle and emerging powers in the region find themselves contemplating the
need to join hands.

www.ibpsguide.com https://estore.ibpsguide.com Page 248 of 331


791). Which of the following should be the THIRD sentence after the rearrangement?
(a) (E) (b) (C) (c) (A) (d) (B) (e) (D)

792). Which of the following should be the LAST sentence of the passage after rearrangement?
(a) (E) (b) (C) (c) (A) (d) (B) (e) (D)

793). Which of the following should be the SECOND sentence of the passage after rearrangement?

(a) (E) (b) (C) (c) (A) (d) (B) (e) (D)

794). Which of the following should be the FIRST sentence of the passage after rearrangement?

(a) (E) (b) (C) (c) (A) (d) (B) (e) (D)

795). Which of the following should be the FOURTH sentence of the passage after rearrangement?
(a) (E) (b) (C) (c) (A) (d) (B) (e) (D)

Directions (796-800): A set of six sentences is given below. The first sentence of the passage is given in bolds.
Rearrange the remaining sentences in a logical sequence and then answer the following questions

Uber anticipated making more deals with rivals, but said it had no plans to do another sale in which it consolidates
its operations in exchange for a minority stake in a rival
A) “One of the potential dangers of our global strategy is that we take on too many battles across too many
fronts and with too many competitors
B) India accounts for more than 10% of Uber’s trips globally, but the company is not making money there yet.

C) It is fair to ask whether consolidation is now the strategy of the day, given this is the third deal of its kind ...

The answer is no,” Khosrowshahi said in a note to employees that was shared with Reuters
D) Southeast Asia was really difficult for Uber. In India, that competition is not across so many different fronts,

“said Lauria a founding partner at Southeast Asia’s Golden Gate Ventures.


E) A source familiar with Uber’s strategy said the company was going to step up its battle with Ola in India,
another competitive and costly market where rivals have heavily subsidized rides in an effort to gain

market share. Uber has close to 60% of the market there, by some estimates

796). Which of the following should be the THIRD sentence after the rearrangement?
(a) (E) (b) (C) (c) (A) (d) (B) (e) (D)

www.ibpsguide.com https://estore.ibpsguide.com Page 249 of 331


797). Which of the following should be the LAST sentence of the passage after rearrangement?
(a) (E) (b) (C) (c) (A) (d) (B) (e) (D)

798). Which of the following should be the SECOND sentence of the passage after rearrangement?
(a) (E) (b) (C) (c) (A) (d) (B) (e) (D)

799). Which of the following should be the FIRST sentence of the passage after rearrangement?
(a) (E) (b) (C) (c) (A) (d) (B) (e) (D)

800). Which of the following should be the FOURTH sentence of the passage after rearrangement?
(a) (E) (b) (C) (c) (A) (d) (B) (e) (D)

Directions (801-805): A set of six sentences is given below. The first sentence of the passage is given in bolds.
Rearrange the remaining sentences in a logical sequence and then answer the following questions

Sundar Pinchai, chief executive officer of Google, expects China to play a crucial role in artificial intelligence (AI)
as he keeps expanding the search giant’s workforce in the country, even as many of its services are blocked.

A) Google has invested in Chinese startups, forged a patent alliance with Tencent Holdings Ltd and is
pushing its Tensor Flow AI tools in the country despite key services such as search and email remaining

blocked
B) The Mountain View, California-based company recently opened a research lab in Beijing focused on AI, a
blossoming field but one at the centre of tensions between China and the US.
C) “China is already playing a big part in how AI will shape our futures,” Pichai said at the annual China

Development Forum in Beijing. “When we build together we get to better ideas faster
D) We’ve already got a small team doing research there and look forward to expanding it,” he told an

audience at the conference that was attended by senior government officials and executives including

Apple CEO Tim Cook


E) In 2010, Google refused to censor search results with China’s internet censors later blocking access. Now
Google and Facebook Inc. are among foreign giants who regularly visit in an effort to curry favour and
regain access to the lucrative market

801). Which of the following should be the THIRD sentence after the rearrangement?

www.ibpsguide.com https://estore.ibpsguide.com Page 250 of 331


(a) (E) (b) (C) (c) (A) (d) (B) (e) (D)

802). Which of the following should be the LAST sentence of the passage after rearrangement?
(a) (E) (b) (C) (c) (A) (d) (B) (e) (D)

803). Which of the following should be the SECOND sentence of the passage after rearrangement?

(a) (E) (b) (C) (c) (A) (d) (B) (e) (D)

804). Which of the following should be the FIRST sentence of the passage after rearrangement? (a) (E)

(b) (C) (c) (A) (d) (B) (e) (D)

805). Which of the following should be the FOURTH sentence of the passage after rearrangement?
(a) (E) (b) (C) (c) (A) (d) (B) (e) (D)

Directions (806-810): A set of six sentences is given below. The first sentence of the passage is given in bolds.
Rearrange the remaining sentences in a logical sequence and then answer the following questions

Like in Australia, in India too there is a growing wariness around President Donald Trump’s vacillations on China.

A) There is a perceptible fear in New Delhi of the prospect that Trump’s “deal-making” temperament could
lead to an American quid-pro-quo arrangement with the Chinese.

B) This would be detrimental to Indian national and regional interests.


C) This would be detrimental to Indian national and regional interests. Therefore, in the pursuit of strategic
autonomy—the holy grail of India’s foreign policy—the logical course of action for India would be to
diversify its security partners, and enter into bilateral and trilateral defense arrangements to augment its

existing strategic reach and capability


D) Allowing Australia to join this year’s edition would go a long way in strengthening bilateral relations as well

laying the foundations of a strong Quad.

E) A solid partnership with Australia should figure prominently in this context. Although India does hold a
bilateral naval exercise, AUSINDEX, with Australia, it is nowhere close to the Malabar exercise in terms of
its scale and depth.

806). Which of the following should be the THIRD sentence after the rearrangement?
(a) (E) (b) (C) (c) (A) (d) (B) (e) (D)

www.ibpsguide.com https://estore.ibpsguide.com Page 251 of 331


807). Which of the following should be the LAST sentence of the passage after rearrangement?
(a) (E) (b) (C) (c) (A) (d) (B) (e) (D)

808). Which of the following should be the SECOND sentence of the passage after rearrangement?
(a) (E) (b) (C) (c) (A) (d) (B) (e) (D)

809). Which of the following should be the FIRST sentence of the passage after rearrangement?
(a) (E) (b) (C) (c) (A) (d) (B) (e) (D)

810). Which of the following should be the FOURTH sentence of the passage after rearrangement?
(a) (E) (b) (C) (c) (A) (d) (B) (e) (D)

Directions (811-815): A set of six sentences is given below. The first sentence of the passage is given in bolds.
Rearrange the remaining sentences in a logical sequence and then answer the following questions

Two years after an India-led boycott of a South Asian regional summit that was to be hosted by Pakistan,
Islamabad seems to be working up support among smaller countries in South Asia for the meeting.

A) But the idea of a Saarc summit is unlikely to appeal to India that holds Pakistan responsible for many
terrorist acts in the country, said analysts

B) Pakistan was to host the 19th Saarc summit in November 2016. A series of attacks on Indian military
installations, however, in 2016, starting with the strike on Pathankot in January, and another in Uri, put
paid to some nascent efforts to get the India-Pakistan peace dialogue—stalled since 2013—back on track.
India pulled out of the summit followed by Bangladesh, Bhutan, Afghanistan and others in the region

C) In recent weeks, Sri Lanka and Nepal have voiced support for the South Asian Association of Regional
Cooperation or Saarc summit in Islamabad this year.

D) India pulled out of the summit followed by Bangladesh, Bhutan, Afghanistan and others in the region.

E) In October, in a rather pointed message to Pakistan, India invited leaders of the BIMSTEC regional
grouping—straddling South and Southeast Asia—for an outreach meeting with Brics leaders in Goa

811). Which of the following should be the THIRD sentence after the rearrangement?
(a) (E) (b) (C) (c) (A) (d) (B) (e) (D)

www.ibpsguide.com https://estore.ibpsguide.com Page 252 of 331


812). Which of the following should be the LAST sentence of the passage after rearrangement?

(a) (E) (b) (C) (c) (A) (d) (B) (e) (D)

813). Which of the following should be the SECOND sentence of the passage after rearrangement?

(a) (E) (b) (C) (c) (A) (d) (B) (e) (D)

814). Which of the following should be the FIRST sentence of the passage after rearrangement?

(a) (E) (b) (C) (c) (A) (d) (B) (e) (D)

815). Which of the following should be the FOURTH sentence of the passage after rearrangement?

(a) (E) (b) (C) (c) (A) (d) (B) (e) (D)

Directions (816-820): A set of six sentences is given below. The first sentence of the passage is given in bolds.
Rearrange the remaining sentences in a logical sequence and then answer the following questions

Like in Australia, in India too there is a growing wariness around President Donald Trump’s vacillations on China.
A) There is a perceptible fear in New Delhi of the prospect that Trump’s “deal-making” temperament could
lead to an American quid-pro-quo arrangement with the Chinese

B) A solid partnership with Australia should figure prominently in this context. Although India does hold a
bilateral naval exercise, AUSINDEX, with Australia, it is nowhere close to the Malabar exercise in terms of
its scale and depth

C) Allowing Australia to join this year’s edition would go a long way in strengthening bilateral relations as well
laying the foundations of a strong Quad.
D) This would be detrimental to Indian national and regional interests.

E) Therefore, in the pursuit of strategic autonomy—the holy grail of India’s foreign policy—the logical course
of action for India would be to diversify its security partners, and enter into bilateral and trilateral defense

arrangements to augment its existing strategic reach and capability.

816). Which of the following should be the THIRD sentence after the rearrangement?
(a) (E) (b) (C) (c) (A) (d) (B) (e) (D)

817). Which of the following should be the LAST sentence of the passage after rearrangement?

(a) (E) (b) (C) (c) (A) (d) (B) (e) (D)

www.ibpsguide.com https://estore.ibpsguide.com Page 253 of 331


818). Which of the following should be the SECOND sentence of the passage after rearrangement?
(a) (E) (b) (C) (c) (A) (d) (B) (e) (D)

819). Which of the following should be the FIRST sentence of the passage after rearrangement?
(a) (E) (b) (C) (c) (A) (d) (B) (e) (D)

820). Which of the following should be the FOURTH sentence of the passage after rearrangement?
(a) (E) (b) (C) (c) (A) (d) (B) (e) (D)

Directions (821-825): A set of six sentences is given below. The first sentence of the passage is given in bolds.
Rearrange the remaining sentences in a logical sequence and then answer the following questions

Defense minister Nirmala Sitharaman has said that India was “alert” and ready to deal with any “unforeseen
situation” in Dolman.
A) We will maintain our territorial integrity,” she told reporters on the sidelines of a function at the CM
residence in Dehradun on Sunday to honor families of distinguished Army men, war widows and veterans.

B) The defense minister, who was on a visit to Uttarakhand for the first time after assuming office, also said
that the government is constantly working towards modernization of forces
C) We are constantly working towards the modernization of our forces
D) “We are alert and ready for any unforeseen situation in dollar.

E) Addressing NDA and IMA cadets, she said they were going to be part of a highly modernized force as a
number of new initiatives have been taken by Prime Minister Narendra Modi to modernize the Army, Navy
and the Air Force.

821). Which of the following should be the THIRD sentence after the rearrangement?
(a) (E) (b) (C) (c) (A) (d) (B) (e) (D)

822). Which of the following should be the LAST sentence of the passage after rearrangement?
(a) (E) (b) (C) (c) (A) (d) (B) (e) (D)

823). Which of the following should be the SECOND sentence of the passage after rearrangement?
(a) (E) (b) (C) (c) (A) (d) (B) (e) (D)

www.ibpsguide.com https://estore.ibpsguide.com Page 254 of 331


824). Which of the following should be the FIRST sentence of the passage after rearrangement?

(a) (E) (b) (C) (c) (A) (d) (B) (e) (D)

825). Which of the following should be the FOURTH sentence of the passage after rearrangement?

(a) (E) (b) (C) (c) (A) (d) (B) (e) (D)

Directions (826-830): A set of six sentences is given below. The first sentence of the passage is given in bolds.
Rearrange the remaining sentences in a logical sequence and then answer the following questions

Donald Trump’s combative new foreign policy team should look at two precedents as the US president pursues
summits with the leaders of Russia and North Korea, according to former diplomats and historians.

A) One is now considered an historic success, the other an unmitigated disaster.


B) An earlier encounter in 1961 between John F. Kennedy and Nikita Khrushchev in Vienna came close to
causing World War III.
C) With a stalled arms-control regime, a broken US-Russian relationship and peace on the Korean peninsula
at stake, Trump’s meetings could help dictate whether the world slides further back toward the days of
missile proliferation and proxy conflicts.

D) Incoming National Security adviser John Bolton, who was named to the post last week, and CIA director
Mike Pompeo, Trump’s pick for secretary of state, have called for regime change in North Korea and are
hawks on Russia

E) Negotiations between Ronald Reagan and Mikhail Gorbachev in Reykjavik in 1983 helped reverse the
nuclear arms race and usher in an end to superpower confrontation.

826). Which of the following should be the THIRD sentence after the rearrangement?
(a) (E) (b) (C) (c) (A) (d) (B) (e) (D)

.
827). Which of the following should be the LAST sentence of the passage after rearrangement?
(a) (E) (b) (C) (c) (A) (d) (B) (e) (D)

828). Which of the following should be the SECOND sentence of the passage after rearrangement?
(a) (E) (b) (C) (c) (A) (d) (B) (e) (D)

www.ibpsguide.com https://estore.ibpsguide.com Page 255 of 331


829). Which of the following should be the FIRST sentence of the passage after rearrangement?

(a) (E) (b) (C) (c) (A) (d) (B) (e) (D)

.830). Which of the following should be the FOURTH sentence of the passage after rearrangement?

(a) (E) (b) (C) (c) (A) (d) (B) (e) (D)

Directions (831-835): A set of six sentences is given below. The first sentence of the passage is given in bolds.
Rearrange the remaining sentences in a logical sequence and then answer the following questions

Shashank Sinha’s vision for Strides Shasun Ltd, the Indian drug maker he runs, resembles a three-act play: It

opens with investment, then shifts to growth, and the grand finale: a possible sale.
A) Chief executive officer (CEO) Sinha estimates the current one, focused on selling generic drugs on
pharmacy shelves in the US and Australia, is now in its second act, though the ending for this one could
be the same as the last.
B) Strides has already built one business, sold it for about $1.65 billion, and is now ramping up its second.
C) “This is not a business that is being built for posterity, or for the third or the fourth generation of the
founders to run this company,” he said in an interview
D) The overall philosophy for this is that there will be a point in time when the value in this business is
optimized and then we are obviously open to all options.”
E) The drug maker’s current market capitalization stands at around $940 million. Sinha declined to put a
timeline on when the company could begin weighing a sale, and said he is focused on growth.

831). Which of the following should be the THIRD sentence after the rearrangement?
(a) (E) (b) (C) (c) (A) (d) (B) (e) (D)

832). Which of the following should be the LAST sentence of the passage after rearrangement?

(a) (E) (b) (C) (c) (A) (d) (B) (e) (D)

833). Which of the following should be the SECOND sentence of the passage after rearrangement?

(a) (E) (b) (C) (c) (A) (d) (B) (e) (D)

834). Which of the following should be the FIRST sentence of the passage after rearrangement?

(a) (E) (b) (C) (c) (A) (d) (B) (e) (D)

www.ibpsguide.com https://estore.ibpsguide.com Page 256 of 331


835). Which of the following should be the FOURTH sentence of the passage after rearrangement?
(a) (E) (b) (C) (c) (A) (d) (B) (e) (D)

Directions (836-840): A set of six sentences is given below. The first sentence of the passage is given in bolds.
Rearrange the remaining sentences in a logical sequence and then answer the following questions

The Bharatiya Janata Party’s (BJP’s) defeat in the recent UP Lou Sabha by polls has been mainly attributed to
the successful alliance between two erstwhile arch-rivals: the Samajwadi Party (SP) and the Bahujan Samaj Party

(BSP).

A) Evidence from post-poll surveys conducted by the Lokniti research programme at the New Delhi-based
Centre for the Study of Developing Societies (CSDS) suggests that the narrowing social base of both
parties since 2014 has made it politically pragmatic for them to bury differences and come together.

B) The brute arithmetic in favour of the alliance proved to be unassailable for the BJP even in Gorakhpur, a
seat held by the party since 1991, vacated recently by chief minister Yogi Adityanath.

C) The smooth sailing of the SP-BSP alliance in the UP bypolls and evident bonhomie among the leaders of
the two parties after tasting victory have raised expectations of a similar front being put up against the BJP
in Uttar Pradesh in the 2019 Lok Sabha elections

D) Our analysis also suggests that the combined electoral strength of the two parties will pose a formidable
challenge for the BJP in the country’s largest state, which sends 80 Lok Sabha members to the 543-strong

Lower House.

E) The decision of the SP and BSP to explore the possibility of an electoral alliance seems to be driven by
the threat that the BJP poses to their respective electoral bases.

836).Which of the following should be the THIRD sentence after the rearrangement?
(a) (E) (b) (C) (c) (A) (d) (B) (e) (D)

837). Which of the following should be the LAST sentence of the passage after rearrangement?
(a) (E) (b) (C) (c) (A) (d) (B) (e) (D)

838). Which of the following should be the SECOND sentence of the passage after rearrangement?
(a) (E) (b) (C) (c) (A) (d) (B) (e) (D)

www.ibpsguide.com https://estore.ibpsguide.com Page 257 of 331


839). Which of the following should be the FIRST sentence of the passage after rearrangement?

(a) (E) (b) (C) (c) (A) (d) (B) (e) (D)

840). Which of the following should be the FOURTH sentence of the passage after rearrangement?

(a) (E) (b) (C) (c) (A) (d) (B) (e) (D)

Directions (841-845): A set of six sentences is given below. The first sentence of the passage is given in bolds.
Rearrange the remaining sentences in a logical sequence and then answer the following questions

For the last three years, a few questions have been bothering the national psyche

A) Were the 39 Indians missing from Mosul in Iraq alive, or had the Islamic State (Islamic State) killed them in
their quest for the blood of innocents?
B) That 39 people were shot in front of his eyes, but Masih escaped saying that he was a Muslim
C) Harjit Masih, who returned from Mosul, claimed he, too, had been held hostage with them
D) His claim appeared to be flimsy
E) Do the killers from Islamic State spare someone just on the basis of being a Muslim? This is unlikely. In
fact, they have killed more Muslims than kafirs (non-believers). Religion has never been an obstacle to

their extremist frenzy and will never be one, since they are themselves involved in irreligious acts.

841).Which of the following should be the THIRD sentence after the rearrangement?
(a) (E) (b) (C) (c) (A) (d) (B) (e) (D)

842). Which of the following should be the LAST sentence of the passage after rearrangement?
(a) (E) (b) (C) (c) (A) (d) (B) (e) (D)

843). Which of the following should be the SECOND sentence of the passage after rearrangement?
(a) (E) (b) (C) (c) (A) (d) (B) (e) (D)

844). Which of the following should be the FIRST sentence of the passage after rearrangement?
(a) (E) (b) (C) (c) (A) (d) (B) (e) (D)

845). Which of the following should be the FOURTH sentence of the passage after rearrangement?

www.ibpsguide.com https://estore.ibpsguide.com Page 258 of 331


(a) (E) (b) (C) (c) (A) (d) (B) (e) (D)

Directions (846-850): A set of six sentences is given below. The first sentence of the passage is given in bolds.
Rearrange the remaining sentences in a logical sequence and then answer the following questions

The NDA government’s scheme to provide health cover of ₹5 lakh per year to 10 crore poor and vulnerable
families through the Ayushman Bharat-National Health Protection Mission has taken a step forward with the
Union Cabinet approving the modalities of its implementation
A) Considering the small window, just over a year, available before the term of the present government ends,

urgent action is needed to roll out such an ambitious scheme


B) For a start, the apex council that will steer the programme and the governing board to operationalize it in
partnership with the States need to be set up.
C) The States, which have a statutory responsibility for provision of health care, have to act quickly and form
dedicated agencies to run the scheme
D) This could be on the lines of legislation governing the rights to food and information

E) Since the NHPM represents the foundation for a universal health coverage system that should eventually
cover all Indians, it needs to be given a sound legal basis, ideally through a separate law.

846).Which of the following should be the THIRD sentence after the rearrangement?

(a) (E) (b) (C) (c) (A) (d) (B) (e) (D)

847). Which of the following should be the LAST sentence of the passage after rearrangement?

(a) (E) (b) (C) (c) (A) (d) (B) (e) (D)

848). Which of the following should be the SECOND sentence of the passage after rearrangement?
(a) (E) (b) (C) (c) (A) (d) (B) (e) (D)

849). Which of the following should be the FIRST sentence of the passage after rearrangement?

(a) (E) (b) (C) (c) (A) (d) (B) (e) (D)

850). Which of the following should be the FOURTH sentence of the passage after rearrangement?

(a) (E) (b) (C) (c) (A) (d) (B) (e) (D)

www.ibpsguide.com https://estore.ibpsguide.com Page 259 of 331


ANSWERS:

701. Correct Answer is: (a)


Explanation: the correct sequence is –ADEBC
702. Correct Answer is: (b)
Explanation: the correct sequence is –ADEBC
703. Correct Answer is: (e)

Explanation: the correct sequence is –ADEBC.

704. Correct Answer is: (c)


Explanation: the correct sequence is –ADEBC

705. Correct Answer is: (d)


Explanation: the correct sequence is –ADEBC
706. Correct Answer is: (c)
Explanation: the correct sequence is – CDABE.
707. Correct Answer is: (a)
Explanation: the correct sequence is – CDABE.

708. Correct Answer is: (e)


Explanation: the correct sequence is – CDABE.
709. Correct Answer is: (b)

Explanation: the correct sequence is – CDABE.


710. Correct Answer is: (d)
Explanation: the correct sequence is – CDABE.

711. Correct Answer is: (b)


Explanation: the correct sequence is –BECDA

712. Correct Answer is: (c)

Explanation: the correct sequence is –BECDA


713. Correct Answer is: (a)

Explanation: the correct sequence is –BECDA.

714. Correct Answer is: (d)


Explanation: the correct sequence is –BECDA

715. Correct Answer is: (e)

www.ibpsguide.com https://estore.ibpsguide.com Page 260 of 331


Explanation: the correct sequence is –BECDA

716. Correct Answer is: (b)


Explanation: the correct sequence is –BACED
717. Correct Answer is: (e)

Explanation: the correct sequence is –BACED


718. Correct Answer is: (c)
Explanation: the correct sequence is –BACED
719. Correct Answer is: (d)
Explanation: the correct sequence is –BACED

720. Correct Answer is: (a)

Explanation: the correct sequence is –BACED


721. Correct Answer is: (e)

Explanation: the correct sequence is – CADBE


722. Correct Answer is: (a)
Explanation: the correct sequence is – CADBE
723. Correct Answer is: (c)
Explanation: the correct sequence is – CADBE
724. Correct Answer is: (b)

Explanation: the correct sequence is – CADBE


725. Correct Answer is: (d)
Explanation: the correct sequence is – CADBE

726. Correct Answer is: (b)


Explanation: the correct sequence is –BDCAE
727. Correct Answer is: (a)

Explanation: the correct sequence is –BDCAE


728. Correct Answer is: (e)

Explanation: the correct sequence is –BDCAE

729. Correct Answer is: (d)


Explanation: the correct sequence is –BDCAE

730. Correct Answer is: (c)

Explanation: the correct sequence is –BDCAE


731. Correct Answer is: (b)

Explanation: the correct sequence is – ABCDE.

www.ibpsguide.com https://estore.ibpsguide.com Page 261 of 331


732. Correct Answer is: (a)

Explanation: the correct sequence is – ABCDE.


733. Correct Answer is: (d)
Explanation: the correct sequence is – ABCDE.

734. Correct Answer is: (c)


Explanation: the correct sequence is – ABCDE.
735. Correct Answer is: (e)
Explanation: the correct sequence is – ABCDE.
736. Correct Answer is: (c)

Explanation: the correct sequence is- EDABC

737. Correct Answer is: (b)


Explanation: the correct sequence is- EDABC

738. Correct Answer is: (e)


Explanation: the correct sequence is- EDABC.
739. Correct Answer is: (a)
Explanation: the correct sequence is- EDABC
740. Correct Answer is: (d)
Explanation: the correct sequence is- EDABC

741. Correct Answer is: (d)


Explanation: the correct sequence is – CABDE
742. Correct Answer is: (a)

Explanation: the correct sequence is – CABDE


743. Correct Answer is: (c)
Explanation: the correct sequence is – CABDE.

744. Correct Answer is: (b)


Explanation: the correct sequence is – CABDE

745. Correct Answer is: (e)

Explanation: the correct sequence is – CABDE


746. Correct Answer is: (b)

Explanation: the correct sequence is – ABCDE

747. Correct Answer is: (a)


Explanation: the correct sequence is – ABCDE

748. Correct Answer is: (d)

www.ibpsguide.com https://estore.ibpsguide.com Page 262 of 331


Explanation: the correct sequence is – ABCDE

749. Correct Answer is: (c)


Explanation: the correct sequence is – ABCDE
750. Correct Answer is: (e)

Explanation: the correct sequence is – ABCDE


751. Correct Answer is: (c)
Explanation: the correct sequence is – BCADE
752. Correct Answer is: (a)
Explanation: the correct sequence is – BCADE

753. Correct Answer is: (b)

Explanation: the correct sequence is – BCADE


754. Correct Answer is: (d)

Explanation: the correct sequence is – BCADE


755. Correct Answer is: (e)
Explanation: the correct sequence is – BCADE
756. Correct Answer is: (d)
Explanation: the correct sequence is – DABEC.
757. Correct Answer is: (b)

Explanation: the correct sequence is – DABEC.


758. Correct Answer is: (c)
Explanation: the correct sequence is – DABEC.

759. Correct Answer is: (e)


Explanation: the correct sequence is – DABEC.
760. Correct Answer is: (a)

Explanation: the correct sequence is – DABEC.


761. Correct Answer is: (a)

Explanation: the correct sequence is –BAECD

762. Correct Answer is: (e)


Explanation: the correct sequence is –BAECD.

763. Correct Answer is: (c)

Explanation: the correct sequence is –BAECD.


764. Correct Answer is: (d)

Explanation: the correct sequence is –BAECD.

www.ibpsguide.com https://estore.ibpsguide.com Page 263 of 331


765. Correct Answer is: (b)

Explanation: the correct sequence is –BAECD.


766. Correct Answer is: (d)
Explanation: the correct sequence is CABDE

767. Correct Answer is: (a)


Explanation: the correct sequence is CABDE
768. Correct Answer is: (c)
Explanation: the correct sequence is CABDE
769. Correct Answer is: (b)

Explanation: the correct sequence is CABDE

770. Correct Answer is: (e)


Explanation: the correct sequence is CABDE

771. Correct Answer is: (a)


Explanation: the correct sequence is CAEDB
772. Correct Answer is: (d)
Explanation: the correct sequence is CAEDB
773. Correct Answer is: (c)
Explanation: the correct sequence is CAEDB.

774. Correct Answer is: (b)


Explanation: the correct sequence is CAEDB
775. Correct Answer is: (e)

Explanation: the correct sequence is CAEDB


776. Correct Answer is: (b)
Explanation: the correct sequence is – ABCDE.

777. Correct Answer is: (a)


Explanation: the correct sequence is – ABCDE.

778. Correct Answer is: (d)

Explanation: the correct sequence is – ABCDE.


779. Correct Answer is: (c)

Explanation: the correct sequence is – ABCDE.

780. Correct Answer is: (e)


Explanation: the correct sequence is – ABCDE.

781. Correct Answer is: (b)

www.ibpsguide.com https://estore.ibpsguide.com Page 264 of 331


Explanation: the correct sequence is BACDE.

782. Correct Answer is: (a)


Explanation: the correct sequence is BACDE.
783. Correct Answer is: (c)

Explanation: the correct sequence is BACDE.


784. Correct Answer is: (d)
Explanation: the correct sequence is BACDE
785. Correct Answer is: (e)
Explanation: the correct sequence is BACDE.

786. Correct Answer is: (d)

Explanation: the correct sequence is CABDE.


787. Correct Answer is: (a)

Explanation: the correct sequence is CABDE.


788. Correct Answer is: (c)
Explanation: the correct sequence is CABDE.
789. Correct Answer is: (b)
Explanation: the correct sequence is CABDE.
790. Correct Answer is: (e)

Explanation: the correct sequence is CABDE.


791. Correct Answer is: (c)
Explanation: the correct sequence is EDACB

792. Correct Answer is: (d)


Explanation: the correct sequence is EDACB
793. Correct Answer is: (e)

Explanation: the correct sequence is EDACB


794. Correct Answer is: (a)

Explanation: the correct sequence is EDACB

795. Correct Answer is: (b)


Explanation: the correct sequence is EDACB

796. Correct Answer is: (a)

Explanation: The correct sequence is CAEBD.


797. Correct Answer is: (e)

Explanation: The correct sequence is CAEBD.

www.ibpsguide.com https://estore.ibpsguide.com Page 265 of 331


798. Correct Answer is: (c)

Explanation: The correct sequence is CAEBD.


799. Correct Answer is: (b)
Explanation: The correct sequence is CAEBD.

800. Correct Answer is: (d)


Explanation: The correct sequence is CAEBD.
801. Correct Answer is: (d)
Explanation: The correct sequence is CABDE.
802. Correct Answer is: (a)

Explanation: The correct sequence is CABDE.

803. Correct Answer is: (c)


Explanation: The correct sequence is CABDE.

804. Correct Answer is: (b)


Explanation: The correct sequence is CABDE.
805. Correct Answer is: (e)
Explanation: The correct sequence is CABDE.
806. Correct Answer is: (b)
Explanation: The correct sequence is ABCED.

807. Correct Answer is: (e)


Explanation: The correct sequence is ABCED.
808. Correct Answer is: (d)

Explanation: The correct sequence is ABCED..


809. Correct Answer is: (c)
Explanation: The correct sequence is ABCED.

810. Correct Answer is: (a)


Explanation: The correct sequence is ABCED.

811. Correct Answer is: (d)

Explanation: The correct sequence is CABDE


812. Correct Answer is: (a)

Explanation: The correct sequence is CABDE

813. Correct Answer is: (c)


Explanation: The correct sequence is CABDE.

814. Correct Answer is: (b)

www.ibpsguide.com https://estore.ibpsguide.com Page 266 of 331


Explanation: The correct sequence is CABDE

815. Correct Answer is: (e)


Explanation: The correct sequence is CABDE
816. Correct Answer is: (d)

Explanation: The correct sequence is ADBCE


817. Correct Answer is: (a)
Explanation: The correct sequence is ADBCE.
818. Correct Answer is: (e)
Explanation: The correct sequence is ADBCE.

819. Correct Answer is: (c)

Explanation: The correct sequence is ADBCE.


820. Correct Answer is: (b)

Explanation: The correct sequence is ADBCE.


821. Correct Answer is: (b)
Explanation: the correct sequence is BDCAE
822. Correct Answer is: (a)
Explanation: the correct sequence is BDCAE
823. Correct Answer is: (e)

Explanation: the correct sequence is BDCAE


824. Correct Answer is: (d)
Explanation: the correct sequence is BDCAE

825. Correct Answer is: (c)


Explanation: the correct sequence is BDCAE
826. Correct Answer is: (d)

Explanation: The correct sequence is AEBCD.


827. Correct Answer is: (e)

Explanation: The correct sequence is AEBCD.

828. Correct Answer is: (a)


Explanation: The correct sequence is AEBCD.

829. Correct Answer is: (c)

Explanation: The correct sequence is AEBCD.


830. Correct Answer is: (b)

Explanation: The correct sequence is AEBCD.

www.ibpsguide.com https://estore.ibpsguide.com Page 267 of 331


831. Correct Answer is: (b)

Explanation: The correct sequence is BACDE.


832. Correct Answer is: (a)
Explanation: The correct sequence is BACDE.

833. Correct Answer is: (c)


Explanation: The correct sequence is BACDE.
834. Correct Answer is: (d)
Explanation: The correct sequence is BACDE.
835. Correct Answer is: (e)

Explanation: The correct sequence is BACDE.

836. Correct Answer is: (c)


Explanation: The correct sequence is BCADE.

837. Correct Answer is: (a)


Explanation: The correct sequence is BCADE.
838. Correct Answer is: (b)
Explanation: The correct sequence is BCADE.
839. Correct Answer is: (d)
Explanation: The correct sequence is BCADE.

840. Correct Answer is: (e)


Explanation: The correct sequence is BCADE.
841. Correct Answer is: (d)

Explanation: The correct sequence is ACBDE


842. Correct Answer is: (a)
Explanation: The correct sequence is ACBDE

843. Correct Answer is: (b)


Explanation: The correct sequence is ACBDE.

844. Correct Answer is: (c)

Explanation: The correct sequence is ACBDE


845. Correct Answer is: (e)

Explanation: The correct sequence is ACBDE

846. Correct Answer is: (b)


Explanation: The correct sequence is ABCED

847. Correct Answer is: (e)

www.ibpsguide.com https://estore.ibpsguide.com Page 268 of 331


Explanation: The correct sequence is ABCED.

848. Correct Answer is: (d)


Explanation: The correct sequence is ABCED.
849. Correct Answer is: (c)

Explanation: The correct sequence is ABCED.


850. Correct Answer is: (a)
Explanation: The correct sequence is ABCED.

Jumbled Sentence Type - 4

Direction (851-880): Five sentences denoted by A, B, C, D and E have been given. Identify the odd sentence and
arrange rest of the four sentences in such a way that it makes a meaningful paragraph.

851). A) The recent equity-market gyrations by themselves give little cause for concern. The world economy
remains in fine fettle, buoyed by a synchronized acceleration in America, Europe and Asia.
B) On balance, hasty tightening is the greater risk. New to his role, Mr Powell may be tempted to establish his
inflation-fighting chops—and his independence from the White House—by pushing for higher rates faster. That
would be a mistake, for three reasons.
C) The violence of the repricing was because of newfangled vehicles that had been caught out betting on low
volatility. However, even as they scrambled to react to its re-emergence, the collateral damage to other markets,
such as corporate bonds and foreign exchange, was limited.
D) After years in which investors could rely on central banks for support, the safety net of extraordinarily loose
monetary policy is slowly being dismantled.
E) Despite the plunge, American stock prices have fallen back only to where they were at the beginning of the

year. Yet this episode does signal just what may lie ahead

(a) BEAC (b) CEBA (c) DBEA (d) ACED (e) BDAE

852). A) Voters will surely share their despondency. Having hammered the “grand coalition” parties in last
September’s inconclusive elections, they have been telling pollsters that their support for Mrs Merkel’s CDU/CSU
alliance and the SPD is sliding.

B) The Berlin Wall stood for 28 years, two months and 27 days; as of this week it has been down for longer. Just
as Germany’s “post-Wall” era has come to an end, so the cozy politics of the past three decades looks as if it is
running out of inspiration.

www.ibpsguide.com https://estore.ibpsguide.com Page 269 of 331


C) On February 6th news came that the Christian Democrat alliance (CDU/CSU) and the Social Democrats
(SPD), had agreed on yet another grand coalition. Germany is desperate for political renewal, but all that its
politicians have been able to come up with is a dreary sort of continuity that has left everyone unhappy.
D) The new government will increase child benefits, cut taxes modestly and limit immigration.

E) The coalition agreement sets out some modest ambitions. There are spending pledges on infrastructure,
where wealthy Germany is surprisingly deficient.

(a) BEAC (b) CEBA (c) DBEA (d) EBAD (e) BCED

853). A) Mr Musk is not the only billionaire entrepreneur with grand ambitions to improve the future of mankind.
Mark Zuckerberg, the founder of Facebook, wants to “cure, prevent or manage” all diseases by the end of the
century.
B) NOTHING declares world changing ambition like a space rocket. This week’s spectacular test confirmed the
Falcon Heavy as the planet’s most powerful operational launch vehicle
C) In the latest twist younger billionaires like Mr Zuckerberg, who made their fortunes in their 20s or 30s, have
switched from a serial model of philanthropy, in which you make money first and then retire and give it away, to a
parallel one, where you start giving the money away while it is still coming in
D) It also testified to the outsized vision of Elon Musk, its creator. To ensure humanity’s long-term survival he
wants both to colonise Mars and to wean the Earth off fossil fuels.
E) Bill Gates, having made his fortune at Microsoft, wants to eradicate polio and malaria, as part of a broader goal
of improving health and alleviating poverty

(a) BDAE (b) CEBA (c) DBEA (d) EBAD (e) CDEA

854). A) The 30-page paper was widely criticized, and quickly dismissed by experts and regulators. Protecting the
network from Chinese hacking, the main reason for the proposal, does not require the state to run the entire
network.
B) So there was much surprise at a recent memo, written for the White House by an official at the National
Security Council, which argued that the next generation of mobile network, “5G” for short, should be built and run
by the American government.
C) For more than three decades, telecoms policy, at least in rich countries, has been a one way street: more

deregulation and more privatization in order to foster more competition.

D) The 30-page paper was widely criticized, and quickly dismissed by experts and regulators. Protecting the
network from Chinese hacking, the main reason for the proposal does not require the state to run the entire
network.

www.ibpsguide.com https://estore.ibpsguide.com Page 270 of 331


E) This direction was set by America in 1984, when it broke up AT&T, its telephone monopoly.

(a) BEAC (b) CEBA (c) DBEA (d) ECBA (e) BDAE

855). A) Professional athletes pay a high price for their pursuit of excellence and glory. Training to the limit tears
muscles and wears out joints.
B) Gymnasts often need hip replacements when barely into middle age. Few footballers make it to the end of their
careers with their knees intact.
C) Fixing doping means fixing incentives. WADA needs money, and to be independent of the sports officials who
currently call the shots.

D) But many also run a darker risk: doping. The Winter Olympics in Pyeongchang, in South Korea, starts this
week in its shadow. Years after whistle-blowers first revealed wholesale doping in Russia, the International
Olympic Committee (IOC) at last decided to bar it from taking part.

E) It has allowed many Russians to compete as individuals. And on the eve of the competition the Court of
Arbitration for Sport said that 28 others should receive a more lenient penalty from the IOC, further muffling the
anti-doping message.

(a) BEAC (b) CEBA (c) ABDE (d) ECBA (e) BDAE

856). A) Elon Musk can seem flakily up himself. His newish tunneling business appears to be a case in point.
B) The project has a cute name (the Boring Company), a wacky way of raising money (an “Initial Hat Offering”
raised almost $1m by selling baseball caps), a physicist-knows-best approach to a social problem (putting private
cars on high-speed underground trolleys to reduce urban congestion) and a quirky, memorable goal (to produce a
tunnelling machine that goes faster than a snail, in this case a snail called Gary).
C) He then created a culture that emphasized experimentation, rapid learning and incremental improvements,

along with a system of sticks and carrots that pushed people to squeeze out inefficiencies.
D) He reduces thorny problems to what he sees as their essence—typically expressed in terms of physics—and
then extends his analysis to technologies, business systems, human psychology and design in an attempt to
solve the issue
E) But it also showcases the techniques that have made Mr Musk a success. Chris Anderson, the curator of TED,
a non-profit organisation that spreads ideas, says that Mr Musk is “uniquely good at system-design thinking”

(a) BEAC (b) CDBA (c) ABED (d) ECBA (e) BDAE

www.ibpsguide.com https://estore.ibpsguide.com Page 271 of 331


857). A) More than 800 jihadists died alongside 163 soldiers and at least47 civilians. The rebuilding, especially of
the heavily damaged eastern half of the city, has barely begun
B) The silence is startling. The only sound is the slight creaking of the metal strips peeling off bombed buildings
like bandages.
C) The conflict between fighters linked with Islamic State (IS) and the Philippine armed forces ended in October,
after five months of destruction.
D) The displaced say they are tired of eating handouts of rice and want to go home. One woman explains that
when the fighting broke out, she told her mother to pack only three changes of clothes because they thought they
would not be away for long.
E) A fancy light fixture hangs askance in what might have been a dining room. Elsewhere dirty toys lie in piles
defecated on by dogs. The animals are healthier here than elsewhere in Marawi, says one local, because they ate

the bodies of those killed in the fighting last year.

(a) BECA (b) CEBA (c) DBE (d) ECBA (e) BDAE

858). A) The verdict formalises the collapse of Bangladesh’s two-party system and the demise of the Zia dynasty.
The BNP and the Awami League (AL), the party currently in power, used to alternate in government.
B) It was Mrs Zia’s first conviction, for stealing cash in 1991 from a trust for orphans founded in memory of her
late husband, Ziaur Rahman, a coup leader who became president before being killed in a coup himself
C) But Mrs Zia is 72, is in ill health and, as a result of the verdict, may not be able to contest future elections. And
Tarique Rahman, her son and political heir, is in exile.
D) Khaleda Zia has been in and out of the courts for over a decade. She has been charged in 37 different cases,
most concerning corruption or abuse of power during her two stints as prime minister, in 1991-96 and 2001-06.
But the verdict reached on February 8th was momentous.
E) Mrs Zia, who leads the Bangladesh Nationalist Party (BNP), currently in opposition, was sentenced to five

years in jail. Although she may yet be freed pending appeals to the High Court and the Supreme Court, her fate
appears sealed.

(a) BEAC (b) CEBA (c) DBEA (d) ECBA (e) BDAE

859). A) That cannot be quite true, as quite a lot ofit is devoted to social media, most notably Twitter.
B) And then there is Narendra Modi, India’s prime minister, who assures his followers on Instagram: “Every
moment of my life is devoted to the welfare of India.”
C) Hun Sen, Cambodia’s strongman, apparently dedicates most of his time to posing for selfies with adoring

young Cambodians, if his Facebook page is to be believed.

www.ibpsguide.com https://estore.ibpsguide.com Page 272 of 331


D) Of course, all this sharing can backfire. Hun Sen, who has run Cambodia for more than 30 years, was mocked
in 2016 when it became obvious he was buying “likes” for his Facebook page.
E) When he is not lifting minuscule weights or catering to the whims of his cats, Najib Razak somehow finds time

to be Malaysia’s prime minister—or so his feed on Instagram, a photo-sharing app, implies

(a) BEAC (b) CEBA (c) DBEA (d) ECBA (e) BDAE

860). A) The picturesque wine country of Hawkes Bay is hardly a classic gangland. Tourists come here to ogle art
deco buildings or slurp merlot.
B) But its less affluent suburbs are divided between bitter rivals: Black Power and the Mongrel Mob, New
Zealand’s biggest gangs. This underworld occasionally rear sits head, with, say, gunfire at a rugby game, or an
assault outside a winery
C) Unlike counterparts in other countries, they thrive in rural areas as well as cities. Almost a quarter of people
living in the shabby bungalows of Flaxmere, a suburb in Hawkes Bay, are said to be linked to Black Power.
D) A clenched fist is the symbol of Black Power; a bulldog or the Nazi salute “Sieg Heil” are the marks of the
Mongrels
E) For a sleepy country, NewZealand has a peculiar problem with gangs. Police count over 5,300 members or
“prospects” lining up to join one of its 25 listed groups, which together makes them a bigger force than the army.

(a) BEAC (b) CEBA (c) DBEA (d) ECBA (e) ABEC

861). A) Except this anti-TTIP protest took place outside the Commission’s London office. A full 500,000
signatories were British.
B) It was erected by campaigners bearing 3m signatures from Europeans who wanted to scupper the
Transatlantic Trade and Investment Partnership (TTIP), a sprawling free-trade deal between the European Union
and America.
C) For trade-deal boosters, this makes new and awkward political alliances necessary. Liberal Brexiteers must

win over those who voted Remain, who tend to be more open when it comes to trade.
D) STANDING eight meters tall, the inflatable Trojan horse outside the European Commission office a couple of
years ago was difficult to miss.
E) For Brexiteers, such a scene seems ideal to help explain why Britain has to strike out on its own; outside the
EU, Britain would no longer be held back by continental trade luddites.

(a) BEAC (b) CEBA (c) DBEA (d) ECBA (e) BDAE

www.ibpsguide.com https://estore.ibpsguide.com Page 273 of 331


862). A) In Japan tattoos are associated with criminals. Many yakuza mobsters spend hundreds of hours under
an inky gun having their entire bodies painted, as a sign of gang membership and to show they can endure pain
B) The sign, says Yuichi Ohama, the spa’s manager, is directed mainly at the gangsters who haunt the local
area, a dense warren of brothels, cabaret bars and striptease clubs.
C) Yet the staff increasingly find themselves turning away tourists, too, he laments: “We’re surprised by how
many have body art.”
D) AWARM aroma of citrus bath salts wafts through the lobby of the Thermae-yu spa in Tokyo’s Kabukicho
district. The instructions at the entrance are chillier: drunks and people with tattoos should stay out.
E) In 2013 a Maori woman taking part in a conference on indigenous languages was barred from entering a bath
house in Hokkaido because of her traditional facial tattoo.

(a) BEAC (b) CEBA (c) DBEA (d) ECBA (e) BDAE

863). A) The outcry from conservatives who see the northern regime as an implacable foe was predictable.
Protesters set fire to North Korean flags and photos of Kim Jong Un, the North’s blood-drenched despot.
B) One conservative MP accused the government of hosting the “Pyongyang Olympics”, single-handedly
undermining South Korea’s long campaign to distinguish between the Olympic city and the North Korean capital.
C) WHEN Moon Jae-in, South Korea’s president, agreed to allow North Korean athletes not only to attend the
Winter Olympics in Pyeongchang, but also to march alongside South Korea’s team at the opening ceremony on
February 9th, and to form a unified women’s ice-hockey team with the South, he knew not all South Koreans

would be happy.
D) The hawks railed at the exemptions that had to be made to local and American laws to allow a plane from the
South to take skiers to the North for training, and to permit a ship from the North to ferry the 140-piece Samjiyon
orchestra to the South.
E) But South Koreans in their 20s and 30s grew up at a time of worsening relations as the North developed

missiles and nuclear bombs.

(a) BEAC (b) CABD (c) DBEA (d) ECBA (e) BDAE

864). A) The court, he claims, was paving the way for a coup by nefarious forces
B) Now he has suspended much of the constitution and declared a 15-day state of emergency. Mr Yameen may
have become a full-blown dictator, but he seems to see himself as the victim of a monstrous injustice.
C) Not even the monetary authority has any handle on the debts the Maldives is amassing, but thinks three-fifths

are owed to China. Any default, and China can extract concessions, such as a base on the Indian Ocean.

www.ibpsguide.com https://estore.ibpsguide.com Page 274 of 331


D) Abdulla Yameen has racked up many accomplishments since becoming president of the strategic
archipelago in 2013, from befriending China and Saudi Arabia to hounding both the opposition and leaders of his
own coalition, intimidating the remains of a free press and, earlier this month, shutting parliament.
E) First soldiers and police surrounded the Supreme Court in Malé, the claustrophobic, sea-girt capital of the

Maldives. Then, earlier this week, they hauled off the chief justice and two associates in the dead of night.

(a) BEAC (b) CEBA (c) DBAC (d) EDBA (e) BDAE

865). A) She was a young teenager before she was able to move back to her parents’ home. Ms Li is now 26 and
runs her own private tutoring business
B) Hidden away from the authorities, and at first kept in the dark herself, Ms Li says she was just starting primary
school when she found out that the kindly aunt and uncle who often visited were in fact her biological parents.
C) When Li Dongxia was a baby, her parents sent her to be raised by her grandparents and other family
members half an hour from their home in the northern Chinese province of Shandong.
D) The Communist Party appears to recognise that it needs to do more to lower these barriers. A population-
planning document released last year acknowledged that the low birth rate was problematic and referred to a
vague package of pronatalist measures that it would consider in response.
E) That was not a choice but a necessity: they already had a daughter, and risked incurring a fine or losing their
jobs for breaking a law that prevented many couples from having more than one child.

(a) BEAC (b) CEBA (c) DBEA (d) ECBA (e) BDAE

866). A) In July 2016 the trio was found guilty of breaking into a government compound and of inciting others to
follow suit
B) Ms Chow’s disqualification drew criticism from Britain, Canada and the European Union. Most damningly, two
heavyweight backers of the government in Beijing ventured that the rules are unclear. One of them, Jasper Tsang
Yok-sing, a for
C) They have become a familiar sight standing on courtroom steps. Since their pro-democracy protests in 2014,
the young leaders of the Umbrella Movement—Joshua Wong, Nathan Law and Alex Chow—have bobbed in and

out of court, and sometimes into prison.

D) A magistrate sentenced Mr Wong and Mr Law to community service and Mr Chow to a three-week stint in
prison, suspended for a year.
E) This week they appeared again, after an appeal overturned controversial custodial sentences handed down

last year. But their mood was sombre.

(a) BEAC (b) CEAD (c) DBEA (d) ECBA (e) BDAE

www.ibpsguide.com https://estore.ibpsguide.com Page 275 of 331


867). A) Good economic news is not always good for everyone. On February 2nd it was revealed that average
hourly wages grew by 2.9% in the year to January—the fastest growth since 2009, at the end of the recession
B) Investors fretted that inflation might rise, forcing the Federal Reserve to raise interest rates further and faster
than expected.
C) America is poised to stimulate an economy that is already growing strongly, at a time of historically low
unemployment
D) America’s economy was very different in the 1960s. Almost a third of workers belonged to trade unions.
Around one in four had wage agreements indexing their pay to inflation.
E) Whether the jitters are justified, however, depends on how an extraordinary experiment in economic policy
plays out

(a) BEAC (b) DEBA (c) ABEC (d) ECBA (e) BDAE

868). A) As the storms died down, wildfires ignited in the West. In October 43 people died and nearly 9,000
structures were destroyed in northern California
B) America suffered several natural disasters last year. Over the course of a few months, hurricanes devastated
parts of Texas, Puerto Rico and Florida, which are home to 8% of the country’s population.
C) The past few months have been difficult for Racelle LaMar, a veterinarian who lives and works in the northern

California town of Santa Rosa.


D) First her practice burned to the ground in one of several fires that scorched 245,000 acres (99,000 hectares)
of homes and vineyards in the wine region north of San Francisco last October Then, when she requested aid
from the Federal Emergency Management Agency (FEMA), things took a curious turn. Someone had already
applied for money using her name, address and Social Security number.
E) In December the Federal Bureau of Investigation launched a taskforce to investigate wildfire fraud complaints
in northern California.

(a) BEAC (b) CEBA (c) DBEA (d) CDBA (e) BDAE

869). A) He also claims that “Investigators of the FBI and the Justice Department have politicized the sacred
investigative process in favor of Democrats and against Republicans.”
B) His actions risk inflicting great damage on the country he leads. On Twitter, Mr Trump calls the former heads of
the FBI and CIA, as well as an ex-director of national intelligence and Democrats on Congress’s intelligence
committees, “liars and leakers”.

www.ibpsguide.com https://estore.ibpsguide.com Page 276 of 331


C) But America’s institutions are not political foes. As an investigation led by Robert Mueller into possible links

between Russia and Mr Trump’s presidential campaign grinds on, the president has flung ever more insults in the
direction of law enforcement.
D) The memo quotes Mr Steele saying he was “desperate that Donald Trump not get elected and was
passionate about him not being president” and alleges anti-Trump bias elsewhere at the FBI.
E) Donald Trump seems to think he can best every challenge by insulting the challenger. It worked splendidly

during the presidential campaign

(a) BEAC (b) CEBA (c) DBEA (d) ECBA (e) BDAE

870). A) In 2010 the Great Lakes Restoration Initiative was born to improve water quality, clean up shorelines
and restore habitats and species.
B) Midwesterners woke up to the damage done when the Chicago, Rouge and Detroit rivers caught fire in the
1960s, fuelled by the oily sludge in the lakes and their arteries
C) A new study from McMaster University raises more concerns. It finds that bluegill sunfish, common in North
America, have to burn much more energy to cope with the array of toxins that they typically encounter. They have
less energy left for growth, reproduction and survival. Effluent from waste water treatment plants does not kill the
fish immediately, but its effect is insidious, says Graham Scott, one of the authors of the study
D) The abundant fresh water of the Great Lakes helped turn America’s Midwest into an industrial powerhouse.
Carmakers in Detroit, steelmakers in Cleveland, brewers in Milwaukee and makers of furniture in Grand Rapids
used huge quantities of water to produce their wares.
E) They also abused it. For almost a century they poured wastewater contaminated with metals, oils, paint and
other toxins back into the lakes.

(a) BEAC (b) CEBA (c) DBEA (d) ECBA (e) BDAE

871). A) To fix matters, a clause known as the “Pocahontas exception” was added to the racist law, to exempt
anyone with no more than one-sixteenth Indian blood.
B) Awkwardly, that would include many of the so-called First Families of Virginia, because they traced their
descent to a native American woman, Pocahontas, who had been abducted and married by a member of the
Jamestown colony three centuries before.
C) This ancestry had been considered far from shameful. It was a mark of American aristocracy, the real-life

Pocahontas having been reinvented (she probably did not save the life of a colonist called John Smith) as an

“American princess”.

www.ibpsguide.com https://estore.ibpsguide.com Page 277 of 331


D) Many North American Indians were settled cultivators. The nomadism of the plains was atypical and shaped
by Europeans. The Sioux, formerly farmers, had shifted to hunting the herds of bison that grew in a land
depopulated by imported diseases, using horses they got from the Spanish and guns from the French
E) In Early 1924 the blue-bloods of Virginia found themselves with a problem. To criminalise interracial marriage,

the state had drafted a law that classified anyone possessing even “one drop” of non-white blood as “coloured”.

(a) BEAC (b) CEBA (c) DBEA (d) EBCA (e) BDAE

872). A) On the evening of February 6th the lights went out across most of Caracas, Venezuela’s capital, just as
the city’s rush hour was beginning.
B) Mr Rodríguez refused to look at the document. So there was no deal. The talks were suspended indefinitely
C) Unable to take the metro, tens of thousands of workers were forced to walk the crime-ridden streets. Many
took the power cut as a metaphor for the country’s snuffed-out democracy and lost prosperity.
D) At precisely the same time, Jorge Rodríguez, Venezuela’s expensively dressed communications minister,
was arriving at a meeting in the Dominican Republic.
E) He has been the chief negotiator for the country’s leftist regime in sporadic talks with the opposition that have
taken place over the past 16 months.

(a) BEAC (b) CEBA (c) DBEA (d) ACDE (e) BDAE

873). A) Tim Hortons’ owner belatedly allowed franchisees to raise prices, but not before blaming a “reckless
few” for sullying the brand’s image. On the campaign trail, Ms Wynne may skip the customary visit
B) Tim Hortons, Canada’s largest coffee and-doughnut chain, is so beloved that politicians campaigning for office
rarely fail to visit one. Its “double-double”, a coffee with two splashes of cream and two sugars, has an entry in the
Canadian Oxford Dictionary.
C) But in January trade-union activists held demonstrations outside Tim Hortons restaurants in Ontario, the
country’s most populous province. Those have died down, but angry letters and phone calls keep coming.

D) The activists’ gripe is about the way the chain handled a sharp increase in Ontario’s minimum wage. On
January 1st the province’s Liberal government raised it from C$11.60 ($9.25) to C$14. That makes it the highest
in Canada. Another rise to C$15 is scheduled for next year.
E) Tim Hortons’ workers have no complaint about that. But they are dismayed by many franchisees’ plans for
covering the cost. Barred from raising prices by Tim Hortons’ owner, a company controlled by 3G Capital, a

Brazilian private-equity firm, some franchisees cut benefits, including paid breaks

(a) BEAC (b) CEBA (c) DBEA (d) ECBA (e) BCDE

www.ibpsguide.com https://estore.ibpsguide.com Page 278 of 331


874). A) Two other countries are question marks. In both Honduras and Bolivia, incumbent presidents have got
the courts to set aside term limits.
B) In fact, democracy has held up surprisingly well in the region. There are only two clear cases of regress.
Venezuela and Nicaragua have abolished term limits and their elected presidents now rule as dictators.
C) It is an anniversary tinged with gloom. Democracy is in retreat worldwide, with scholars identifying more than
two dozen countries that have reverted to authoritarianism in this century. Many worry for its future in Latin
America, too.

D) In Latin America, even as the new is born the old tends not to die. The social democratic PAC wars with the
PLN but has failed to kill it off: with 17 seats, the PLN will be the largest party in the new assembly. Costa Rica
suffers a vicious circle in which the voters seek new political actors who fare as badly as the old ones, says Mr
Casas. The current president, Luis Guillermo Solís, was a once-fresh face who failed to fix the budget or reform
taxes.
E) This year marks the 40th anniversary of the start of the democratic wave that swept over Latin America and
turned military dictators into political flotsam.

(a) BEAC (b) CEBA (c) DBEA (d) ECBA (e) BDAE

875). A) Many suspect that Mr Moussa is only running to create the illusion of a real contest. Instead of talking
about the joke election, Egyptians are talking about inflation, which they do not find funny.
B) A single challenger, Moussa Mustafa Moussa, who until recently was an ardent fan of the president,
announced his candidacy on January 28th and registered it 15 minutes before the deadline the next day, having
somehow gathered 47,000 signatures in record time.
C) With less than two months to go before Egypt’s presidential election, no one is talking about the choice of

candidates, because there is no choice.


D) A simpler system would distort the economy less while helping the poor far more. A study in 2013 by the Cato
Institute, a free-market think-tank in Washington, estimated that if all food and energy subsidies were stopped and
half of the savings used to pay for cash transfers to the poorest 60% of households, each of those households
would receive $622 a year, more than doubling incomes for the bottom 25%.
E) All serious rivals to President Abdel-Fattah al-Sisi, who seized power in a coup in 2013, have been scared off.

(a) BEAC (b) CEBA (c) DBEA (d) ECBA (e) BDAE

876). A) Hizbullah is not ready for another conflict, but it is adding to its arsenal of guided missiles. The IDF may
feel forced to forestall this build-up. Mr Assad also seems more interested
B) In the north Hizbullah would probably like to regroup, after nearly seven years of fighting in Syria. Lebanon
was so badly damaged during the group’s previous war with Israel, in 2006, that Hizbullah’s leaders regret
provoking it.

www.ibpsguide.com https://estore.ibpsguide.com Page 279 of 331


C) A deal between Hamas and the PA was meant to hand administrative control of Gaza to the PA, which in turn
would lift the sanctions
D) Hamas may merely be using its war talk to draw attention to Gaza’s misery. The siege and sanctions
imposed by the Palestinian Authority (PA), which runs the West Bank, has left the enclave short of electricity,
drinking water and food
E) There is talk of impending conflicts on Israel’s northern borders, too. On January 28th the IDF’s spokesman,
Ronen Manelis, wrote an op-ed, published by Lebanese websites, in which he warned Lebanon not to allow Iran
to produce precision missiles in the country.

(a) BEAC (b) CEBA (c) DBEA (d) DCBA (e) BDAE

877). A) Within weeks of Mr Ramaphosa’s elevation to president-in waiting (parliament picks the president, and
the ANC controls parliament), police and prosecutors had opened investigations into several of Mr Zuma’s
friends.
B) Start with the police and prosecutor’s office, which were paralyzed for the best part of a decade under Mr
Zuma, who faces 783 charges of corruption.
C) But if Mr Ramaphosa really wants to halt the rot he will have to push Mr Zuma from the presidency before the
end of his term next year
D) The new era began brightly. Since becoming leader of the ruling African National Congress (ANC) in
December, Cyril Ramaphosa has moved swiftly to stop Jacob Zuma, South Africa’s president, from wrecking the
place more than he already has.
E) It is an immense task. Yet a burst of movement across several areas of government suggests that Mr

Ramaphosa is wasting no time in tackling the corruption that has hollowed out South Africa.

(a) BEAC (b) CEBA (c) DBEA (d) ECBA (e) BDAE

878).A) In December Ciudadanos became the biggest single force in Catalonia at a regional election. Now it is
jostling the ruling conservative People’s Party (PP) at the top of the national opinion polls
B) When Albert Rivera gave a talk at a regular breakfast meeting for business folk at the Ritz Hotel in Madrid last
month, more than 600 people turned up, a record for the event.
C) Mr Rajoy does not have to call a fresh election until 2020, but there will be municipal and regional polls in

May 2019.

D) He has suddenly become Spain’s hottest ticket, almost three years after he leapt into national politics at the
head of Ciudadanos (“Citizens”), a newish liberal party

www.ibpsguide.com https://estore.ibpsguide.com Page 280 of 331


E) There Emmanuel Macron, to whom he feels politically close, swept aside an ossified two-party system last

year.

(a) BEAC (b) CEBA (c) DBEA (d) ECBA (e) BDAE

879). A) Despite spending as much on secondary schooling as other OECD countries, France no longer
achieves corresponding results. Between 2003 and 2012, performance in international maths tests fell compared
with other countries.
B) Instead, the diploma will be reorganised around a “major” of four big exams in the final year, down from
between ten and 15 currently
C) Yet President Emmanuel Macron is now about to announce the most radical overhaul of the exam for over

halfa century.
D) It was by imperial decree that Napoleon founded the French baccalauréat, the country’s school-leaving exam,
in 1808
E) To this day, some 700,000 pupils still take the bac, the great majority of the annual age cohort. It has become
the badge of excellence for a French lycée system that offers a model of globally standardised education,
including to over 900 lycées with a total of 330,000 pupils abroad. Y

(a) BEAC (b) CEBA (c) DECA (d) ACBE (e) BDAE

880). A) On February 6th 1943 Auschwitz received 2,000 Polish Jews from a ghetto in Bialystok, in north-east
Poland. Almost all of them were murdered in the death camp’s gas chambers; just one grisly episode in the six-
year saga of Nazi barbarity in Poland.
B) As he prepared to sign the law Andrzej Duda, Poland’s president, said no Holocaust survivor should feel
scared to give personal testimony. Academics and artists are exempt from its provisions.
C) If Poland’s new law was designed to deflect attention from Polish wrongdoing, it backfired. For weeks foreign

media have been recounting the details of Polish wartime atrocities

D) Poles have long railed against the phrase “Polish death camps”, as Barack Obama learned when he
thoughtlessly deployed it in 2012. But the term reflects clumsiness, not historical revisionism: no one argues that
Poles ran Auschwitz or any of the other camps in Poland.
E) Six million Poles were killed in the second world war, most of them victims of the Third Reich. This week,
exactly 75 years after that routine day in Auschwitz, Poland passed a law that threatens fines and imprisonment
upon anyone who attributes those crimes to the “Polish nation”

(a) BEAC (b) CEBA (c) DBEA (d) AEDB (e) BDAE

www.ibpsguide.com https://estore.ibpsguide.com Page 281 of 331


ANSWERS:
851). Correct Answer is: (d)
Explanation: Sentences ACED form a correct order while sentence B is the odd one out
852). Correct Answer is: (e)
Explanation: Sentences BCED form a correct order while sentence A is the odd one out.
853). Correct Answer is: (b)
Explanation: Sentences CEBA form a correct order while sentence D does not seem fitting in between these.
854). Correct Answer is: (b)
Explanation: Sentences CEBA form a correct order while sentence D is the odd one out.
855). Correct Answer is: (c)
Explanation: Sentences ABDE form a correct order while sentence C is the odd one out.
856). Correct Answer is: (c)
Explanation: Sentences ABED form a correct order while sentence C is the odd one out.
857). Correct Answer is: (a)
Explanation: Sentences BECA form a correct order while sentence D does not seem to fit in between these.
858). Correct Answer is: (c)
Explanation: Sentences DBEA form a correct order while sentence C does not seem to fit in between these.
859). Correct Answer is: (d)
Explanation: Sentences ECBA form a correct order while sentence D does not seem to fit in between these.
860). Correct Answer is: (e)
Explanation: Sentences ABEC form a correct order while sentence D does not seem to fit in between these.
861). Correct Answer is: (c)
Explanation: Sentences DBEA form a correct order while sentence C does not seem to fit in between these.
862). Correct Answer is: (c)
Explanation: Sentences DBEA form a correct order while sentence C does not seem to fit in between these.
863). Correct Answer is: b)
Explanation: Sentences CABD form a correct order while sentence E does not seem to fit in between these.
864). Correct Answer is: (d)
Explanation: Sentences EDBA form a correct order while sentence C does not seem to fit in between these.
865). Correct Answer is: (b)
Explanation: Sentences CEBA form a correct order while sentence D does not seem to fit in between these.
866). Correct Answer is: (b)
Explanation: Sentences CEAD form a correct order while sentence B does not seem to fit in between these.
867). Correct Answer is: (c)
Explanation: Sentences ABEC form a correct order while sentence D does not seem to fit in between these.
868). Correct Answer is: (d)
Explanation: Sentences CDBA form a correct order while sentence E does not seem to fit in between these.

www.ibpsguide.com https://estore.ibpsguide.com Page 282 of 331


869). Correct Answer is: (d)
Explanation: Sentences ECBA form a correct order while sentence D does not seem to fit in between these.
870). Correct Answer is: (c)
Explanation: Sentences DBEA form a correct order while sentence C is the odd one out.
871). Correct Answer is: (d)
Explanation: Sentences EBCA form a correct order while sentence D is the odd one out.
872). Correct Answer is: (d)
Explanation: Sentences ACDE form a correct order while sentence B is the odd one out.
873. Correct Answer is: (e)
Explanation: Sentences BCDE form a correct order while sentence A is the odd one out.
874). Correct Answer is: (d)
Explanation: Sentences ECBA form a correct order while sentence D is the odd one out.
875). Correct Answer is: (b)
Explanation: Sentences CEBA form a correct order while sentence D is the odd one out.
876). Correct Answer is: (d)
Explanation: Sentences DCBA form a correct order while sentence E is the odd one out.
877). Correct Answer is: (c)
Explanation: Sentences DBEA form a correct order while sentence C is the odd one out.
878). Correct Answer is: (e)
Explanation: Sentences BDAE form a correct order while sentence C is the odd one out.
879). Correct Answer is: (c)
Explanation: Sentences DECA form a correct order while sentence B is the odd one out.
880). Correct Answer is: (b)
Explanation: Sentences CEBA form a correct order while sentence D is the odd one out.

Reading Comprehension Type - 1


Directions (881-910): Read the following passage divided into a number of paragraphs carefully and answer the
questions that follow it.

Paragraph-1 Former President Barack Obama said today that negotiations with North Korea on its nuclear
weapons programme are difficult, partly because the country’s isolation minimizes possible leverage, such as
trade and travel sanctions against Pyongyang. “North is an example of a country that is so far out of the
international norms and so disconnected with the rest of the world,” Obama told a packed hall in Tokyo.
Paragraph – 2 He stressed the effort to get North Korea to give up nuclear weapons remains difficult, but
countries working together, including China as well as South Korea and Japan, to combine pressure on North

www.ibpsguide.com https://estore.ibpsguide.com Page 283 of 331


Korea, will be better than nations working alone. He noted that past US efforts on Iran’s nuclear weapons were
more successful because there was more leverage, but there’s little commerce and travel with North Korea to
being with. “That makes them less subject to these kinds of negotiations,” he said of North Korea.
Paragraph -3 Obama was speaking at an event sponsored by a Japanese nonprofit group during his trip to Asia,
which included earlier stops in Singapore, New Zealand and Australia. Obama’s work after leaving office has
been focused on nurturing young leaders. Obama, who was welcomed by a standing ovation, said the U.S.-Japan
alliance remained strong, and the U.S. committed to defending Japan.
Paragraph -4 “North Korea is a real threat. Our view has always been that we would prefer to resolve these issues
peacefully,” he said, adding that otherwise “the cost in terms of human life would be significant.” He
acknowledged progress on a nuclear-free world will likely take a long time as long as Russia and the U.S. could
not agree to start reducing their stockpiles.
Paragraph – 5 Obama also reflected on his 2016 visit to Hiroshima, one of two Japanese cities where the U.S.
dropped atomic bombs in the closing days of World War II. His visit was the first by an American president.
Paragraph – 6 Almost all American presidents tend to be relatively popular in Japan, which views the U.S. as its
most important ally. But many Japanese particularly appreciate Obama’s efforts on denuclearization and
remember with fondness his trip to Hiroshima and his message of working toward a world without nuclear
weapons.
“It was an extraordinarily powerful moment for me,” Obama recalled.

881). why Barak Obama said that the effort to get North Korea to give up nuclear weapons remains difficult?
(I) He acknowledged progress on a nuclear-free world will likely take a long time as long as Russia and the U.S.
could not agree to start reducing their stockpiles.
(II) The country’s isolation minimizes possible leverage, such as trade and travel sanctions against Pyongyang.
(III) North is an example of a country that is so far out of the international norms and so disconnected with the rest
of the world.
(IV) North Korea is a real threat&he would prefer to resolve these issues peacefully.
(a) Only (I) is correct (b) Both (I) and (IV) are correct
(c) Both (II) and (III) are correct (d) Only (I), (III) and (IV) are correct
(e) All are correct

882). What are the possible ways to resolve the issue of nuclear weapons?
(I) If countries works together, including China as well as South Korea and Japan, to combine pressure on
North Korea, & resolving it peacefully since the cost of human life is significant.
(II) commerce and travel with North Korea
(III) Acknowledging the progress on a nuclear-free world.
(IV) By spreading the message of working toward a world without nuclear weapons.
(a) Only (I) is correct (b) Both (I) and (IV) are correct
(c) Both (II) and (III) are correct (d) Only (I), (III) and (IV) are correct

www.ibpsguide.com https://estore.ibpsguide.com Page 284 of 331


(e) None of these

883). Find the incorrect statement in the context of the given passage.
(I) Obama was speaking at an event sponsored by a Japanese nonprofit group during his trip to Asia,
which included earlier stops in Singapore, New Zealand and India.
(II) Almost all American presidents tend to be relatively popular in Japan, which views the U.S. as its most
important ally
(III) “North is an example of a country that is so far out of the international norms and so disconnected with
the rest of the world,” Obama told a packed hall in Tokyo.
(IV) He acknowledged progress on a nuclear-free world will unlikely take a long time as long as Russia and
the U.S. could not agree to start reducing their stockpiles.
(a) Only (I) is in correct (b) Both (I) and (IV) are incorrect
(c) Both (II) and (III) are incorrect (d) Only (I), (III) and (IV) are incorrect
(e) All are correct.

884). Find the correct statement in the context of the given passage.
(I) Former President Barack Obama said today that negotiations with North Korea on its nuclear weapons
programme are difficult, partly because the country’s isolation minimizes possible leverage, such as trade and
travel sanctions against Pyongyang
(II) Obama’s work after leaving office has been focused on nurturing young leaders.
(III) Obama’s visit was the first by an American president.
IV) North is an example of a country that is so far out of the international norms and so disconnected with the rest
of the world
(a) Only (I) is correct (b) Both (I) and (IV) are correct
(c) Both (II) and (III) are correct (d) Only (I), (III) and (IV) are correct
(e) All are correct.

885). what is the perfect title for the given passage.


(I) Nuclear weapon – merits & demerits
(II) The world leader – Barak Obama
(III) North Korea – and its nuclear power.
(IV) Nuclear weapons – a threat to the world
(a) Only (III) is correct (b) Both (I) and (IV) are correct
(c) Both (II) and (III) are correct (d) Only (I), (III) and (IV) are correct
(e) All are correct.

www.ibpsguide.com https://estore.ibpsguide.com Page 285 of 331


Directions (886-888) choose the words that is most nearly the same in meaning of the word as used in the context
of the passage
886). Leverage
(a) Trade on (b) Hold (c) Eminence (d) Impotence

887). Alliance
(a) Connection (b) Affinity (c) Coalition (d) Union

888). Stockpiles
(a) Accumulate (b) Collect (c) Pile up (d) cache

Directions – (889-890) choose the words that is most nearly the oppositein meaning of the word as used in the
context of the passage
889). Recalled
(a) Recalcitrant (b) Ratify (c) Blank out (d) Call up

890). Ally
(a) Foe (b) Accomplice (c) Accessory (d) Conspirator.

Directions (891-900): Read the following passage divided into a number of paragraphs carefully and answer the
questions that follow it.
Paragraph – 1Facebook’s business model is pretty simple. It offers a free service to connect users with loved
ones and others of similar interests. In return, it collects data about what its users do online (and,
increasingly, offline). It then uses this data to sell targeted advertising. The whole thing works brilliantly.
Paragraph -2 Yet the company -- like so many others that profit from selling access to their users’ data -- seems
reluctant to describe so explicitly how it makes money. Instead, in response to news that data from as many as 50
million users was gathered and shared by a third party without their knowledge, Facebook issued a statement that
begins: “Protecting people’s information is the most important thing we do at Facebook.”
Paragraph – 3There is always some distance between what a company says it does and what it actually does. For
Facebook, it’s greater than most. And -- unlike for many companies -- this disconnect is no longer an abstract
concern. As the network assumes a central role in public life in the U.S. and elsewhere, it is warping politics and
misleading voters. Facebook’s cherished myth that it’s simply “bringing the world closer together” is becoming
hard to defend.
Paragraph – 4Its latest scandal offers a case in point. In 2014, a researcher got permission from both Facebook
and about 270,000 users to gather personal data about them. He then shared that data -- and that of about 50
million of their Facebook friends -- with a company called Cambridge Analytica, which used it to create
“psychographic” profiles of voters and later worked for the presidential campaign of Donald Trump.

www.ibpsguide.com https://estore.ibpsguide.com Page 286 of 331


Paragraph - 5Sharing data without users’ consent is “against our policies,” says Mark Zuckerberg, Facebook’s
founder and CEO. But the larger question, for regulators as much as the company itself, is how much control
users should have over Facebook’s vast repository of personal data, which has become an immensely powerful
political tool with few safeguards to speak of.
Paragraph – 6 The Trump team used such tools to great effect. It tested tens of thousands of ad variations on the
network. It used Facebook data to deliver provocative messages to receptive subsets of the electorate -- knowing
the broader public would be none the wiser -- while conducting “major voter suppression operations” to dampen
turnout for Trump’s opponent. As it happens, Russia had the same idea. This stuff only works because Facebook
furtively collects so much intimate information. Its business is selling data about its users.
Paragraph – 7And data has no values or principles of its own. It can help elect a good candidate or a bad one. It
can be used for valuable research or vile psychological experiments. It can reveal stunningly personal details --
and it is now in the hands of a huge range of people, responsible and otherwise.
Paragraph – 8 Facebook has every right to make money. But the company -- and its billions of users, for that
matter, as well as the few billion remaining humans who have yet to join -- shouldn’t delude themselves about
how it does so. Only then will they be able to clearly measure the consequences of their choices

891). How the personal data of the users worked for the presidential campaign of Donald Trump?
(I) The researcher shared that data -- and that of about 50 million of their Facebook friends -- with a company
called Cambridge Analytica, which used it to create “psychographic” profiles of voters.
(II) It used Facebook data to deliver provocative messages to receptive subsets of the electorate -- knowing
the broader public would be none the wiser -- while conducting “major voter suppression operations” to
dampen turnout for Trump’s opponent
(III) As the network assumes a central role in public life in the U.S. and elsewhere, it is warping politics and
misleading voters.
(IV) Facebook furtively collects so much intimate information.
(a) Only (I) is correct (b) Both (I) and (II) are correct
(c) Both (II) and (IV) are correct (d) Only (II), (III) and (IV) are correct
(e) All are correct

892). Why Author says data has no values or principles of its own?
(I) As it is warping politics and misleading voters.
(II) Facebook’s vast repository of personal data, which has become an immensely powerful political tool with
few safeguards to speak of.
(III) It can help elect a good candidate or a bad one.
(IV) It can be used for valuable research or vile psychological experiments.
(a) Only (I) is correct (b) Both (I) and (II) are correct
(c) Both (III) and (IV) are correct (d) Only (II), (III) and (IV) are correct
(e) All are correct

www.ibpsguide.com https://estore.ibpsguide.com Page 287 of 331


893). Why Author saying that – “Facebook’s cherished myth that it’s simply “bringing the world closer together” is
becoming hard to defend”.
(I) Facebook’s vast repository of personal data, which has become an immensely powerful political tool with few
safeguards to speak of.
(II) As the network assumes a central role in public life in the U.S. and elsewhere, it is warping politics and
misleading voters
(III) The Trump team used such tools to great effect. It tested tens of thousands of ad variations on the network.
(IV) And data has no values or principles of its own. It can help elect a good candidate or a bad one. It can be
used for valuable research or vile psychological experiments.
(a) Only (Ii) is correct (b) Both (I) and (II) are correct
(c) Both (III) and (IV) are correct (d) Only (II), (III) and (IV) are correct
(e) All are correct

894). Find the incorrect statement in the context of the given passage.
(I) Facebook’s cherished myth that it’s simply “bringing the world closer together” is becoming hard to defend.
(II) Facebook furtively collects so much intimate information. Its business is selling data about its users.
(III) Yet the company -- like so many others that profit from selling access to their users’ data -- describes
explicitly how it makes money.
(IV) Facebook’s business model is pretty simple. It offers a free service to connect users with loved ones and
others of similar interests. In return, it collects data about what its users do online
(a) Only (III) is in correct (b) Both (I) and (IV) are incorrect
(c) Both (II) and (III) are incorrect (d) Only (I), (III) and (IV) are incorrect
(e) All are correct.

895). Find the correct statement in the context of the given passage.
(I)Facebook issued a statement that begins: “Protecting people’s information is the most important thing we do at
Facebook.”
(II) Facebook’s cherished myth that it’s simply “bringing the world closer together” is becoming hard to defend.
(III) The Trump team used such tools to great effect. It tested tens of thousands of ad variations on the network
(IV) Its latest scandal offers a case in point. In 2014, a researcher got permission from both Facebook and about
270,000 users to gather personal data about them.
(a) Only (I) is correct (b) Both (I) and (IV) are correct
(c) Both (II) and (III) are correct (d) Only (I), (III) and (IV) are correct
(e) All are correct.

Directions – (896-898) choose the words that is most nearly the same in meaning of the word as used in the
context of the passage.

www.ibpsguide.com https://estore.ibpsguide.com Page 288 of 331


896). Explicitly
(a) Ambiguous (b) Equivocal (c) Implicit (d) Unequivocal.

897). Warping
(a) Upgrading (b) Decelerating (c) Distorting (d) Unstrained

898). Consent
(a) Interdiction (b) Injunction (c) Permission (d) Repression.

Directions – (899-900) choose the words that is most nearly the opposite in meaning of the word as used in the
context of the passage

899). Furtively-
(a) Privately (b) Surreptitious (c) Explicitly (d) Clandestinely

900). Delude.
(a) Uncloak (b) Beguile (c) Hoax (d) Deceive.

Directions (901-910): Read the following passage divided into a number of paragraphs carefully and answer the
questions that follow it.

Paragraph 1: As part of its continuing campaign to prevent China from stealing American intellectual property,
President Donald Trump’s administration is considering restrictions on the number of Chinese citizens enrolled at
US colleges and universities. Targeting foreign students will undermine US competitiveness, not enhance it.
Paragraph 2:Of the 1 million foreign nationals enrolled at US schools, nearly one-third are from China—double the
number of any other country. Chinese students receive 10% of all doctorates awarded in the US, most of them in
science and engineering. Some 80% of Chinese doctoral holders stay in the US and work after they earn their
degrees. There are more Chinese engineers working on artificial intelligence at US technology companies than in
all of China.
Paragraph 3:The gainsto the US economy aren’t limited to Silicon Valley. Chinese students spend at least $12
billion a year on tuition and living expenses—money that supports hundreds of thousands of jobs in and around
college campuses. High fees for international students subsidize tuition for US citizens and—until recently—have
helped public universities offset cuts in government funding. Such benefits are largely dismissed by those who
cast Chinese students in the US as national security threats. President Trump’s national security strategy calls for
the government to consider restrictions on “foreign STEM students from designated countries to ensure that
intellectual property is not transferred to our competitors.”

www.ibpsguide.com https://estore.ibpsguide.com Page 289 of 331


Paragraph 4: The US can and should do more to counter Chinese theft of American trade secrets. But only a tiny
fraction of students from China have ever been charged with illegal activity. Singling them out would be both
discriminatory and dangerous, potentially fuelling suspicion of law-abiding Americans of Chinese descent.
Paragraph 5: The most sensible strategy to protect the country’s intellectual property isn’t to keep talented
foreign students out, but to encourage them to stay in the US and put their knowledge to use—by joining the
workforce or starting a business. That’s the purpose of the optional practical training program, which allows
foreign students who earn degrees in technical fields to work in the US for up to three years. The Trump
administration, true to form, wants to cut the program instead. Whether the president imposes new quotas on
Chinese students, the goal of some of his advisers seems clear: to make “designated” foreigners unwelcome on
US campuses. That’s not only un-American, but also self-defeating.

901). According to the passage what is strategy to safeguard the country’s intellectual property?
(I) By providing optical training programme.
(II) By consider restrictions on “foreign STEM students from designated countries to ensure that intellectual
property is not transferred to our competitors.”
(III) By encouraging Chinese students to stay in the U.S.
(IV) By putting the knowledge of Chinese students to use.

(a) Only (I) is correct (b) Both (I) and (II) are correct
(c) Both (III) and (IV) are correct (d) Only (I), (III) and (IV) are correct
(e) All are correct

902). Why it is said that U.S fuelling the suspicion of law-abiding Americans of Chinese descent?
(I)The U.S is countering Chinese theft of American trade secrets.
(II)The president imposes new quotas on Chinese students, the goal of some of his advisers seems clear: to
make “designated” foreigners unwelcome on US campuses
(III) U.S Singling out Chinese students which is both discriminatory and dangerous
(IV)Only a tiny fraction of students from China have ever been charged with illegal activity.
(a) Only (IV) is correct (b) Both (I) and (III) are correct
(c) Only (I) is correct (d) Only (I), (III) and (IV) are correct
(e) None of the above.

903). Why Author is saying Chinese students adds to the economy of the U.S?
(I) Of the 1 million foreign nationals enrolled at US schools, nearly one-third are from China—double the number
of any other country
(II)High fees for international students subsidize tuition for US citizens and—until recently—have helped public
universities offset cuts in government funding.

www.ibpsguide.com https://estore.ibpsguide.com Page 290 of 331


(III)Chinese students spend at least $12 billion a year on tuition and living expenses—money that supports
hundreds of thousands of jobs in and around college campuses.
(IV)There are more Chinese engineers working on artificial intelligence at US technology companies than in all of
China.
(a) Only (I) is correct (b) Both (I) and (II) are correct
(c) Both (III) and (IV) are correct (d) Only (I), (III) and (IV) are correct
(e) All are correct

904). Find the incorrect statement in the context of the given passage.
(I)Chinese students receive 10% of all doctorates awarded in the US, most of them in science and engineering.
(II)The US can and should do more to counter Chinese theft of American trade secrets.
(III)Some 80% of Chinese engineers stay in the US and work after they earn their degrees.
(IV)Chinese students spend at least $12 billion a year on tuition and living expenses—money that supports
hundreds of thousands of jobs in and around college campuses
(a) Only (III) is incorrect (b) Both (I) and (II) are incorrect
(c) Both (III) and (IV) are incorrect (d) Only (I), (III) and (IV) are incorrect
(e) All are correct

905). Find the correct statement in the context of the given passage.
(I) Of the 1 million foreign nationals enrolled at US schools, nearly one-third are from China—double the number
of any other country
(II) As part of its continuing campaign to prevent China from stealing American intellectual property, President
Donald Trump’s administration is considering restrictions on the number of Chinese citizens enrolled at US
colleges and universities.
(III)More than 80% of Chinese engineers stay in the US and work after they earn their degrees
(IV). The Trump administration, true to form, wants to cut the program instead.

(a) Only (I) is correct (b) Both (I) and (II) are correct
(c) Both (III) and (IV) are correct (d) Only (I), (III) and (IV) are correct
(e) All are correct

906). What is the meaning of “singling out” as used in the passage?


(I) discrimination
(II) Anoint
(III) Espouse
(IV) Expunge.
(a) Only (I) is correct (b) Both (I) and (II) are correct
(c) Both (III) and (IV) are correct (d) Only (I), (III) and (IV) are correct

www.ibpsguide.com https://estore.ibpsguide.com Page 291 of 331


(e) All are correct

Directions – (907-910) choose the words that is most nearly the same / opposite in meaning of the word as used
in the context of the passage.

907). Fuelling-
(a) Burn (b) Fire (c) Charge (d) Stimulates

908). Undermine-
(a) Hurt (b) Sabotage (c) Compromise (d) Nourish

909). Enrolled
(a) Expel (b) Matriculate (c) Enter (d) Volunteer.

910). Subsidized
(a) Defund (b) Award (c) Grand (d) Establish

Answers:
Direction (881-890):
881). Correct Answer is: (c)
Explanation – As per the context North Korea’s isolation, and it doesn’t follow the international norms. So, it is
difficult to negotiate them in terms of giving up of nuclear weapons.

882). Correct Answer is: (a)


Explanation – As per the context given in the passage we can inferred that option (I) is the possible way to solve
the issue.

883). Correct Answer is: (b)


Explanation –Only option (I) & (IV) is incorrect.

884). Correct Answer is: (e)


Explanation – All are correct as per the given context in the passage.

885). Correct Answer is: (a)


Explanation - Option (III) as per the context the passage talks about North Korea and nuclear weapon.

www.ibpsguide.com https://estore.ibpsguide.com Page 292 of 331


886). Correct Answer is: (a)

887). Correct Answer is: (c)

888). Correct Answer is: (d)

889). Correct Answer is: (c)

890). Correct Answer is: (a)

Direction (891-900):
891). Correct Answer is: (b)
Explanation – Both options (I) & (II) are correct.

892). Correct Answer is: (c)


Explanation – As the data collected from uses through face book can be used anywhere and it can mislead
people in making their decision. As author gives the example of Political campaign – data can help to elect a good
candidate or bad.

893). Correct Answer is: (a)

894). Correct Answer is: (a)

Explanation – The given sentence is incorrect as per the passage, face book doesn’t explain how it makes money.

895). Correct Answer is: (e)

Explanation – The given options are correct as per the context given in the passage.

896). Correct Answer is: (d)

Explicitly – meaning- In a clear & detailed manner.

Synonyms- precise, clear, denotive

Antonyms- ambiguous, equivocal, implicit (unexpressed, unsaid, implied)

www.ibpsguide.com https://estore.ibpsguide.com Page 293 of 331


897). Correct Answer is: (c)

Warping – meaning- torturing, harming, corrupting

Synonyms- distortion, defaming, twist.

Antonyms- upgrading, valuing.

898). Correct Answer is: (c)

Consent–allowing, granting

Synonyms- permission, accord, sufferance, assent

Antonyms- interdiction, forbid

899). Correct Answer is: (c)

Synonyms- sneaky, concealed

Antonyms- explicitly, overtly, publicly.

900). Correct Answer is: (a)


Synonyms- deceit, fraud, mislead
Antonyms- honest, open up, clear, guide

Direction (901-910):
901). Correct Answer is: (c)

Explanation – Option ‘c’ is correct. As per the passage the best way to protect the intellectual property rights is by
encouraging Chinese students to stay in the U.S, putting their knowledge to use.

902). Correct Answer is: (a)

Explanation – Since U.s is targeting only a fraction of Chinese students who are well behaved, keeping the law
which is discriminatory.

903). Correct Answer is: (e)

Explanation – Chinese students spending $ 12 billion a year on tuition that supports the jobs, there are many
Chinese working engineers working on artificial intelligence all these helps in the growth of the U.S economy.

www.ibpsguide.com https://estore.ibpsguide.com Page 294 of 331


904). Correct Answer is: (a)
Explanation- Only (III) option is incorrect as per the context of the passage rest are correct.

905). Correct Answer is: (d)


Explanation- Only option- d is correct.

906). Correct Answer is: (a)


Meaning of singling out – to discriminate, to choose one from a group.

907). Correct Answer is: (d) –stimulates is a synonym of fuelling

908). Correct Answer is: (b)


Meaning of undermine is to weaken, erode.

909). Correct Answer is: (a)


Meaning of enrolled – to enlist, register. And Expel is an antonym of enroll.

910). Correct Answer is: (a)


Meaning of subsidized- to provide financial support, fund. Capital, endow. And Defund is the antonym of
subsidized.

Cloze Test – Type – 1

Direction (911-920): In the passage given below there are some blanks, each followed by a word given in bold.

Every blank has four alternative words given in options (a), (b), (c) and (d). You have to tell which word will best
suit to the respective blank. Mark (e) as your answer if the word given in the bold after the blank is your answer
i.e. "No change required".
We are in a (911) multilateral health crisis, and it grows worse by the year. By 2030 almost half the world’s

population will be overweight or obese if (912) these trends continue, the World Health Organization has warned.
There are already 124 million obese children: a more than tenfold increase in four decades. More than a million of

these live in the UK, which has the worst obesity rates in western Europe. This is a social (913) obstacle, both in

www.ibpsguide.com https://estore.ibpsguide.com Page 295 of 331


cause and consequence. The chief executive of NHS England, Simon Stevens, has warned that obesity could

(914) bankrupt the health service. Yet the government’s response has been as modest and (915) limp as these
figures are shocking. Medical experts (916) explains its childhood obesity strategy as weak, embarrassing and

even insulting. (917) Before it inherited a tax on sugary drinks – which comes into force this year – from George
Osborne, it rowed (918) off from restrictions on price-cutting promotions and junk food marketing or advertising.
(919) Indeed, the strategy relies heavily on measures such as school activity (920) promotions.

911). ?
(a) broad (b) global (c) whole (d) general (e) No change required.

912). ?
(a) remaining (b) extant (c) gradual (d) current (e) No change required.

913). ?
(a) impediment (b) hitch (c) problem (d) epidemic (e) No change required.

914). ?
(a) hinder (b) stop (c) wipe (d) interlude (e) No change required.

915). ?
(a) inadequate (b) slow (c) vivacious (d) underhand (e) No change required.

916). ?
(a) opts (b) outlined (c) describe (d) paints (e) No change required.

917). ?

(a) After (b) When (c) As (d) Though (e) No change required.

918). ?

(a) back (b) away (c) in (d) beneath (e) No change required.

919). ?

(a) Therefore (b) Instead (c) Still (d) Presumably (e) No change required.

www.ibpsguide.com https://estore.ibpsguide.com Page 296 of 331


920). ?
(a) jobs (b) projects (c) programmes (d) operations (e) No change required.

Direction (921-930): In the passage given below there are some blanks, each followed by a word given in bold.
Every blank has four alternative words given in options (a), (b), (c) and (d). You have to tell which word will best
suit to the respective blank. Mark (e) as your answer if the word given in the bold after the blank is your answer
i.e. "No change required".
Campaigners (921) concluded that would not be enough; now research ( 922) proves they were right – even when

such initiatives tackle both diet and exercise, and make efforts to (923) pay out to families. Children in schools in

the West Midlands were (924) appointed a year of extra physical activity sessions, a healthy eating programme
and cookery workshops ( 925) by their parents. It failed to have any (926) sensible effect on children’s weight. The

researchers’ conclusion was clear: much more ambitious action is needed. The (927) product of the obesity

epidemic are multiple and complex, as the landmark Foresight report produced over a decade ago (928) squinted
: we live in an obesogenic environment, and some (929) many so than others (more than twice as many children

in deprived areas are obese as in affluent areas). TVs and smartphones in bedrooms and (930) assurance on
cars play their part; so too do food deserts, where fruit and vegetables are expensive or inaccessible.

921). ?
(a) estimated (b) warned (c) decided (d) assumed (e) No change required.

922). ?
(a) justifies (b) acknowledges (c) explained (d) relates (e) No change required.

923). ?
(a) reach (b) phase (c) bag (d) ace (e) No change required.

924). ?
(a) incorporated (b) established (c) specified (d) given (e) No change required.

925). ?
(a) along (b) through (c) with (d) together (e) No change required.

www.ibpsguide.com https://estore.ibpsguide.com Page 297 of 331


926). ?

(a) significant (b) desirable (c) beneficial (d) prudent (e) No change required.

927). ?

(a) outcome (b) causes (c) effect (d) aftermath (e) No change required.

928). ?
(a) discerned (b) gaped (c) underscored (d) indicted (e) No change required.

929). ?

(a) far (b) more (c) much (d) greater (e) No change required.

930). ?
(a) driving (b) sitting (c) credit (d) reliance (e) No change required.

Direction (931-940): In the passage given below there are some blanks, each followed by a word given in bold.
Every blank has four alternative words given in options (a), (b), (c) and (d). You have to tell which word will best
suit to the respective blank. Mark (e) as your answer if the word given in the bold after the blank is your answer

i.e. "No change required".

It is (931) justified to fill a hungry child with doughnuts than with apples. But one factor leaps (932) in: greed. The

problem is not (933) friable by a generation of Augustus Gloops but the (934) pismire of the Willy Wonkas who
press junk food on consumers, (935) then profess surprise at the results. The tactics of big food are strikingly

similar to those of big tobacco over the years. But big food has the advantage that everyone needs to eat, while

no one needs to smoke, and that a biscuit does not (936) thwart health as a cigarette does. Thus, these
companies tell us that we should not (937) hitch individual freedom; that it is up to people to show self-discipline;

and that their (938) products are fine as occasional (939) snubs. Never mind that they (940) count family-size

packs as if they are suitable for individuals.

931). ?

(a) good (b) advisable (c) cheaper (d) wise (e) No change required.

932). ?

www.ibpsguide.com https://estore.ibpsguide.com Page 298 of 331


(a) onto (b) away (c) off (d) out (e) No change required.

933). ?
(a) gluttony (b) fugacious (c) defalcate (d) peripatetic (e) No change required.

934). ?
(a) otiose (b) avarice (c) hegemony (d) adumbrate (e) No change required.

935). ?

(a) to (b) and (c) hence (d) of (e) No change required.

936). ?

(a) alter (b) hamper (c) damage (d) erodes (e) No change required.

937). ?
(a) control (b) silence (c) restrict (d) harness (e) No change required.

938). ?

(a) recipes (b) items (c) techniques (d) pieces (e) No change required.

939). ?

(a) flits (b) consciences (c) indulgences (d) convalescences (e) No change required.

940). ?

(a) include (b) present (c) conclude (d) suggest (e) No change required.

ANSWERS:

911). Correct Answer is: (b)


Multilateral means 'agreed upon or participated in by three or more parties, especially the governments of different

countries' which is irrelevant with respect to the sentence. The correct word is 'global' which means ' relating to

the whole world; worldwide.'


912). Correct Answer is: (d)

www.ibpsguide.com https://estore.ibpsguide.com Page 299 of 331


The correct word to fit in appropriately in the sentence is 'current' which means 'belonging to the present time;

happening or being used or done now.'


913). Correct Answer is: (c)
The correct word to fit in the blank appropriately is 'problem' which means 'a matter or situation regarded as

unwelcome or harmful and needing to be dealt with and overcome.'


Impediment- a hindrance or obstruction in doing something.
Hitch- a temporary difficulty or problem.
Epidemic- a widespread occurrence of an infectious disease in a community at a particular time.
914). Correct Answer is: (e)

Here 'bankrupt' means completely lacking in a particular good quality.

915). Correct Answer is: (a)


Limp means walk with difficulty, typically because of a damaged or stiff leg or foot. This is contextually

inappropriate with respect to the sentence. The correct word to fit in the sentence is 'inadequate' which means
'lacking the quality or quantity required; insufficient for a purpose.'
916). Correct Answer is: (c)
Since the subject is plural (medical experts), the verb cannot be singular. The correct word to fit in the sentence is
'describe' which means 'give a detailed account in words of.'
917). Correct Answer is: (d)

The correct word to fit in the sentence appropriately is 'Though'.


918). Correct Answer is: (a)
The correct phrasal verb is 'rowed back' which means to reverse an earlier decision or previously held opinion.

919). Correct Answer is: (b)


The correct word to fit in the sentence is 'Instead'.
920). Correct Answer is: (c)

The correct word to fit in the blank is 'programmes' which means 'a planned series of future events or
performances.'

921). Correct Answer is: (b)

The correct word to fit in the sentence is 'warned' which means 'to inform someone in advance of a possible
danger, problem, or other unpleasant situation.'

922). Correct Answer is: (e)

The word is correct with respect to the sentence.


923). Correct Answer is: (a)

www.ibpsguide.com https://estore.ibpsguide.com Page 300 of 331


The correct phrasal verb to use here is 'reach out' which means 'to try to communicate with a person or a group of

people, usually in order to help or involve them.


924). Correct Answer is: (d)
The correct word to fit in the sentence is 'given' as it means to allow (someone) to have (a specified amount of

time) for an activity or undertaking.


925). Correct Answer is: (c)
The correct word to go with the sentence is 'with'.
926). Correct Answer is: (a)
The correct word to go with the sentence is 'significant' as it means sufficiently great or important to be worthy of

attention; noteworthy.

927). Correct Answer is: (b)


The use of plural tense 'are' suggests that the subject should also be plural. Hence, 'causes' is the right word to fit

in the sentence.
928). Correct Answer is: (c)
Squinted means to look at someone or something with one or both eyes partly closed in an attempt to see more
clearly or as a reaction to strong light. ; this is not contextually appropriate with the sentence. The correct word to
go with the sentence is 'underscored' which means emphasized.
Discerned-recognize or find out.

Gaped-be or become wide open.


Indicted-formally accuse of or charge with a crime.
929). Correct Answer is: (b)

The correct word to use in the sentence is 'more'.


930). Correct Answer is: (d)
The correct word to fit in the sentence is 'reliance' as it means dependence on or trust in someone or something.

931). Correct Answer is: (c)


Since, there is a comparison in the latter part of the sentence, the correct word to fit in the sentence is 'cheaper'.

932). Correct Answer is: (d)

The correct phrasal here is 'leaps out' which means to be readily noticed.
933). Correct Answer is: (a)

Friable means to be easily crumbled. This is contextually inappropriate with respect to the sentence. The correct

word to go with the sentence is gluttony as it means habitual greed or excess in eating.
Fugacious- tending to disappear; fleeting.

Defalcate- embezzle (funds with which one has been entrusted).

www.ibpsguide.com https://estore.ibpsguide.com Page 301 of 331


Peripatetic- travelling from place to place, in particular working or based in various places for relatively short

periods.
934). Correct Answer is: (b)
Pismire means an ant. This word is wrong in the sentence. The correct word to go with the sentence is 'avarice'

which means an extreme greed for wealth or material gain.


Otiose- serving no practical purpose or result.
Hegemony- leadership or dominance, especially by one state or social group over others.
Adumbrate- foreshadow (a future event).
935). Correct Answer is: (e)

The word is correct and needs no replacement.

936). Correct Answer is: (c)


Thwart means to prevent (someone) from accomplishing something. This is contextually inappropriate with the

sentence. The correct word to go with the sentence is 'damage' as it means physical harm that impairs the value,
usefulness, or normal function of something.
937). Correct Answer is: (c)
Hitch means a temporary difficulty or problem. This is not appropriate to use in the sentence. the correct word
which goes with the sentence is 'restrict' as it means put a limit on; keep under control.
938). Correct Answer is: (e)

The word is correct and needs no replacement.


939). Correct Answer is: (c)
Snub means to rebuff, ignore, or spurn disdainfully. This is contextually inappropriate to use in the sentence. The

correct word that goes with the sentence is 'indulgences' which means the action or fact of indulging.
Flits- move swiftly and lightly.
Consciences- a person's moral sense of right and wrong, viewed as acting as a guide to one's behaviour.

Convalescences- time spent recovering from an illness or medical treatment; recuperation.


940). Correct Answer is: (b)

The correct word to go with the sentence is 'present' as it means to represent (someone or something) to others in

a particular way.

www.ibpsguide.com https://estore.ibpsguide.com Page 302 of 331


Theme Detection Type - 1

Directions (941-970): In each questions below, a Theme has been given followed by two passage. You have to
determine which passage is based on the given theme and mark it as your option. More than one passage can be

based or not based on the given theme that is highlighted in bold.

941. Is Britain’s Anglosphere a Delusion?


a. The third idea is that “global Britain” means the Anglosphere. This embraces countries around the world that
share a common culture because they were once part of the British empire. “Outside the EU, the world is our
oyster”, a Brexiteer once put it poetically. “And the Commonwealth remains that precious pearl within.” Supporters

of this idea argue that the Anglosphere has deep roots in British history: in “The History of the English-Speaking
Peoples”, Winston Churchill argues that England is a global island, scattering its people around the world. But
they also point out that it is attractively modern. It is global where the EU is regional, networked where the EU is

bureaucratic, bottom-up where the EU is top-down.


b. The phrase “global Britain” is well intentioned, designed to send a message that Britain is not withdrawing from
the world by leaving the EU. It remains open for business, active on the world stage, bouncily cosmopolitan. But
Britain needs to do more than remain open for business. It needs to work out ways of engaging without
overstretching its abilities and of embracing globalisation without forgetting that it has downsides as well as
upsides. Talking Anglosphere isn’t going to help.

(a) Only a (b) Only b (c) Both a & b (d) Neither a nor b (e) Cannot be determined

942. What will be repercussions of the journalists’ murder?


a. Slovaks have mobilized in numbers unseen since the 1989 Velvet Revolution, and on March 9th there were
parallel protests in more than 30 other towns and cities, many considered the heartland of Mr Fico’s nationalist
Smer party. In the eastern city of Presov, Zlatica Kusnirova, mother of Martina Kusnirova, who was shot alongside

Mr Kuciak, addressed a crowd of 7,000.


b. The fallout from the murder on February 25th of an investigative journalist, Jan Kuciak, and his fiancée this
week brought down Robert Fico, who served as Slovakia’s prime minister for ten of the past 12 years. Mr Fico
had put up a fight. Drawing on Viktor Orban’s playbook from neighboring Hungary, he had hoped to defuse the

crisis by blaming a conspiracy of foreigners including George Soros, a billionaire financier, for the political
upheaval. But after 50,000 demonstrators (one in nine residents) took to the streets in Bratislava calling for his

resignation on March 9th, it was clear that he had failed.

www.ibpsguide.com https://estore.ibpsguide.com Page 303 of 331


(a) Only a (b) Only b (c) Both a & b (d) Neither a nor b (e) Cannot be determined

943. “What’s the difference between Martin Selmayr and God?”

a. MrSelmayris also happy to deploy the dark arts in pursuit of grander goals. Lurid accusations sit at his door,
from bullying commissioners to threatening miscreant journalists with violence. He carefully controls the flow of
information, excluding officials he considers unhelpful or incapable, and is a master of the strategic leak. Many

distrust his federalist vision for Europe’s future; he isin constant battle with officials who serve the EU Council,
where members of national governments sit. Now his foes sniff vulnerability. And that demarcates the lines
against God.

b. “God does not think he’s Selmayr.” As political gags go, it is not exactly a side-splitter (its origins are said to be
German). But it simultaneously captures the strange blend of fear and scorn with which Mr Selmayr is regarded in
Brussels, and the obsessions of a sometimes-parochial town with a Eurocrat who is barely known outside it.

(a) Only a (b) Only b (c) Both a & b (d) Neither a nor b (e) Cannot be determined

944. How is Conor Lamb helpful for the democracts?


a. If Democrats could repeat Conor Lamb’s extraordinary result in Pennsylvania when the mid-terms roll round,
they would win enough seats in Congress to override a presidential veto. The former marine and federal
prosecutor won a district that President Donald Trump took by almost 20 percentage points, the electoral
equivalent of defying gravity. Democrats can now look forward to special elections in Arizona’s 8th district (which

Mr Trump won by 21 points) and Ohio’s 12th district (which Mr Trump won by 11 points) with relish.
b. Mr Trump travelled to Pennsylvania twice to prop up Mr Saccone. During a 75- minute freewheeling speech on

March 10th he talked about himself, insulted several adversaries and mentioned Mr Saccone only in passing.
(“Personally, I like Rick Saccone. I think he’s handsome.”) As his polling lead disappeared Mr Saccone grew more
strident. Appearing next to Donald Trump junior, the president’s son, on the eve ofthe election, he claimed that

“the other side” hated the president and the country. “And I tell you some more,” he said. “They have a hatred for
God.”

(a) Only a (b) Only b (c) Both a & b (d) Neither a nor b (e) Cannot be determined

945. A storied ride is being remodeled. Again

www.ibpsguide.com https://estore.ibpsguide.com Page 304 of 331


a. Since1967, the Pirates of the Caribbean attraction at Disneyland in Anaheim has immersed visitors in the world

of swashbuckling buccaneers. After boarding small boats in a landing lit by the glow of fireflies, riders float past
Dead Man’s Cove, where the skeletons of unlucky marauders sit guarding their booty even in death. They watch

as cannonballs whistle through the air and behold as rambunctious pirates loot villages and set things aflame

b. After next month, one 50-year-old scene will be updated to reflect more sensitive times. Disney will temporarily
close the Pirates of the Caribbean ride to tweak the “Wench Auction” scene, where a full-busted animatronic

woman with flame-red hair, a red corset and a lacy petticoat stands on an auction block to be sold. “We wants the
redhead,” riders can hear the pirates shout as they glide by. Behind her stand other wenches, their eyes downcast
and pleading, their waists tied to one another with a leather strap. Standing guard over them is a rotund pirate

with a lewd grin and a pistol tucked in his waistband.

(a) Only a (b) Only b (c) Both a & b (d) Neither a nor b (e) Cannot be determined

946. Losing their religion


a. As soon as he stepped of fthe plane on a family holiday to Kenya, Mahad Olad knew something was wrong. His
mother, a “very devout, very conservative, very Wahhabi” woman, was acting strangely— furtively taking phone
calls when she thought he was out of earshot. His suspicions would soon be proved correct. Mr Olad’s family,
Somali immigrants to America and devout Muslims, had discovered that he had not only renounced Islam but was
also gay. The holiday was a ruse, an intervention to save his soul.
b. While the penalties for apostasy can be high in the West, they are much more severe in the Muslim world. In
Pakistan, blasphemy carries a death sentence. In Bangladesh, atheist writers have been hacked to death by

machete-wielding vigilantes. An atheist who recently appeared on Egyptian television to debate a former deputy
sheikh from Al-Azhar University was dismissed by the host and told that he needed to see a psychiatrist.

(a) Only a (b) Only b (c) Both a & b (d) Neither a nor b (e) Cannot be determined

947. Mike Pompeo is where foreign-policy realism meets America First

a. E IGHT years ago Mike Pompeo was a 47-year-old greenhorn congressional candidate in Wichita, Kansas,
with a modest business career and unremarkable limited-government views. He was best known for being the

biggest recipient of campaign donations from Wichita’s largest private-sector employer, Koch Industries. No one,
save possibly that company’s conservative mega-donor owners, marked him out for greatness. Yet with the
Senate’s blessing, which is likely to be forthcoming, Mr Pompeo will shortly be the first person to have occupied

the offices of both CIA director and secretary of state.

www.ibpsguide.com https://estore.ibpsguide.com Page 305 of 331


b. Mr Pompeo seems in many ways a reliable foreign-policy realist: pessimistic about alliances and supportive of

free trade, albeit with a hawkish enthusiasm for using military power. He has advocated a tougher line against
Russian expansionism, in Ukraine and Syria, than Mr Trump has. He sounds as admiring of China’s strongman

leader, Xi Jinping, as the president does. He appears deeply skeptical that North Korea’s rogue regime is open to

negotiation. Yet he also holds some of the irrational views that made his party so vulnerable to Mr Trump’s
confabulations and conspiracy theories.

(a) Only a (b) Only b (c) Both a & b (d) Neither a nor b (e) Cannot be determined

948. Rise of Patanjali – Marketing lessons to learn.

a. A quarter of a century ago the town of Tuxtla Gutiérrez, the capital of Chiapas State in Mexico’s deep south,
was the setting of a spate of horrific killings of transgender prostitutes. Nine of them were murdered in two years,
shot execution-style with up to a dozen bullets from high-calibre revolvers. Police claimed that in two cases they
were murdered after having had sex with their killers. The deaths caused a stir in Mexico, not least because of
speculation that a police death squad was involved and because the authorities framed clearly innocent people.
The Mexican interior minister at the time, Patrocinio González, when previously governor of Chiapas, had closed
down discos frequented by the sex workers, forcing them onto the street.
b. Bello was reminded of his reporting trip to cover that long-ago outrage by “A Fantastic Woman”, a Chilean film
which has just won an Oscar. Its protagonist is Marina, a trans woman (someone who has transitioned from male
to female) who has a conventional life as a waitress and aspiring classical singer. It tells the story of what

happens when her lover, an older businessman with whom she lives, dies suddenly. The film is remarkable for
Marina’s dignity in the face of psychological violence. This derives from the fear her transgressive identity
provokes. “What are you?” spits the dead man’s son. In interviews Daniela Vega, the film’s star, who is herself a
trans woman, says optimistically that attitudes are changing in Chile, until recently a socially conservative country.

That applies more broadly. A survey last year by ILGA, a pressure group, found that some 70% of respondents in
Latin America agreed that gays and trans people should enjoy the same legal rights as anyone else, the highest

figure anywhere. “The region has seen huge change in terms of its willingness to talk about this,” says Javier
Corrales, a political scientist at Amherst College in Massachusetts

(a) Only a (b) Only b (c) Both a & b (d) Neither a nor b (e) Cannot be determined

949. Why Constitutions Matter?

www.ibpsguide.com https://estore.ibpsguide.com Page 306 of 331


a. A BIT like President Donald Trump, Tanzania’s president, John Magufuli, likes to fire employees on television.

In November Mr Magafuli used a live broadcast from a small town in the north of the country summarily to dismiss
two officials after they failed to remember instantly details in their budgets. When one protested that she couldn’t

reasonably be expected to be able to recall every figure, Mr Magufuli told her, “You can’t talk to me like that.”

Sacking minor officials in front of an audience is only one part of Mr Magufuli’s authoritarian populism.
b. Mr Magufuli is both head of state and chairman of the party, with the power to hire and fire civil servants,

including judges, as he pleases. On taking office, he quickly filled important posts in the government and the party
with his own allies. Few are willing to speak up against the presidency, says one CCM MP. There is little hope of
change coming through the ballot box since the opposition is crushed and the next election, in 2020, will probably

be rigged. Nor is there much hope that the party can restrain MrMagufuli. Some hope that Jakaya Kikwete and
Benjamin Mkapa, two former presidents, can persuade him to change course. Others dream, seemingly forlornly,
that the party will revolt.

(a) Only a (b) Only b (c) Both a & b (d) Neither a nor b (e) Cannot be determined

950. Grow your rice and eat it


a. Standing ankle-deep in water between neatly spaced rice plants, an instructor shows a group of about 100
farmers in Kebbi, a state in north-west Nigeria, how to apply herbicide. The training session, arranged by TGI

Group, a Nigerian conglomerate that runs a large rice mill nearby, has an enthusiastic audience. Hussein Ahmed,
a farmer, says the yield from his small field has increased by about 50% since he started using chemicals and
carefully spacing the seedlings. Another farmer boasts of marrying a second wife thanks to the extra money he is

earning from growing rice


b. The government says its policies are working and that Nigeria will no longer need to import rice by the end of
the year. But its numbers do not add up. It says that rice production has doubled since 2015 (and will increase by

50% again this year), but there are scant data to support such ambitious claims. Nigerians eat 5.3m-7m tonnes of
rice a year. Imports account for 2m-3m tonnes, a figure that has barely budged in recent years. Nigeria’s

information minister, Lai Mohammed, points to statistics from Thailand showing that its exports to Nigeria have

slumped by 97% in two years.

(a) Only a (b) Only b (c) Both a & b (d) Neither a nor b (e) Cannot be determined

951. Why poor doesn’t always mean cheap?

www.ibpsguide.com https://estore.ibpsguide.com Page 307 of 331


a. The World Bank publishes rough estimates of price levels in different countries, showing how far a dollar would

stretch if converted into local currency. On this measure, Kenya is more expensive than Poland. This is surprising.
The cost of living is generally higher in richer places, a phenomenon best explained by the economists Bela

Balassa and Paul Samuelson. They distinguished between goods that can be traded internationally and many

services, like hairdressing, that cannot. In rich countries, manufacturing is highly productive, allowing firms to pay
high wages and still charge internationally competitive prices

b. Mr Hassan thinks that low agricultural productivity explains the puzzle. In much of Africa farmers scratch away
at thin soils, with little fertilizer and no irrigation. An Asian-style Green Revolution is only slowly taking root. Weak
infrastructure also drives up prices, as can be seen in Wakulima, a wholesale food market in Nairobi. Moses

Mungai has driven a maize lorry for four hours to get here, from a border town in the foothills of Kilimanjaro. But
he says it took four days to collect the crop from local farms. When the rains come he has to hire a tractor to
navigate soupy roads. Counties charge levies on commodities passing through. Middlemen take a cut.

(a) Only a (b) Only b (c) Both a & b (d) Neither a nor b (e) Cannot be determined

952. Gender Gap, Disturbingly wide


a. Police in Haldwani, a small town where the Gangetic Plain bumps against the Himalayas, shrugged when
Kamini Sen lodged charges against her husband last October. What could be more ordinary than wife-beating

and dowry extortion? The case got hotter when the 29- year-old, who holds masters degrees in both English and
psychology, added that her spouse had also secretly married a younger woman. But only the final twist, revealed
later when police tracked and caught Krishna “Sweety” Sen, shocked India. The two-timing swindler, said to like

cursing, smoking and motorbikes, confessed that he was not a man but a woman.
b. Of all the hurdles Indian women still face, sex and marriage may be the highest. The vast majority of marriages
are still arranged by families, and some 94% of them among Hindus are within the same caste. Women are

expected, by and large, to become part of the husband’s family, and to bring along a sweetener. In-laws can
press their demands in unsubtle ways. Aside from an annual average of around 8,000 “dowry deaths”—wives

killed because they have not coughed up enough money—recent newspaper reports tell of such persuasive

methods as beating with hockey sticks, stealing a kidney and blackmailing with sex tapes. More broadly, violence
against women in various forms appears to be on the increase, although this may chiefly be the result of an
increased tendency to report such crimes. If so, the change is desperately needed. A research paper from 2014

that compared police records of sexual violence with survey results estimated that less than 6% of such crimes
outside the home, and less than 1% of sexual assaults by husbands, are officially reported.

www.ibpsguide.com https://estore.ibpsguide.com Page 308 of 331


(a) Only a (b) Only b (c) Both a & b (d) Neither a nor b (e) Cannot be determined

953. What to make of the killing of cuddly marsupials?


a. Australians are not, as they sometimes joke, the only people to eat their iconic national animal. Swedes munch

on moose; in Spain, bull-tail stew is a delicacy. But the culling of kangaroos divides opinion Down Under. Many
view the marsupials as pests which destroy pasture and cause crashes by hopping in front of cars. Animal-rights

types counter that killing them is inhumane, and that kangaroo meat is rife with bacteria. Both sides are hopping
mad about a new film, which shows botched slaughters and suggests that hunting is diminishing the population.
“We’ve learned how polarizing the subject is in Australia,” Mick McIntyre, one of its makers, told a local paper

b. Annual aerial surveys suggest that there are more than 47m kangaroos bounding through the outback, making

them some of the most abundant large vertebrates on earth. Their natural predators, such as dingoes, are scarce,
so when the vegetation they eat is abundant, their numbers jump. State governments have long set “harvesting”

quotas to keep the four most populous species in check. But some ecologists suggest that the culls are
damaging, and that the population estimates are over-optimistic.

(a) Only a (b) Only b (c) Both a & b (d) Neither a nor b (e) Cannot be
determined

954. What could go wrong in the great summit?


a. Negotiated with America, starting with the foreign minister, Ri YongHo. MrTrump has not nominated an
ambassador to South Korea and the top North Korean envoy at the State Department recently resigned. Mr

Trump sacked his Secretary of State, Rex Tillerson, without consulting him about his decision to agree to a
summit. American experts on Korea and concerned foreign allies alike expect their advice to be shrugged off by
the president, a man bored by briefings, suspicious of alliances and focused (his own advisers admit) on quick

wins that make him look good.


b. Daniel Russel, a career diplomat, ran North Korea policy under President Barack Obama. If briefing Mr Trump,

he would stress that America is strengthened by its alliances with South Korea and Japan. When North Korea

talks about America needing to abandon “hostile policies”, that is a trap, he adds: code for removing American
troops from Korean soil and ditching defence treaties with Asian allies. Korea-hands worry that it is de-escalation

that Kim Jong Un is selling, not denuclearization. They fear he wants his country to be accepted as a nuclear-
armed power, in keeping with his yearning to meet America’s president as an equal. One possible ploy might be
to for swear missiles that can hit American cities but to preserve a small nuclear arsenal, perhaps under

www.ibpsguide.com https://estore.ibpsguide.com Page 309 of 331


international supervision. Such a deal could cause other Asian countries to seek nuclear arms and raise the

spectre of nuclear smuggling

(a) Only a (b) Only b (c) Both a & b (d) Neither a nor b (e) Cannot be determined

955. As regulations are tighten, unregistered congregations steel themselves


a. Xu Yonghai’s flock gathers weekly to worship in his small studio apartment in west-central Beijing. On a chilly

winter morning a dozen people climb the concrete stairs to his door, dump their coats on his Snoopy bed sheets
and gather around a table laid with tea and Bibles. The service begins with some devotional songs, accompanied

by music from a battery-powered speaker. The pocket-sized gadget packs up halfway through the medley, forcing

the pastor to dig out a spare. Many tight-knit services such as this one take place across China each week. The
small congregation meets without the permission of the Three-Self Patriotic Movement, a government umbrella

under which all China’s Protestant congregations are supposed to huddle. It meets on Fridays rather than
Sundays, an arrangement considered less likely to provoke officials.
b. Although Christians are growing more numerous, the wriggle room allowed to them is shrinking. Of most recent

concern is a revised set of religious regulations that came into force in February. The old rules had stopped short
of explicitly outlawing informal religious gatherings, but the new ones state more clearly that unregistered
churches are beyond the pale. Fearing a clampdown, some bigger churches have split their congregations into
small house groups that they think officials will find less bothersome, says Fan Yafeng, a pastor and legal scholar.
Others are appointing chains of substitute ministers and managers to keep things running should the main ones
be arrested.

(a) Only a (b) Only b (c) Both a & b (d) Neither a nor b (e) Cannot be determined

956. States are finding new ways of killing enemies abroad, and of justifying their acts
a. As Europeans know to their lamentable cost, assassinations can start wars, even world wars. A bullet fired by a

Serbian nationalist, killing Austria’s archduke in June 1914, sparked the calamitous First World War which

arguably paved the way to the second. Earlier assassinations may have drastically altered the course of history,
too. The bomb thrown in 1881 at Tsar Alexander II, who had emancipated the serfs, woefully stymied reform in

Russia. More recently, the murder in 1961 of Patrice Lumumba, the Congolese prime minister, often blamed on
the CIA, helped set that country on its path to mayhem. The killing in 1994 of Rwanda’s president, Juvé- nal
Habyarimana, set off Africa’s worst genocide. The murder of Israel’s Prime minister, Yitzhak Rabin, by a Jewish

fanatic the following year dimmed the prospect of peace between Israelis and Palestinians. The assassination in

www.ibpsguide.com https://estore.ibpsguide.com Page 310 of 331


2007 of Benazir Bhutto, when she was bidding to become Pakistan’s prime minister, stalled her country’s efforts

to build democracy
b. Yet an attack does not have to be on a head of state to prove a political shock. The phenomenon of state-

sanctioned attacks on perceived enemies at home, but especially abroad, has recently concentrated the minds of

lawyers and policymakers. The neurochemical attack this month on Sergei Skripal, a retired Russian double
agent, in Salisbury, a sleepy British cathedral city, is just the latest in a line of brazen incidents. On March 12th

Britain’s prime minister, Theresa May, told Parliament that the Russian state was “highly likely” to have been the
perpetrator. Two days later she announced the expulsion of23 Russian diplomats

(a) Only a (b) Only b (c) Both a & b (d) Neither a nor b (e) Cannot be determined

957. Alliance between two biggest carmakers - The tricky task of drawing closer together.

a. Renault unveiled the EZ-GO, a concept for a robotaxi, at the Geneva motor show, which opened on March 5th.
Nissan, in conjunction with DeNA, a Japanese software firm, recently began trials of driverless taxis in Japan. The
two companies are pursuing their own paths towards the future of mobility. Yet both are bound together in a close
alliance, which celebrates its 20th anniversary next year. In 2016 they were joined by Mitsubishi. Last year the trio
sold 10.6m cars between them, one in every nine worldwide. It is a unique car making liaison, neither a full
merger nor as loose as the many tie ups forged to spread the cost of developing pricey pieces of technology.

Each firm remains autonomous but shares a growing number of links in the supply chain with the other two. It all
looks hugely successful. In 2017 Renault-Nissan-Mitsubishi overtook Volkswagen (VW) as the world’s biggest car
company

b. Yet enthusiasm for the alliance among petrol heads and analysts is muted. Despite making some sporty
models—Renault even runs a Formula 1 team—the group lacks a brand such as VW’s Porsche to set car-buyers’
pulses racing. Those who pore over its financial performance use words like “decent” and “reasonable”. The mass
market is competitive and margins are low. Investors cringe at a complex structure. Renault owns a controlling
43.4% of Nissan; Nissan has a non-voting 15% stake in Renault. Mitsubishi is controlled by Nissan through a 34%

stake. Carlos Ghosn is chairman of all three firms. Last year he stepped down as boss of Nissan but still runs

Renault, plus the alliance itself, with its own board and executives.

a) Only a b) Only b c) Both a & b d) Neither a nor b e) Cannot be determined

958. Protectionism and competition fears may impede carrier partnerships

www.ibpsguide.com https://estore.ibpsguide.com Page 311 of 331


a. As Amercia’s oldest airline still aloft, Delta makes much of its southern roots. At its biggest hub, Atlanta airport,

the company museum recounts how it became the world’s second-biggest carrier. The answer: by buying up
domestic rivals. With few takeover targets left at home, Delta’s chief executive, Ed Bastian, is looking abroad. But

his plans for more foreign joint ventures (JVs) face regulatory headwinds. Last year Mr Bastian announced a flurry

ofJVs. In May Delta launched one with Aeromexico and in June another with Korean Air.
b. Expansion plans like Mr Bastian’s may also run up against rising protectionism. After Britain leaves the EU, it

will have to negotiate a new open-skies agreement with America. On March 6th the Financial Times revealed that
the Trump administration wants stricter terms for Britain. Under America’s proposed ownership rules, if a planned
purchase by Air France of 31% of Virgin goes ahead, it may no longer be British enough to qualify for the open-

skies agreement and thus for its JV with Delta. Although JVs are falling out of favor with authorities, airlines are
still keen on consolidation. With scale, “I don’t think we’re ever going to lose money again”, American’s boss
recently proclaimed. The carrier has duly resubmitted its JV with Qantas to regulators. At the Atlanta museum,
displays of crew uniforms and other items from the 40 defunct carriers Delta has swallowed hold a lesson for

airlines

a) Only a b) Only b c) Both a & b d) Neither a nor b e) Cannot be determined

959. Two German rivals make a pact

a. When Johannes Teyssen took control of E.ON in 2010, it was Germany’s second-biggest company after
Siemens, an industrial giant. From its headquarters in chic Düsseldorf, the utility looked down on RWE, its

longtime rival, based in Essen, a down-at-heel former coal-and-steel town 40 minutes’ drive away. The illusion of
superiority did not last. The following year Angela Merkel, Germany’s chancellor, reacted to the meltdown at
Fukushima in Japan by starting a process to shut down Germany’s nuclear power plants, on which both
companies relied.

b. The main criticism of the plan is that it should have happened two years ago. Uniper soared in value once spun
off, but Innogy struggled. After a profit warning in December, it jettisoned its chief executive (and architect of the

spin-off). Earlier this month it was rocked by an unexplained acid attack on Bernhard Günther, its chief financial

officer. Yet without the initial spin-offs, it hard to see how the deal could have happened. Markets have reacted
well, suggesting regulators and activist investors are not expected to put up a fight.

(a) Only a (b) Only b (c) Both a & b (d) Neither a nor b (e) Cannot be determined

960. Going Dutch

www.ibpsguide.com https://estore.ibpsguide.com Page 312 of 331


a. Proudly overlooking the River Thames, Unilever House looks more royal palace than office building. Built on

the site of a Tudor estate, for nine decades it has been the London home to Unilever, one of the world’s largest
consumer goods firms. Since a merger of British soap makers and Dutch margarine merchants in1929, Unilever

has been a dual nationality company. It is legally domiciled in Britain and the Netherlands, with headquarters in

both the London building and in Rotterdam. The appeal of dual citizenship has faded. After a year-long review, on
March 15th Unilever’s board announced plans to move its legal base to Rotterdam.

b. Many in the City of London finger Britain’s decision to leave the European Union for the move. But Graeme
Pitkethly, the firm’s chief financial officer, insisted Brexit was “absolutely not a factor” in the decision. Over the
past decade Unilever has been shifting its production facilities nearer to its customers in other parts of the world.

Even Britain’s possible exit from the EU’s customs union would not affect its operations as much as it once would.
Instead the company claims that the new structure will make it “simpler, more agile and more focused”. Under its
current structure, the shares of Unilever NV cannot be exchanged for those in Unilever PLC, its British sibling.
This makes it much harder to buy or spin off its businesses. Unilever’s chief executive, Paul Polman, wants to

boost profits by selling low-margin foods divisions and moving into more profitable areas such as personal-care
products. That makes a single entity alluring.

(a) Only a (b) Only b (c) Both a & b (d) Neither a nor b (e) Cannot be determined

961. A Silicon Valley darling is accused of misleading investors


a. “The Next Steve Jobs” is how Inc., an American business magazine, described Elizabeth Holmes when her
photograph appeared on its cover in 2015. They may share an affinity for black turtlenecks but the reputations of

Ms Holmes and Apple’s celebrated late boss could not be more different. On March 14th Ms Holmes was accused
of fraud by America’s Securities and Exchange Commission (SEC). She has agreed to pay a $500,000 fine, not
serve as an officer of a public company for ten years and turn over much of her stake in Theranos, the startup she

founded (she has neither admitted nor denied wrongdoing).


b. Ms Holmes wooed investors while sharing few details about how exactly her technology worked. Today they

are being more inquisitive and cautious, especially in health care. The SECis eyeing Silicon Valley’s firms more

closely for foul play, too, for example by asking to see how venture-capital firms mark their investments and how
startups value their private shares. The time when plucky wannabe tech titans could do no wrong is gone. Ms
Holmes has certainly left a mark on Silicon Valley.

(a) Only a (b) Only b (c) Both a & b (d) Neither a nor b (e) Cannot be determined

www.ibpsguide.com https://estore.ibpsguide.com Page 313 of 331


962. Online starlets are refashioning Chinese e-commerce

a. In November Ms Fang launched her own clothing line. Part of her motivation, she says, was that the brands
she endorsed did not always match the trends she was sharing with her followers. By creating her own marque,
Becky’s Fantasy, she retains full control of the quality. She also gains a new revenue stream. For the time being

only 3-5% of wanghong follow Ms Fang’s entrepreneurial example, iiMedia Research reckons. But it expects the
model to become an industry in its own right, straddling entertainment and e-commerce, and driven by online data
b. An industry is indeed springing up to assist the internet starlets. Dedicated wanghong incubators, of which

China now has around 50, seek out promising candidates, help them hone online business models and act as
supply-chain managers, as well as agents. They help with hiring designers, sourcing fabrics and finding factories.

In 2016 Alibaba, China’s biggest e-commerce group and owner of the Taobao emporium, invested 300m yuan in

Ruhnn, an incubator which was valued at 3.1bn yuan at the time and has since inked contracts with a few dozen
influencers.

(a) Only a (b) Only b (c) Both a & b (d) Neither a nor b (e) Cannot be determined

963. Which firms make the biggest killing from America’s health-care system?
a. Every year America spends about $5,000 more per person on health care than other rich countries do. Yet its
people are not any healthier. Where does all the money go? One explanation is waste, with patients wolfing down

too many pills and administrators churning out red tape. There is also the cost of services that may be popular
and legitimate but do nothing to improve medical outcomes. Manhattan’s hospitals, with their swish reception
desks and menus, can seem like hotels compared with London’s bleached Victorian structures.

b. In crude terms, the health-care labyrinth comprises six layers, each involving the state, mutual organizations
and private firms. People and employers pay insurance companies, which pay opaque aggregators known as
pharmacy-benefit managers and preferred provider organizers. They in turn pay doctors, hospitals and

pharmacies, which in turn pay wholesalers, who pay the manufacturers of equipment and drugs. Some
conglomerates span several layers. For example on March 8th Cigna, an insurance firm, bid $67bn for Express

Scripts, a benefit manager. A system of rebates means money flows in both directions so that the real price of

products and services (net of rebates) is obscured.

(a) Only a (b) Only b (c) Both a & b (d) Neither a nor b (e) Cannot be determined

964. America’s public markets are perking up. Can it last?

www.ibpsguide.com https://estore.ibpsguide.com Page 314 of 331


a. For years, discussions of America’s public markets have usually featured a lament for their dwindling appeal.

According to Jay Ritter of the University of Florida, the number of publicly listed companies peaked in 1997 at
8,491. By 2017 it had slumped to 4,496. True, many of the companies that went public in the internet’s early days

should never have done so. But the decline worries anyone who sees public markets as the best way for ordinary

investors to benefit from the successes of corporate America.


b. The mood right now is more buoyant. Bankers and lawyers who usually chat with journalists in their offices are

on the road hunting for business, offering only snatched interviews from airports in cities that they are unwilling to
disclose. “There are plenty of signs that IPO activity is about to surge,” says Kathleen Smith of Renaissance
Capital, a research firm. The line-up of listings spans countries and industries. The biggest offering in America so

far this year has been that of PagSeguro Digital, a Brazilian e-commerce platform. Among those imminent is
Dropbox, a file-sharing service. If rumour is right, Lyft, a ride-sharing app, maysoon follow. Next month Spotify, a
Swedish music streaming service, is due to make its debut on the New York Stock Exchange in an unusual “direct
listing”.

(a) Only a (b) Only b (c) Both a & b (d) Neither a nor b (e) Cannot be determined

965. A primer on blockchain-based versions of central-bank money.


a. Bitcoin, Ethereum, XRP, Stellar, Cardano: the infant world of cryptocurrencies is already mind-bogglingly
crowded. Amid the cacophony of blockchain-based would-be substitutes for official currencies, central banks from
Singapore to Sweden have been pondering whether they should issue digital versions of their own money, too.
None is about to do so, but a report prepared by central bank officials from around the world, published by the

Bank for International Settlements on March 12th—a week before finance ministers and central-bank heads from
G20 countries meet in Buenos Aires— offers a guide to how to approach the task.

b. For a start, it matters who will be using these central bank digital currencies (CBDCs). Existing central-bank
money comes in two flavors: notes and coins available to anyone; and reserve and settlement accounts open only
to commercial banks, already in electronic form (though not based on blockchain) and used for interbank

payments. Similarly, CBDCs could be either widely available or tightly restricted. A CBDC open to all would in
effect allow anyone to have an account at the central bank.

(a) Only a (b) Only b (c) Both a & b (d) Neither a nor b (e) Cannot be determined

966. Do credit booms foretell crises?

www.ibpsguide.com https://estore.ibpsguide.com Page 315 of 331


a. On the morning of December 7th 1941, George Elliott Junior noticed “the largest blip” he had ever seen on a

radar near America’s naval base at Pearl Harbour. His discovery was dismissed by his superiors, who were thus
unprepared for the Japanese bombers that arrived shortly after. The mistake prompted urgent research into

“receiver operating characteristics”, the ability of radar operators to distinguish between true and false alarms. A

similar concern motivates research at the Bank for International Settlements (BIS) in Basel, Switzerland. Its
equivalent to the radar is a set of economic indicators that can potentially detect the approach of financial crises

b. One obvious explanation for China’s resilience is that its credit is mostly homegrown, extended by domestic
banks and other Chinese lenders. By contrast the crisis-struck emerging economies mostly relied on inflows of
foreign capital to finance their current-account deficits with the rest of the world. Looking at both current-account

gaps and credit gaps may provide better predictions, says Michael Spencer of Deutsche Bank. He calculates that
China’s risk of a financial crisis this year is less than 8% (assuming a credit gap of under 13%) partly because it
runs a current-account surplus of about 1.4% of GDP. If China’s government keeps credit stable as a share of
GDP this year, this crisis-risk could fall to about 5%.

(a) Only a (b) Only b (c) Both a & b (d) Neither a nor b (e) Cannot be determined

967. Why so much land has no official owner?


a. The government is considering a few moves that might help. One is to make the registration of inherited land
compulsory. That, at least, would force new owners to pay taxes. Faced with vast, complicated public-works
projects such as rebuilding Tohoku, it has lowered the legal barriers to seizing vacant land, and become more
aggressive in doing so. This is simply an acknowledgment that the legal system is creaking, says Mr Masumoto.

Yet more action will be needed. Overgrown lots, many hosting decrepit houses, are popping up in towns and even
some big cities across Japan, says Hiroya Masuda, a former minister of internal affairs who helped draw up the

report. As more people die and pass on titles to unwitting or unwilling relatives—the number of deaths in Japan is
expected to peak at 1.67m in 2040—the growing swathes of unclaimed land could overwhelm the state, he fears.
b. The countryside is littered with vacant plots and empty houses. Some date from Japan’s great post-war

migration to coastal cities; others were abandoned more recently, as urbanization continued and as the population
has shrunk and aged. Ownership has often passed through several generations and the thread may have been
lost. Some titles in Tohoku had not changed since the1860s.

(a) Only a (b) Only b (c) Both a & b (d) Neither a nor b (e) Cannot be determined

968. A political coalition in favor of protectionism

www.ibpsguide.com https://estore.ibpsguide.com Page 316 of 331


a. The post-war system of global trade has been close to expiring, seemingly, for most of the post-war period. It

tottered in the 1980s, when Ronald Reagan muscled trading partners into curbing their exports to America. It
wobbled with the end of the fruitless Doha round of trade talks. The system now faces the antediluvian economics

of President Donald Trump, who seems bent on its destruction. Mr Trump’s mercantilism is gaining steam.

Straight after saying he would slap hefty tariffs on aluminium and steel imports, he is setting his sights on China, a
favorite stump-speech bogeyman. This week he blocked the takeover of an American chip maker by a

Singaporean rival, because of fears of Chinese technological leadership. He is poised to act against China over
its theft of intellectual property and its trade surplus.
b. When discussing trade-policy trade-offs, economists typically focus on conflicts between producers and

consumers. They see consumers benefiting from liberal trade rules, enjoying foreign wines and cheap Chinese
electronics. But households, they reckon, are rarely animated enough about trade fights to mount serious
opposition to producers, who are assumed to favor protection and who are highly motivated and organized in their
lobbying for tariffs and other barriers. Yet in practice, interests diverge across industries and regions. In America,

as Douglas Irwin describes in his magisterial history of trade policy, “Clashing over Commerce”, battles between
blocs determined trade strategy. Before the civil war Democratic, export-oriented southern states held the political
upper hand over the pro-tariff, industrializing states of the north, which tended to vote for the Republican Party
and its precursors

(a) Only a (b) Only b (c) Both a & b (d) Neither a nor b (e) Cannot be determined

969. Creation hardier varieties of coral.

a. By some estimates, half of the world’s coral has been lost since the 1980s. Corals are delicate animals, and are
succumbing to pollution and sediment from coastal construction. Also to blame are sewage, farmland run-off and

fishing, all of which favor the growth of the big, fleshy algae that are corals’ main competitors for space. (The first
two encourage algal growth and the third removes animals that eat those algae.) But the biggest killer is warming
seawater. Ocean heat waves in 2015, 2016 and 2017 finished off an astonishing 20% of the coral on Earth

b. One way to do this would be selective breeding. Most species of coral spawn on just one or two nights a year, a
process regulated by the lunar cycle, the time of sunset and the temperature of the water. The sperm and eggs
released during spawning meet and unite, and the results grow into larvae that search for places where they can

settle down and metamorphose into the stone-encased sea-anemone-like polyps that are the adult form. In the
wild, the meeting of sperm and egg is random. Some researchers, however, are trying to load the dice. By starting
with wild specimens that have survived a period of heat which killed their neighbors, they hope to breed heat
resistance into the offspring.

www.ibpsguide.com https://estore.ibpsguide.com Page 317 of 331


(a) Only a (b) Only b (c) Both a & b (d) Neither a nor b (e) Cannot be determined

970. More experienced judges are more generous

a. Students are widely judged on their abilities before being allowed to enter top universities. Athletes are
assessed on their physical prowess before being awarded medals. And academic papers, like those reported in
this section, must run the gauntlet of peer review before being published. In making their determinations,

evaluators study that which they are judging in a sequence, one student, athlete or paper after another, and apply
standardized criteria. This approach is supposed to afford equal treatment to all. But research just published in
Psychological Science by Kieran O’Connor and Amar Cheema of the University of Virginia suggests that it is

actually biased in favor of those who are judged late in the process
b. On the face of it, the chance of any of that debris hitting someone is low. For one thing, the world is mostly
ocean. For another, even on land people are small and scarce compared with the available area. Nor, indeed, is

anyone known to have been injured by re-entering debris since the space age began (though someone has been
hit but not hurt). The risk of such injury cannot, however, be ruled out—and the chance of damage to property,
which occupies a larger fraction of Earth’s surface than people do, is proportionately higher.

(a) Only a (b) Only b (c) Both a & b (d) Neither a nor b (e) Cannot be determined

ANSWERS:
941. Correct Answer is: (a)

942. Correct Answer is: (c)


943. Correct Answer is: (b)
944. Correct Answer is: (a)

945. Correct Answer is: (c)


946. Correct Answer is: (c)

947. Correct Answer is: (b)

Statement b aptly describes the ‘foreign policy meets America first.’


948. Correct Answer is: (a)

949. Correct Answer is: (b)

Statement b tells us why constitutions are required as it exemplifies the problems and benefits regarding the same
950. Correct Answer is: (a)

951. Correct Answer is: (c)

www.ibpsguide.com https://estore.ibpsguide.com Page 318 of 331


952. Correct Answer is: (b)

953. Correct Answer is: (d)


Both mention the situation but don’t lay out anything regarding the repercussions.
954. Correct Answer is: (c)

955. Correct Answer is: (a)


Only a talk of how they steel themselves.
956. Correct Answer is: (a)
Only a talks of the theme mentioned in a focused manner.
957. Correct Answer is: (b)

Only b talks of the said tricky situation

958. Correct Answer is: (b)


Only B correctly focuses on the theme

959. Correct Answer is: (a)


960. Correct Answer is: (c)
961. Correct Answer is: (a)
962. Correct Answer is: (c)
963. Correct Answer is: (b)
964. Correct Answer is: (c)

965. Correct Answer is: (b)


966. Correct Answer is: (d)
967. Correct Answer is: (a)

968. Correct Answer is: (b)


969. Correct Answer is: (b)
970. Correct Answer is: (d)

Synonyms and Antonyms Type - 1

Directions (971 - 1000): In each of the following questions four words are given, of which two words are most

nearly the same or opposite in meaning. From the given options, find out the pair of words which is similar or
opposite to each other.

971). A. Murkiest

www.ibpsguide.com https://estore.ibpsguide.com Page 319 of 331


B. Dismay

C.Obscure
D. Scuff.
(a) A-B (b) B-D (c) A-C (d) C-D (e) B-C

972) .A. Ostracize


B. Emotive
C. Patronize
D. Unwholesome.

(a) A-C (b) B-D (c) A-B (d) C-D (e) B-C

973). A) Menace

B) Purgative
C) Angst
D) Laxative.
(a) A-C (b) B-D (c) A-B (d) C-D (e) B-C

974). A) Adept

B) Adopt
C) Adapt
D) Adroit.

(a) A-C (b) B-D (c) A-B (d) A-D (e) B-C

975). A) Badinage

B) Debacle
C) Entangle

D) Innocuous.

(a) A-C (b) B-D (c) A-B (d) A-D (e) B-C

976) . A) Effulgent

B) Affluent
C) Dazzling

D) Echelon

www.ibpsguide.com https://estore.ibpsguide.com Page 320 of 331


(a) A-D (b) B-D (c) A-B (d) A-C (e) B-C

977). A) Flummox
B) Perplexed

C) Enlighten
D) Bandy
(a) A-D (b) B-D (c) A-B (d) A-C (e) B-C

978). A) Malcontent

B) Pliable

C) Brittle
D) Exultant.

(a) B-C (b) B-D (c) A-B (d) A-C (e) A-D

979) . A) Choke
B) Snub
C) Ignore
D) Clinch

(a) C-D (b) B-C (c) A-B (d) A-D (e) B-D

980). A) Tar

B) Arrogate
C) Abrogate
D) Annul

(a) B-C (b) C-D (c) A-B (d) A-C (e) A-D

981). A) Mendacious

B) Memoir
C) Mendicant

D) Veracious

(a) B-C (b) C-D (c) A-B (d) A-C (e) A-D

982). A) Stymied

www.ibpsguide.com https://estore.ibpsguide.com Page 321 of 331


B) Impede

C) Infamy
D) Stylish
(a) B-C (b) C-D (c) A-B (d) A-C (e) A-D

983). A) Huddle
B) Maltreat
C) Liberate
D) Coddle.

(a) B-C (b) B-D (c) A-B (d) A-C (e) A-D

984). A) Sturdy

B) Flimsy
C) Pamper
D) Content
(a) B-C (b) B-D (c) A-B (d) A-C (e) A-D

985). A) Zigzag

B) Strained
C) Meandering
D) Deterrent

(a) B-C (b) B-D (c) A-B (d) A-C (e) A-D

986). A) Bigoted

B) Partisan
C) Peril

D) Despicable

(a) B-C (b) B-D (c) A-B (d) A-C (e) A-D

987). A) Suspicion

B) Connivance
C) Intrigue

D) Set up

www.ibpsguide.com https://estore.ibpsguide.com Page 322 of 331


(a) B-C (b) B-D (c) A-B (d) A-C (e) A-D

988). A) Paradigm
B) Spirit

C) Exemplar
D) Precedent
(a) B-C (b) B-D (c) A-B (d) A-C (e) A-D

989). A) Relegate

B) Disarrange

C) Lowering
D) Avoiding

(a) B-C (b) B-D (c) A-B (d) A-C (e) A-D

990). A) Orchestrate
B) Sequence
C) Disarrange
D) Obscure

(a) B-C (b) B-D (c) A-B (d) A-C (e) A-D

991). A) Jettison

B) Assent
C) Deliberate
D) Discard

(a) B-C (b) B-D (c) A-B (d) A-C (e) A-D

992). A) Grotesque

B) Whimsical
C) Reluctant

D) Liberate

(a) B-C (b) B-D (c) A-B (d) A-C (e) A-D

993). A) Deterrent

www.ibpsguide.com https://estore.ibpsguide.com Page 323 of 331


B) Apprehensive

C) Paltry
D) Prophylactic
(a) B-C (b) B-D (c) A-B (d) A-C (e) A-D

994). A) Belittle.
B) Magnify
C) Hazardous
D) Nonchalant.

(a) B-C (b) B-D (c) A-B (d) A-C (e) A-D

995). A) Façade

B) Extol
C) Abhor
D) Superficial.
(a) B-C (b) B-D (c) A-B (d) A-C (e) A-D

996). A) Nefarious

B) Virtuous
C) Precarious
D) Gregarious.

(a) B-C (b) B-D (c) A-B (d) A-C (e) A-D

997). A) Fracas

B) Scrutiny
C) Withstand

D) Scuffle.

(a) B-C (b) B-D (c) A-B (d) A-C (e) A-D

998). A) Strained

B) Safeguard
C) Amiable

D) Uncontrived

www.ibpsguide.com https://estore.ibpsguide.com Page 324 of 331


(a) B-C (b) B-D (c) A-B (d) A-C (e) A-D

999) . A) Beguile
B) Dazzle

C) Deluge
D) Devious.
(a) B-C (b) B-D (c) A-B (d) A-C (e) A-D

1000). A) Repercussions

B) Aftermath

C) Resentment
D) Vicious

(a) B-C (b) B-D (c) A-C (d) A-B (e) A-D

ANSWERS:

971. Correct Answer is: (c)


Murkiest - Questionable, suspicious, mysterious, dark, indistinct, dishonest
Obscure- Dark, faint, hidden, inconspicuous. Enigmatic, esoteric.Both of the words are similar in meaning.
So, Ans is option -(c) as both of the words are synonyms of each other.
Dismay- A sudden or complete loss of courage & firmness in the face of trouble /danger.

Scuff– scrape, to hit lightly,rub, scrape, brush the surface (of an object, ball) against something

972. Correct Answer is: (a)


Ostracize – reject, exile, shun, Shove.
Patronize- support, sponsor, consent, acquiesce.

So, Ans is option (a) as both the words are antonyms of each other.

Emotive - inflammatory, controversial, contentious, emotional.


Unwholesome- Not characterized by or conducive to health or moral well-being.

973. Correct Answer is: (b)


Purgative- emotional, healthy, laxative.

Laxative- Evacuant.

www.ibpsguide.com https://estore.ibpsguide.com Page 325 of 331


So, Ans is option - (b) as both of the words are synonyms of each other.

Angst- feeling of extreme pain.


Menace- danger, peril, risk.

974. Correct Answer is: (d)


Adept- skillful, adroit.
So, Ans is option - (d) as both of the words are synonyms of each other.

Adopt- legally take (another's child) and bring it up as one's own, accept, ratify, arrogate.
Adapt- to change & make suitable for a new use, to adjust to a new situation.

975. Correct Answer is: (a)


Badinage- Playful, teasing, talk, banter
Entangle- mire, involve, slack, difficulty.

So, Ans is option - (a) as both of the words are antonyms of each other.
Debacle- Failure, Disintegration
Deluge- Surround.

976. Correct Answer is: (d)

Effulgent- shining brilliantly, radiant, dazzling


So, Ans is option - (d) as both of the words are synonyms of each other.

Affluent- moneyed, rich, wealthy.


Echelon- Degree, place, situation.

977. Correct Answer is: (c)

Flummox- confuse, perplexed, perplexed.


So, Ans is option - (c) as both of the words are synonyms of each other.

Enlighten- apprise, advice, acquaint.

Bandy- to discuss ideas, argue in a causal way.

978. Correct Answer is: (e)

Malcontent- a rebel, dissatisfied, restless, disgruntled.


Exultant- filled with or expressing great joy or triumph: jubilant.

So, Ans is option - (e) as both of the words are antonyms of each other.

www.ibpsguide.com https://estore.ibpsguide.com Page 326 of 331


Pliable- flexible

Brittle- rigid, hard.

919. Correct Answer is: (b)

Snub - means an act of rebuffing or ignoring someone or something. Both snub and ignore are similar in meaning.
Choke- have severe difficulty in breathing because of a constricted or obstructed throat or a lack of air.
Clinch- confirm or settle (a contract or bargain).

980. Correct Answer is: (b)

Abrogate- Repeal, cancel, nullify, negate.

Annul- Repeal, negate.


So, Ans is option - (b) as both of the words are synonyms of each other.
Tar- cover, surface, finish
Arrogate- to claim or seize without justification.

981. Correct Answer is: (e)


Mendacious- lying, deceitful
Veracious- Honesty.

So, Ans is option - (e) as both of the words are antonyms of each other.
Mendicant- a beggar

Memoir- Personal reminiscences.

982. Correct Answer is: (c)


Stymied- Impede, hamper, hinder, prevent, inhibit.
Impede- frustrate, spoil, prevent.
So, Ans is option - (c) as both of the words are synonyms of each other.

Infamy- bad name, bad reputation, condemnation.

Stylish- fashionable, modish.

983. Correct Answer is: (b)

Maltreat- mishandle, treat badly, handle/treat roughly, knock


Coddle- content, delight, humor, indulge, pamper.

So, Ans is option - (b) as both of the words are antonyms of each other.

www.ibpsguide.com https://estore.ibpsguide.com Page 327 of 331


Huddle- Gather, Squeeze.

Liberate- set free, free, release, let out, let go, discharge, set/let loose, deliver, save, rescue, extricate;

984. Correct Answer is: (c)

Sturdy- marked by the ability to withstand stress without structural damage or distortion, bombproof.
Flimsy- Breakable, Unstable.
So, Ans is option - (c) as both of the words are antonyms of each other.

Pamper- gratify, palpable


Content- ideas, matter, satisfied, pleased.

985. Correct Answer is: (d)


Zigzag- Diffuse, wandering, roaming, digressive.
Meandering- winding, zigzag, leaping.
So, Ans is option - (d) as both of the words are synonyms of each other.
Strained- exhausted
Deterrent- safeguard.

986. Correct Answer is: (c)

Bigoted- biased, one sided, partial.


Partisan- Prejudiced, bigoted.

So, Ans is option - (c) as both of the words are synonyms of each other

Peril- danger
Despicable- shameful.

987. Correct Answer is: (a)


Connivance- conspiring, engaged in a secret plan, intrigue

Intrigue - complicity, connivance.

So, Ans is option - (a) as both of the words are synonyms of each other
Suspicion - doubt, a state of mental uneasiness and uncertainty.

Set up- the preparation and adjustment of machines for an assigned task, project.

988. Correct Answer is: (d)

Paradigm- example, pattern, model

www.ibpsguide.com https://estore.ibpsguide.com Page 328 of 331


Exemplar- An exemplar is someone or something that is considered to be so good that they should be copied or

imitated
So, Ans is option - (d) as both of the words are synonyms of each other

Spirit- a supernatural being or essence, a special attitude or frame of mind


Precedent - Prior in time, order, significance.

989. Correct Answer is: (c)


Relegate- assign, delegate.
Disarrange- to disturb the arrangement or order of, disorganize.

So, Ans is option - (c) as both of the words are antonyms of each other

Lowering- to lower oneself


Avoiding - to neglect, ignore.

990. Correct Answer is: (d)


Orchestrate- Organize, arrange, plan
Disarrange- mess up.
So, Ans is option - (d) as both of the words are antonyms of each other
Sequence - succession, order.

Obscure – complex, difficult to understand.

991. Correct Answer is: (e)

Jettison- dump, discard.


Discard- to get rid of especially as useless or unwanted

So, Ans is option - (e) as both of the words are synonyms of each other.
Deliberate- meant, planned.
Assent- consent, permission.

992. Correct Answer is: (c)


Grotesque- distorted, whimsical, twisted, bent.

So, Ans is option - (c) as both of the words are synonyms of each other.

Reluctant- hesitate.
Liberate- to set free.

www.ibpsguide.com https://estore.ibpsguide.com Page 329 of 331


993. Correct Answer is: (e)

Deterrent – safeguard, protect.


Prophylactic - deterrent, safeguard.
So, Ans is option - (e) as both of the words are synonyms of each other.

Paltry- small. Insufficient.


Apprehensive- anxious, concerned, worried.

994. Correct Answer is: (c)


Belittle- to cause (a person or thing) to seem little or less, deprecate, depreciate, derogate, diminish.

Magnify -to increase in significance, to praise, laud.

So, Ans is option - (c) as both of the words are antonyms of each other.
Hazardous- dangerous, harmful
Nonchalant- cool, calm, indifferent.

995. Correct Answer is: (e)


Façade-a false, superficial, or artificial appearance or effect
So, Ans is option - (e) as both of the words are synonyms of each other.

Extol- to praise, to laud


Abhor – abominate. Hate.

996. Correct Answer is: (c)

Nefarious- wicked, vile, atrocious.


Virtuous- Celibate, moral, ethical.

So, Ans is option - (c) as both of the words are antonyms of each other.
Precarious- insecure, shady
Gregarious- outgoing, Friendly.

997. Correct Answer is: (e)


Fracas- scuffle, disturbance, quarrel

So, Ans is option - (e) as both of the words are synonyms of each other.
Scrutiny - study, search, investigation.
Withstand- to stand up against.

www.ibpsguide.com https://estore.ibpsguide.com Page 330 of 331


998. Correct Answer is: (d)

Strained- exhausted
Amiable- friendly, sociable, and congenial
So, Ans is option - (d) as both of the words are antonyms of each other.

Uncontrived- not showing the effects of planning or devising: having an artificial or unnatural appearance or
quality
Safeguard - to protect.

999. Correct Answer is: (c)

Beguile- Attract, Dazzle, Engage

So, Ans is option - (c) as both of the words are synonyms of each other.
Deluge- Besiege, surround.

Devious-Dishonest, Tortuous

1000. Correct Answer is: (d)


Repercussions - after-effects, by-product; consequences, effects, aftermaths.
So, Ans is option - (d) as both of the words are synonyms of each other.
Resentment - bitterness.
Vicious - cruel.

www.ibpsguide.com https://estore.ibpsguide.com Page 331 of 331

You might also like